The Polity Compendium - Disha Publication PDF

Download as pdf or txt
Download as pdf or txt
You are on page 1of 220

Downloaded From : www.EasyEngineering.

net

ww
w
The
.Ea
Polity
ngi
syE
Compendium
neer
i ng.
n et

Downloaded From : www.EasyEngineering.net


Downloaded From : www.EasyEngineering.net

• Head Office : B-32, Shivalik Main Road, Malviya Nagar, New Delhi-110017

• Sales Office : B-48, Shivalik Main Road, Malviya Nagar, New Delhi-110017
Tel. : 011-26691021 / 26691713

ww
Typeset by Disha DTP Team

w.E
a syE
ngi
nee
r ing
.ne
DISHA PUBLICATION
ALL RIGHTS RESERVED
t
© Copyright Publisher

No part of this publication may be reproduced in any form without prior permission of the publisher. The author and the
publisher do not take any legal responsibility for any errors or misrepresentations that might have crept in. We have
tried and made our best efforts to provide accurate up-to-date information in this book.

For further information about the books from DISHA,


Log on to www.dishapublication.com or email to [email protected]

Downloaded From : www.EasyEngineering.net


Downloaded From : www.EasyEngineering.net

CONTENTS
1. Constitutional Framework and Citizenship P-1-P-22
Historical Background; Making Salient & Features Of Indian Constitution; Parts Of Constitution

2. Fundamental Rights, Fundamental Duties and Directive Principles of State Policy P-23-P-40
Fundamental Rights; Cultural and Educational Rights; Directive Principles of the State Policy;
Fundamental Duties

3. Union Government P-41-P-92



ww
President; Vice President; Prime Minister; Council of Ministers; Attorney General of India;
Parliament Houses; Legislative Procedures; Parliamentary Committees


w.E
4. State Government
Introduction; Governor; Chief Minister; Council of Ministers (COM); Advocate General ;
P-93-P-118

a syE
Legislative Assembly (Vidhan Sabha); Centre-state Relations

5. Local Government P-119-P-136



ngi
Introduction; Panchayati Raj; 73rd Ammendment Act; Municipalities; 74th Amendment Act;
Commissions and Acts

6. Judiciary nee P-137-P-164



r ing
Introduction; Union Judiciary; Supreme Court; High court; Subordinate Courts; Other Local
Courts; Criminal Law (Amendment) Act, 2013

7. Miscellaneous Topics
.ne P-165-P-195
Introduction; Planning Commission; NITI Aayog; National Development Council (NDC); Finance
Commission; Election Commission; Political Parties and Pressure Group; National Commission
for SCs & STs; UPSC; SSC; Law Commission; The Lokpal and Lokayuktas
t
8. Amendments & Articles of the Indian Constitution P-196-P-216
Introduction; Amendment List; Article List

Downloaded From : www.EasyEngineering.net


Downloaded From : www.EasyEngineering.net

ww
w.E
a syE
ngi
nee
r ing
.ne
t

Downloaded From : www.EasyEngineering.net


Downloaded From : www.EasyEngineering.net

POLITY

CONSTITUTIONAL FRAMEWORK
1
AND CITIZENSHIP
Chapter
ww
Introduction
w.E
a syE
The term constitution is derived from latin word “constituere” which means to “to establish”.
Constitution means a document having a special legal sanctity, which sets out the framework, principles and functions of the
government. The idea of constitutionalism suggests ways and means to work out a government form, which exercises power

ngi
and ensures, at the same time, individual freedom and liberty.

nee
Indian Constitutional Framework

Historical Background
Making of
Indian Constitution r
Salient Features
of Constitution
ing Parts of Constitution

.ne
About
Constitution
Important
Acts
Sources Brief
description
of Important
features
Schedules
t
Important objective Important
Events Resolution Committees

Preamble Union Citizenship


and its
Territory

Related State Territory


Articles Reorganisation of India
Act

Downloaded From : www.EasyEngineering.net


Downloaded From : www.EasyEngineering.net
P-2 Constitutional Framework and Citizenship

HISTORICAL BACKGROUND
Constitutional
Important Provisions
Landmark
Regulating Act, 1773 • British government to regulate affairs of East India Co.
• Designated Governor of Bengal as Governor General of Bengal. Warren Hastings was the first
Governor General.
• Established a Supreme Court at Calcutta.
Pitts India Act, 1784 • Indian affairs under direct control of British government.
• Board of Control was established.
Charter Act of 1793 • Salary of company to drawn from the Indian exchequer.
• The Governor General and governors to override the decision of Councils.
• Company got monopoly of trade with India for another 20 years.
Charter Act, 1833 • Governor General of Bengal became Governor General of India.

ww •


Lord William Bentinck was the first Governor General of India.
The Act centralized British rule in India.
Created Government of India, with authority over all of British India.

Charter Act, 1853


w.E •

East India Co. lost its monopoly of tea trade and China trade.
The Indian Civil Services was founded.
• Separated legislative & executive functions of Governor General’s Council.

1858
a • Open competition for Indian Civil Services.

syE
• Patronage of the directors of the company ends.
Government of India, Act, • Act known as the Act for the Good Government of India.
• Company rule was replaced by British crown.

ngi
• Secretary of State for India was appointed to exercise the power of the Crown. He was a member
of British cabinet, responsible to it and was assisted by Council of India with 15 members.

nee
• Governor General became the agent of the crown and now known as Viceroy of India.
• Lord Canning became the first Viceroy of India.

Indian Councils Act, 1861 •





Parliamentary system started in India.

r
Indians became non-official members of the legislature.
Started decentralization of power.
ing
Recognition to the ‘Portfolio’ system, was introduced by Lord Canning in 1859.

Indian Councils Act, 1892 • Introduced indirect elections. Governor General could nominate members.
.ne
• Functions of Legislative Council Expanded to discuss the budget and ask questions to the executive.

Indian Councils Act, 1909


(Morley-Minto Reforms.
Lord Morley was then the
majority.
• Provincial legislative councils had a majority of non-official members.
t
• Central Legislative Council became imperial Legislative Council with officials forming the

secretary of state for India • Satyendra Prasad Sinha became the first Indian to join the Viceroy’s Executive Council.
and Lord Minto was then • Introduced communal representation for muslims with a separate electorate system. Legalized
the viceroy of India). communalism. Lord Minto created a communal electorate.
Government of India • Separated central subjects from provincial subjects.
Act, 1919/ Montague • Provincial subjects were transferred and reserved.
- Chelmsford Reforms. • Transferred subjects were administered by Governor with the help of ministers who were
Montague was the responsible to the legislature.
secretary of state for India • Reserved subjects were administered by Governor and Executive Council who were not
and Lord Chelmeford was responsible to the legislature.
the viceroy of India. • Dyarchy/ Dual system of government was introduced.
• Bicameral legislature with upper and lower houses were formed with direct elections.
• Majority of members in both houses were directly elected.
• 3 of the 6 members of governor-general’s council had to be Indians.

Downloaded From : www.EasyEngineering.net


Downloaded From : www.EasyEngineering.net
Constitutional Framework and Citizenship P-3

Government of India Act, • Established All India federation of provinces & princely states as constituent units.
1935 • It provided for the establishment of Reserve Bank of India, Federal Court, Public Service
Commission, Provincial Public Service Commission & Joint Public Service Commission for
two or more Provinces.
• Federal, provincial and concurrent were lists introduced.
• Abolished dyarchy in provinces which now had provincial autonomy.
• Introduced dyarchy at the centre and bicameralism in the provinces.
• Introduced responsible governments in provinces.
The August Offer, 1940 • Expansion of the Governor-General’s Executive Council to include more Indians.
• Establishment of an Advisory War Council.
Cripps Proposals, 1942 • Provision made up for participation of Indian States in the Constitution making body.
• The leaders of the principal sections of the Indian people were invited to take active and
effective participation in the councils of their country.
C.R. Formula, 1944 • C.R. Raja Gopalchari presented the ‘C.R. Formula’ by which the Muslim League would
support the Congress demand for complete freedom.
Wavell Plan, 1945 • Under this plan, India was to be granted Dominion status in the near future.
Lord
ww Attlee’s • On 15 March 1946, Lord Attlee declared that as the tide of nationalism was surging ahead in
Announcement, March 1946 India, it was in British interest to take positive action.

w.E
Cabinet Mission Plan, 1946 • There should be a Union of India, embracing both British India and the states which should
deal with foreign affairs.
• To set up Interim Government.
• A Constituent Assembly should be set up to draw up the future Constitution of the country.

1947 a
Indian Independence Act, • Declared India as independent & sovereign state.

syE
• Established responsible government at the Center & Provinces.
• Designated Governor General of India & Provincial Governors as Constitutional heads or
nominal heads.

ngi
• Lord Mountbatten became the first Governor General of free India. The first & last Indian
Governor General was C. Rajagopalachari.

S. No
Interim Government (1946)

Members Portfolios Held


nee
First Cabinet of Free India (1947)
Members Portfolios Held

1. Pt. Jawaharalal Nehru External Affairs and


Commonwealth Relations r
Pt. Jawaharlal Nehru

ing
Prime Minister, External Affairs
and Commonwealth Relation;
Scientific Research
2.

3.
Sardar Vallabhbhai
Patel
Dr Rajendra Prasad
Home, Information and
Broadcasting
Food and Agriculture
Sardar Vallabhbhai Patel
States
.ne
Home, Information and Broadcasting;

4.
5.
6.
Dr John Mathai
Jagijvan Ram
Sardar Baldev Singh
Industries and Supplies
Labour
Defence
Dr. Rajendra Prasad
Maulana Abul Kalam Azad Education
Dr. John Mathai
t
Food and Agriculture

Railways and Transport


RK Shanmugham Chetty Finance
Dr. BR Ambedkar Law
7. CH Bhabha Works, Mines and Power
Jagjivan Ram Labour
8. Liaquat Ali Khan Finance
Sardar Baldev Singh Defence
9. Abdur Rab Nishtar Posts and Air
10. Asaf Ali Railways and Transport Raj Kumari Amrit Kaur Health
11. C Rajagopalachari Education and Arts C.H. Bhabha Commerce
12. II Chundrigar Commerce Rafi Ahmed Kidwai Communication
13. Ghaznafar Ali Khan Health
Dr. Shyama Prasad Mukherji Industries and Supplies
14. Joginder Nath Mandal Law
V.N. Gadhil Works, Mines and Power

Downloaded From : www.EasyEngineering.net


Downloaded From : www.EasyEngineering.net
P-4 Constitutional Framework and Citizenship

Sources of Indian Constitution Indian Constitution.


• Out of 389 members, 296 were indirectly elected from
India constitution has borrowed its provisions from following British India and 93 were nominated by princely states.
sources. The Constituent Assembly had both nominated and
elected members. The elected members were indirectly
Country Provisions Borrowed
elected by members of the provincial assemblies.
Government of Federal scheme. ▪▪ First meeting of Constituent Assembly was held on
India Act, 1935 Declaration of emergency powers. Dec 9, 1946.
Ordinance defining the power of the ▪▪ Muslim League boycotted the Constituent Assembly.
President and Governors. ▪▪ Dr. Sachidanand Sinha, the senior most member of
Office of the Governor. the assembly, was elected as the temporary president
Power of federal judiciary. of the assembly.
Administration at the Centre and state level. ▪▪ Dr. Rajendra Prasad was elected as the permanent
president of the Assembly.
United Parliamentary system. ▪▪ Sir B. N. Rau was appointed as the legal advisor to
Kingdom Bicameral parliament. the Assembly.
Prime minister. ▪▪ An Objective Resolution’ was moved by Jawaharlal
Council of ministers. Nehru on Dec. 13, 1946, which later became the

ww Single citizenship.
Office of CAG.
Writ jurisdiction of courts.
Rule of law.
Preamble to the Constitution.
▪▪ On the 26th November, 1949 the Constitution
was declared as passed after the President of the
Assembly signed the document. Thus on 26th
USA
w.E
Written Constitution.
Fundamental rights.
Supreme Court.
November, 1949 the Constitution of India was
adopted. The commencement of the Constitution
occurred on 26th Jan. 1950.

a
President as executive head of the state.

syE
Impeachment of the President, removal of
SC and HC judges.
Vice President as chairman of Rajya Sabha.
▪▪ Provisions relating to citizenship, elections,
provisional parliament, and temporary provisions
became effective from 26th November, 1949.
▪▪ On January 24, 1950 the Constituent Assembly held
its final session. It had continued as a provisional

Australia
Judicial review, independence of judiciary.
Concurrent list. ngi •
parliament from 26 January, 1950 till the formation
of new parliament in May, 1952.
First ‘Draft Constitution of India’ was published in Feb,

USSR
Cooperative federalism.
Joint sitting of two houses of parliament.
Fundamental duties. • nee
1948. It was prepared by Sir B. N. Rau, Constitutional
Advisor to the Constituent Assembly.
Dr. B. R. Ambedkar is considered the father of the Indian
Weimer
Constitution of
Germany
Suspension of fundamental rights during
emergency.
Ballot system.

r
Constitution.

ing
The Constituent Assembly took almost 3 years (2 years,
11 months & 18 days) to draft the Constitution for
Independent India.
Canada Federal system.
Residuary powers.
Appointment of Governor.

.ne
It held 11 sessions covering a total of 165 days.

Objective resolution

South Africa

Ireland
Advisory jurisdiction of SC.
Procedure of constitutional amendment.
Electing member to Rajya Sabha.
Concept of directive principles of state
t
On January 22, 1947, the Constituent Assembly adopted
Objective Resolution proposed by Jawahar Lal Nehru. The
Objectives Resolution contained the fundamental propositions
of the Constitution and set forth the political ideas that should
policy. guide its deliberations.
Nomination of members to Rajya Sabha by The main principles of the resolution were:
the President. (i) India is an Independent, Sovereign, Republic;
Presidential election. (ii) India shall be a Union of erstwhile. British Indian
territories, Indian states and the parts outside British
MAKING OF INDIAN CONSTITUTION India and Indian states as are willing to be a part of the
Union.
• The idea of Constituent Assembly for making the (iii) Territories forming the Union shall be autonomous units
Constitution was first mooted by M. N. Roy in 1934. and exercise all powers except those assigned to or vested
Indian National Congress (INC) officially demanded the
in the Union.
formation of Constituent Assembly in 1935. The demand
was accepted, in principle, for a Constituent Assembly in (iv) All powers and authority of sovereign and independent
August offer of 1940. India and its Constitution shall flow from the people.
• Under the Cabinet Mission Plan, 1946 a Constituent (v) All people of India shall be guaranteed and secured
Assembly was constituted in Nov. 1946 for framing the social, economic and political justice; equality of status

Downloaded From : www.EasyEngineering.net


Downloaded From : www.EasyEngineering.net
Constitutional Framework and Citizenship P-5

and opportunities before law; and fundamental freedoms the parliament as well as ratification of not less than one-
of talk, expression, belief, faith, worship, vocation, half of the state legislatures (Article 368). Again, some
association and action - subject to law and public morality. provisions of the Constitution can be amended by the
(vi) The minorities, backward and tribal areas, depressed and parliament alone by a two-third majority.
other backward classes shall be provided adequate safe (iii) Parliamentary System of Government
guards. The Constitution provides for a parliamentary system of
(vii) The territorial integrity of the republic and its sovereign government under which the real executive power rests
rights on land, sea and air shall be maintained according with the Council of Ministers and the President is only a
to justice and law of civilized nations. nominal head. The Council of Ministers stay in office as
(viii) The land would make full and willing contribution of the long as they enjoy the confidence of the Parliament.
promotion of world peace and welfare of mankind. The framers of the Constitution decided to adopt a
parliamentary system of government for several reasons.
Important Committees Firstly, the system was already in existence in India and
people were well acquainted with its working.
COMMITTEE CHAIRMAN
Secondly, the vast size of the country and the diversity
Drafting Committee Dr. B.R. Ambedkar of its culture necessitated the adoption of a parliamentary
Members: (Father of the Indian form of government.

ww
• Alladi Krishnaswamy Ayyar
• N. Gopala Swami Ayyangar
• Dr. KM Munshi
Constitution) Thirdly, the desire to avoid conflicts between the
Executive and the Legislature, which was a common
features in America, also induced the members of the

• N Madhava Rao
• TT Krishna Machari
w.E
• Syed Mohammad Saadullah


Constituent Assembly to opt for a parliamentary system.
(iv) Federal System with a Unitary Bias
The Indian Constitution provides for a federation with
a strong centre. It is noteworthy that the Constitution
Flag Committee
a
Union Constitution Committee
J. B. Kriplani

syE
Jawaharlal Nehru
has not used the word ‘federation’, anywhere, and has
described India as a ‘Union of States’ which implies that
the Indian federation is not the result of any agreement
among the units and the units cannot secede from it. India
Provincial Constitution Committee

Union Powers Committee


Sardar Vallabh Bhai
Patel
Jawaharlal Nehru ngi possesses most of the federal features but also several of
the unitary features. The Indian federal structure acquires
a unitary character during emergency, when the normal
Committee on Fundamental Rights Sardar Vallabh Bhai
and Minorities Patel

nee
distribution of powers between the centre and the states
undergoes vital changes.
Therefore, it is said by KC wheare that India has a
Special Committee to Examine the (Chairman: Alladi
Draft Constitution
Finance & Staff Committee
Krishnaswamy Iyer)
Dr. Rajendra Prasad
r
quasi federal set up.

ing
Morris Jonnes called it as Bargaining Federalism &
Granville Austine called it as Cooperative Federalism.

Ad-hoc
Court
Committee on Supreme S. Varadachariar characteristics:
▪▪ Division of power. .ne
• India is a distinct federation with following

Ad-hoc Committee on National Flag Dr. Rajendra Prasad


Committee on Chief Commissioners’ B. Pattabhi
▪▪ Bicameral legislature.
▪▪ Supremacy of the constitution.
▪▪ Written constitution.
▪▪ Independency of Judiciary.
t
Provinces Sitaramayya • Constitution has a unitary bias with:
▪▪ Appointment of Governors by the centre.
SALIENT FEATURES OF INDIAN ▪▪ Parliament’s power to legislate in national
CONSTITUTION interest.
▪▪ Parliament’s power to form new states, change
(i) Bulkiest Constitution of the World their names and alter boundaries of existing
Indian Constitution is the one of longest Constitution in states.
the world. Originally it contained 395 Articles, 22 Parts ▪▪ Emergency provisions.
and 8 Schedules. After amendments till date, there are ▪▪ Single constitution.
more than 447 Articles, 24 Parts and 12 Schedules. ▪▪ Single citizenship.
(ii) Combination of Rigidity and Flexibility ▪▪ Integrated judiciary.
The Indian Constitution is a combination of rigidity and ▪▪ Comptroller and Auditor General.
flexibility. While some provisions of the Constitution (v) Fundamental Rights
can be amended by the Parliament by a simple majority, The Constitution contains an elaborate list of Fundamental
others require a two-third majority of the members of Rights. The state cannot make laws which take away or

Downloaded From : www.EasyEngineering.net


Downloaded From : www.EasyEngineering.net
P-6 Constitutional Framework and Citizenship

abridge any of the fundamental rights of the citizens. (xii) Emergency Powers
If it does so, the courts can declare such a law as The Constitution vests extraordinary powers in the
unconstitutional. It may be noted that the fundamental President during emergencies arising out of armed
rights granted by the Constitution are not absolute and rebellion or external aggression; emergency due to the
are subject to certain restrictions. In other words, the breakdown of constitutional machinery in the state;
Constitution seeks to strike a balance between individual and financial emergency when the credit of the country
liberty and social interests. is threatened. In fact, during emergency the federal
(vi) Fundamental Duties Constitution can virtually be converted into a unitary
The constitution also contains a list of 11 fundamental Constitution.
duties of the citizens. While ten of these duties (xiii) Single Citizenship
were added to the Constitution by the Forty Second
It provides single citizenship. All persons residing in
Amendment in 1976, the eleventh duty was added by the
different parts of the country are treated as Indian citizens
86th Constitutional Amendment Act (2002). These duties
and are entitled to the same rights of citizenship. There is
serve as constant reminders to the citizens that they have
no separate citizenship of different states.
to observe certain basic norms of democratic conduct.
(vii) Directive Principles of State Policy (xiv) Bicameral Legislature

ww
The Constitution outlines certain Directive Principles of
State Policy which the government has to keep in mind
while formulating any policy. These principles seek to
It provides a bicameral legislature at the Centre consisting
of the Lok Sabha and the Rajya Sabha. The former
contains representative of the people, while the latter

w.E
provide social and economic basis for democracy and the
establishment of a welfare state. Unlike the Fundamental
Rights, the Directive Principles of State Policy are non-
contains representatives of the states.
(xv) Special Provision of Minorities
The Consitution makes special provision for minorities,

a
justiciable, which implies that no action can be brought

syE
against the State before a court of law for its failure of
implementing the Directive Principles of States policy.
(viii) Secular State
Scheduled Castes, Scheduled Tribes, etc. It not only
reserves seats for them in the Parliament and State
legislatures, but also grants them certain special rights
and privileges.
The Constitution makes India a secular state. This means
that there is no state religion and the state is completely
detached from religious dogmas. It also implies that ngi (xvi) Panchayati Raj
The Constitution provides constitutional basis to
Panchayati Raj institutions as well as urban local bodies.
citizens are free to profess, practise and propagate any
religion. However, freedom of religion is not absolute
and the same can be regulated in the interest of the public.
nee
This was achieved through the seventy-third and
seventy-fourth amendments to the Constitution carried

(ix) Independent & Integrated Judiciary


The Constitution provides an independent judiciary which
ensures that the government is carried on in accordance
(xvii)
r
out in December 1992.

ing
Strike Balance between Constitutional Supremacy
and Parliamentary Sovereignty

with the provisions of the Constitution. Judiciary acts as


the guardian of the liberties and fundamental rights of the

.ne
The Indian Constitution combines two seemingly
contradictory principles of supremacy of Constitution,
(as obtains in USA), and Parliamentary sovereignty


citizens. It also determines the limits of the powers of the
centre and the states.
The Constitution provides a single integrated judiciary
with the Supreme Court at the top. Below the Supreme
t
(as obtains in Britain). The Supreme Court through its
power of judicial review can declare the laws passed by
the Parliament as unconstitutional. On the other hand, the
Parliament can amend major portions of the Constitution.
Court, there are high Courts at the state level. Under the
High Court, there are Subordinate courts. (xviii) Basic Structure
(x) People as Source of Authority Certain features of the Constitution are beyond the
The Constitution draws its authority from the people and amending powers of the Parliament. All laws and
has been promulgated in the name of the people. This is constitutional amendments which transgress the basic
evident from the Preamble which states ‘We, the people structure are liable to be struck down. Some of the
of India... do hereby adopt, enact and give to ourselves
major features of the basic structure include; supremacy
this Constitution.’
of the Constitution, republican form of government,
(xi) Universal Adult Franchise
secularism, federal character, sovereignty of the country,
The Constitution introduces universal adult franchise
parliamentary democracy, fundamental rights, directive
and accords the right to vote to all citizens above 18
years of age without discrimination. However, it makes principles etc. In Kesavananda Bharati Vs State of
reservation of seats for Scheduled Castes and Scheduled Kerala case 1973, Supreme Court propounded the
Tribes to provide them adequate representation. Doctrine of Basic Structure of the Constitution.

Downloaded From : www.EasyEngineering.net


Downloaded From : www.EasyEngineering.net
Constitutional Framework and Citizenship P-7

Schedules in Constitution of India


Numbers Subject Matter Articles Covered
First Schedule 1. Names of the States and their territorial jurisdiction 1 and 4
2. Names of the Union Territories and their extent.
Second Schedule Provisions relating to the emoluments, allowances, privileges and so on : 59, 65, 75, 97, 125, 148, 158,
164, 186 & 221
1. The President of India
2. The Governors of States
3. The Speaker and the Deputy Speaker of the Lok Sabha
4. The Chairman and the Deputy Chairman of the Rajya Sabha
5. The Speaker and the Deputy Speaker of the Legislative Assembly in
the states
6. The Chairman and the Deputy Chairman of the Legislative Council in
the states
7. The Judges of the Supreme Court
8. The Judges of the High Courts

ww
Third Schedule
9. The Comptroller and Auditor-General of India
Forms of Oaths or Affirmations for: 75, 84, 99, 124, 146, 173, 188
and 219

w.E
1. The Union ministers
2. The candidates for election to the Parliament
3. The members of Parliament

a
4. The judges of the Supreme Court

syE
5. The comptroller and Auditor-General of India
6. The state ministers

ngi
7. The candidates for election to the state legislature
8. The members of the state legislature

Fourth Schedule
9. The judges of the High Courts
nee
Allocation of seats in the Rajya Sabha to the states and the union territories 4 and 80
Fifth Schedule

Sixth Schedule
and scheduled tribes
r
Provisions relating to the administration and control of scheduled areas 244

ing
Provisions relating to the administration of tribal areas in the states of 244 and 275

Seventh Schedule
Assam, Meghalaya, Tripura and Mizoram.
Division of powers between the Union and the States in terms of List I 246
(Union List), List II (State List) and List III (Concurrent List). Presently, .ne
Eighth Schedule
the Union List Contains 100 subjects (originally 97), the state list contains
61 subjects (originally 66) and the concurrent list contains 52 subjects
(originally 47).
Languages recognized by the Constitution. 344 and 351
t
Originally, it had 14 languages but presently there are 22 languages.
They are: Assamese, Bengali, Bodo, Dogri (Dongri), Gujarati, Hindi,
Kannada, Kashmiri, Konkani, Mathili (Maithili), Malayalam, Manipuri,
Marathi, Nepali, Oriya, Punjabi, Sanskrit, Santhali, Sindhi, Tamil,
Telugu and Urdu. Sindhi was added by the 21st Amendment Act of 1967;
Konkani, Manipuri and Nepali were added by the 71st Amendment Act
of 1992; and Bodo, Dongri, Maithili and Santhali were added by the 92nd
Amendment Act of 2003.
Ninth Schedule Acts and Regulations (originally 13 but presently 282)19 of the state 31-B
legislatures dealing with land reforms and abolition of the Zamindari
system and of the Parliament dealing with other matters. This schedule
was added by the 1st Amendment (1951) to protect the laws included in
it from judicial scrutiny on the ground of violation of fundamental rights.
However, in 2007, the Supreme Court ruled that the laws included in this
schedule after April 24, 1973, are now open to judicial review.

Downloaded From : www.EasyEngineering.net


Downloaded From : www.EasyEngineering.net
P-8 Constitutional Framework and Citizenship

Tenth Schedule Provisions relating to disqualification of the members of Parliament and 102 and 191
State Legislatures on the ground of defection. This schedule was added
by the 52nd Amendment Act of 1985, also known as Anti-defection Law.
Eleventh Schedule Specifies the powers, authority and responsibilities of Panchayats. It has 243-G
29 matters. This schedule was added by the 73rd Amendment Act of
1992.
Twelfth Schedule Specifies the powers, authority and responsibilities of Municipalities. It 243-W
has 18 matters. This schedule was added by the 74th Amendment Act of
1992.

PARTS OF CONSTITUTION Justice


The preamble speaks of social, economic, and political justice.
Preamble Social justice implies that discrimination on the basis of caste,
The term Preamble refers to the introduction or preface to the race, sex or religion should cease.
Constitution. Economic justice implies that the gap between the rich and the
The Preamble is not enforceable in a court of law and, generally, poor is bridged, and that exploitation ceases.

ww
not considered a part of the Constitution, it provides a key to
the understanding and interpretation of the Constitution, it has,
therefore, been described as the soul of the Constitution. In
Political justice implies that all citizens should have equal
opportunity to participate in the political system.

w.E
cases of doubt the Supreme Court has referred to the Preamble
to elucidate vague aspects of the Constitution.
The Constitution of India is preceded by a Preamble which
Liberty
Democracy is closely connected with the idea of liberty.
Certain minimal rights must be enjoyed by every person in a

a
(i) indicates the source from which it derives authority; and

syE
(ii) state the objective which the constitution seeks to achieve.
It has been amended by the 42nd Amendment Act 1976 which
added 3 new words– Socialist, Secular & Integrity.
community for a free and civilised existence.

Equality
Rights have no meaning if they cannot be enjoyed equally by
‘We, the people of India having solemnly resolved to
constitute India into a sovereign, socialist, secular democraticngi all members of the community.

Fraternity
republic and to secure to all its citizens:
JUSTICE social, economic and political;
LIBERTY of thought, expression, belief, faith and worship; nee
Fraternity means a sense of brotherhood. Fraternity is also
sought to be promoted by ensuring equal rights to all. Fraternity
EQUALITY of status and of opportunity and to promote
among them all ;
FRATERNITY assuring the dignity of the individual and the
and respected.
r
is not possible unless the dignity of each individual is presented

The Union and its Territory ing


unity and integrity of the nation.
In our Constituent Assembly this twenty-sixth day of
.ne
Part 1 of the Constitution comprises four Articles concerned
with the territory of India. Article 1 stipulates that India, that
November, 1949 we do hereby adopt, enact and give to
ourselves this Constitution.

The word sovereign means that India is both internally as


t
is Bharat, shall be a Union of States. The states and territories
of India are to be specified in the First Schedule. It is to be
noted that the expression, ‘Union of India’, is not synonymous
with ‘the territory of India’; the ‘Union’ includes only the
States which are members of the federal system and share a
well as externally free and is not dependent upon any outside
distribution of powers with the Union while the ‘territory of
authority. The term ‘socialist’ in the Preamble (inserted by the
India’ includes the entire area over which the sovereignty of
42nd Amendment) refers to some form of ownership of means
India extends.
of production and distribution by the state. However, the Indian
The makers of the Indian Constitution, gave the Union
brand of socialism is quite different and holds faith in a mixed
Parliament the power to reorganise the States by a simple
economy. Secularism implies that the state is only concerned
procedure. In the original Constitution there were four
with relations between various citizens and is not concerned
categories of States and Territories. But since the Seventh
with relations of man with God. Further, it means that the state
Amendment Act, 1956, all the States (except for Jammu
has no religion of its own. The term Democratic implies that
and Kashmir) belong to one class and all the constitutional
the government draws its authority from the people. The rulers
are elected by the people and are accountable to them. The provisions relating to States apply to all of them in the same
word republic implies that the head of the state in India shall manner. As for the administration of the certain Scheduled
be an elected person and shall hold office for a fixed term. The Areas and Tribal Areas within the States, the provisions are
President of India is the chief executive head of India. specially listed in the Fifth and Sixth Schedules. The Union

Downloaded From : www.EasyEngineering.net


Downloaded From : www.EasyEngineering.net
Constitutional Framework and Citizenship P-9

Territories are centrally administered according to provisions 8. Haryana 23. Sikkim


contained in Part VII of the Constitution. They are governed
9. Himachal Pradesh 24. Tamil Nadu
by the President through an administrator appointed by him.
At present, there are 29 states & 7 Union Territories. 10. Jammu and Kashmir 25. Telangana
11. Jharkhand 26. Tripura
Reorganisation of States 12. Karnataka 27. Uttarakhand
Article 2 empowers Parliament to admit into the Union, or
13. Kerala 28. Uttar Pradesh
establish, new States on such terms and conditions as it thinks
fit. By Article 3, Parliament has the power by law to form a 14. Madhya Pradesh 29. West Bengal
new State from the territory of any State or by uniting two or 15. Maharashtra
more States, increase or decrease the area of any State, or alter
Union Territories
the boundaries or the name of any State. The only conditions
laid down for the making of such a law are that 1. Andaman & Nicobar Islands
(i) such a Bill must be introduced only on the recommendation 2. Chandiyarh
of the President, and 3. Dadra and Nagar Haveli
(ii) before recommending the Bill, the President is to refer it to 4. Daman and Diu
the concerned State Legislature which would express its views 5. Delhi (National Capital Territory)

ww
within a specified period. The President is not, however, bound
by the views of the State Legislature. However, in the case
of Jammu and Kashmir, the consent of the State Legislature
6. Lakshadweep
7. Puducherry

w.E
is required before a Bill on such alterations is introduced
in Parliament. A simple majority and ordinary legislative
procedure is enough for Parliament to form new States or alter
Andhra
Pradesh
Formation of states after 1950s
Formed by the State of Andhra Pradesh Act,
1953 by carving out some areas from the State

a
existing State boundaries. Thus the will of the Union Executive

the territories of the units of the federation.

States Reorganisation Act


syE
and Legislature prevails in matters of altering or redistributing Kerala
of Madras.
Formed by the State Reorganisation Act,
1956. It comprised Travancore and Cochin
areas.

The Government appointed a commission under S.K. Dhar to


ngi
examine the feasibility of reorganisation of States on a linguistic
Karnataka Formed from the princely state of Mysore
by the state Reorganisation Act, 1956, It was

basis. The S.K. Dhar Commission preferred reorganisation for


administrative convenience rather than on a linguistic basis. nee
renamed Karnataka is 1973 from Mysore.
Gujarat and The State of Bombay was divided into two
Maharashtra states namely, Maharashtra and Gujarat by the
The Dhar Commission’s report created much resentment
and led to the appointment of another Linguistic Provinces
Committee in December 1948 to examine the Linguistic basis
Nagaland
r
Bombay (Reorganisation) Act, 1960.

ing
State of Nagaland Act, 1962 created the State
of Nagaland, which was carved out of Assam
of States reorganisation. This Committee consisted of J.L.
Nehru, Vallabhbai Patel and Pattabhi Sitaramayya and hence
was popularly known as JVP Committee.
Haryana
(now Asom).

.ne
It was carved out from the State of Punjab by
the Punjab (Reorganisation) Act, 1966.
However, in 1953, the first linguistic State came into being
was Andhra Pradesh, created by separating the Telugu
speaking areas from the State of Madras. This compelled the
Govt. of India to appoint a 3 member States Reorganization
Himachal
Pradesh t
State of Himachal Pradesh Act, 1970 elevated
the Union Territory of Himachal Pradesh to
the status of state.
Meghalaya First carved out as a sub-state within the State
Commission in 1953 to look into the question of redrawing of Assam by 23rd Constitutional Amendment,
of the boundaries of States. This commission was chaired by 1969. Later, in 1971, it received the status of
Fazal Ali. full-fledged State by the North-Eastern Areas
(Reorganisation) Act, 1971.
Territory of India States
Manipur and Both these states were elevated from the status
1. Andhra Pradesh 16. Manipur Tripura of Union Territories by the North-Eastern
2. Arunachal Pradesh 17. Meghalaya Areas (Reorganisation) Act, 1971.
3. Asom 18. Mizoram Sikkim Sikkim was given first the status of associate
state by the 35th Constitutional Amendment
4. Bihar 19. Negaland Act, 1974. 36th Amendment Act, 1975,
5. Chhattisgarh 20. Odisha uplifted its status to a full state in 1975. From
6. Goa 21. Punjab 1947 to 1974, Sikkim was a protectorate of
7. Gujarat 22. Rajasthan India, with India being responsible for its
defence, external affairs and communications.

Downloaded From : www.EasyEngineering.net


Downloaded From : www.EasyEngineering.net
P-10 Constitutional Framework and Citizenship

Mizoram It was uplifted to the status of a full state by ordinarily residing in India for at least five years before
the State of Mizoram Act, 1986. the commencement of the Constitution, provided they
had not voluntarily acquired the citizenship of some
Arunachal It received the status of a full state by the State foreign country.
Pradesh of Arunachal Pradesh Act, 1986.
Goa It was separated from the Union Territory of Acquisition and Termination of Citizenship
Goa, Daman and Diu and was made a full- Rules regarding acquisition and termination of Indian
fledged state by the Goa, Daman and Diu citizenship have been laid down in the Citizenship Act of 1955.
Reorganisation Act, , 1987. But, Daman and A person can acquire citizenship of India in five ways.
Diu remained as Union Territory till date.
Chhattisgarh Created, as 26th State of India, in 2000, by
1. Citizenship by Birth
dividing Madhya Pradesh on 1st November, A person born in India on or after January 1950 is treated as
2000. citizen of India by birth.
Uttarakhand Earlier known as Uttaranchal, it was formed 2. Citizenship by Descent
in 2000 by dividing Uttar Pradesh on 9th
A person who was residing outside India on or after 26 January
November, 2000. In January 2007, the name
of the state was officially changed from 1950 is treated as a citizen of India by descent if his father was

ww Uttaranchal to Uttarakhand. It is 27th state


of the Union of India.
Jharkhand Created as 28th State, in 2000, by dividing
citizen of India at the time of his birth.
3. Citizenship by Registration

Telangana w.E
Bihar on 15th November, 2000.
Created as 29th State on 2nd June, 2014. It
was separated from Andhra Pradesh with
The following categories of persons can be registered as
citizens of India on application by the prescribed authority:
(a) Persons of Indian origin who are ordinarily resident

a
Hyderabad as the capital of both state for the
period of 10 years.
syE
On the recommendation of Shri Krishna
Committee Report, the new State Telangana is
in India for five years before filing of application for
registration.
(b) Persons of Indian origin who are ordinarily resident in
any country or place outside India.

Citizenship
carved out from Andhra Pradesh.
ngi (c) Women who are married to citizens of India.
(d) Minor’ children of persons who are citizens of India.
(e) Persons of full age and capacity who are citizens of
Part II of Constitution (Articles 5-11) contains provisions
relating to citizenship of India. In brief:
Article 5 - citizenship at the commencement of the Constitution:
nee
Commonwealth countries or the Republic of Ireland.
4. Citizenship by Naturalisation
Article 6 - rights of citizenship of certain persons who have
migrated to India from Pakistan;
Article 7 - rights of citizenship of certain migrants to Pakistan;
r ing
A person can acquire citizenship of India through naturalisation
if he (a) belongs to a country where the citizens of India are
allowed to become subjects or citizens of that country by

Article 8 - right of citizenship of certain persons of India origin


residing outside India:
naturalisation;

.ne
(b) renounces the citizenship of his country and intimates the
renunciation to the government of India;
Article 9 - Persons voluntarily acquiring citizenship of a foreign
state not be citizens:
Article 10 - continuance of the rights of citizenship:
Article 11 - parliament to regulate the right of citizenship by
t
(c) has been residing in India or serving the government for 12
months before the date of making application for naturalisation;
(d) possesses a good character;
(e) possesses workable knowledge of an Indian language;
law.
(f) intends to reside in India or to serve under the Government
• The Constitution of India provides for single citizenship. of India after naturalisation. However, the Government of
All persons residing in different parts of the country India can waive any or all of the above conditions in case of a
enjoy Indian citizenship (Article 5). There is no separate person who has rendered distinguished service in the cause of
citizenship of states. According to the Constitution, the philosophy, science, art, literature, world peace and the like.
following three categories of persons are entitled to 5. By Incorporation of Territory
citizenship:
1. Person domiciled in India, If any new territory is added to India, the Government of India
can specify the persons of the territory who shall be citizens of
2. Refugees who migrated to India from Pakistan, India by reasons of their connection with that territory.
3. Indians living in other countries.
• Domiciled persons include those with permanent homes Citizenship can be terminated in three ways
in India, persons born in India, persons either of whose (a) A citizen may voluntarily renounce his citizenship
parents was born in Indian territory, and persons by making necessary declaration to this effect in the

Downloaded From : www.EasyEngineering.net


Downloaded From : www.EasyEngineering.net
Constitutional Framework and Citizenship P-11

prescribed form. Usually citizenship is renounced by India. There is no separate state citizenship as in other federal
a citizen who wants to become the national to another countries. This implies that every citizen has same rights,
country. privileges and immunities, no matter in which state he resides.
(b) The citizenship can be terminated if a person voluntarily
Dual Citizenship for People of India Origin
acquires the citizenship of any other country by
naturalisation, registration or otherwise. (PIOs)
(c) The Central Government can deprive a naturalised citizen • In December 2003 a new law was passed which permits
of his citizenship, if it is satisfied that the citizenship was the people of Indian origin residing in 16 countries, viz.
acquired by fraud, false representation or concealment of Australia, Canada, Finland, France, Greece, Ireland,
material facts; or if the person shows disloyalty towards Israel, Italy the Netherlands, New Zealand, Portugal,
the Indian Constitution or indulges in trade with enemy Cyprus, Sweden, Switzerland, UK and United States,
countries during war; or if the person has been sentenced to have dual citizenship status. This will enable them to
to imprisonment for a period of two years or more within participate in the economic activities and acquire real
five years of his registration of naturalisation or if he has estate.
been continuously residing out of India for more than • As of January, 2006, the Indian government has
seven years. introduced the “Overseas Citizenship of India” (OCI) so

ww
Amendment of Citizenship Act
In 1986, the Citizenship Act was amended and acquisition
as to allow a limited form of dual citizenship. A citizenship
(Amendment) ordinance 2005, was promulgated in
June 2005 and an Act was passed in August 2005. The

w.E
of citizenship by persons coming to India as refugees from
Bangladesh, Sri Lanka and other countries was made difficult.
It provided that persons born in India
amended Act enables the central government to register,
as an Overseas Citizen of India (OCI), any person of full
age and capacity.

a syE
(a) on or after January 26, 1950 but prior to 26 November
1986, (b) on or after the commencement of the Amending Act,
1986, shall be citizens of India by right only if either of their
• Who is a citizen of another country now, but was a
citizen of India at the time of, or at any time after the
commencement of the Constitution of India on 26th
January 1950.
the period for acquisition of citizenship through registration
from six months to five years. ngi
parents is a citizen of India at the time of his birth. It increased
• Who is a citizen of another country, but was eligible to
become a citizen of India at the time of common cement

In December 2003 the Citizenship Act 1955 was amended to


facilitate the re-acquisition of the Indian citizenship by persons
of full age who are children of Indian citizens and former

nee
of the Constitution.
Who is a citizen of another country, but belonged to a
territory that became a part of India after the 15th of
Indian citizens. However, the Act made the acquisition of
Indian citizenship and naturalisation more stringent to prevent
illegal migrants from becoming eligible for Indian citizenship.

r
August 1947.

ing
Who is child or a grand child of such a citizen; or a minor
child of a person mentioned in all the above. No person

Recently Adan Sami, A Pakistani singer, who has been staying


in India since 2001, will be given Indian citizenship under .ne
who is or has been citizen of Pakistan or Bangladesh
shall be eligible to be registered as an OCI.
Section 6 of the Indian Citizenship Act, 1955. The provision
allows citizenship, through a certificate of “naturalisation”,
to applicants with distinguished contribution in the field of
science, philosophy, art, literature, world peace or human
Pravasi Bharatiya Divas (PBD)
t
It is celebrated on 9th January every year to mark the
contribution of Overseas Indian Community in the development
progress.
of India. 9th January, was chosen as the day to celebrate this
Commonwealth Citizenship occasion, since it was on this day in 1915 that Mahatma
Gandhi, the greatest Pravasi, returned to India from South
Every person who is a citizen of a Commonwealth country, by
virtue of that citizenship enjoys the status of Commonwealth Africa, led India’s freedom struggle and changed the lives of
citizenship in India. The Indian Citizenship Act 1955 Indians forever.
empowers the Central Government to make provisions on the PBD conventions are being held every year, since 2003.
basis of reciprocity for the enforcement of all or any of the These conventions provide a platform to the overseas Indian
rights of a citizen of India on the citizens of U.K. Australia, community to engage with the government and people of the
Canada, Ceylon (Sri Lanka), New Zealand, Pakistan, and land of their ancestors for mutual beneficial activities.
other Commonwealth countries.
The decision to celebrate the Pravasi Bharatiya Divas was
Single Citizenship taken in accordance with recommendations of the High Level
Another notable feature of the Indian citizenship is that the Committee (HLC) on the Indian Diaspora set-up by Government
Constitution recognises only one citizenship, viz. that of of India under the Chairmanship of Dr. LM Singhvi.

Downloaded From : www.EasyEngineering.net


Downloaded From : www.EasyEngineering.net
P-12 Constitutional Framework and Citizenship

Exercise - 1
1. Constituent Assembly which framed India’s Constitution (c) Indian Councils Act of 1861
was set up by: (d) Indian Councils Act of 1882
(a) Indian Independence Act 12. Which act provided for direct control of Indian affairs by
(b) Government of India Act, 1935 the British Government?
(c) Cabinet Mission Plan - 1946 (a) Charter Act of 1858
(d) Queen’s Proclamation (b) Regulating Act of 1773
2. ______________ headed the Drafting Committee of the (c) Pitts India Act of 1784
Constitution ? (d) Charter Act of l833
(a) Sachchidanand Sinha (b) B.N. Rao 13. Which one of the following pairs is correctly matched?
(c) Jawaharlal Nehru (d) B.R. Ambedkar (a) 1919 Act - Dyarchy at the Centre
3. _________ is borrowed from the Weimar Constitution of (b) 1861 Act - Portfolio system

ww
Germany by the Indian Constitution
(a) A federation with a strong centre
(b) System of presidential elections
(c) Directive Principles of State Policy.


(c) 1935 Act
(d) 1853 Act
- Bicameralism
- Governor-General of India
14. The Constituent Assembly, which framed the Indian
Constitution, was set up in

w.E
(d) Suspension of fundamental rights during Emergency
4. Preamble borrows the ideals of “liberty, equality and
fraternity from:
(a) 1945
(c) 1947
(b) 1946
(d) 1949
15. Which one of the following pairs is not correctly matched?


a
(a) Russian Revolution (b) Irish Revolution

5. Preamble enshrines ideals spelt out in:


syE
(c) French Revolution (d) American Constitution

(a) Speech by Nehru on the banks of Ravi calling for


Purna Swaraj.




(a) States Reorganization : Andhra Pradesh Act
(b) Treaty of Yandabu : Assam
(c) State of Bilaspur : Himachal Pradesh
(d) Year 1966 : Gujarat becomes a state
16. Constitutional government stand for
(b) Nehru Report
(c) Resolution adopted at the Karachi session of the
Congress Party ngi

(a) limited government
(b) a government limited by the desires and capacities
of those who exercise power.
(d) Objectives Resolution of the Constituent Assembly
6. States Reorganisation Act created ___ states and ___
Union Territories.


nee
(c) a government run according to general laws known
to the people
(d) a government run by people’s representatives.
(a) 14; 7
(c) 15; 7

citizens.
(b) 14; 6
(d) 15; 6
7. The Constitution bestows ____ citizenship on Indian r ing
17. Which items is wrongly matched?
(a) December 9, 1947 Constituent Assembly’s
First meeting
(a) Single
(c) Federal
(b) dual
(d) Three




.ne
(b) November 26, 1949 The people of India adopted,
enacted and gave to
themselves the Constitution
8. Pravasi Bharatiya Diwas coincides with:
(a) Gandhiji’s becoming a lawyer in South Africa
(b) Gandhiji’s return from South Africa
(c) Beginning of civil disobedience movement
(d) India’s proclamation as a republic
(c) January 24, 1950



t
The Constitution was signed
by the members of the
Constituent Assembly
(d) January 26, 1950 the date of commencement of
9. Which portfolio was held by Dr. Rajendra Prasad in the the Constitution
Interim Government formed in the year 1946? 18. Which features and source are WRONGLY matched?
(a) Defence (a) Judicial review - British practice
(b) External Affairs and Commonwealth Relations (b) Concurrent List - Australian Constitution
(c) Food and Agriculture (c) Directive Principles - Irish Constitution
(d) None (d) Fundamental Rights - US Constitution
10. Which of the following is correctly matched? 19. Which one among the following has the power to regulate
(a) 1909 Act - Principle of election the right of citizenship in India?
(b) 1919 Act - Provincial autonomy (a) The Union Cabinet (b) The Parliament
(c) 1935 Act - Dyarchy in states (c) The Supreme Court (d) The Law Commission
(d) 1947 Act - Responsible government 20. What is extradition ?
11. Statutory recognition to the portfolio system was accorded (a) Delivering a national of another country for the trial
by: of offences
(a) Indian Councils Act of 1892 (b) To block the trade of other countries with a particular
(b) Indian Councils Act of 1871 country

Downloaded From : www.EasyEngineering.net


Downloaded From : www.EasyEngineering.net
Constitutional Framework and Citizenship P-13

(c) Forcing a person to leave the country of which he is 31. Which one of the following is not related to the
a citizen constitutional Government ?
(d) To force foreign national to leave the country (a) Coalition Government
21. “Referendum” has an integral relationship with (b) Presidential Government
(a) Indirect democracy (c) Hierarchy Government
(b) Limited monarchy (d) Federal Government
(c) Direct democracy 32. The Indian constitution is divided into
(d) People’s courts (a) 16 chapters (b) 22 chapters
(c) 24 chapters (d) 25 chapters
22. Which of the following statements best describe in
33. Which of the states of Indian federation has a separate
systems of liberal democracy? constitution ?
(a) Liberal democratic political systems only occur in (a) Goa
highly industrialised societies. (b) Tamil Nadu
(b) The same political party holds power continuously (c) Jammu & Kashmir
in liberal democratic regimes. (d) Himachal Pradesh
(c) Liberal democratic governments prevent their 34. Which one of the following is not a part of the ‘basic
citizens from travelling abroad. structure’ of the Indian constitution ?
(d) Liberal democratic governments protect basic civil (a) Rule of law

ww liberties.
23. The mention of the word ‘justice’ in the preamble to the
constitution of India expresses
(b) Secularism


(c) Republican form of government
(d) Parliamentary form of government
35. The philosophical foundation of the Indian constitution is




w.E
(a) social, political and religious justice
(b) social, economic and cultural justice
(c) social, economic and political justice
(d) economic and political justice



(a) Directive principle of state policy
(b) Fundamental rights
(c) Federal Structure



(a) Golak Nath case
a
‘Basic Structure of Constitution’?

(b) Shankari Prasad case


syE
24. In which case the supreme court evolved the concept of
(d) Preamble
36. Which article of Indian constitution declares Devnagri



Hindi as an official language of India?
(a) Article 343
(c) Article 154
(b) Article 348
(d) Article 156


(c) Kishana Nanda Bharti case
(d) Minerva Mills case ngi 37. Which of the following gave the idea of constituent


Assembly for India?
(a) Simon Commission

nee
25. By which amendment of the constitution, the Word
‘Socialist’ was incorporated in the preamble of the (b) Rajaji Formula
constitution? (c) Cabinet Mission Plan
(a) 42nd Amendment (b) 44thAmendment (d) Wavell Plan
th
(c) 25 Amendment (d) 24th Amendment
26. Which one of the following rights has been described by
Dr. Ambedkar as ‘The heart and soul of the constitution’?
r
38. Who amongst the following was not a member of the

ing
drafting committee of the constitution?
(a) Mohammod Sadullah



(a) Right of Equality
(b) Right to freedom
(c) Right to property



(b) K.M. Munshi
(c) A.K. Ayyar
(d) J.L. Nehru .ne


(d) Right to Constitutional Remedies
27. Which schedule of Indian Constitution is related to
panchayti Raj ?
(a) II Schedule (b) VIII Schedule




(a) Anti-defection Legislation
(b) Panchayti Raj
(c) Land Reforms
t
39. The tenth schedule of Indian constitution deals with

(d) Distribution of powers between the union and states


(c) X Schedule (d) XI Schedule
40. The number of official languages recognized under 8th
28. Which term is not used in the preamble of the Indian
schedule of the Indian Constitution is
constitution ?
(a) Sixteen (b) Seventeen
(a) Republic (b) Integrity
(c) Twenty (d) Twenty Two
(c) Federal (d) Socialist
41. How many articles are there in the Indian constitution?
29. The source of the basic structure theory of the constitution
(a) 395 (b) 396
of India is
(c) 398 (d) 399
(a) the constitution
(b) opinion of jurists 42. The feature of Indian constitution borrowed from South
(c) indicial interpretation African constitution is:
(d) parliamentary statutes (a) Procedure established by law
30. The provision of the sixth schedule shall not apply in (b) Procedure for amendment of constitution and
which one of the following states? election of Rajya Sabha members
(a) Meghalaya (b) Tripura (c) Method of election of President
(c) Mizoram (d) Goa (d) Joint Sitting of the Parliament

Downloaded From : www.EasyEngineering.net


Downloaded From : www.EasyEngineering.net
P-14 Constitutional Framework and Citizenship

43. The words justice in the form of social, economic and 53. Which schedule of the Constitution of India contains the three
political justice present in the Preamble has been taken lists that divide powers between the Union and the sates?
from (a) Fifth (b) Sixth
(a) American Revolution (c) Seventh (d) Eigth
(b) French Revolution 54. In which part of the Constitution, details of citizenship
(c) Russian Revolution are mentioned?
(d) None of the above (a) I (b) II
44. The terms not introduced in the Preamble through the (c) III (d) IV
42nd amendment are: 55. Who among the following was the first Law Minister of
(a) Socialist (b) Secular India?
(c) Integrity (d) None of the above (a) Jawaharlal Nehru
45. Which case determined that the Preamble is a part of the (b) Maulana Abdul Kalam Azad
constitution (c) Dr BR Ambedkar
(a) Berubari Union case (d) T Krishnamachari
(b) Kesavananda Bharati case 56. Who among the following was the Finance Minister of
(c) Golaknath case India in the Interim Government during 1946-1947?
(a) R K Shanmukham Chetty
(d) Minerva Mills case
(b) John Mathai
46. Which statements are correct?


ww
(a) Originally the constitution had 14 languages in the
8th schedule.
(b) Currently there are 22 languages with the last


(c) Liaquat Ali Khan
(d) Chintamanrao Deshmukh
57. Which one of the following rights conferred by the
Constitution of India is also available to non-citizens?


(c) both correct
(d) both wrong w.E
amendment being the 92nd amendment.

47. Which statements are correct?




(a) Freedom of speech, assembly and form association
(b) Freedom to move, reside and settle in any part of the
territory of India


a syE
(a) Article 2 of the constitution gives the right to the
government to create new states into the Union of
India which are not part of India already.
(b) Article 3 gives the power to the government to


(c) Freedom to acquire property or to carry on any
occupation, trade or business
(d) Right to constitutional remedies
58. Which one among the following statements is not correct?
The word ‘socialist’ in the Preamble of the Constitution



change the boundaries of the state within India.
(c) both correct
(d) both wrong ngi
of India read with
(a) Article 39 (d), would enable the court to uphold the
constitutionality of nationalisation laws
48. The parliament can make any law for the whole or any


part of India for implementing international treaties?
(a) with the consent of all the states


nee
(b) Article 14, would enable the court to strike down
a-statute which failed to achieve the socialist goal to
the fullest extent
(c) Article 25, would enable the court to ensure freedom



(b) with the consent of majority of states
(c) with the consent of state concerned
(d) without the consent of any state.

r ing
guaranteed under that Article
(d) Article 23, would enable the court to reduce
inequality in income and status
59. The Constitution of India is republican because it

.ne
49. Under which one of the following constitution amendment
act, four languages were added to the languages under (a) provides for an elected Parliament
the eighth schedule of the constitution of India thereby, (b) provides for adult franchise





raising their number to 22?
(a) 90th Amendment Act
(b) 91st Amendment Act
(c) 92nd Amendment Act
(d) 93rd Amendment Act


(c) contains a bill of rights
(d) has no hereditary elements

because it
(a) uses value loaded words
t
60. The Preamble is useful in constitutional interpretation

50. Who among the following was the advisor of the Drafting (b) contains the real objective and philosophy of the
Committee of the Constituent Assembly? constitution makers
(a) B. Shiva Rao (b) Dr. B.R. Ambedkar (c) is a source of power and limitation
(c) Sachidananda Sinha (d) B.N. Rao (d) gives and exhaustive list of basic features of the
51. The idea of including the Emergency provisions in the Constitution
Constitution of India has been borrowed from the 61. Which one among the following was not a proposal of
(a) Constitution of Canada the Cabinet Mission, 1946?
(b) Weimar Constitution of Germany (a) The Constituent Assembly was to be constituted on
(c) Constitution of Ireland the democratic principle of population strength
(d) Constitution of the USA (b) Provision for an Indian Union of Provinces and
52. Which one of the following Schedules of the Constitution States
of India includes the disqualification of a Legislator on (c) All the members of the Constituent Assembly were
grounds of defection? to be indians
(a) 8th Schedule (b) 7th Schedule (d) British Goverment was to supervise the affairs of the
(c) Schedule (d) 10th Schedule Constituent Assembly

Downloaded From : www.EasyEngineering.net


Downloaded From : www.EasyEngineering.net
Constitutional Framework and Citizenship P-15

62. ‘The Draft Constitution as framed only provides a 69. If a new state of the Indian Union is to be created, which
machinery for the government of the country. It is not a one of the following schedules of the Constitution must
contrivance to install any particular party in power as has be amended?
been done in some countries. Who should be in power is (a) First (b) Second
left to be determined by the people, as it must be, if the (c) Third (d) Fifth
system is to satisfy the test of democracy’. 70. The Ninth Schedule to the Indian Constitution was added by:
The above passage from Constituent Assembly debates is (a) First Amendment
attributed to (b) Eighth Amendment
(a) Pandit Jawaharlal Nehru (c) Ninth Amendment
(b) Dr B R Ambedkar
(d) Forty Second Amendment
(c) Maulana Abdul Kalam Azad
71. Which one of the following schedules of the Indian
(d) Acharya J B Kriplani
63. Which among the following features of a federal system Constitution lists the names of states and specifies their
is not found in the Indian Political System ? territories?
(a) Dual citizenship (a) First (b) Second
(b) Distribution of powers between the Federal and the (c) Third (d) Fourth
State Governments 72. Which one of the following statements correctly describes

ww
(c) Supremacy of the Constitution the Fourth Schedule of the Constitution of India?
(d) Authority of the Courts to interpret the Constitution (a) It contains the scheme of the distribution of powers
64. The Sixth Schedule of the Indian Constitution contains between the Union and the States
provisions for the administration of Tribal areas. Which

?
(a) Assam
w.E
of the following States is not covered under this Schedule

(b) Manipur


(b) It contains the languages listed in the Constitution
(c) It contains the provisions regarding the administration
of tribal areas
(d) It allocates seats in the Council of States
(c) Meghalaya

a
65. In the following quotation,
(d) Tripura

“WE THE PEOPLE OF INDIA, having solemnly


syE
resolved to constitute India into a Sovereign Socialist
73. Which Schedule of the Constitution of India contains :
Special provisions for the administration and control of
Scheduled Areas in several States?


Secular Democratic Republic and to secure to all its
citizens:
JUSTICE, social, economic and political; LIBERTY of ngi (a) Third
(c) Seventh
(b) Fifth
(d) Ninth
74. Under which one of the following Constitution


thought, expression, belief faith and worship;
EQUALITY of status and of opportunity: and to promote
among them all; FRATERNITY assuring the dignity nee
Amendment Acts, four languages were added to the list of
languages under the Eighth Schedule of the Constitution
of India, thereby raising their number to 22?


of the individual and the unity and the integrity of the
Nation.
In our Constituent Assembly this ‘X’ do hereby adopt,
enact and give to ourselves this Constitution.”,



r ing
(a) Constitution (Ninetieth Amendment) Act
(b) Constitution (Ninety-first Amendment) Act
(c) Constitution (Ninety-second Amendment) Act


‘X’ stands for:
(a) twenty-sixth day of January, 1950

.ne
(d) Constitution (Ninety-third Amendment) Act
75. The distribution of powers between the Centre and the



(b) twenty-sixth day of November, 1949
(c) twenty-sixth day of January, 1949
(d) None of the above
66. Which one of the following schedules of the Constitution
of India contains provisions regarding anti-defection


(a) Morley-Minto Reforms, 1909
(b) Montagu-Chelmsford Act, 1919
t
States in the Indian Constitution is based on the scheme
provided in the

Act? (c) Government of India Act, 1935


(a) Second Schedule (b) Fifth Schedule (d) Indian Independence Act, 1947
(c) Eighth Schedule (d) Tenth Schedule 76. Which one of the following Schedules of the Constitution
67. The Constitution of India recognises: of India contains provisions regarding anti-defection?
(a) only religious minorities (a) Second Schedule (b) Fifth Schedule
(b) only linguistic minorities (c) Eighth Schedule (d) Tenth Schedule
(c) religious and linguistic minorities
77. The provisions in Fifth Schedule and Sixth Schedule in
(d) religious, linguistic and ethnic minorities
68. Which one of the following statements correctly describes the Constitution of India are made in order to
the Fourth Schedule of the Constitution of India? [CSAT-2015-1]
(a) It lists the distribution of powers between the Union (a) protect the interests of Scheduled Tribes
and the states (b) determine the boundaries between States
(b) It contains the languages listed in the Constitution (c) determine the powers, authority and responsibilities
(c) It contains the languages listed in the Constitution of Panchayats
(d) It allocates seats in the Council of States (d) protect the interests of all the border States

Downloaded From : www.EasyEngineering.net


Downloaded From : www.EasyEngineering.net
P-16 Constitutional Framework and Citizenship

Exercise - 2
Which of the above is/are correct?
Statement Based MCQ (a) 1 and 2 (b) 1, 2 and 3
(c) 2,3 and 4 (d) 1, 2, 3 and 4
1. Indian Constitution establishes a secular state meaning 7. The features of Government of lndia Act of 1858
that: includes:
1. State treats all religions equally 1. Replacement of Company rule by the Crown rule.
2. Freedom of faith and worship 2. Establishment of a Board of Control over the Court
3. Educational bodies can impart religious instructions of Directors.
4. State does not discriminate on the basis of religion 3. Reaffirmation of the system of open competition.
Which of the following statements is/are correct? 4. Separating the legislative and executive functions of
(a) 1 and 2 (b) 1, 2 and 3 the Governor-General.
(c) 2, 3 and 4 (d) 1, 2 and 4 5. Creation of a new office of the Secretary of State for
2. What facts emerges from the Preamble? India.
1. When the Constitution was enacted



ww
2. Ideals that were to be achieved .
3. System of government
4. Source of authority



Which of the following statements is/are correct?
(a) 1,3 and 4
(c) 1 and 5
(b) 1,2 and 4
(d) 1, 3 and 5
8. The features of Indian parliamentary system are:



(a) 2, 3 and 4
(c) 1, 2 and 3 w.E
Which of the following statements is/are correct?

3. Which statement is correct?


(b) 1 and 2
(d) 1, 2, 3 and 4


1. Independent judiciary.
2. Collective responsibility of the executive to the
legislature.


a
1. Territory of the constituents of the Indian federation

syE
may be altered by the Parliament by a simple
majority.
2. Consent of the State Legislature is necessary before



3. A written Constitution.
4. Presence of de jure and de facto executives.
5. Individual responsibility of the executive to the
legislature.
Which of the following statements is/are correct?
the Parliament alters boundaries.
3. President’s recommendation is necessary for
introducing any Bill on redrawing the boundary of a ngi

(a) 2, 3 and 4
(c) 2, 4 and 5
(b) 1, 2 and 4
(d) 1, 2, 4 and 5
9. Which of the following are the federal features of the


State.
4. President must have State’s opinion before altering
the name of the State.
Which of the following statements is/are correct?


nee
Indian Constitution?
1. Rigid Constitution 2. Bicameralleg legislature
3. Office of the CAG 4. Collective responsibility


(a) 1 only
(c) 1, 3 and 4
(b) 1 and 2
(d) 1 and 3
4. _____ gives the Indian Constitution a unitary bias?



r
5. Office of the Governor
(a) 1, 2 and 3
(c) 1, 2, 3 and 4 ing (b) 1, 2 and 5
(d) 1 and 2



1. Rigidity of the amendment process
2. Written Constitution
3. Same Constitution for Centre and States .ne
10. The forms of oaths or affirmations for which of the
following officials are mentioned in the third schedule of
the Indian constitution?




4. Fixed State boundaries
Which of the above is/are correct?
(a) 2 and 4
(c) 3 and 4
(b) 1, 2 and 4
(d) 3 only
5. Consider the following statements :




1. The Comptroller and Auditor General
2. The Chief Election Commissioner
3. The Chief Justice of a High Court
4. The Attorney General
t
1. The Charter Act 1853 abolished East India Select the correct answer using the codes given below:
Company’s monopoly of Indian trade. (a) 1 and 2 only (b) 1, 2 and 3 only
2. Under the Government of India Act 1858, the (c) 2, 3 and 4 only (d) 1 and 3 only
British Parliament abolished the East India Company 11. Which of the following provisions of the constitution
altogether and undertook the responsibility of ruling reveal the secular character of the Indian State:
India directly. 1. The state shall not deny to any person equality
Which of the statements given above is/are correct ? before the law.
(a) 1 only (b) 2 only 2. No religious instruction shall be provided in any
(c) Both 1 and 2 (d) Neither 1 nor 2 educational institution maintained by the state.
6. The salient features of the Government of India Act, 1935
3. The state shall endeavour to secure for all citizens a
are:
uniform Civil Code.
1. All India Federation
4. Any section of the society shall have the right to
2. Provincial Autonomy
conserve its distinct language.
3. Dyarchy at the Centre
(a) 1, 2, 3 only (b) 2 only
4. Abolition of Dyarchy in the states
(c) 2 & 3 only (d) 1, 2, 3 & 4

Downloaded From : www.EasyEngineering.net


Downloaded From : www.EasyEngineering.net
Constitutional Framework and Citizenship P-17

12. Which of the following feature/s of the constitution of 3. Manipur was the first to become a UT choice
India has/have been borrowed from GOI Act 1935: (a) Only 1 and 2 correct.
1. Office of the Governor. (b) Only 2 and 3 correct.
2. Emergency Provisions. (c) Only 1 and 3 correct
3. Legislative Procedure. (d) all correct.
4. Bicameralism. 19. Consider the following statements:
5. Federation with a strong center. 1. An amendment to the Constitution of India can be
(a) 1, 2 & 4 only initiated by an introduction of a bill in the Lok Sabha
(b) 1, 3, 4 & 5 only only.
(c) 1 & 2 only 2. If such an amendment seeks to make changes in the
(d) None of the above options are correct federal character of the Constitution, the amendment
13. Which of the following term/s was/were added by the also requires to be ratified by the legislature of all
42nd Amendment act in our Preamble: the States of India.
1. Socialist 2. Secular Which of the statements given above is/are correct?
3. Republic (a) 1 only (b) 2 only
(a) 1 & 2 only (b) 1 only. (c) Both 1 and 2 (d) Neither 1 nor 2
(c) 2 & 3 only (d) 1, 2 & 3 20. Which among the following statements is/are the
14. Which of the following statements are incorrect: feature(s) of a Federal state ?


ww
1. The Preamble is a prohibition on the powers of the
legislature.
2. Preamble is not a part of the constitution.


1. The power of the central and the state (Constituent
Unit) Government are clearly laid down.
2. It has an unwritten constitution.



(a) 1 & 2 only
(c) 1 & 3 only
w.E
3. Preamble cannot be amended, as it is a part of the
basic structure of the constitution.
(b) 2 & 3 only.
(d) 1, 2 & 3



Select the correct answer using the code given below:
(a) 1 only
(c) Both 1 and 2
(b) 2 only
(d) Neither 1 nor 2

a syE
15. Which of the following statement/s is/are correct:
1. Indian territory can be ceded to a foreign state
without amending the constitution as mentioned in
article 368.
21. Which of the following statements is/are not violative of


the principle of federalism?
1. The President of India takes over administration of
provinces under the emergency provisions.
2. The Parliament of India has exclusive power to make
2. Resolution of boundary dispute between India and
another country does not require constitutional
amendment. ngi
any law with respect to any matter not enumerated
in the Concurrent List or State List.
3. The distribution of powers between the Union
(a) 1 only
(c) Both
(b) 2 only
(d) None
16. Choose the correct statements from below:
nee
and Provinces is done through three different lists
enumerated in the Constitution of India.
Select the correct answer using the code given below:


1. The Regulating Act of 1773 was the first step by
British govt to regulate the East India Company.
2. The Pitt’s India Act setup the Supreme Court in
Code:


r
(a) 1 and 2
(c) 3 only ing(b) 2 and 3
(d) 1 and 3


Calcutta.
3. The Charter Act of 1833 first recognized the
company’s territories in India as British possessions.
Federal System is/are correct?
.ne
22. Which of the following statements with regard to the

1. In a federation, two sets of governments co- exist




4. Pitt’s India Act created the Board of Control to look
after political affairs.
(a) Only 1,2,3 is correct  (b)  Only 1,3,4 is correct
(c) Only 1,4 is correct (d)  All are correct
17. Choose the correct answer




and there is distribution of power.
2. There is a written constitution.

(a) 1 only
(c) Both 1 and 2
(b) 2 only
t
Select the correct answer using the code given below:

(d) Neither 1 or 2
1. Charter Act of 1833 created the post Governor 23. Which of the following statements regarding the Preamble
General of India. of the Constitution of India is/are correct ?
2. Charter Act, 1833 deprived the Governors of 1. The Preamble is an integral part of the Constitution.
Bombay and Madras their legislative powers. 2. The words ‘secular’ and ‘socialist’ have been a part
3. Charter Act, 1853 introduced open competitive of the Preamble since its inception.
system for recruitment of civil servants. Select the correct answer using the code given below :
4. Macaulay Committee on the Indian Police Service (a) 1 only (b) 2 only
was formed in 1854 and it drafted the Indian Police (c) Both 1 and 2 (d) Neither 1 nor 2
Act,1861. 24. Consider the following statements
(a) Only 1,2 is correct (b) Only 1,3 is correct 1. A person who was born on January, 26th, 1951 in
(c) Only 1,2,3 is correct (d) 1,2,3,4 is correct Rangoon, whose father was a citizen of India by
18. Consider following statements birth at the time of his birth is deemed to be an
1. Chattisgarh was formed by the Uttar Pradesh Indian citizen by descent.
Reorganization Act. 2. A person who was born on July, 1st 1988 in Itanagar,
2. Himachal Pradesh was first UT to become a State whose mother is a citizen of India at the time of his

Downloaded From : www.EasyEngineering.net


Downloaded From : www.EasyEngineering.net
P-18 Constitutional Framework and Citizenship

birth but the father was not, is deemed to be a citizen (a) 1 and 2 (b) 1 and 3
of India by birth. (c) 1,2 and 3 (d) 2 and 3
Which of the statements given above is/are correct? 31. The citizenship means
(a) Only 1 (b) Only 2 1. full civil and political rights of the citizens.
(c) Both 1 and 2 (d) Neither 1 nor 2 2. the right of suffrage for election to the House of the
25. Under which of the following conditions can citizenship People (of the Union) and the Legislative Assembly
be provided in India?
of every state.
1. One should be born in India.
3. the right to become a Member of the Parliament and
2. Either of whose parents was born in India
Member of Legislative Assemblies.
3. Who has been a resident of India for not less than
five years. Select the correct answer using the codes given
Select the correct answer using the codes given below below
(a) 1, 2 and 3 (b) 1 and 2 (a) 1 and 2 (b) 1 and 3
(c) 2 and 3 (d) None of these (c) 2 and 3 (d) All of these
26. Which of the following condition/conditions must 32. Which of the following statement(s) is/are not correct for
be fulfilled by the NRIs to be eligible to vote in the Ninth Schedule of the Constitution of India ?
elections in India? 1. It was inserted by the first amendment in 1951.
1. They must be physically present in their place of 2. It includes those laws which are beyond the purview of


wworigin to exercise their franchise.
2. NRIs whether they have acquired citizenship of
other countries or not are eligible to vote.


judicial review.
3. It was inserted by the 42nd Amendment.
4. The laws in the Ninth Schedule are primarily those



w.E
3. Eligible NRIs have to register by filling up form 6-A
with electoral registration office.
Select the correct answer using the codes given below
(a) 1, 2 and 3 (b) 1 and 3



which pertain to the matters of national security.
Select the correct answer using the code given below :
(a) 1 and 2
(c) 3 and 4
(b) 2 and 3
(d) 3 only


(c) Only 2
a (d) Only 3

a written and an unwritten Constitution?


syE
27. What is/are the major difference/differences between

1. A written Constitution is the formal source of all


33. Which of the following statements is/are not true for the


category of the Overseas Citizens of India (OCI) inserted by
the amendment to the Citizenship Act of India in 2003 ?
1. It gives dual citizenship to Persons of Indian Origin

ngi
Constitutional Laws in the country and the unwritten (PIO) who are citizens of another country.
Constitution is not the formal source. 2. It gives Persons of Indian Origin (PIO) who are citizens
2. A written Constitution is entirely codified whereas of another country, an OCI card without citizenship.

nee
an unwritten Constitution is not. 3. It permits the OCI to vote in general elections in India.
Select the correct answer using the codes given below 4. It allows the OCI to travel to India without visa.
(a) Only 1 (b) Only 2 Select the correct answer using the code given below :
(c) Both 1 and 2 (d) Neither 1 nor 2 (a) 1 and 2 (b) 1 and 3
28. Consider the following statements


1. In India, only two Union Territories have Legislative
Assemblies.
2. Mizoram, Nagaland and Meghalaya, the three

r
(c) 3 only

ing
(d) 2 and 4
34. Consider the following statements:
1. Article 371 A to 371 I were inserted in the Constitution

North-Eastern States of India, have only one seat


each in the Lok Sabha. .ne
of India to meet regional demands of Nagaland,
Assam, Manipur, Andhra Pradesh, Sikkim, Mizoram,
Arunchal Pradesh and Goa.



Which of the statement(s) given above is/are correct?
(a) Only 1
(c) Both 1 and 2
(b) Only 2
(d) Neither 1 nor 2
29. The Government of India Act, 1919
1. Established a bicameral legislature at the centre.


single citizenship. t
2. Constitution of India and the United States of America
envisage a dual policy (The Union and the States) but a

3. A naturalized citizen of India can never be deprived of


his citizenship.
2. Introduced dyarchy in the provincial executive. Which of the statements given above is/are correct?
3. Introduced a Federal System of Government in (a) 1, 2 and 3 (b) 1 and 3
India. (c) 3 only (d) 1 only
Select the correct answer using the codes given
35. Consider the following statements:
below
1. The Constitution of India has 40 parts.
(a) 1 and 3 (b) 2 and 3
2. There are 390 Articles in the Constitution of India in
(c) 1,2 and 3 (d) 1 and 2
all.
30. Among the following ideals and philosophy, identify
3. Ninth, Tenth, Eleventh and Twelfth Schedules were
those enshrined in the Preamble to the Constitution of
added to the Constitution of India by the Constitution
India. (Amendment) Acts.
1. Sovereign democratic republic. Which of the statements given above is/are correct?
2. Socialism and secularism. (a) 1 and 2 (b) 2 only
3. Capitalism and free trade. (c) 3 only (d) 1, 2 and 3
Select the correct answer using the codes given 36. Match the following
below

Downloaded From : www.EasyEngineering.net


Downloaded From : www.EasyEngineering.net
Constitutional Framework and Citizenship P-19

List-I List-II List-I List-II


(Schedule in the (Subject) (Person) (Role in making of the
Constitution of India) Constitution of India)”
A. Tenth Schedule 1. Languages A. Rajendra Prasad 1. Member. Drafting Committee
B. Eighth Schedule 2. Provisions as to disqualification B. T T 2.  Chairman Constituent Assembly
on the grounds of defection Krishanamachari
C. H C Mukherjee 3. Chairman. Drafting Committee
C. First Schedule 3. Validation of certain Acts and
Regulations D. B R Ambedkar 4. Vice Chairman. Constituent
Assembly
D. Ninth Schedule 4. The States
Codes:
Codes: A B C D A B C D
A B C D (a) 2 1 4 3 (b) 2 4 1 3
(a) 4 3 2 1 (c) 3 4 1 2 (d) 3 1 4 2
(b) 2 1 4 3
(c) 4 1 2 3
(d) 2 3 4 1 Matching Based MCQ

ww
37. Match the following
List-I
(Provision of the
List II
(Source)
40. Match List I (Acts of colonial Government of India) with
List II (Provisions)
Constitution of India)
A. Amendment
Constitution
of w.E
the Constitution of Germany
List I

Britain to fully East
List II
(A) Charter Act, 1813 (1) Set up a board of
control of regulate the

B. Directive Principles
a
C. Emergency Power of Constitution of South Africa
the President
syE
Constitution of Canada
India company


(B) Regulating Act

was ended
(2) Company’s trade
monopoly in India

D. The Union-State Relations Irish Constitution


ngi


(C) Act of 1858 (3) The power to govern
was transferred from
East Indian the
Codes :
A B C D
(a) 1 2 4 3

nee
British Crown
(D) Pitt’s India Act
Company the to

(4) The Company’s


(b) 3 4 1 2
(c) 1 4 2
(d) 3 7 4 2
3

38. Match the following




r
correspondence and
ing
directors were asked
to present to the
British Government all

List-I
(Act)
List-II
(Feature)

.ne
documents pertaining to
the administration of

A.

B.
The Indian Councils Introduction of provincial
Act, 1892 autonomy

The Indian Councils Introduction of the


the company




(a) A – 2 ; B – 4 ; C – 3 ; D – 1
(b) A – 1 ; B – 3 ; C – 4 ; D – 2
(c) A – 2 ; B – 3 ; C – 4 ; D – 1
(d) A – 1 ; B – 4 ; C – 3 ; D – 2
t
Act, 1909 Indian principle of election 41. Match List-I with List-II and select the correct answer:
List-I (Item in the List-II (Country from
C. The Government of Introduction of diarchy in Indian Constitution) which it was derived)
India -Act, 1919 provinces A. Directive Principles 1. Australia
of State Policy
D. The Government of Introduction of separate B. Fundamental Rights 2. Canada
India Act, 1935 electorate for the Muslims C. Concurrent List in 3. Ireland
Union-State Relations
Codes: D. India as a Union of 4. United Kingdom
A B C D States with greater
(a) 2 4 3 1 powers to the Union
(b) 1 3 4 2 5. United States of America
Codes:
(c) 2 3 4 1
(a) A-5, B-4, C-1, D-2 (b) A-3, B-5, C-2, D-1
(d) 1 4 3 2
(c) A-5, B-4, C-2, D-1 (d) A-3, B-5, C-1, D-2
39. Match the following

Downloaded From : www.EasyEngineering.net


Downloaded From : www.EasyEngineering.net
P-20 Constitutional Framework and Citizenship

EXERCISE-1 49. (c) Under the constitutional (Ninety-second amendment)


1. (c) 2. (d) 3. (d) 4. (c) 5. (d) Act, four languages Bodo, Dogri, Santhali and Maithali
6. (b) 7. (a) 8. (b) 9. (c) 10. (d) were added to languages under the eight schedule
11. (c) 12. (c) 13. (b) 14. (d) 15. (d) of the constitution of India thereby raising the total
16. (a) 17. (a) 18. (a) number of languages listed in the schedule to 22.
19. (b) The parliament may regulate the right of citizenship 50. (d) The advisor of the drafting committee of the con-
of India through the amendment of citizenship act. stituent assembly was Sir B.N. Rao.
20. (a) 21. (c) 51. (b) the idea of including the emergency provisions in the
22. (d) Civil liberties are the rights held by citizens of a Constitution of India has been borrowed from the
country. In liberal democracies citizens have rights Weimar Constitution of Germany.
such as freedom of speech, freedom of association 52. (d) The 10th Schedule to the Indian Constitution is
known as Anti-Defection Law. It was inserted by
e.g. to join a trade union, freedom to choose their
the 52nd Amendment Act 1985 to the Constitution.

wwpolitical representatives.
23. (c) The mention of the world ‘justice’ in the preamble to
the constitution of India expresses social, economic
It sets the provisions for disqualification of elected
members on the grounds of defection to another po-
litical party.

w.E
and political justice.
24. (c) 25. (a) 26. (d) 27. (d) 28. (d)
29. (c) 30. (d) 31. (c) 32. (b)
53. (c) 7th Schedule gives allocation of powers and func-


tions between Union & States. It contains 3 lists:
Union List (97 Subjects)

a
33. (c) The state of Jammu & Kashmir of Indian federation

syE
has a separate constitution. Article 370 of the Indian
constitution, which is of a temporary nature, grants
special status to Jammu and Kashmir. Under Part


States List (66 subjects)
Concurrent List(52 subjects)
54. (b) Details of Citizenship are mentioned in part ll(Article
5-11) of the constitution.

ngi
XXI of the Constitution of India, which deals with 55. (c) Jawaharlal Nehru took charge as the first Prime Min-
“Temporary, Transitional and Special provisions”, ister of Indiaon 15 August 1947, and chose 15 other
the State of Jammu and Kashmir has been accorded members for his cabinet in which B. R. Ambedkar
special status under Article 370.
34. (d) 35. (d) 36. (a)
37. (c) Cabinet mission plan gave the idea of constituent nee
was the first law minister of India.
56. (c) Liaquat Ali Khan became the first Finance Minister
of India in the Interim Government during 1946-
Assembly for India. In 1946, British Prime Minister
Clement Attlee formulated a cabinet mission to India
to discuss and finalize plans for the transfer of power
r
1947.

ing
57. (c) Freedom to acquire property or to carry on any oc-
cupation trade or business is also available to non
citizens.
from the British Raj to Indian leadership as well as
provide India with independence under Dominion
status in the Commonwealth of Nations. The Mission .ne
58. (d) Traffic in human beings and beggars and other simi-
lar forms of forced labour are prohibited and any

38. (d)
discussed the framework of the constitution and laid
down in some detail the procedure to be followed by
the constitution drafting body.
t
contravention of this provision shall be an offence
punishable in accordance with law . The terms ‘So-
cialist’ was added by the 42nd Amendment and assert
that the government must adopt socialistic policies to
ensure decent life for all Indian citizens. Thus, the
39. (a) The tenth schedule of Indian constitution deals with
word Socialist in the preamble of the Constitution of
anti-defection Legislation. Tenth Schedule (Articles India read with Article 23.
102(2) and 191(2))—”Anti-defection” provisions for 59. (a) The constitution of India is republican because it
Members of Parliament and Members of the State provides for an elected parliament.
Legislatures. 60. (b) The preamble is useful in constitutional interpreta-
40. (d) 41. (a) tion because it contains the real objective and phi-
42. (b) 43. (c) 44. (d) 45. (b) 46. (c) losophy of the constitution makers.
47. (c) 61. (d) The Cabinet mission arrived on March 24, 1946.
48. (d) Act 253- Parliament has power to make any law for The objective of this mission was to:
the whole or any part of the country or territory • Devise a machinery to draw up the constitution
of India for implementing any treaty, agreement or of Independent India.
convention with any other country or any decision • Make arrangements for interim Government.
made at any international conference, association or • Thus the mission was like a declaration of India’s
other body without the consent of any state. independence.

Downloaded From : www.EasyEngineering.net


Downloaded From : www.EasyEngineering.net
Constitutional Framework and Citizenship P-21

• British Government was to supervise the affairs Schedule deals with administration and control of
of the constituent Assembly, was not the aim of tribal areas in the state of Assam, Meghalaya,
the mission. Mizoram and Tripura.
62. (b) On 29 August 1947, the Drafting Committee was 74. (c) 92nd Amendment Act 2003 added Bodo, Santhali,
appointed with Dr B. R. Ambedkar as the Chairman Maithali and Dogri languages in the 8th Schedule of
along with six other members assisted by a constitu- the Constitution. Originally there were 14 languages
tional advisor. These members were Pandit Govind in the 8th schedule. 21st amendment act added Sin-
Ballabh Pant, K M Munshi, Alladi Krishnaswamy dhi language. 71st amendment act added Konkani,
Iyer, N Gopala swami Ayengar, B L Mitter and Md.
Manipuri and Nepali languages.
Saadullah. A Draft Constitution was prepared by the
75. (c) Distribution of power between the Centre and the
committee and submitted to the Assembly on 4 No-
States in the Indian Constitution is based on the Gov-
vember 1947. This is Ambedkar’s second argument
rested on the legitimacy of the democratic system. ernment of India Act. 1935.
63. (a) Dual Citizenship is not found in the Indian political 76. (d) The 52nd Amendment act of 1985, added 10th
system. The Indian Constitution does not allow dual schedule to the Constitution. This is often referred
citizenship. Automatic loss of Indian citizenship to as anti-defection law.
covered in Section 9(1) of the Citizenship act 77. (a) The Fifth Schedule of the Constitution of India deals

ww 1955, provides that any citizen of India who by


naturalisation or registration acquires the citizenship
of another country shall cease to be a citizen of
with administration and control of scheduled areas
and scheduled tribes in these areas. The Sixth Sched-
ule to the Constitution of India contains provisions

w.E
India. Indian Government has started OCI (Overseas
citizen of India) Scheme in 2005. According to the
scheme if you are already a Person of Indian origin
(POI) and have taken up citizenship abroad, you can
concerning the administration of tribal areas in the
States of Assam, Meghalaya, Tripura and Mizoram.

a syE
take up benefits of OCI scheme, which gives you
the same travel and residence privileges like other
Indians but you are not allowed to vote and take up
jobs in Government sector.
1. (d)
6. (d)
EXERCISE-2
2. (d) 3. (d) 4. (d) 5. (b)
7. (d) 8. (c) 9. (d)
10. (d) The forms of oaths or affirmations, in the third
64. (b) Tribal areas generally mean areas having preponder-
ance of tribal population. However, the Constitu- ngi schedule of the Indian constitution, are mentioned
for the given officials. Third Schedule (Articles
75(4), 99, 124(6), 148(2), 164(3), 188 and 219).

nee
tion of India refers tribal areas within the States of
Assam, Meghalaya, Tripura & Mizoram, as those 11. (d) 12. (c) 13. (a) 14. (d) 15. (b)
areas specified in Parts I, II, IIA & III of the table 16. (c) 17. (c) 18. (b) 19. (d) 20. (a)
21. (b) The Parliament of India has exclusive power to make
65. (b)

66. (d)
appended to paragraph 20 of the Sixth Schedule.
The Constitution of India was enacted on Nov. 26,
1949 but it was commenced on Jan. 26, 1950.
Tenth schedule was added by the 52nd Constitu-
r ing
any law with respect to any matter not enumerated
in the Concurrent List or State List. The distribution
of powers between the Union and Provinces is

67. (c)
tional Amendment Act, 1985. It provides for anti-
defection law.
The Constitution of India recognizes religious and
Constitution of India.
.ne
done through three different lists enumerated in the

68. (d)
linguistic minorities under article 29 and 30 (Cul-
tural and Educational rights). However it does not
define the term Minority.
Fourth schedule allocates seats in the Council of
t
22. (c) Federal government is the common government
of a federation. There is more than one level of
government which run according overlapping and
shared power is prescribed by a constitution.
23. (a) The preamble to the Constitution of India is a brief
States i.e. Rajya Sabha.
introductory statement that sets out the guiding
69. (a) First schedule contains names of the States and UTs,
that’s why it should be amended, if a new state is purpose and principles of the document.
created. 24. (a) A person born in India on or after 26th January
70. (a) Ninth Schedule was added by First Amendment Act 1950 but before 1st July 1987 is a citizen of India
of 1951, which relates to Land Reforms. by birth irrespective of the nationality of his parents,
71. (a) The ‘First Schedule’ of the constitution deals with considered citizen of India by birth if either of his
list of States and Union Territories. parents is a citizen of India at the time of his birth.
72. (d) Fourth schedule allocates seats in the Council of The citizenship of India is mentioned in Articles 5 to
States i.e. Rajya Sabha (Upper House of Parlia- 11 (Part II).
ment). 25. (a) Under article 5 of the Indian Constitution Citizenship
73. (b) Fifth schedule relates to the control and adminis- at the commencement of the Constitution every
tration of scheduled areas in states other than Tri- person who has his domicile in the territory of India
pura, Assam, Meghalaya and Mizoram. While Sixth and

Downloaded From : www.EasyEngineering.net


Downloaded From : www.EasyEngineering.net
P-22 Constitutional Framework and Citizenship

(i) who was born in the territory of India; or Naturalization or by Registration. They can be
(ii) either of whose parents was born in the territory of deprived of citizenship if they acquired citizenship
India; or by using fraudulent means.
(iii) who has been ordinarily resident in the territory 35. (c) The Constitution of India has 24 parts, 12
of India for not less than five years preceding such schedules and more than 444 articles at present.
commencement, shall be a citizen of India. In the original constitution, there were 22 parts,
26. (a) All statements are correct. 8 schedules and 395 articles. Ninth Schedule was
27. (c) A written constitution is one which is found in one or added by 1st Constitutional Amendment Act, 1951.
more than one legal documents duly enacted in the Tenth Schedule was added by 52nd Constitutional
form of laws. It is precise, definite and systematic Amendment Act, 1985. Eleventh Schedule was
and codified. An unwritten constitution is one in added by 73rd Constitutional Amendment Act, 1992.
which most of the principles of the government have Twelfth Schedule was added by 74th Constitutional
never been enacted in the form of laws. It consists Amendment Act, 1992.
of customs, conventions, traditions and some written 36. (b) Schedule X was added by 52nd amendment in 1985.
laws bearing different dates. It is not codified. It is It contains provisions of disqualification on the
unsystematic, indefinite and unprecise. So, Written grounds of defection.

ww constitution is the formal source of all constitutional


laws and Unwritten Constitution is not the formal
source.


Schedule Vlll contains List of 22 languages of India
recognized by Constitution.
Schedule l deals with the List of States & Union

ministers. w.E
28. (a) Delhi and Puducherry have their own elected
legislative assembliesand the executive councils of

29. (d) This act made the central legislature bicameral.



Territories.
Schedule X Contains acts & orders related to land
tenure, land tax, railways, and industries. Added by
Ist amendment in 1951. Laws under Schedule IX are

a
The first house which was central legislature with

syE
145 members (out of which 104 elected and 41
nominated) was called central Legislative Assembly
and second with 60 members (out of which 33 elected
beyond the purview of judicial review even though
they violate fundamental rights enshrined under part
III of the Constitution.
37. (b) Constitution of India is unique in itself. Many
and 27 nominated) was called Council of States.
It Introduced Diarchy in the provincial executive. ngi features of our constitution are borrowed from
various sources around the world.
Federal system of Government was introduced in
India by The Government of India Act 1935.
30. (a) According to the preamble of Indian Constitution,

nee
1. Amendment of the constitution - Constitution of
South Africa
2. Directive Principles- Irish Constitution
India is aSovereign, Socialist, Secular and
Democratic republic. Capitalism and free trade is not
enshrined in the preamble of the Indian constitution.
31. (d) All statements are true. Citizenship is covered in

r
3. Emergency Powers of the President- Constitution
of the Germany
ing
4. The Union State Relations- Constitution of Canada

.ne
38. (a) The Indian Council Act 1892- Introduction of the
Part II of the Indian constitution (articles 5-11). principle of election
32. (a) The Ninth Schedule was added by the 1st Amendment The Indian Council Act 1909- Introduction of
1951 to protect the laws included in it from judicial
scrutiny on the ground of violation of fundamental
rights. However in 2007 Supreme Court ruled that
the laws included in it after 24 April 1973 are now


separate electorate for the Muslims
t
The government of India act 1919- Introduction of
diarchy in provinces
The government of India Act 1935- Introduction of
open to judicial review. provincial autonomy.
33. (c) It does not permit the OCI to vote in general election 39. (a) The Constitution of India was drafted by the
in India. On the occasion of first Pravasi Bhartiya constituent assembly and it was set up under the
Diwas on 9th January 2003, former Prime Minister cabinet Mission plan on 16 May 1946. Dr. Rajendra
AtalBihari Vajpayee had made an announcement Prasad then became the President of the Constituent
for grant of dual citizenship to PIOs. The necessary Assembly. Tiruvellore Thattai Krishnamachari was
Legislation was introduced by the Government of a member of drafting committee. Harendra Coomar
India in Parliament in May 2003. Mookerjee was the Vice-president of the Constituent
34. (d) Article 371 A to I deals with special provisions Assembly of India for drafting the Constitution of
to -Nagaland, Assam, Manipur, Andhra Pradesh, India. Bhimrao Ambedkar was appointed Chairman
Sikkim, Mizoram, Arunachal Pradesh and Goa. of the Constitution Drafting Committee.
The Constitution of India envisages a single policy 40. (a)
for both Union and the States. A naturalized 41. (d) Borrowed features from different countries are
citizen is one who acquires citizenship either by correct.

Downloaded From : www.EasyEngineering.net


Downloaded From : www.EasyEngineering.net

FUNDAMENTAL RIGHTS, FUNDAMENTAL


DUTIES AND DIRECTIVE PRINCIPLES OF
2
STATE POLICY Chapter

ww
Introduction

w.E
Fundamental Rights are defined as basic human freedoms, which every Indian citizen has to enjoy for a proper and harmonious
development of personality. It contains six rights. Fundamentals Rights are justiciable and are protected by the judiciary. The
Constitution seeks to strike a balance between individual freedom and social interests through the Fundamental Duties. To make

a
fundamental right and duties of citizens of India safe and effective, there are certain guanidines for the governance of the country

syE
given in the Directive Principles of state policy.

Parts of Constitution

ngi
Part III

FRS
Part IV

DPS nee
Part IV A

FDS

Magna Carta Part III (Article 12 Borrowed from


r
Can be Suspended
during Emergency ing FRs available only
to citizens & Not to
Foreigners- Arts.
of India to 35) USA & France except Art. 20 & 21
.ne
15, 16, 19, 29, 30

Right to Equality Right to Freedom


Fundamental Rights

Right Against Right to Freedom of Curtural and


t
Right to
Exploitation Religion Educational Constitutional
Rights Remedies

Articles Articles Articles Articles Articles Articles


14-18 19-22 23-24 25-28 29-30 32
(14) Equality before (19) 6 Rights: (23) Prohibition (25) Freedom of (29) Protection of Includes writs:
Law & Equal Freedom of of traffic in conscience & interests of (1) Habeas Corpus
protection of speech & human beings free profession, minorities. (2) Mandamus
law. expression, & forced practice & (3) Prohibition
Assembly, labour. propagation of
(4) Certiorari
Association, religion.
Movement, (5) Quo–warranto
Residence &
Profession.

Downloaded From : www.EasyEngineering.net


Downloaded From : www.EasyEngineering.net
P-24 Fundamental Rights, Fundamental Duties and Directive Principles of State Policy

(15) Prohibition of (20) Protection (24) Prohibition of (26 ) Manage (30) Right of
discrimination in respect of employment religious minorities to
on grounds of conviction for of children in affairs. establish &
religion, race, offences. factories, etc. administer
sex, etc. educational
institutions.
(16) Equality of (21) Protection of (27) Freedom
opportunity life & personal from payment
in public liberty. of taxes for
employment. promotion of
any religion.
(17) Abolition of (22) Protection (28) Freedom from
untouchability. against arrest attending
& detention in religious
certain cases. instruction
or worship
in certain
educational

ww
(18) Abolition of
titles.
institutions.

Important Feature
w.E
Fundamental Rights (Part III) • Fundamental rights which impose absolute limitations
upon the legislative power cannot be regulated by the
legislature are covered by Articles 15, 17, 18, 20 and 24.

a
Articles 12-35 of the Constitution deal with fundamental

syE
rights. Constitution does not define fundamental rights but
says that they are fundamental and superior to ordinary
laws. Fundamental rights cannot be altered except with


All fundamental rights are guaranteed against state action.
If Rights under Article 19 and Article 21 are violated
by an individual, legal remedies but not Constitutional
remedies are available.


constitutional amendment.
Fundamental Rights incorporate provisions of “American
Bill of Rights” and are very elaborate and comprehensive. ngi • Article 12 defines the State which includes:
1. Government and Indian parliament i.e. executive
and Legislature of the Union.
They provide for absolute rights with judiciary having
powers to impose limitations. Legislature can impose
limitations on fundamental rights during national
nee
2. Government and legislature of the states.
3. All local and other authorities within the territory
of India.


emergency.
These can be negative obligations on the state not to
encroach on individual liberty in its activities.


r
4. All local and other authorities under the control of GOI.

ing
Article 13 confers the power of judicial review to the
courts of all legislative acts.
Supreme Court of India and State High Courts have this


They promote political democracy and prevent
authoritative rule.
In USA legislature is competent to interfere with .ne
power under Article 32 and 226. They can declare a law
unconstitutional if it is inconsistent with Part III of the


individual rights if they endanger safety of the state and
collective interests.
Originally the Constitution classifies Fundamental Rights
under 7 groups.

Constitution.

of Laws
t
Power of judicial review makes the constitution legalistic.
: Equality before law/Equal protection

• Now there are only 6 groups as Right to Property has • Equality before law and equal protection of laws are
been eliminated by the 44th Amendment Act, 1978 and different.
transposed to a new Article – Article 300 A, which
Equality Before Law Equal Protection of Law
is outside Part-III of the Constitution and has been
labelled in as ‘Chapter IV’ of Part XII – but that is not a Negative concept Positive concept
fundamental right. Absence of special privilege Right to equality of treatment
due to birth, creed or like in the in similar circumstances.
• Fundamental rights under Articles 15, 16, 19, 29 & 30
favour of any person. There is
are applicable to Indian citizens.
equal treatment before law.
• Fundamental rights under Articles 14, 20, 21, 23, 25,
Dicey’s concept of rule of law. Treated as due process of law.
26, 27 and 28 are available to all resident of India both
citizens and foreigner. Established law in England. An American concept.
• Some fundamental rights are negatively worded as ‘Rule of Law’ is the “Basic Feature” of the Constitution which
prohibitions on the State like Articles - 14, 15(1), 16(2), cannot be destroyed even by constitutional amendment under
18(1), 20, 22(1) and 28(1) Article 368.

Downloaded From : www.EasyEngineering.net


Downloaded From : www.EasyEngineering.net
Fundamental Rights, Fundamental Duties and Directive Principles of State Policy P-25

Exception to the Rule of Law ARTICLE 17 & 18: Abolition of Untouchability and
Art Explanation
Titles
361 Immunity to the President of India and State 17 Abolition of Untouchability. The parliament has
Governors. passed protection of civil rights act, 1955 to abolish
untouchability.
361(1) President of India and state Governors are 18 Abolition of Titles except military and academic
not answerable to any court for exercise and titles, i.e, Bharat Ratna, Padma Vibhushan, Padma
performance of their powers and duties. Shri and National Awards.
361 (2) No criminal proceedings can be initiated against
ARTICLE 19
President of India and state Governors during their
term of office. • Provides for 6 fundamental rights in the nature of freedoms.
These are guaranteed to Indian citizens with reasonable
361 (4) No civil proceeding can be instituted during the term restrictions.
of office in respect of any act done by President of
India and State Governor in their personal capacity Freedom Restrictions imposed
before or after they enter office till 2 months after 19 (1) (a) Speech & 19 (2) Integrity and

ww the notice has been delivered to the President.


The foreign ambassadors and diplomats enjoy
immunity from criminal and civil proceedings.
Expression

Freedom
8 Grounds sovereignty of India
Security of the state
Friendly relations with


immunity.
w.E
The UNO and its agencies enjoy the diplomatic

Concept of equality provides protection against


of Press &
Media.
People’s
Right to
foreign states
Public order
Decency and morality



arbitrariness.

a syE
Concept of equality promotes natural justice.
Right to Equality incorporates Equal pay for equal.
work [Art 39(d)]. This is not a fundamental right but a
Know Contempt of court
Defamation
Incitement to an offence
constitutional goal under Articles 14, 16 and 39(d).
ARTICLE 15: Prohibition of discrimination
ngi 19 (1) (b) Assembly 19(3) Assembly must be peaceful.
3 Grounds Assembly must be unarmed
Restriction under Art 19
15 (1) No discrimination on grounds of religion, race,
caste, sex or place of birth can be made. It applies
to matters under the control of the state. nee (3):
Sovereignty and integrity
of India
15 (2) Prohibits discrimination at public places (shops,
public hotels, restaurants, well, tanks, bathing
ghats etc.) and applies both to state and private
r
19 (1) (c) Forming
Association ing
19 (4)
Public order
Sovereignty and integrity
3 Grounds of India

15 (3)
individual.
Provisions for protection of women & children. Morality .ne
Public order

15 (4) Provisions to protect interests of backward classes,


1st amendment Act, 1951.

ARTICLE 16: Equality of opportunity in public


19 (1) (d) Freedom of
Movement
19 (5)
t
Interest of general public
2 Grounds Protection of interests of
any Scheduled Tribe
employment 19 (1) (e) Freedom of 19 (5) Interest of general public
Residence 2 Grounds Protection of interests of
16 (1)&(2) No discrimination in public employment on
grounds of religion, race, caste, sex, descent, Scheduled Tribe.
place of birth or residence. 19 (1) (f) Freedom of 19 (6) By the state making any
16 (3) Residence within a state is a qualification for Profession, law relating to:
appointment for any government post. Occupation, Protecting Public interest.
16 (4) For reservation of posts in govt. jobs in favour of Trade or Establishing professional/
any backward class. Business technical qualifications for
16 (5) Provides for the incumbent of any office, in a profession/ occupation,
connection with the affairs of any religious or trade or business.
denominational institution or any member of the Enabling state to conduct
governing body shall be a person professing a any trade or business
particular religion or belonging to a particular excluding citizens wholly
denomination is not a violation of the Constitution. or partially.

Downloaded From : www.EasyEngineering.net


Downloaded From : www.EasyEngineering.net
P-26 Fundamental Rights, Fundamental Duties and Directive Principles of State Policy

ARTICLE 20 : Protection in respect of conviction qualified to be appointed as judge of a High Court has
for offences reported before the expiration of the said period of 3
• Provides 3 safeguards to persons accused of crimes: month that their is in its option sufficient cause for such
▪▪ Article 20 (1): Ex-Post facto law – no person shall detention.
be convicted of any offence except for the violation • This right is not available to an enemy, an alien and a
of ‘law in force’. Such protection does not apply in person arrested and detained under Preventive Detention.
case of Preventive Detention. ARTICLE 23 and 24: Right Against Exploitation
▪▪ Article 20 (2): Double Jeopardy – no person shall
• Article 23: protects individual against actions of the
be prosecuted and punished for the same offence
state and private citizens. This right is available to both
more than once.
citizens and non-citizens.
▪▪ Article 20 (3): Prohibition against Self • Article 23(i): prohibits traffic in human beings and
Incrimination – no person accused of an offence forced labour,
shall be compelled to be a witness against himself. • Article 23(ii): nothing in this article shall prevent state
ARTICLE 21 and 22 : Protection of Life and from imposing compulsory services for public purpose
Personal Liberty and in imposing such service the state shall not make
any discrimination on grounds of religion, race, caste or
• Constitution provides for a two fold guarantee:

ww
▪▪ No person shall be deprived of his life and personal
liberty except according to the law (Article 21).
▪▪ Safeguards against arbitrary arrest and detention

class or any of them.
Article 24: prohibits employment of children below
14 years of age in a dangerous occupation, factory and

• w.E
(Article 22).
Prior to Menaka Gandhi Case (1978), Article 21
guaranteed the Right to Life and Liberty against arbitrary •
mines.
ARTICLE 25 and 28: Right to Freedom of Religion
India, under the Constitution, is a “Secular State”,


It includes – a
action of the executive. Article 21 now protects Right to

Right to live with human dignity. syE


Life and Personal Liberty even from legislative action.

i.e. a state which observes an attitude of neutrality and
impartiality towards all religions.
There is no “State religion” in India. State will not
establish a religion of its own, nor will it patronize any



Right to livelihood.
Right to privacy.
Right to shelter. ngi religion. This is implicit from:
▪▪ State will not compel any citizen to pay any tax for
promotion or maintenance of a religion or religious



Right to health and Medical Assistance.
Right to free legal aid.
Right against solitary confinement.
nee
institution (Article 27).
▪▪ No religious instruction shall be provided in
an educational institution run completely by


Available to ‘citizens’ and ‘non-citizens’.
Right to Education is a Fundamental Right under
Article 21-A (86th Constitutional Amendment 2002).
r
government funds.

ing
▪▪ Religious instruction can be imparted in educational
institutions recognized by or receiving aid from the


Provision of Compensation if Article 21 is violated.
Right to Death is not a fundamental right under
Article 21. .ne
state, no person attending such institution shall be
compelled to receive such religious instructions
(Art 28).
ARTICLE 22
Article 22 grants protection to persons who are arrested or
detained. Detention is of two types – punitive and preventive.


religion
t
Article 25 guaranteed the Freedom of Conscience and
Freedom to Profess, Practice and Propagate personal

Right to ‘Propagate’ under Article 25 gives the right


Punitive Detention is to punish a person for an offence only to disseminate the tenets of religion but it would not
committed by him after trial and conviction in a Court. include the Right to Convert.
Preventive means detention of a person without trial and • States have made it a penal offence to convert or attempt
conviction by a person for a past offence, but to prevent him to convert a person by means of “force, fraud or
from committing an offence in the near future. allurement’.
• It includes • Volunteer conversion with free consent is allowed.
▪▪ Right to be informed about the ground of arrest. • Article 26 provides rights to every religious group or
▪▪ Right to be defended by a lawyer of his own choice. denominations:-
▪▪ Right to be produced before a magistrate within 24 ▪▪ To establish and maintain institutions for religious
hours. and charitable purposes.
▪▪ No detention beyond 24 hrs except by order of the ▪▪ To manage its own religious affairs.
magistrate. ▪▪ To own and acquire movable and immovable
No law providing for preventive detention shall authorize property.
the detention of a person for a longer period than 3 months ▪▪ To administer such property in accordance with
unless - an advisory board consisting of persons who are national laws.

Downloaded From : www.EasyEngineering.net


Downloaded From : www.EasyEngineering.net
Fundamental Rights, Fundamental Duties and Directive Principles of State Policy P-27

Cultural and Educational Rights Writs Under Article 32


• Article 29 (1) guarantees to citizens having a distinct Habeas corpus Order to the person who has detained
language, script or culture of its own, the right to means ‘to produce another to produce the detainee before
conserve the same. the body of’ the court. This is issued to let the court
know the grounds of confinement. This
• Article 30 provides for the right to religious and protects individual liberty.
linguistic minorities to establish and maintain educational It is a powerful safeguard against
institution to conserve their language, script or culture. arbitrary Acts not only of private
• Article 30 (2) prohibits the state from discriminating individual but also of the Executive.
against any educational institution in grant of aid. No Mandamus means Commands a public or quasi-public
citizen shall be denied admission to educational institutions ‘a Command’ legal person to perform his duty.
maintained by the state or receiving aid out of state The writ of mandamus can be issued
by the court to enforce Fundamental
funds on grounds of religion, race, caste and language.
Rights: whenever a public officer or
It is compulsory for unaided private institutions to give a Government has committed an Act
reservations to backward classes. Minority institutions violating a person’s Fundamental
are exempted from such obligation. Rights, the court can restrain that


ww
Article 29 applies only to citizens.
Article 30 applies to both citizens and non-citizens.

ARTICLE 31-A, 31-B & 31-C


authority from enforcing such orders or
committing such an act.
Prohibition means Issued by Supreme Court or High

31 A
Art
w.E
Relates To
Facilitate agrarian reforms
Amendment
1st Amendment, 1951
‘to forbid’ Court to a lower court forbidding it
continue proceedings in a case beyond
its jurisdiction or exercise jurisdiction
which is not vested with it legally.
31 B
a
None of the acts mentioned in 1st Amendment, 1951
9th Schedule shall be deemed
to be void on the ground that
they are inconsistent with
syE
The Supreme Court can issue the writ
only where a Fundamental Right is
affected because of jurisdictional defect
in their proceedings.
Part III of the Constitution.
Legislature is competent to
amend and repeal these acts.
ngi Certiorari means Issued to a lower court after a case has
‘to be certified’ been decided by it quashing the decision
or order. It ensures that the jurisdiction

31 C Empowers legislature to 25th Amendment, 1971


enact laws for implementing nee of an inferior court or tribunals is
properly exercised.
While prohibition is available during
the pendency of the proceedings and
Directive Principles of State
Policy under Articles 39 (b)
& (c). Only Articles 39(b)
& (c) have over-riding effect
r ing
before the order is made, certiorari can
be issued only after the order has been
made under similar circumstances.


over fundamental rights.
Supreme Court has viewed that 9th Schedule must come
Quo Warranto
means ‘what is
your authority’ .ne
Issued by the court to enquire into the
legality of claim which a person asserts
to a public office.


under Judicial Review. Sates have passed acts regarding
reservations, and placed them under Schedule 9 to make
them non-justiciable.
Right to Property which was a fundamental right ▪▪
t
The writ of quo warranto enables the
public to see that a public office is not
usurped by an unlawful claimant.
Article 226 gives power to High Court to issue writs.
under Article 31, was repealed by 44th Constitutional ▪▪
Article 32 is used for the enforcement of fundamental
Amendment, 1978. It was made a Constitutional Right rights only, Article 226 is helpful for “any other
under ordinary law under Article 300-A. purpose” also. Therefore the power of the High
Court is wider under Article 226 than the power
ARTICLE 32: Right to Constitutional Remedies conferred on Supreme Court under Article 32.
• It provides for machinery for the effective enforcement ▪▪ Courts also issue Injunction, which is not mentioned
of Fundamental Rights. in the Constitution. It is issued against private persons.
• It empowers a person to approach the Supreme Court ▪▪ Public Interest Litigation (PIL)
The traditional rule to apply for redressal of breach
directly for the enforcement of his Fundamental Rights.
of fundamental rights. The person whose right has
• Right to Constitutional Remedies cannot be suspended been breached can approach the court (locus standi).
except otherwise provided in the Constitution, i.e. during ▪▪ Public Interest Litigation (PIL), borrowed from USA,
Emergency. is being applied for achieving larger public interest.
• Dr. Ambedkar calls this article “the very soul and Any public-spirited person can go to the court for
heart of the Constitution.” redressal of breach of fundamental rights.

Downloaded From : www.EasyEngineering.net


Downloaded From : www.EasyEngineering.net
P-28 Fundamental Rights, Fundamental Duties and Directive Principles of State Policy

Right to Information basic structure of the Constitution has been pronounced


The right to information has been granted to citizens under the by the Supreme Court.
Information Act passed by the Indian Parliament in 2005. • The Parliament is not authorized to limit the operation
The Act entitles every citizen to have access to information of Articles 14, 19 and 21 which form the part of basic
controlled by public authorities of both the Union and the State structure of the Constitution.
governments. The main objective of this right is to make the
government open, transparent, responsive and accountable Directive Principles of the State Policy
to the people. According to this law people can seek any • Contained in Part-IV of the Constitution (Articles 36-51),
information from the government, which is duty bound to borrowed from Ireland.
provide the requisite information within a specified period of • These are fundamental guidelines in the governance of
30 days. If the concerned authorities do not provide correct and the country.
timely information, complaint can be lodged with the Central
• They promote the concept of a welfare state.
Information Commission/State Information Commission. If
• They are classified into 3 categories:
the requested information is denied to a person, he/she must
be informed of the reasons for refusal. It may further be noted • Socialistic or Welfare Principles: Arts 38, 39, 41-43.
that the law does not apply to Jammu and Kashmir, or security • Gandhian Principles: Arts 40, 43, 46-49
• Liberal - Intellectual Principles: Arts 44-45, 50-51

ww
agencies like IB, RAW and BSE. The other areas which have
been excluded from the jurisdiction of the law include cabinet
papers, legal advice relating to decision making, information
• While fundamental rights aim at political freedoms,
DPSP aim at securing economic and social justice

w.E
likely to breach the privileges of the Parliament and state through appropriate state action.
legislatures, etc. • They impose obligations on the state and give directions
to take positive action to promote social welfare.
Suspension of Fundamental Rights • These are the ideals to be kept in mind while formulating


a
Fundamental rights are not absolute and have limitations

syE
regarding their exercise. The limitations strike a balance
between individual liberty and social need.
Article 358 provides that during national emergency,


policy or pass a law.
Directives are non-justiciable. Courts cannot be
approached for their enforcement (Article 37).
Articles 38 & 39 embody the Jurisprudential doctrine of
President under Article 352, fundamental freedoms
guaranteed by Article 19 are suspended and cannot
be revived during emergency. Things done during ngi •
“Distributive Justice”.
42nd Amendment Act (1976) added Articles 39-A, 43-A,


emergency cannot be challenged even after it is over.
Article 359 empowers the President to suspend the Right
to move a Court for the enforcement of rights conferred
nee
48-A. Provision for “Creation of Opportunities
for healthy development of children” in Article
39 (A).


by Part III of the Constitution (except Article 20 & 21)
during an emergency.
Under Article 358, rights conferred by article 19 are

r
Dr. B.R. Ambedkar described the DPs as “Instrument
of Instruction.”
Art ingRelates To
suspended. Suspension under article 359 can only be by
an order of the President.
38

.ne
Social order based on justice and to minimise
inequalities in income, status, facilities and
opportunities.
Can Fundamental Rights be Amended


Whether fundamental rights are amendable is a matter of
controversy.
In Golaknath case, 1967, Supreme Court questioned
39

• Means of Livelihood to all.


t
Principles of policy to be followed by the State for
securing economic justice in the form of:

• Use of resources for common good.


the validity of amendments to the Constitution by the • Prevention of concentration of wealth.
• Equal pay for equal work.
parliament. It held the amendments to be invalid.
• Protection of workers.
• 24th Amendment 1971, added word ‘Power’ to Article
• Protection of children and youth.
368 and described specifically the power of Parliament to
39 A Equal justice and free legal aid to the poor. Legal
amend the Constitution and laid down procedures.
aid and speedy trial are fundamental rights under
• This was challenged in Keshavanand Bharati case (1973)
Article 21 of the Constitution available to all
in which “Basic Structure” doctrine was established by prisoners and are enforceable by the courts.
the Supreme Court. Parliament had the power to amend
40 Organization of village panchayats.
the Constitution without harming its Basic structure. But
as to what forms the basic structure is not clear. 41 Right to Work, Education and Public assistance
in some cases.
• 42nd Amendment was passed by the parliament in 1976.
It was called the “Mini Constitution”. It gave unlimited 42 Just and humane conditions of work.
amending powers to the Parliament. The validity of this 43 Living wage for workers. Living wage and not
amendment was tested in Minerva Mills case, 1980. The minimum wage.

Downloaded From : www.EasyEngineering.net


Downloaded From : www.EasyEngineering.net
Fundamental Rights, Fundamental Duties and Directive Principles of State Policy P-29

43 A Participation of workers in management. Fundamental Duties


44 Uniform civil code. • Covered under Article 51-A (PART IV A) of the
45 Free and compulsory education for children. Constitution.
• Added to the Constitution by 42nd amendment, 1976.
46 Educational and economic interests of SC, ST and • Added on the recommendations of “Swaran Singh Committee.”
weaker sections. • Originally 10 duties, but now increased to 11.
47 Standard of living and improvement in health. 51-A (k) was added by 86th Amendment Act, 2002.
(93rd amendment bill).
48 Agriculture and animal husbandry. • Borrowed from the Constitution of undivided USSR.
48 A Protection of environment, forests and wildlife.
Article Relates to
49 Protection of monuments, places and objects of 51 A (a) Abide by the Constitution and respect National
national importance. Flag and National Anthem.
50 Separation of judiciary from the executive. b Follow ideals of the freedom struggle.
51 Promotion of international peace and security. c Protect sovereignty & integrity of India.
d Defend the country and render national services
Directives in other Parts of the
ww
Constitution
Besides the directives in part IV, there are certain other
e
f
when called upon.
Spirit of common brotherhood.
Preserve composite culture.

w.E
Directives in the Constitution, also non-Justiciable. These
include Article 350 A which calls upon the state to provide
adequate facilities for instruction in mother tongue at primary
g
h
i
j
Protect natural environment.
Develop scientific temper.
Safeguard public property.
Strive for excellence.

a
school level to children from linguistic minority groups.

syE
Article 351 calls upon the Union to promote the spread and
development of Hindi to enable it to become the medium of
expression of all the parts of the composite culture of India.
k Duty of all parents and guardians to send their
children in the age group of 6-14 years to school.
Verma Committee on Fundamental Duties
Article 335 supports the claims of the Scheduled Castes and
Scheduled Tribes to appointments in government service,
subject to the maintenance of efficiency of administration. ngi In 1999, the Verma Committee on Fundamental Duties of the
citizens identified and pointed out the legal provisions for the
enforcement of certain Fundamental Duties. The following are
Fundamental Rights and Directive Principles: Difference
There is no doubt that both the Fundamental Rights and the nee
some of them:
(a) The Prevention of Insults to National Honour Act (1971)
Directive Principles of State Policy are important feature of the
Constitution. However, they differ from each other in certain
points. r
prevents disrespect to the Constitution of India, the

ing
National Flag and the National Anthem.
(b) The Indian Penal Code declares the imputations and
assertion prejudicial to national integration as punishable
Differences –
(i) The Fundamental Rights seek to protect the individual
offences.
.ne
(c) The Wildlife (protection) Act 1972 prohibits trade in rare
from state encroachment; the Directive Principles are
aimed at the promotion of the general welfare of society.
(ii) The Fundamental Rights constitute limitations upon State
action; The Directive Principles are positive instructions
and endangered species.
t
(d) The Representation of People Act (1951) has provided
for the disqualification of members of the Parliament or
a State Legislature for indulging in corrupt practice of
to the government to take steps to establish a just social,
soliciting votes on the ground of religion or promoting
economic and political order.
enmity between different sections of people on grounds
of religion, caste, race and language, etc.

Downloaded From : www.EasyEngineering.net


Downloaded From : www.EasyEngineering.net
P-30 Fundamental Rights, Fundamental Duties and Directive Principles of State Policy

Exercise - 1
1. Fundamental Right to ... has been deleted by the ... 9. Entitlement of legal aid is a concomitant right that arises
Amendment Act. out of Fundamental Right:
(a) Form associations; 44th (a) To freedom
(b) Property; 44th (b) Against exploitation
(c) against exploitation; 42nd (c) To equality before law or equal protection of laws
(d) All of the above
(d) private property; 42nd
10. The Fundamental Duties were added to the Constitution
2. Regarding equality before law, the Indian Constitution
(a) to make the fundamental rights more effective
makes exceptions in the case of (b) to check anti-national, subversive and unconstitutional
(a) President or a Governor agitations
(b) Foreign sovereigns only (c) to accord priority to the directive principles over
(c) President only fundamental rights
(d) None of the above (d) to achieve all the above objectives

ww
3. Constitution permits preventive detention but stipulates
that:
(a) no one should be detained beyond three months
11. A fundamental Right guaranteed in the Indian


Constitution can be suspended only by
(a) a proclamation of national emergency

detention.
w.E
unless an Advisory Board authorises further

(b) grounds for detention should be conveyed to the





(b) an Act passed by the Parliament
(c) an amendment of the Constitution
(d) the judicial decision of the Supreme Court
12. “The state shall strive to promote the welfare of people by


a
person before arresting him

within 24 hours of the arrest


(d) All of the above syE
(c) the person must be produced before a magistrate
securing and protecting as effectively as it may, a social
order in which justice–social, economic and political–
shall inform all the institutions of the national life.” This
provision is contained in which of the following articles
4. ______ can impose reasonable restrictions on


Fundamental Rights of Indians?
(a) Supreme Court ngi

of the Indian Constitution?
(a) Article 39
(c) Article 38
(b) Article 46
(d) Article 37
(b) Parliament
(c) President on the advice of the Council of Ministers
(d) None of these as restrictions are provided in the
in
nee
13. The Directive Principles of State Policy are incorporated

(a) Part III (Articles 36-51)


Constitution
5. Enforcement of Directive Principles depends on
(a) Courts


r
(b) Part IV (Articles 36-51)

ing
(c) Part V (Articles 19-21)
(d) Part II (Articles 36-56)
14. A writ of prohibition is an order issued by the supreme
(b) Effective opposition in the Parliament
(c) Resources available to the Government
(d) Public cooperation

court or high court which
.ne
(a) affects the production and consumption of liquor

6. If the State implements a Directive Principle calling for



equitable distribution of material resources it
(a) is put in the Ninth Schedule
(b) is not void even if it violates the rights in Articles 14


t
(b) prohibits the police from arresting a person
(c) forbids the administrative authority from taking a
particular action
(d) prohibits a quasi-judicial authority from proceeding
with a case
and 19 15. The 44th Amendment of the Indian Constitution withdrew
(c) can be struck down by the Supreme Court on the Fundamental Right:
(a) to freedom of religion
grounds of violating Fundamental Rights
(b) to constitutional remedies
(d) is not void if it violates a fundamental right
(c) to property
7. A new fundamental duty was included in Part IV A (d) against exploitation
under Article 51 A through Constitutional Amendment 16. Which one of the following pairs is not correctly matched?
numbered (a) Power of : Creating a new
(a) Forty-Second (b) Forty-Fourth Parliament State
(c) Eighty-Sixth (d) Ninety-Second (b) Power of State : Altering the name
8. Right to Information Act, 2005 is : Legislature of a State
(a) A fundamental right under the Constitution (c) Equality before : Both Indian and
(b) Available only to citizens the law non-Indian citizens
(c) Applicable for all public and private bodies (d) Equality of : Indian citizen
(d) Applicable for all states opportunity

Downloaded From : www.EasyEngineering.net


Downloaded From : www.EasyEngineering.net
Fundamental Rights, Fundamental Duties and Directive Principles of State Policy P-31

17. Which one of the following is not included in Article 19 (a) Prohibition, Certiorari & Mandamus.
of the constitution of India, pertaining to the Right to (b) Certiorari & Mandamus.
Freedom? (c) Prohibition & Mandamus.
(a) Right to reside and settle in any part of the territory (d) Prohibition & Certiorari.
of India 24. The Preventive Detention Act has a restraining effect on:
(b) Right to form associations or unions (a) Right of Equality
(c) Right of minorities to establish and administer (b) Right to Freedom
educational institutions (c) Right to Religion
(d) Right to assemble peaceably and without arms (d) Right to Constitutional Remedies
18. What are the Directive principles State policy? 25. Reasonable restriction on the rights of Indian citizens can
(a) The main objectives of the framers of the Constitution be imposed by:
(b) The principles that are expected to guide the state in (a) the President (b) the Parliament
the governance of the country (c) the Supreme Court (d) None of the above
(c) The ideals of a Welfare State that should be accept- 26. Under which Article of the Constitution can an individual
able to all right-thinking persons. move the Supreme Court directly in case of any violation
(d) Social rights are opposed to personal rights, of a Fundamental Right?
enumerated in Part III. (a) Article 13 (b) Article 14

ww
19. Which one of the following is not a correct statement in
relation to the provisions of the Indian Constitution?
(a) No person shall be convicated of any offence except

27.
(c) Article 32 (d) Article 34
Freedom of speech under the Indian constitution is subject
to reasonable restriction on the grounds of protection of


w.E
for violation of law in force at the time of commission
(b) No person shall be denied bail
(c) No person shall be punished for the same offence
more than once




(a) sovereignty and integrity of the country
(b) the dignity of the office of the Prime Minister
(c) both (a) and (b)
(d) none of the above

a
(d) No person accused of an offence shall be compelled
to be a witness against himself
syE
20. Which of the following Fundamental Duties have been
enumerated in our Constitution for the citizens?
28.


Cultural and Educational rights include:
(a) Right of minorities to establish and administer their
educational enstitutions
(b) Right of minorities to promote their language
(a) To abide by the Constitution and respect its ideals
and institutions, the National Flag and the National
Anthem
ngi (c) Right against discrimination for admission to
educational institutions on the grounds of religion,
race or caste


(b) To cherish and follow the noble ideals which inspired
our national struggle for freedom
(c) To uphold and protect the sovereignty, unity and

29. nee
(d) All of these
The granting of fundamental rights to citizens aims at
ensuring:


integrity of India
(d) All of the above
21. Which one of the following is incorrect?



r ing
(a) an independent judiciary
(b) socialistic government
(c) individual liberty
(a) Our Constitution does not discriminate against any
citizen on the ground of his religion, in matters of
employment to public services or holding public

30.
(d) none of these

.ne
Which fundamental right is concerned with abolition of
social distinctions?


offices
(b) Our Constitution does not prohibit the imparting of
religlous instruction in educational institutions run
by the Government




(a) Right to equality
(b) Right against exploitation
(c) Right to life and liberty
(d) Cultural and educational rights
t
(c) The Constitution of India guarantees to all the 31. The Fundamental Rights have the sanction of:
citizens of India the right to profess, practise and (a) the Supreme Court
propagate a religion of their choice (b) the Constitution
(d) No person can be made to pay taxes for the promotion (c) majority opinion
of any particular religion under our Constitution (d) the Government
22. Article 21 declares that – 32. How can the Fundamental Rights be suspended?
“No person shall be deprived of his life or personal liberty (a) If Parliament passes a law by two-thirds majority
except according to procedure established by law.” (b) If the Supreme Court orders it
The protection under article 21 is: (c) If the President orders it in the time of National
(a) Against arbitrary legislative action. Emergency
(b) Against arbitrary executive action. (d) They can never be suspended
(c) Both A & B. 33. Which one of the following writs literally means ‘we
(d) None command’?
23. Which of the following writs can be issued against (a) Habeas Corpus (b) Mandamus
administrative authorities? (c) Quo Warranto (d) Certiorari

Downloaded From : www.EasyEngineering.net


Downloaded From : www.EasyEngineering.net
P-32 Fundamental Rights, Fundamental Duties and Directive Principles of State Policy

34. In India, Mandamus will lie against: (a) the Preamble and the Fundamental Rights
(a) officers bound to do a public duty (b) the Preamble and the Directive Principles of State
(b) Government Policy
(c) both officers and the Government (c) the Fundamental Rights and the Directive Principles
(d) none of these of State Policy
35. The writ of prohibition issued by the Supreme Court or a (d) None of the above
High Court is issued against: 44. Which of the following Articles of the Constitution of
(a) judicial or quasi-judicial authorities India makes a specific mention of village panchayats?
(b) administrative and judicial authorities (a) Article 19 (b) Article 21
(c) administrative authorities only (c) Article 40 (d) Article 246
(d) administrative authorities and government 45. The Instrument of Instructions on the Government of
36. Which of the following Fundamental Rights have been India Act 1935 has been incorporated in the Constitution
deleted from our Constitution through a constitutional of India in the year 1950 as
(a) Fundamental Rights
amendment?
(b) Directive Principles of State Policy
(a) Right against exploitation
(c) Fundamental Duties
(b) Right to freedom of religion
(d) Emergency Provisions
(c) Right to property

37.
ww
(d) Freedom of speech and expression
The authority to issue writs for the enforcement of
Fundamental Rights rests with whom?
46. Freedom of the press in India is


(a) available to the people under the law of the
Parliament
(b) specifically provided in the Constitution


w.E
(a) All the courts in India
(b) The Parliament
(c) The Supreme Court


(c) implied in the right of freedom of expression
(d) available to the people of India under executive
order

38.


a
(d) The President of India

syE
How can the Fundamental Rights be protected by a citizen?
(a) By approaching the Supreme Court which will issue
appropriate writs against the authority
(b) Parliament will take note of such violations and tell
47. Right to Information in India is a



(a) Fundamental Right
(b) Legal Right
(c) Both Fundamental and Legal Right


the courts
(c) The Executive will inform the Courts ngi (d) Neither Fundamental nor Legal Right
48. Which one among the following statements is correct ?
The press in democracy must

39.
(d) It is automatically protected
Can Parliament amend or modify any of the Fundamental
Rights given in the Constitution?


nee
(a) be free and impartial
(b) be committed to the policies of the government
(c) highlight the achievement of the government without




(a) Parliament can do so only after a referendum
(b) Parliament can amend them by special majority
(c) Parliament cannot amend any
(d) Only the President can issue directions to amend

r
criticizing its policies

ing
(d) criticize the policies of the government
49. Which of the following would be construed as a
reasonable restriction of the right to freedom’?

40.
them
Right to participate in government and equal opportunity


votes in the name of religion
.ne
(a) When the state disallows a candidate from securing

(b) When the state disallows citizens from forming a



to occupy the highest office based on qualification gives
the citizens:
(a) national liberty
(c) natural liberty
(b) political liberty
(d) civil liberty
(c) When the Government of Nagaland disallowst
club out of State funds that denies access to women

temporary residents to buy immovable property in


Nagaland.
41. A citizen’s freedom of speech and expression may be
(d) All of the above
subjected to reasonable restriction on the grounds of all
50. Which one of the following is not a correct description of
except:
the Directive Principles of State Policy?
(a) sovereignty of India (b) public order (a) Directive Principles are not enforceable by the
(c) contempt of court (d) unbecoming criticism courts
42. The Indian Constitution declares that protection of life (b) Directive Principles have a political sanction
and liberty: (c) Directive Principles are declaration of objective for
(a) can never be taken away in any condition State Legislation
(b) can be taken away only according to procedure (d) Directive Principles promise equal income and free
established by law health care for all Indians
(c) can be taken away during the Emergency through 51. Which one of the following International Human Rights
Presidential order Instruments has been signed by India but not yet ratified?
(d) none of the above (a) Convention on the right of the child.
43. ‘Economic Justice’ as one of the objectives of the Indian (b) Convention on the Elimination of all forms of
Constitution has been provided in discrimination against women

Downloaded From : www.EasyEngineering.net


Downloaded From : www.EasyEngineering.net
Fundamental Rights, Fundamental Duties and Directive Principles of State Policy P-33

(c) Convention on the political rights of women 58. Which one among the following is a Fundamental Duties
(d) Convention on the nationality of married women of citizens under the Constitution of India?
52. Which one of the following is a human right as well as a (a) To provide friendly co-operation to the people of the
Fundamental Rights under the Constitution of India? neighbouring countries
(a) Right to Information (b) Right to Education (b) To protect monuments of national importance
(c) Right to Work (d) Right to Housing (c) To defend the country and render national service
53. Which one of the following Fundamental Rights is also when called upon to do so
available to a foreigner on Indian soil? (d) To know more and more about the history of India
(a) Prohibition of discrimination on grounds of religion, 59. Which one among the following writs literally means you
race, caste, sex or place of birth many have the body?
(b) Equality of opportunity in matters of public (a) Certiorari (b) Habeas Corpus
employment (c) Mandamus (d) Quo Warranto
(c) Protection of life and personal liberty according to 60. Which one among the following is not a fundamental
procedure established by law duty of the citizen of India?
(d) To practice any profession or to carry on any (a) To develop scientific temper, humanism and the
occupation, trade or business spirit of inquiry and reform
54. Which one among the following statements regarding the (b) To safeguard public property and to abjure violence


ww
constitutionally guaranteed Right to Education in India is
correct?
(a) This right covers both child and adult illiteracy and


(c) To uphold and protect the sovereignty, unity and
integrity of India
(d) To practice family planning and to control population

w.E
therefore, universally guarantees education to all
citizens of India.
(b) This right is a child right covering the age group of
61. The writ of Prohibition is issued by a superior court
(a) to prevent an inferior court or tribunal from
exceeding its jurisdiction or acting contrary to the
rules of natural justice
2015.
a
6 to 14 years and becomes operational from the year

syE
(c) This right has been taken from the British
Constitution which was the first Welfare State in the
world.


(b) to an inferior court or body exercising judicial or
quasijudicial functions to transfer the record to
proceedings in a case for its review
(c) where it can call upon a person to show under what
(d) This right has been given to all Indian children
between the ages of 6 to 14 years under the 86th ngi
authority he/she is holding the office
(d) to an authority to produce an illegally detained
person before the court for trial
Constitutional Amendment Act.
55. Which one among the following is not guaranteed by the
Constitution of India? nee
62. Which one among the following is not included in the
Fundamental Rights embodied in the Constitution of
India?



(a) Freedom to move freely throughout the country
(b) Freedom to assemble peacefully without arms
(c) Freedom to own, acquire and dispose property
anywhere in the country



r
(a) Right to Equality
(b) Right to Freedom
ing
(c) Right against Exploitation
(d) Freedom to practice any trade or profession
56. The purpose of Directive Principles of State Policy is to
(d) Right to Information

.ne
63. Which one among the following is a Fundamental Duties
of citizens under the Constitution of India?



(a) lay down positive instructions which would guide
State Policy at all levels.
(b) implement Gandhiji’s idea for a decentralised state
(c) check the use of arbitrary powers by the government



t
(a) To provide friendly cooperation to the people of the
neighbouring countries
(b) To visit the monuments of national importance
(c) To defend the country and render national service
(d) promote welfare of the backward sections of the when called upon to do so
society (d) To know more and more about the religions of India
57. Suppose a Legislation was passed by the Parliament 64. Which one among the following statements is not correct?
imposing certain restrictions on newspapers. These (a) The right conferred by Article 32 cannot be
included page ceiling, price and advertisements. The suspended except by virtue of Article 359 (1) of the
legislation is included in the Ninth Schedule to the Constitution of India
Constitution of India. In this context, which one among (b) The enforcement of Articles 20 and 21 cannot be
the following statements is correct? suspended
(a) The legislation is invalid as it violates the Freedom (c) Punishments can be prescribed by a State Legislation
of Press for offences under Part III of the Constitution of
(b) The legislation is valid by virtue of Article 31 B India
(c) The legislation is invalid as it imposes unreasonable (d) The Fundamental Rights can be abrogated by law
restrictions under Article 19 (2) of the Constitution made by the Parliament with regard to members of
(d) The legislation is valid as the Press is not a citizen the forces charged with the maintenance of public
under Article 19 of the Constitution order

Downloaded From : www.EasyEngineering.net


Downloaded From : www.EasyEngineering.net
P-34 Fundamental Rights, Fundamental Duties and Directive Principles of State Policy

65. Which of the following freedoms is not specifically work in any factory or mine or engaged in any other
mentioned in the Constitution of India as a Fundamental hazardous employment’?
Right but has been subsequently upheld by the Supreme (a) Article 24 (b) Article 45
Court as such? (c) Article 330 (d) Article 368
(a) Freedom of trade, occupation and business 69. Under the Constitution of India, which one of the
(b) Freedom to reside and settle in any part of the following is not a fundamental duty ?
country (a) To vote in public elections
(c) Freedom of association and union (b) To develop the scientific temper
(d) Freedom of the press (c) To safeguard public property
66. Which one of the following categories of persons is not (d) To abide by the Constitution and respect its ideals
treated at par so far as the availability of Fundamental 70. According to the Constitution of India, which of the
Rights is concerned ? following are fundamental for the governance of the
(a) Members of the armed forces country?
(b) Members of the forces charged with the responsibility (a) Fundamental Rights
of maintenance of public order (b) Fundamental Duties
(c) Members of the forces employed in connection with (c) Directive Principles of State Policy
the communication systems set up in the country (d) Fundamental Rights and Fundamental Duties
71. The purpose of the inclusion of Directive Principles of

ww
(d) Members of the forces employed in connection with
the communication systems set up for maintenance
of public order
67. Which of the following is not true of Article 32 of the


the State Policy in the Indian Constitution is to establish:
(a) political democracy
(b) social democracy

w.E
Indian Constitution ?
(a) It gives the Supreme Court and the High Courts
the power to issue writs for the enforcement of


(c) Gandhian democracy
(d) social and economic democracy
72. In the Constitution of India, promotion of international
peace and security is included in the [CSAT 2014-I]


a
Fundamental Rights.

syE
(b) It is included in Part III of the Indian Constitution
and is therefore itself a Fundamental Right.
(c) Dr. Ambedkar called it the ‘very soul of the Indian




(a) Preamble to the Constitution
(b) Directive Principles of State Policy
(c) Fundamental Duties
(d) Ninth Schedule

Constitution’.
(d) An aggrieved person has no right to complain under
Article 32 where a Fundamental Right has not been ngi 73. The ideal of Welfare State’ in the Indian Constitution is
enshrined in its 
(a) Preamble
[CSAT 2015-I]

violated.
68. Which Article of the Constitution of India says, ‘No child
below the age of fourteen years shall be employed to


nee
(b) Directive Principles of State Policy
(c) Fundamental Rights
(d) Seventh Schedule

Exercise - 2 r ing
Statement Based MCQ
3.

________ are wrongly matched?
.ne
1. Habeas corpus – literally means ‘to have a body’
1.



What does not explain the reason for the rights included
in Part III of the Constitution being called ‘fundamental’?
1. They are available to both citizens and aliens
2. They are above ordinary law


case
t
2. Mandaums – commands a person to perform a public
duty issued to a lower court to stop proceedings in a

3. Quo warranto – ‘by what authority’ issued against


a person holding a public office or governmental
3. They are fundamental to governance.
4. They are not absolute privilege.
Which of the above is/are correct? 4. Prohibition - issued to a lower court quashing a
(a) 2 and 4 (b) 1, 2 and 3 decision or order
(c) 1, 3 and 4 (d) 3 and 4 Which of the following statements is/are correct?
2. _____ comes under Right to Equality? (a) 2, 3 and 4 (b) 1 and 2
1. Non-discrimination by State on grounds of religion (c) 3 only (d) 3 and 4
or race 4. ____ is the “socialistic” Directive Principles
2. Equal protection before law 1. Prevention of concentration of wealth
3. Equal pay for equal work 2. Right to work
4. Equality of opportunity in employment provided by 3. Separation of judiciary and executive
the State 4. Organising agriculture along scientific research
Which of the above is/are correct? Which of the above is/are correct?
(a) 1, 2 and 3 (b) 2, 3 and 4 (a) 1 and 2 (b) 1, 2 and 3
(c) 2 and 4 (d) 1, 2 and 4 (c) 2 and 4 (d) 1, 2, 3 and 4

Downloaded From : www.EasyEngineering.net


Downloaded From : www.EasyEngineering.net
Fundamental Rights, Fundamental Duties and Directive Principles of State Policy P-35

5. Which statement does not indicate the difference between 10. Which of the following are envisaged by the right against
Fundamental Rights and Directive Principles? exploitation in the constitution of India?
1. Directive Principles aim at promoting social welfare, 1. Prohibition of traffic in human beings and forced
while Fundamental Rights protect individuals from labour
State encroachment 2. Abolition of untouchability
2. Fundamental Rights put limitations on State action but 3. Protection of the interests of minorities
Directive Principles are positive instructions to the 4. Prohibition of employment of children in factories
Government to move towards a just socio-economic and mines.
order Which of the following statements is/are correct?
3. Fundamental Rights were the terminate six months at (a) 1 and 2 (b) 1 and 3
most the period till the date for presidential election (c) 2, 3 and 4 (d) 1 and 4
is notified included in the original constitution, 11. Which of the followings is/are among the Fundamental
but Directive Principles were added by the first Duties of citizens laid down in the Indian Constitution?
amendment
1. To preserve the rich heritage of our composite
4. Fundamental Rights can be amended but Directive
culture
Principles cannot be amended
2. To protect the weaker sections from social injustice
Which of the following statements is/are correct?
(a) 1 and 2 (b) 2 and 3 3. To develop the scientific temper and spirit of inquiry

ww
(c) 3 and 4 (d) 1, 2 and 4
6. Which are among the Fundamental Duties in the
Constitution?



4. To strive towards excellence in all spheres of
individual and collective activity
Select the correct answer using the codes given below :
(a) 1 and 2 only (b) 2 only




w.E
1. To preserve the heritage of our composite culture
2. To abide by the Constitution
3. To strive for excellence in scientific research
4. To render national service
(c) 1, 3 and 4 only (d) 1, 2, 3 and 4
12. In which of the following cases the six rights guaranteed
by article 19 can be suspended?



(a) 1, 2 and 3
(c) 1, 2 and 4 a
Which of the above is/are correct?
(b) 1 and 2

syE
(d) 1, 2, 3 and 4
7. Consider the following statements :




1. External Aggression.
2. Internal Emergency.
3. When Martial Law is in force.
(a) 1 only (b) 2 & 3 only
1. Free and compulsory education to the children
of 6-14 years age group by the State was made a
Fundamental Right by the 76th Amendment to the ngi (c) 1 & 3 only (d) 1, 2 & 3
13. Which of the following rights is/are available to foreigners
in India.


Constitution of India.
2. Sarva Shiksha Abhiyan seeks to provide computer
education even in rural areas.
3. Education was included in the Concurrent List by


nee
1. Right to Education.
2. Right to Information.
(a) 1 only (b) 2 only


the 42nd Amendment, 1976 to the Constitution of
India.
Which of the statements given above are correct?
(c) Both

r ing
(d) None
14. Which of the following statement/s is/are correct:
1. Right to Information is a fundamental right enshrined
in article 19 (1) of the constitution.


(a) 1, 2 and 3
(c) 2 and 3
(b) 1 and 2
(d) 1 and 3
8. The correct statements about the Directive Principles of


RTI act.
(a) 1 only (b) 2 only.ne
2. Supreme Court of India is not under the purview of




State Policy are:
1. They are borrowed from the Irish Constitution.
2. They are incorporated in Part V of the Constitution.
3. They seek to provide social and economic base to
democracy.
(c) Both (d) None
t
15. Article 32 confers the right to remedies for the
enforcement of the FR of an aggrieved citizen. Consider
the following statements w.r.t article 32.
4. The state must compulsorily implement them. 1. Parliament can suspend this right during national
5. All of them are Gandhian in nature. emergency.
Which of the following statements is/are correct? 2. Only SC shall have the power to issue writs for the
(a) 1, 2, 3 and 5 (b) 1, 3 and 5 enforcement of any of the FR.
(c) 1,3,4 and 5 (d) 1 and 3 Correct statement(s) is/are:
9. The correct statements about Fundamental Rights are: (a) 1 only (b) 2 only
1. They are enforceable in the court of law. (c) Both. (d) None
2. These rights are absolute. 16. Which of the following right(s) is/are enshrined in Article
3. They can be suspended during national emergency, 21 – “No person shall be deprived of his life or personal
except some. liberty except according to procedure established by
4. They are available only to Indian citizens. law.”
5. They are contained in Part IV of the Constitution. 1. Right to speedy trial.
Which of the following statements is/are correct? 2. Right against delayed execution.
(a) 1, 3, 4 and 5 (b) 1, 2, 3 and 5 (a) 1 only (b) 2 only
(c) 1 and 3 (d) 1, 3 and 5 (c) Both (d) None

Downloaded From : www.EasyEngineering.net


Downloaded From : www.EasyEngineering.net
P-36 Fundamental Rights, Fundamental Duties and Directive Principles of State Policy

17. The directive principles were made non – justifiable and Select the correct answer using the codes given below
legally non – enforceable because: (a) 1 and 2 (b) 2 and 4
1. The country did not possess sufficient financial (c) 2 and 3 (d) 3 and 4
resources to implement them. 24. Which of the following statements is/are correct?
2. There was widespread backwardness in the country 1. In India, the constitutional remedy under Article 32
that could stand in the way of implementation. is available only in case of Fundamental Rights, not
(a) 1 only (b) 2 only in the case of rights which follow from some other
(c) Both (d) None provision in the Constitution.
18. Which of the following statement/s is/are correct. 2. Both the Supreme Court and High Courts can issue
1. The directive principles are meant to establish the writs of habeas corpus, mandamus, prohibition,
Political Democracy. certiorari and quo warranto only for the purpose of
2. The directive principles are meant to establish Social enforcement of Fundamental Rights.
Democracy. Select the correct answer using the codes given below
3. The directive principles are meant to establish  (a) Only 1 (b) Only 2
Economic Democracy. (c) Both 1 and 2 (d) Neither 1 nor 2
(a) 1 only (b) 2 & 3 only 25. Which of the following statements regarding writ of
(c) 1 & 3 only (d) 1, 2 & 3 certiorari is/are correct?


ww
19. Which of the following statement/s is/are correct.
1. Fundamental Rights enjoy legal supremacy over
Directive principles.
2. The Parliament can amend the Fundamental Rights
1. There should be court, tribunal or an officer having
legal authority to determine the questions of deciding
Fundamental Rights with a duty to act judicially.


(c) Both
w.E
for implementing the directive principles.
(a) 1 only (b) 2 only
(d) None



2. Write of certiorari is available during the tendency
of proceedings before a subordinate court.
Select the correct answer using the codes given below
(a) Only I (b) Only 2


a
20. Which of the following statements is/are correct?

syE
Article 26 of the Constitution of India states that subject
to public order, morality and health, every religious
denomination or any section there of shall have the right.
1. to establish and maintain institutions for religious
(c) Both 1 and 2 (d) Neither 1 nor 2
26. Which of the following are envisaged as being part of
the ‘Right against Exploitation’ in the Constitution of
India?


and charitable purposes.
2. to manage its own affairs in matters of religion. ngi


1. Prohibition of traffic in human beings and forced
labour.
2. Abolition of untouchability.



3. to own and acquire movable and immovable
property.
Select the correct answer using the codes given below
(a) Only 1 (b) 1 and 3

nee
3. Protection of the interests of the minorities.
4. Prohibition of employment of children in factories
and mines.

21.

(c) 1 and 3 (d) All of the above
Consider the following statements.
1. Article 46 of the Constitution of India provides for



r
(a) 1 and 2
(c) 1 and 4 ing
Select the correct answer using the codes given below
(b) 1 and 3
(d) 2, 3 and 4


free legal aid to Scheduled Castes and Scheduled
Tribes.
2. Article 14 of the Constitution of India provides for
27.


the issue of writ of quo warranto?
.ne
Which among the following conditions are necessary for

1. The office must be public and must be created by a




22.
equality before law.
Which of the statements given above is/are correct?
(a) Only 1
(c) Both 1 and 2
(b) Only 2
(d) Neither 1 nor 2
In which of the following years the Fundamental Duties

Statute or by the constitution itself.
t
2. The office must be a substantive one and not merely
the function or employment of a servant at the will
and during the pleasure of another.
have been added to the existent Fundamental Rights in the 3. There has been a contravention of the Constitution
Constitution of India? or a Statute or Statutory Instrument, in appointing
(a) 1965 (b) 1976 such person to that office.
(c) 1979 (d) 1982 Select the correct answer using the codes given below
23. Which of the following statements regarding the Fundamental (a) 1 and 2 (b) 1 and 3
Duties contained in the Constitution of India are correct? (c) 2 and 3 (d) All of these
1. Fundamental Duties can be enforced through writ 28. The Rights to Information means and includes
jurisdiction. 1. Inspection of documents.
2. Fundamental Duties have formed a part of the 2. Taking out files from office to any place desired by
Constitution since its adoption. the applicant.
3. Fundamental Duties became a part of the Constitution 3. Taking photograph of files.
in accordance with the recommendations of the 4. Obtaining information in tapes.
Swaran Singh Committee. Select the correct answer using the codes given below
4. Fundamental Duties are applicable only to the (a) 1 and 3 (b) 1, 2 and 3
citizens of India. (c) 2 and 4 (d) All of these

Downloaded From : www.EasyEngineering.net


Downloaded From : www.EasyEngineering.net
Fundamental Rights, Fundamental Duties and Directive Principles of State Policy P-37

29. Which among the following provisions of the Constitution 1. The Principles spell out the socio-economic
of India is/are fulfilled by the National Social Assistance democracy in the country.
Programme launched by the Government of India? 2. The provisions contained in these Principles are not
1. Fundamental Rights enforceable by any court.
2. Fundamental Duties Which of the statements given above is / are correct?
3. Directive Principles of State Policy (a) 1 only (b) 2 only
Select the correct answer using the codes given below: (c) Both 1 and 2 (d) Neither 1 nor 2
(a) 1 and 2 (b) 1 and 3
(c) Only 3 (d) All of these Matching Based MCQ
30. Consider the following statements
1. Forming a cooperative society is a Fundamental
DIRECTIONS (Qs. 36 to 40) : Match List-I with List-II and
Right in India.
select the correct answer using the codes given below the
2. Cooperative societies do not fall within the ambit of
lists.
the Right to Information Act, 2005.
Which of the statements given above is/are correct? 36. List-I List-II
(a) Only 1 (b) Only 2 (Writs) (Provisions)
(c) Both 1 and 2 (d) Neither 1 nor 2 (A) Habeas (1) directs public
31. Which of the following is/are included in the Directive Corpus servant to perform some


ww
Principles of the State Policy?
1. Prohibition of traffic in human beings and forced
labour


(B) Mandamus
public duty refused to have
been performed by him
(2) directs an individual


w.E
2. Prohibition of consumption except for medicinal
purposes of intoxicating drinks and of other drugs
which are injurious to health
Select the correct answer using the code given below:



(private or executive) to
produce a detainee before
the court
Code:


(a) 1 only
(c) Both 1 and 2
a (b) 2 only
syE
(d) Neither 1 nor 2
32. Consider the following provisions under the Directive



(C) Prohibition


(3) issued by high court
forbidding an inferior
court from continuing
proceedings in a particular


India :
1. Securing for citizens of India a uniform civil code ngi
Principles of State Policy as enshrined in the Constitution of



(D) Certiorari
case
(4) enquires into the legality
of the claim which a party



2.
3.
4.
Organizing village Panchayats
Promoting cottage industries in rural areas
Securing for all the workers reasonable leisure and


nee
asserts to a public office
(5) Issued to a lower court
quashing its decision in a


cultural opportunities
Which of the above are the Gandhian Principles that are
reflected in the Directive Principles of State Policy?
(a) 1, 2 and 4 only (b) 2 and 3 only


r particular case

ing
(a) A – 5; B – 1; C – 3; D – 2; E – 4
(b) A – 2; B – 1; C – 3; D – 5; E – 4
(c) A – 4; B – 2; C – 3; D – 1; E – 5
(c) 1, 3 and 4 only (d) 1, 2, 3 and 4
33. Which of the following provisions of the Constitution of
India have a bearing on Education?

37.
(d) A – 3; B – 4; C – 5; D – 2; E – 1
List-I List-II .ne




1. Directive Principles of State Policy
2. Rural and Urban Local Bodies
3. Fifth Schedule
4. Sixth Schedule




(Writs) (Literal meanings)
(A) Mandamus
authority’
(B) Habeas Corpus (2) ‘We command’
(C) Quo warranto
t
(1) ‘By what warrant or

(3) ‘To be certified’


5. Seventh Schedule
Select the correct answer using the codes given below : (D) Certiorari (4) ‘You may have the body’ or
(a) 1 and 2 only (b) 3, 4 and 5 only ‘To have the body of
(c) 1, 2 and 5 only (d) 1, 2, 3, 4 and 5 (a) A – 2; B – 3; C – 4; D – 1
34. Which of the following is/are among the Fundamental (b) A – 2; B – 4; C – 3; D – 1
Duties of citizens laid down in the Indian Constitution? (c) A – 1; B – 4; C – 2; D – 3
1. To preserve the rich heritage of our composite culture (d) A – 2; B – 4; C – 1; D – 3
2. To protect the weaker sections from social injustice 38. Which one of the following pairs is not correctly matched?
3. To develop the scientific temper and spirit of inquiry (a) Power of : Creating a new
4. To strive towards excellence in all spheres of Parliament State
individual and collective activity (b) Power of State : Altering the name
Select the correct answer using the codes given below : Legislature of a State
(a) 1 and 2 only (b) 2 only (c) Equality before : Both Indian and
(c) 1, 3 and 4 only (d) 1, 2, 3 and 4 the law non-Indian citizens
35. Consider the following statements regarding the Directive (d) Equality of : Indian citizen
Principles of State Policy: opportunity

Downloaded From : www.EasyEngineering.net


Downloaded From : www.EasyEngineering.net
P-38 Fundamental Rights, Fundamental Duties and Directive Principles of State Policy

39. Match List-I (Article of Indian Constitution) with List -II Codes :
(Provisions) and select the correct answer using the codes (a) A-2, B-4, C-3, D-1 (b) A-3, B-1, C-2, D-4
given below the lists: (c) A-2, B-1, C-3, D-4 (d) A-3, B-4, C-2, D-1
List-I List-II 40. Match List I (Articles of the Constitution of India) with
(Article of Indian (Provisions) List II (Provision) and select the correct answer using the
Constitution) codes given below the lists:
A. Article 16 (2) 1. No person shall be deprived List-I List-II
of his property save by the A. Article 14 1. The State shall not discriminate
authority of law against any citizen on grounds
only of religion, race, caste, sex
B. Article 29 (2) 2. No person can be discriminated
place of birth or any of term.
against in the matter of public B. Article 15 2. The State shall not deny to any
appointment on the ground of person equality before the law
race, religious or caste or the equal protection of laws
C. Article 30 (I) 3. All minorities whether based within the territory of India.
on religion or language shall C. Article 16 3. ‘Untouchability’ is abolished
have to establish and administer and its practice in any form is
educational institutions of their forbidden.
choice D. Article 17 4. There shall be equality of

ww
D. Article 31 (I) 4. No citizen shall be denied
admission into any educational
institution maintained by the
opportunity for all citizens in
matters relating to employment
or appointment to any office
under the State.

w.E State, or receiving State aid, on


grounds of religion, race, caste,
language or any of them



Codes:
(a) A-2, B-4, C-1, D-3 (b) A-3, B-1, C-4, D-2
(c) A-2, B-1, C-4, D-3 (d) A-3, B-4, C-1, D-2

a syE
1. (b)
EXERCISE-1
2. (a) 3. (a) 4. (b) 5. (c) ngi 47. (a)
48. (a) A press in democracy must be free and impartial.
6. (b) 7. (a) 8. (b) 9. (c) 10.
11. (a) 12. (c) 13. (b) 14. (d)
15. (c) The 44th Amendment of the Indian constitution
(b) 49. (d)

nee
50. (d) The Directive Principles of State Policy are
guidelines for creating a social order characterized
removed the right to property from the list of
fundamental right and made it an ordinary right.
16. (b) In the case of altering the name of a state, requires
approval of Union Cabinet and Parliament under
r
by social, economic, and political justice, liberty,

ing
equality, and fraternity as enunciated in the
constitution’s preamble. It does not promise equal
income and free healthcare for all Indians.
article 3 and 4 of the constitution.
17. (c) 18. (b) 19. (b) 20. (d) 21. (b) .ne
51. (d) Convention on the nationality of married women has
been signed by India but not yet ratified.
52. (b) The Constitution (86th Amendment) Act, 2002
22. (c) 23. (b) 24. (b) 25. (b) 26. (c)
27. (a) 28. (d) 29. (c) 30. (a) 31. (b)
32. (c) 33. (b) 34. (c) 35. (a) 36. (c)
37. (c) 38. (a) 39. (b) 40. (a) 41. (d)
t
inserted Article 21-A in the Constitution of India
to provide free and compulsory education of all
children in the age group of six to fourteen years
as a Fundamental Right. This is recognized in the
42. (b)
International Covenant on Economic, Social and
43. (b) The Preamble to the Constitution of India in its Cultural Rights as a human right that includes the
introductory statement says- “Justice- Social, right to free, compulsory primary education for all.
Economic and Political” and the Directive Principles
53. (c) According to article 21 of the Indian Constitution,
of state policies aim to create social and economic no person shall be deprived of his life or personal
condition under which the citizens can lead a good liberty except according to procedure established by
life. They also aim to establish social and economic law. The Supreme Court of India on 19 June 2013 in
democracy through a welfare state. its decision established that right to life and liberty,
44. (c) enshrined under Article 21 of the Constitution, is
45. (d) The instrument of instructions on the government available to foreign nationals also.
of India act 1935 has been incorporated in the 54. (d) The 86th amendment to the Constitution approved
constitution of India in the year 1950 as an in 2002 providing free and compulsory education
emergency provisions. to all children age 6 to 14 years has been notified.
46. (c) Freedom of the press in India is implied in the right It included Article 21(a) in the Indian constitution
of Freedom of Expression. making education a fundamental right.

Downloaded From : www.EasyEngineering.net


Downloaded From : www.EasyEngineering.net
Fundamental Rights, Fundamental Duties and Directive Principles of State Policy P-39

55. (c) The Indian Constitution does not recognize property 63. (c) One of the fundamental duties is to “defend the
right as a fundamental right. In the year 1978, the country and render national service when called
44th amendment eliminated the right to acquire, hold upon to do so.”
and dispose of property as a fundamental right. 64. (c) The State shall not make any law which takes away
56. (a) The purpose of Directive Principle of State Policy is or abridges the rights conferred by this Part lll and
to lay down positive instructions which would guide any law made in contravention of this clause shall,
State Policy at all levels. The Directive Principles of to the extent of the contravention, be void.
State Policy contained in Part IV, Articles 36-51 of 65. (d) The constitution of India does not specifically
the Indian constitution. The Directive Principles may mention the freedom of press. Freedom of press is
be said to contain the philosophy of the constitution. implied from the Article 19(a)(a) of the Constitution.
The Directive principles are broad directives given 66. (a) Members of Armed Forces are not treated at par
to the state in accordance with which the legislative so far as the availability of Fundamental Rights is
and executive powers of the state are to be exercised. concerned.
57. (b) Article 31B of the Constitution of India ensured 67. (d) Under Article 226,a High Court can issue these
that any law in the 9th Schedule could not be writs not only for the purpose of enforcement of the
challenged in courts and Government can rationalize fundamental rights but also for the redress of any
other injury or illegality, owing to contravention of
its programme of social engineering by reforming
the ordinary law.

ww land and agrarian laws. In other words laws under


Ninth Schedule are beyond the purview of judicial
review even though they violate fundamental rights
68. (a) Article 24 of the constitution states that, no child
below the age of fourteen years shall be employed to
work in any factory or mine or engaged in any other

w.E
enshrined under part III of the Constitution. On the
one hand considerable power was given to legislature
under Article 31B and on the other hand the power
of judiciary was curtailed, this is the starting point
hazardous employment.
69. (a) To vote in public elections is not a fundamental
duty.

a
of tussle between legislature and judiciary.

syE
58. (c) The Forty Second Constitution Amendment Act,
1976 has incorporated ten Fundamental Duties in
Article 51(a) of the constitution of India. The 86th
70. (c) Directive Principles of State Policy are guidelines to
the central and State government of India to be kept
in mind while framing laws and policies. DPSPs
aim to create social and economic conditions under
which the citizens can lead a good life. They also
Constitution Amendment Act 2002 has added one
more Fundamental Duty in Article 51(a) of the
constitution of India. As a result, there are now 11 ngi aim to establish social and economic democracy
through a welfare state. They act as a check on
the government. It is a yardstick in the hands of
Fundamental Duties of the citizen of India.
59. (b) Habeas corpus writs literally means you should have
the body. It is writ that a person may seek from a
nee
the people to measure the performance of the
government. It shall be the duty of the state to apply
these principles in making laws.
court to obtain immediate release from an unlawful
confinement.
60. (d) To practise family planning and to control
r
71. (d) The purpose of Directive Principles of State

ing
Policy is to establish the social and economic
democracy. Political democracy is established by
the Fundamental Rights.
population, is not a fundamental duty of the citizen
of India. The Fundamental Duties of citizens were
added to the Constitution by the 42nd Amendment .ne
72. (b) Promotion of international peace and security
is included in the Directive Principles of State as
in 1976. Originally ten in number, the Fundamental
Duties were increased to eleven by the 86th
Amendment in 2002, which added a duty on every
parent or guardian to ensure that their child or ward
t
Article 51 of constitution that mentions to promote
international peace and security and maintain just
an honourable relations between nations between
nations; to foster respect for international law and
treaty obligations, and to encourage settlements of
was provided opportunities for education between
international disputes by arbitration.
the ages of six and fourteen years.
73. (b) Directive Principles of State Policy (DPSPs) aim to
61. (a) The writ of Prohibition is an order from a superior
create social and economic conditions under which
court to a lower court or tribunal directing the judge the citizens can lead a good life. They also aim to
and the parties to cease the litigation because the establish social and economic democracy through
lower court does not have proper jurisdiction to hear a welfare state. The Directive Principles of State
or determine the matters before it. Policy is guidelines/principles given to the central
62. (d) RTI act was passed by Parliament on 15 June 2005 and state governments of India, to be kept in mind
and came fully into force on 12 October 2005. while framing laws and policies.
It has been given the status of a fundamental right
EXERCISE-2
under Article 19(a) of the Constitution. Article 19
(a) under which every citizen has freedom of speech 1. (c) 2. (d) 3. (d) 4. (a) 5. (d)
and expression and have the right to know how the 6. (d) 7. (c) 8. (d) 9. (c) 10. (d)
government works, what role does it play, what are 11. (c) 12. (c) 13. (a) 14. (a) 15. (d)
its functions and so on. 16. (c) 17. (c) 18. (b) 19. (c)

Downloaded From : www.EasyEngineering.net


Downloaded From : www.EasyEngineering.net
P-40 Fundamental Rights, Fundamental Duties and Directive Principles of State Policy

20. (d) Article 26 of the Indian Constitution states freedom necessary for the issue of a writ of quo warranto are
to manage religious affairs subject to public order, as follows :
morality and health, every religious denomination (i) The office must be public and it must be created
or any section thereof shall have the right- by a statute or by the constitution itself.
(a) to establish and maintain institutions for (ii) The office must be a substantive one and not
religious and charitable purposes; merely the function or employment of a servant
(b) to manage its own affairs in matters of religion; at the will and during the pleasure of another.
(c) to own and acquire movable and immovable (iii) There has been a contravention of the
property; and constitution on or a statute or statutory
(d) to administer such property in accordance with instrument, in appointing such person to that
law office.
21. (b) Article 46 deals with Promotion of educational and 28. (c) The right to information includes
economic interests of the weaker sections of the • Any document, manuscript and file
people, and in particular, of the Scheduled Castes • Any microfilm, microfiche and facsimile copy of
and the Scheduled Tribes and shall protect them a document;
from social injustice and all forms of exploitation. It • Any reproduction of image or images embodied in
does not provide free legal aid to them. According such microfilm (whether enlarged or not).
to Article 14, “the State shall not deny to any person • Any other material produced by a computer or
equality before the law or equal protection of the any other device

wwlaws within the territory of India”.


22. (b) The Fundamental Duties were added to the
Constitution by the 42nd Amendment in 1976 on the
recommendations of the Swaran Singh Committee.
It does not include taking photograph of files.
29. (c) The National Social Assistance Programme(NSAP)
represents a significant step towards the fulfilment
of the Directive Principles in Article 41 of the

w.E
They were Originally ten in number, but by the 86th
Amendment in 2002 they were increased to eleven.
23. (d) The fundamental duties are defined as the moral
obligations of all citizens to help promote a spirit
Constitution. It came into effect from 15th August
1995.
30. (c) With the enactment of the 97th amendment to the
Constitution of India and its inclusion in Part IX of

a
of patriotism and to uphold the unity of India. The

syE
Fundamental Duties of citizens were added to the
Constitution by the 42nd Amendment in 1976,
upon the recommendations of the Swaran Singh
Committee. The fundamental duties are contained
the Constitution, formation of cooperative societies
has become one of the fundamental rights of an
Indian citizen. Cooperative societies have thus come
under the ambit of The Right to Information Act.
in Art. 51APart IV(a). The fundamental duties
however are non-justifiable in character. This
means that no citizen can be punished by a court for ngi Cooperative societies normally include cooperative
banks, credit societies, sugar factories, distilleries,
handloom-power loom factories, distilleries, milk
violation of a fundamental duty.
24. (c) According to Article 32, when an individual feels that
he has been “unduly deprived” of his fundamental nee
producing societies, water supply societies etc.
31. (b) Statement 1 is incorrect as it is a Fundamental
Right under article 23 of Part III of the constitution.
Statement 2 corresponds to Directive Principles of
rights, he can move the Supreme Court and seek
justice. Apart from the Supreme Court, the High
Courts also have the power to protect fundamental
rights. Like the apex court, they also can issue writs
r
State Policy under Article 47 under Part IV of the
Constitution.
ing
32. (b) Organizing village Panchayats and promoting cottage
industries in rural areas are the Gandhian principles
for the enforcement of fundamental rights of the
citizens. The both courts can issue five different
writs - Certiorari, Habeas Corpus, Mandamus,
Policy.
33. (c) 1, 2 and 5 are correct. .ne
that are reflected in the Directive Principles of State

Prohibition, and Quo Warranto.


25. (b) If a lower court or tribunal gives its decision but
based on wrong jurisdiction the affected party
can move this writ to a higher court like supreme
court or High Court. The writ of certiorari issued
t
34. (c) All the statements except 2 regarding the
Fundamental Duties of citizens are correct.
35. (c) The directive principles ensure that the State shall
strive to promote the welfare of the people by
promoting a social order in which social, economic
to subordinate judicial or quasi judicial body when
they act. and political justice is informed in all institutions
• Without or in excess of jurisdiction of life. The provisions of The Directive Principles
• In violation of the prescribed procedure of State Policy are not enforceable by any court,
• In contravention of principles of natural justice but the principles laid down therein are considered
• Resulting in an error of law apparent on the face fundamental in the governance of the country,
of record. making it the duty of the State to apply these
26. (c) Articles 23 and 24 of the Indian Constitution principles in making laws to establish a just society
safeguard women and children and others against in the count.
exploitation of various forms. 36. (b) 37. (d)
Article 23 declares slave trade, prostitution and 38. (b) In the case of altering the name of a state, requires
human trafficking a punishable offence. approval of Union Cabinet and Parliament under
Article 24 of the Indian Constitution prohibits article 3 and 4 of the constitution.
employment of children below the age of 14 years 39. (a) These are Fundamental Rights under Part III of the
in dangerous jobs like factories and mines. Constitution, runs from Art 14 – 32.
27. (d) All the conditions given in the question are necessary 40. (c) These are Fundamental rights under Part III of the
for issuing writ of quo warranto. The conditions Constitution.

Downloaded From : www.EasyEngineering.net


Downloaded From : www.EasyEngineering.net

UNION GOVERNMENT
3
Chapter

ww
Introduction

w.E
The Government of India or the central or the union Government is divided into three main sections, namely the executive,
Legislature and the judiciary.
The Union Government

a
•• PART V (ARTICLES 52 TO 151) deals with the Executive, Parliament (Legislature), Union Judiciary and the Comptroller
and Auditor General of India.
syE
•• Union Executive: Articles 52 to 78 (Part V).
•• Union Executive in India consists of the President, Vice-President, Prime minister and his/her Council of ministers and
the Attorney General of India.
ngi
Union Government

nee
Executive

r Legislature

ing
President Vice
President
Prime
Minister
Council
of Ministers
Attorney
General of
India
Houses Legislative
Procedures
.ne
Parliamentary
Committees

Rajya
Sabha
Lok
Sabha Adhoc t Standing

Bills Budget Sessions Fund and


and Motions Grants

UNION EXECUTIVE
The President Election of the President (Articles 54 & 55)
•• Article 52: There shall be a President of India. He shall •• Article 54 provides that President shall be elected by an
be the head of the state. electoral college consisting of:-
•• Article 53: Executive powers of the Union shall be vested (a) Elected members of both houses of parliament.
in the President, exercised by him either directly or
through the officers subordinate to him, who give aid and (b) Elected members of the legislative assemblies of the
advice to the President for the exercise of powers. states.

Downloaded From : www.EasyEngineering.net


Downloaded From : www.EasyEngineering.net
P-42 Union Government

•• Word “State” includes “National Capital Territory of (d) Not hold any office of profit under GOI, or any state
Delhi” and UT of Pondicherry (Punducherry). This was government or under any local or other authority
added by 70th Amendment Act, 1992. subject to the control of the government.
•• Under Article 59, the President cannot be a member of
Members of legislative councils (in case of the bicameral
either house of parliament or any state legislature. If such
legislature in state) do not participate in presidential election.
a member is elected President, he shall be deemed to have
Nominated members of both the Houses at the Centre and vacated his seat in that house on the date which he enters
the States do not have voting rights in the election of the the office of President.
President. •• His emoluments, allowances and privileges are determined
•• Article 55 (3) states that the election of the President shall by the parliament by law. Salary and allowances cannot be
be held in accordance with the system of proportional diminished during his term of office. Monthly emoluments
representation by means of single transferable vote. are Rs. 1.5 lakh and the pension is Rs. 75,000 per month.
Voting is done through secret ballot. •• Oath or affirmation of President’s office is administered
•• Article 55(1) provides that as far as practicable there shall by the Chief Justice of India (Article 60) or by the senior
be uniformity in the scale of representation among the most judge of the Supreme Court.
states as well as parity between the states as a whole and •• Term of office of President is 5 years from the date on
the Union in the election of the President. which he/she enters upon his/her office. The president is

ww
•• To secure uniformity among states and parity between the
Union and states following formula is adopted:
Value of the vote of an MLA =________________________
Total population of the state
••
eligible for re-election.
Termination from office is possible before the term of 5
years ends either of the two ways:



w.E Elected members of the state
legislative assembly × 1000
Total value of votes of all
(a) By resignation in writing under addressed to Vice-
President of India who shall communicates it to the
speaker, Lok Sabha.

a
(b) By removal by Impeachment (Article 61). The only
Value of vote of an MP= MLAs
of all states
_____________________

•• syE
Total Nos. of elected MPs
Population data used for these calculations are of 1971
census. 42nd amendment, 1976 froze the “last preceding
ground for impeachment specified in Article 61 (1) is
‘Violation of the Constitution’.

Impeachment against the President

••
census” to 1971, till the first census after 2000.
In 2000, the Union cabinet decided to extend the freeze ngi ••

••
Impeachment is a quasi-judicial procedure mentioned in
Article 61.

nee
on fresh delimitation of parliamentary and assembly Impeachment charge against the President may be initiated
constituencies up to 2026. The argument was that the by either houses of the parliament.
states which had better population control thought that
Impeachment Process
••
such a revision would reduce their seats in the parliament.
After calculating the value of vote of MLAs and MPs,
a complex system of calculating the quota of individual
candidates is used which is based on the order of
r ing
Charge must be in the form of a proposal/ resolution signed by
not less than 1/4th of the total members of the house and moved
after giving at least 14 days’ advance notice to the President.
preference of candidates.

Disputes on election of the President .ne


This resolution must be passed by a majority of not less than

•• Article 71 provides that all disputes arising out of the


election of President or Vice-President shall be ‘inquired’
into and ‘decided’ by the Supreme Court whose decision
shall be final.
t
2/3rd of the total membership of the initiating house.

Charge is then investigated by the other house. The President


has right to appear and to be represented at the investigation.
•• If the election of President is declared void by the Supreme
Court, the acts performed by President before the date of If the other house, after investigations, passes a resolution
such decision of court remain valid. by 2/3rd majority of the total membership declaring that the
•• Article 71(4) declares that the election of President or charge is proved, the President is removed from the office
Vice-President cannot be challenged on the ground of any from the date on which the resolution is passed.
vacancy in the electoral college which elects him. In this context, two things should be noted:
(a) the nominated members of either House of Parliament can
Qualifications for the Office of President participate in the impeachment of the President though
•• Under Article 58, he must be: they do not participate in his election;
(a) Citizen of India (b) the elected members of the legislative assemblies of states
(b) Completed 35 years and the Union Territories of Delhi and Pondicherry do
(c) Qualified for election as a member of the Lok Sabha, not participate in the impeachment of the President though
i.e. he must be registered as a voter in a parliamentary they participate in his election.
constituency. No President has so far been impeached.

Downloaded From : www.EasyEngineering.net


Downloaded From : www.EasyEngineering.net
Union Government P-43

Vacancy in the office of President [Article 65(1)] most judge of the Supreme Court available) acts as the
President or discharges the functions of the President.
•• This may be caused by:
(a) On the expiry of the term (5 years) •• When any person, i.e Vice-President, Chief Justice of
India, or the seniormost judge of the Supreme Court is
(b) By his/her death
acting as the President or discharging the functions of the
(c) By his/her resignation
President, he enjoys all the powers and immunities of the
(d) On his/her removal by impeachment President and is entitled to such emoluments, allowances
•• When he becomes disqualified to hold office or when his and privileges as are determined by the Parliament.
election is declared void.
•• If the vacancy is caused by ending of the term, election Privileges of the President (Article 361)
to fill the vacancy must be completed before the expiry. (a) He is Not answerable to any court for the exercise
Outgoing President continues to hold office even if his/ of powers and duties of his office. However during
her term has expired until his/her successor enters his/ investigations to the charges of impeachment, conduct of
her office. the President may be reviewed by any court, tribunal or
•• If there is some other reason of vacancy other than expiry body appointed by either house of Parliament.
of term, election to fill the vacancy must be held within
(b) During his term of office, no criminal proceedings, no
the 6 months from the date of occurrence of vacancy. The

•• ww
Vice-President shall act as President [Article 65(1)].
If the President is temporarily unable to discharge his/her
duties due to an absence from India, illness or any other
process for arrest or imprisonment can be undertaken.
(c) No civil proceeding until:-
(a) A notice in writing has been given to the President 2

••
w.E
such cause, Vice-President shall discharge his functions
until the President resumes his duties [Article 65(2)].
In case the office of Vice-President is vacant, the Chief

months in advance.
(b) The notice states the nature of proceeding, cause of
action, name, residence and description of the party

a
Justice of India (or if his office is also vacant, the senior

syE President
taking the proceedings and the relief claimed.

Position Term Oath Emolument


ngi
Qualification Election Vacancy Powers
Executive
• Executive • 5 years

the state Re-election


• By chief
Head of • Eligible for justice
of India
• ` 1.5 lakh
per month
• Pension
• Citizen of
India
• Completed 35 nee
• Elected indirectly
by electoral
college consisting
• On expiry of
term
• By death
Legislative
Judicial
Emergency

r
• First or senior ` 75,000 years of age • By resignation
elected members
citizen
of the
country
most
judge
of the
per month • Qualified for
election as a
member of
of both houses
of parliament &
state’s legislative ing
• By impeachment

Supreme
Court
Lok Sabha
• Not hold any
office of profit
assemblies
• Following formula
is adopted first .ne
Value of vote of an MLA
under GOI stage:

Total population of state


×
1
Total no.of elected members in the state legislative assembly 1000
t
Total value of votes of all MLAs of all states
Value of vote of an MP =
Total no.elected MPs
•• At second stage, a complex system of calculating Quota of individual candidate is used which is based on the
order of preference of candidates.
Powers and Duties of the President •• After the 42nd Constitutional Amendment it became
obligatory for the President to seek advice of Council of
•• “Executive power of the Union shall be vested in the
Ministers. 44th Amendment gave him the power to send
President” (Article 53).
back the advice for reconsideration. But if the Council of
•• There are Constitutional limitations on the exercise of Ministers sent back the same advice, President had to act
powers by the President. according to such advice.
•• All executive powers are exercised by the •• “Executive Power” refers to the power exercised by the
President with the advice of the Council of Ministers Council of Ministers in the name of the President. The
[Article 74(1)]. Council of Ministers is the “real executive”.

Downloaded From : www.EasyEngineering.net


Downloaded From : www.EasyEngineering.net
P-44 Union Government

Legislative Powers (iii) Bill involving expenditure from consolidated fund of


India even though it may not be a money bill.
•• President is a part of the Parliament. He exercises
legislative powers with ministerial advice [Article 74(1)]. (iv) State bills restricting Freedom of Trade (Article
304).
•• When a bill is presented to the President for his consent,
he can take following 3 steps: •• Presidential veto’s can be of following types:
(a) He may declare his assent to the bill. The Veto power enjoyed by the executive in modern
states can be classified into the following four types
(b) He may declare that he withholds his ascent to the
bill. 1. Absolute Veto
(c) He may, in case of bills other than money bills, It is the power to say no to a Bill passed by both
return the bill for reconsideration of the houses. Houses of Parliament. Such as Bill never becomes
an act. The power cannot be overridden by the
•• A money bill cannot be returned for reconsideration.
legislature. The Indian President has this power in
•• But if a bill (other than money bill) is passed again by relation in Bills except Money Bills.
both houses of Parliament with or without amendment and
2. Suspensive Veto
again presented to President, he has to give assent to it.
The President exercises this veto, when he returns a
•• In case of state bills reserved by Governor for consideration
Bill for reconsideration of the Parliament. However,
of President, President has power of absolute veto, i.e.

ww
withholding of assent to the bill. Reservation is compulsory
in the case where the law in question would derogate the
powers of the High Court under the Constitution.
if the Bill is passed again by the Parliament, with
or without amendments and again presented to the
President, it is obligatory for the President to give
••

••
w.E
In case of money bill of states so reserved, President may
either declare his assent or withhold his assent.
He has power to summon, prorogue the Parliament and

his assent to the Bill.
3. Pocket Veto
In this case, the President neither assents nor rejects
nor returns the Bill, but simply keeps the Bill pending

a
he can dissolve the Lok Sabha (Article 85). He shall have

syE
the power to summon a joint sitting of both houses of
Parliament in the case of deadlock over an ordinary bill
presided over by Speaker (Article 108).
for an indefinite period. This power of the President
not to take any action (either positive or negative)
on the Bill is known as the Pocket Veto. Since, the
Constitution of India does not specify a time limit
•• Addresses both houses of Parliament assembled together,

ngi
at first session after each general election to the Lok Sabha
and at the commencement of first session of each year.
for the President to give assent to a Bill, the Indian
President can exercise Pocket veto.

•• President has right to address either house or their joint


sitting at any time, and to require the attendance of
members for this purpose. President has right to send

nee
4. Qualified Veto
It is the power of veto which can be overridden by
the legislature by a higher majority. The American

••
messages to either house of parliament either in regard to
any pending bill or to any other matter.
Nomination of members to both the houses of the
r
President may return a Bill within 10 days specifying

ing
his objections to the Bill. If both the houses pass the
Bill again with 2/3rd majority (present and voting)
the veto is overridden. If the requisite majority cannot
Parliament:
ƒƒ For Rajya Sabha, 12 members are nominated. These
persons are having the special knowledge or practical
Qualified veto. .ne
be mustered, the veto stands. In India, there is no

experience of Literature, Science, Art and Social


Service [Article 80(1)].
ƒƒ For Lok Sabha, not more than 2 members can be
nominated by the President from Anglo-Indian
pocket veto.

Financial Powers
••
t
•• President of India enjoys Absolute, suspensive and

He causes to be laid before the Parliament the “Annual


community, if he thinks this community is not Financial Statement” or the “Budget”.
adequately represented in Lok Sabha. [Article 331]. •• Money Bill can be introduced in the Parliament only after
•• He decides on questions of disqualification of MPs in his approval.
consultation with the Election Commission. •• No demand for grant can be made except on his
•• He lays certain reports and statements before Parliament recommendations.
like: Reports of CAG, UPSC, Finance Commission, •• Money is released from the Contingency Fund after his
Special Officers for SCs/STs, Linguistic minorities, sanction.
Commission on backward classes and Annual Financial
•• He constitutes a “Finance Commission” after a gap of 5
Statement (Budget).
years to recommend the distribution of revenues between
•• Prior recommendation of President for introducing Centre and states.
legislation is required on:
(i) Bill for formation of new states or alteration of Executive Powers
boundaries of existing states (Article 3). •• All executive work is performed in the name of the
(ii) Money bill [Article 117(1)]. President.

Downloaded From : www.EasyEngineering.net


Downloaded From : www.EasyEngineering.net
Union Government P-45

•• He appoints the Prime Minister and on his advice •• The President uses his pardoning powers on the advice of
ministers of the Union; judges of the Supreme Court and Union Government.
high courts, Governors of the states, Attorney General,
Comptroller & Auditor General, Chairman and members Military Powers
of Public Service Commission, members of the Finance President is the supreme commander of the armed forces of
Commission, other official commissions, special officers the country. The exercise of this power is regulated by law
for SCs & STs, commission to report on administration (Article 53). He appoints the chiefs of the Army, Navy and
of scheduled areas, Inter state council, Commission to Air force. He can declare war or conclude peace subject to the
investigate the condition of backward classes, special approval of the parliament.
officers for linguistic minorities.
•• These officials hold their office during the pleasure of Diplomatic Powers
the President. They can be removed by following the He represents India in international forums. He sends and
procedure laid down in the Constitution. He/she exercises receives ambassadors and diplomatic representatives. All
these powers with the advice of Council of Ministers. treaties and international agreements are negotiated and
•• He makes rules specifying the manner in which the orders concluded in his name though subject to approval of the
and other instruments made and executed in his name Parliament.
shall be authenticated.
••

••
ww
He directly administers the Union Territories through
administrator appointed by him.
He appoints the Chief Justices and the other judges of
National Emergency
•• The President can proclaim emergency in the entire
country or in any part of it on the grounds of war, external

•• w.E
Supreme Court and high courts.
He can seek the advice of the Supreme Court on questions
of law or fact. The advice tendered by the Supreme Court
••
aggression or armed rebellion.
Term ‘armed rebellion’ was inserted by the 44th
constitutional amendment act (1978), replacing the

Judicial Powers
••
a
is not binding on him.

syE
He appoints the Chief Justices and the other judges of
••

••
original term ‘internal disturbance’.
The President can proclaim this emergency only after
receiving a written recommendation from the cabinet.
The proclamation of emergency must be approved by the

••
Supreme Court and high courts.
He can seek the advice of the Supreme Court on questions
of law or fact. The advice tendered by the Supreme Court ngi ••
parliament (both houses) within one month. If approved,
the emergency shall continue for six months.
It can be extended for an indefinite period with an approval

••
is not binding on him.
Under Article 72, President has the power to grant:
ƒƒ Pardons which completely absolve the offender
••
nee
of the parliament for every six months.
A national emergency has been proclaimed three times so
far in 1962, 1971 and 1975.
from all punishments.
ƒƒ Reprieves or stay on the execution of the sentence
for a temporary period.
••
r ing
During national emergency President can
ƒƒ Give directions to any state with regard to the manner
in which its executive power is to be exercised.
ƒƒ Respites or awarding lesser punishment on special
grounds. year at a time.
.ne
ƒƒ Extend the normal tenure of the Lok Sabha by one

ƒƒ Remission or reduction of sentence without changing


it’s character.
ƒƒ Commutation or substitution of one form of
punishment for another form which is lighter.
t
ƒƒ Modify the pattern of the distribution of financial
resources between the union and the states.
ƒƒ Suspend the fundamental rights of citizens except
the right to life and personal liberty (article 21)
and the right to protection in respect of conviction
•• To suspend, remit or commute the sentence of any person
convicted of any offence – for offences (article 20). Moreover, the right to
ƒƒ By court martial. six freedoms (article 19) can only be suspended in
case of external emergency (i.e. on the grounds
ƒƒ An offence against any law relating to any matter to
of war or external aggression) and not in case of
which executive power of the union extends.
internal emergency (i.e. on the grounds of an armed
ƒƒ In all cases of death sentence.
rebellion).
•• He is the only authority for commuting a death sentence.
•• It should also be mentioned here that the Parliament can
•• Pardoning powers on the President is to correct possible make laws on items mentioned in the State List during
judicial errors. No human system is totally free from the period of national emergency. Such laws become
imperfections. ineffective six months after the emergency.
•• Power of President is an executive power and independent
•• Emergency can be declared even if imminent danger is
of judiciary. He is not a court of appeal and cannot be
there. Actual occurrence isn’t necessary for its proclamation.
compelled to give a hearing to a petitioner. Courts cannot
interfere in the exercise of this power. •• If a notice in writing signed by not less than 1/10th of
total members of Lok Sabha describing their intention

Downloaded From : www.EasyEngineering.net


Downloaded From : www.EasyEngineering.net
P-46 Union Government

to disapprove the continuation of emergency, served •• In S.R. Bommai Vs UOI case, Supreme Court said that
to Speaker of House or to President if house is not in the satisfaction of President can be scrutinized by the
session, special sitting shall be held within 14 days from courts.
date of such notice. •• First time proclaimed in Punjab in 1951 followed by
•• Satisfaction of President can be challenged on grounds of PEPSU in 1953 and A.P. in 1954.
malafide intention.
•• In Minerva Mills Vs Union of India it was held that there is Financial Emergency
no bar to judicial review of the validity of proclamation of •• The President can proclaim financial emergency if he is
emergency issued by President under Article 352(1). But satisfied that the financial stability or credit of India or any
court’s powers are confined to check whether limitations part thereof, is threatened. Such a proclamation must be
conferred by constitution are complied with or not. approved by the parliament within two months.
•• President acquires the following extraordinary powers:-
State Emergency/President’s Rule
ƒƒ He can give directions to the states to observe the
•• Also known as a constitutional emergency, it can be canons of financial propriety.
proclaimed by the President on the following grounds:
ƒƒ He can require that all money bills and other financial
(i) Failure of constitutional machinery in the states
bills passed by the state legislature be reserved for
(article 356) or

••
ww
(ii) Failure to comply with or to give effect to directions
given by the union (article 365)
Imposed when the President is satisfied (on the basis of
his consideration.
ƒƒ He can issue directions for the reduction of salaries
and allowances of all or any class of person serving

w.E
either a report of the state governor or otherwise), that the
governance of a state cannot be carried on in accordance
with the provisions of the constitution.
••
in connection with the affairs of the union and the
states, including the judges of the Supreme Court and
high courts.
••

a
The above proclamation in a State should be approved

syE
by the Parliament (both houses) within two months. If
approved, it remains in force for six months. It can be
extended for a maximum period of three years with the
This type of emergency has not been declared so far.

Miscellaneous Powers/Residuary Powers


•• Can make rules and regulations relating to matters not
mentioned in the Constitution.
approval of parliament every six months. However,
beyond one year, it can be extended by six month at a
time only when the following two conditions are fulfilled. ngi •• He has final legislative power for all Union Territories
except Pondicherry and Delhi.
(i) Proclamation of national emergency should be in
operation in the entire country, or in the whole or
any part of the concerned state; and
••

nee
He has special power for Scheduled Areas, Tribes and
Tribal Areas.

Ordinance Making Power of the President


••
(ii) The election commission must certify that the general
elections to the concerned state cannot be held on
account of difficulties.
(Article 123)
••
r ing
Ordinances can be issued when both the houses of


The President acquires the following extraordinary
powers during state emergency:-
(i) Assign to himself all or any of the functions of the .ne
parliament are not in session and the President is satisfied
that circumstances exist which make it necessary to take
‘immediate action’.


state government and powers of the governor.
(ii) Declare that the powers of the state legislature
shall be exercisable by or under the authority of the
parliament.
••

••
••
t
This power is exercised on the advice of Council of
Ministers. Therefore it is not for ‘individual satisfaction’.
It is the most important legislative power of the President.
Ordinances issued by the President have the force of an act
(iii) Authorize (when the Lok Sabha is not in session)
of the Parliament. Ordinance can be retrospective, may
expenditure from the consolidated fund of the state
pending the sanction of such expenditure by the amend or repeal any act of the parliament or a previous
parliament. ordinance itself.
(iv) Promulgate ordinances for the administration of the •• Ordinances must be laid before both houses of the
state when the parliament is not in session. parliament and shall not operate at the expiry of 6 weeks
•• In brief, the President dissolves the state council of from the date of re-assembly of the Parliament, unless
ministers headed by the chief minister and the state disapproved earlier by the Parliament. If the houses are
legislature. The parliament passes the state budget and summoned to re-assemble on different dates, the period of
legislation bills. The state Governor, on behalf of the 6 weeks is counted from the later of those dates.
President, carries on the state administration of the state •• Though the ordinance making power of the President is
with the help of advisors appointed by the President. coextensive with legislative power of the Parliament, it is
•• However, the President can’t assume to himself power not a parallel power of legislation. It is the power to meet
vested in high court/ suspend operation of any provision emergent needs for legislation when the Parliament is not
relating to high court. in session.

Downloaded From : www.EasyEngineering.net


Downloaded From : www.EasyEngineering.net
Union Government P-47

•• An ordinance is void if it makes a provision which •• Ordinance making power is a relic of Act of 1935.
under the Constitution is beyond the competence of the •• Ordinance making powers of President (Article 123) &
Parliament. An ordinance cannot violate fundamental Governors (Article 213) are corollary to each other.
rights. Judicial interference is possible to check any
malafide intention in an ordinance.
Powers of President

Executive Legislative Judicial Emergency Miscellaneous

Appoints PM, Directly Dissolution or Prorogation Seek Advice Power


Council of ministers, administers of Parliament of Supreme to grant
Attorney_General, UTs Address & Send messages Court pardon
CAG, Judges of to houses
Supreme Court & High Nominates Members of
Courts, Governors, Parliament
Administrators

ww
of UTs, Chairman &
Members of Public
service Commission,
Assent on Bills passed by
Parliament
Laying down some reports
National Failure of Financial
Emergency Constitutional Emergency
(Art. 352) Machinery (Art. 360)

Chairman & Members


of Finance Commission,
Chief Election
w.E before Parliament

Issue Ordinance
in States
(Art. 356)

Supreme Conclude
Commissioner to &
Election commissioners,
Inter - State Council,
Commission to
a syE
Commander of
Armed Forces
& Approve
International
Treaties &
Agreements
investigate the condition
of SCs, STs & OBCs.
Position of Indian President ngi
The position of the Indian President is somewhat difficult to
categorise. Like the American president, he is elected for a
V.V. Giri

Justice M
nee 03.05.1969 20.07.1969 First acting President
of India.
20.07.1969 24.08.1969 Was also the Chief
fixed term, and like his American counterpart he is removable
by the legislature through the process of impeachment. But
the Indian Constitution makers preferred not to go completely
the American way because absence of coordination between
Hidayat-ul-lah
V.V. Giri r ing
24.08.1969 24.08.1974
Justice of India.

the legislature and the executive is a source of weakness of he


American political system. Political analysts prefer to use the
F. Ali Ahmed
BD Jatti .ne
24.08.1974 11.02.1977 Died in Office
11.02.1977 25.07.1977 Acting President
more dignified term of “Constitutional head” the president has
thus been made a formal or constitutional head of the executive
and the real executive powers are vested in the Ministers or
the Cabinet.
N Sanjeeva
Reddy t
25.07.1977 25.07.1982 Youngest President
(64 years)
Giani Zail Singh 25.07.1982 25.07.1987 First Sikh President
List of President of India R Venkataraman 25.07.1987 25.07.1992 Oldest President
Name Tenure Important Facts (76 years)
From To Dr SD Sharma 25.07.1992 25.07.1997 —
Dr. Rajendra 26.01.1950 13.05.1962 First President and
KR Narayanan 25.07.1997 25.07.2002 First Dalit
Prasad also had the longest
President
tenure (12 years).
Dr. S. 13.05.1962 13.05.1967 Was also the first Dr APJ Abdul 25.07.2002 25.07.2007 First Scientist to
Radhakrishnan Vice-President of Kalam become President
India.
Mrs Pratibha 25.07.2007 25.07.2012 First Woman to
Dr. Zakir 13.05.1967 03.05.1969 Shortest tenure; First Patil become President
Hussain Muslim President;
First President to Pranab 25.07.2012 Till Date —
die in harness Mukherjee

Downloaded From : www.EasyEngineering.net


Downloaded From : www.EasyEngineering.net
P-48 Union Government

Vice President The Prime Minister and Council of


•• Article 63 of the Constitution provides that there shall be Ministers
a Vice-President of India.
Real Executive Authority
•• Article 66 says that the Vice-President is elected by the
member of both houses of Parliament in a joint session by As the President of India is a constitutional executive head, the
secret ballot with the system of proportional representation real executive authority of the Union is exercised by the Prime
by means of single transferable vote. States have no role Minister and his Council of Ministers. The Indian PM has often
to play in his election. been designated as primes inter pares (first among equals)
•• Qualifications of the Vice-President are same as those of & interstellar Lunar linares (moon among the stars).
President except that he must be eligible for election to
Rajya Sabha. The Prime Minister
•• His term of office is 5 years. He may resign from his The office of the Prime Minister has been created by the
office before the expiry of normal term by writing to the Constitution. The Prime Minister is appointed by the President
President. (Article 75). Generally the President has no choice in the
•• He can be removed from the post of the Vice President appointment of the Prime Minister and invites the leader of the
by a resolution of the Rajya Sabha, passed by a majority majority political party in the Lok Sabha for this office. The

ww
of all the members of the house and agreed to by a simple
majority of the Lok Sabha. Such a resolution can be
moved only after giving 14 day’s notice of the intention
Prime Minister theoretically holds office during the pleasure
of the President. But the Prime Minister actually stays in
office as long as he enjoys the confidence of the Parliament

w.E
to move the resolution. While resolution for impeachment
of the President can be moved in the either of the houses,
the resolution for the removal of Vice-President can only
be moved in the Rajya Sabha.
especially the Lok Sabha. The normal term is five years but it
is automatically reduced if the Lok Sabha is dissolved earlier.
•• The Prime Minister gets the same salary and allowances
••
a
Vice-President gets a salary of Rs. 125,000 per month

syE
and other emoluments as chairman of Rajya Sabha besides
a daily allowance while he presides over Rajya Sabha as
admissible to a members of the Parliament.
which are paid to the members of Parliament. He also
receives a constituency allowance like other MPs. In
addition, he is also entitled to a sumptuary allowance,
free official residence, free travel medical facilities, etc.

Functions of The Vice President


•• No functions are attached to the office of the Vice
ngi
•• Powers of Prime Minister. The Prime Minister enjoys


extensive powers which are as follows-
1. The President convenes and prorogues all sessions of

••
President.
He is the ex-officio chairman of Rajya Sabha (Article 64).
nee
the Parliament in consultation with him.
2. He can recommend the dissolution of Lok Sabha to
the President before expiry of its normal term.
••

••
He presides over the meetings of Rajya Sabha but is not a
member of Rajya Sabha. He has no right to vote.
During his discharge of his functions as a President, in case
that post falls vacant on account of the death resignation
r ing
3. All the members of the Council of Ministers are
appointed by the President on the recommendations
of the Prime Minister.
or removal of the President (Art 65). The Vice-President
shall have all powers and immunities of the President.

.ne
4. He allocates portfolios among the various ministers
and reshuffles them. He can ask a minister to resign
and can even get him dismissed by the President.
•• He is entitled to such emoluments, allowances and
privileges as may be determined by parliament by law,
mentioned in Second Schedule.

Vice-Presidents of India
t
5. He presides over the meetings of the Council of
Ministers and exercises a strong influence on its
decisions.
6. He exercises general supervision over the working of
Vice-Presidents Tenure other ministers and ensures that they work as a team.
Dr. Sarvapalli Radhakrishnan 1952-1962 7. The Prime Minister can bring about the fall of the
Dr. Zakir Hussain 1962-1967 Council of Ministers if he resigns. He is the pivot
around which the Council of Ministers revolves.
Varahagiri Venkatagiri 1967-1969
8. The Prime Minister is the chief channel of
Gopal Swarup Pathak 1969-1974
communication between the President and the
BD Jatti 1974-1979
Council of Ministers and keeps the former informed
Justice Mohammad Hidayat-ul-lah 1979-1984 about all the decisions of the Council.
R Venkataraman 1984-1987 9. He assists the President in the appointment of all
Dr Shanker Dayal Sharma 1987-1992 high officials.
KR Narayanan 1992-1997 10. He can recommend to the President, with the
Krishan Kant (Died) 1997-2002 concurrence of other cabinet ministers, to proclaim
Bhairon Singh Shekhawat 2002-2007 a state of emergency on grounds of war, external
Mohammad Hamid Ansari 2007 till date aggression or armed rebellion.

Downloaded From : www.EasyEngineering.net


Downloaded From : www.EasyEngineering.net
Union Government P-49

11. He advises the President about imposition of 12. He is the chairman of the Planning Commission,
Presidential Rule in the states on grounds of National Development Council & Inter-State
breakdown of constitutional machinery or imposition Council.
of an emergency due to financial instability.
List of Prime Ministers
Name Tenure Note Party (Alliance)
From To
Jawarharlal Nehru 15.08.1947 27.05.1964 First Prime Minister of India, died in office; INC
also had the longest tenure (17 years)
Gulzari 27.05.1964 09.06.1964 First Acting Prime Minister INC
Lal Nanda
Lal Bahadur Shastri 09.06.1964 11.01.1966 Only Prime minister to die abroad during an INC
official tour
Gulzari Lal Nanda 11.01.1966 24.01.1966 First to become Acting Prime Minister twice INC
Indira Gandhi 24.01.1966 24.03.1977 First woman Prime Minister of India; First INC

ww
Morarji Desai
Prime Minister to lose an election
24.03.1977 28.07.1979 Oldest Prime Minister (81 years) and the first Janata
to resign from office
Charan Singh
w.E Party
28.07.1979 14.01.1980 Only Prime Minister who did not face the Janata Party (Secular) with INC
Parliament
Indira Gandhi
Rajiv Gandhi
VP Singh
a
14.01.1980 31.10.1984 First Prime Minister to be assassinated

syE
31.10.1984 01.12.1989 Youngest Prime Minister (40 years)
INC
INC
21.12.1989 10.11.1990 First Prime Minister to step down after vote Janata Dal (National Front)
of no-confidence
Chandra Shekhar
PV Narasimah Rao
10.01.1990 21.06.1991
ngi —
21.06.1991 16.05.1996 First Prime Minister from Southern India
Samajwadi Party with INC
INC
Atal Bihari Vajpayee
HD Deva Gowada
IK Gujral
01.06.1996 20.04.1997
21.04.1997 19.03.1998


nee
16.05.1996 01.06.1996 Shortest tenure of a Prime Minister BJP
Janata Dal (United Front)
Janata Dal
Atal Bihari Vajpayee
Atal Bihari Bajpayee
19.03.1998 13.10.1999
13.10.1999 22.05.2004

— r ing
BJP (NDA)
BJP (NDA)
Dr Manmohan Singh

Narendra Modi
22.05.2004 26.05.2014 First Sikh Prime Minister, longest tenure after INC (UPA)
JL Nehru
26.05.2014 Till date First PM born after Independence BJP (NDA) .ne
Council of Ministers
•• “There shall be a Council of Ministers with Prime
Minister as its head to aid and advice the President, who
••

••
t
The salaries of the ministers and their salaries and
allowances are determined by the Parliament.`
Constitutional duties of the Prime Minister as provided
shall in exercise of his functions act in accordance with in Article 78 is to communicate to the President “all
such advice” (Article 74(1) after 42nd amendment, 1976. decisions” of the Council of Ministers relating to:
•• The Prime Minister is appointed by the President, ƒƒ Administration of the affairs of the Union.
Ministers are appointed by the President on advice of the ƒƒ Proposal for legislation.
Prime Minister [Article 75(1)].
ƒƒ To furnish information relating to the administration
•• The ministers hold office during the pleasure of President
of affairs of the Union and proposals for legislation
[Article 75(2)].
as the President may call for.
•• There is no constitutional bar for a nominated member to
be appointed as a union minister. ƒƒ If the President so requires to submit for the
•• There is no bar on the appointment of a person from consideration of Council of Ministers any matter on
outside the legislature as minister, but he cannot continue which a decision has been taken by a minister but
as minister for more than 6 months unless he secures a seat which has not been considered by the council.
in either house of Parliament by election or nomination. •• Allocation of the portfolios among ministers is done by
[Article 75(5)]. the Prime Minister.

Downloaded From : www.EasyEngineering.net


Downloaded From : www.EasyEngineering.net
P-50 Union Government

Powers is left to the President to make rules as to how his orders


are to be authenticated. Further the courts are barred from
The Council of Ministers formulates and implements the
inquiring into the nature of advice rendered by the ministers.
policy of the country. It introduces most of the important bills
As such, if the President’s act is authenticated by a secretary
and resolutions in the Parliament and steers them through.
to the government of India no minister can be hold legally
It prepares and presents the budget to the Parliament for its
responsible for it even through he may have advised it.
approval; and generally it is passed in the form in which it is
presented. Composition of Council of Ministers
The foreign policy of the Government is determined by the
There are 3 categories of Ministers:
Council of Ministers. It plays a vital role in recognition of new
states and advises the President with regard to appointments of 1. Cabinet ministers
diplomats. 2. Minister of state
3. Deputy ministers
Collective Responsibility •• Cabinet ministers head ministries in the Government.
•• Council of Ministers are collectively responsible to the They are members of the Cabinet, attend its meetings and
Lok Sabha [Article 75(3)]. The ministry resigns if it loses decide policies.
the confidence of the Lok Sabha. •• Ministers of state can have an independent charge or can
•• be attached to cabinet ministers. They are not members of

•• ww
Vote of no confidence passed against any minister leads to
the resignation of the entire Council of Ministers.
They work as a team and swim and sink together.
the cabinet nor do they attend its meetings unless invited.
•• Deputy ministers have no separate charges. They assist
the ministers with whom they are attached and perform

•• w.E
Individual Responsibility
The principle embodied in Article 75(2) is that of
individual responsibility. It says that the minister shall
administrative duties.
•• There are also parliamentary secretaries with no
department under their control. They assist senior

••
a
hold office during the pleasure of the President.

syE
Hence a minister can be dismissed even if the ministry has
the confidence of legislature.
ministers in discharging parliamentary duties. Since
1967, no parliamentary secretaries have been appointed.
•• Cabinet is an extra-constitutional body based upon
conventions. It is the supreme policy making body.
Legal Responsibility
The system of legal responsibility of a minister is not
prescribed in the Indian Constitution. The Indian Constitution ngi •• Kitchen cabinet is an extra-constitutional body consisting
of the Prime Minister and a few influential colleagues in
whom he has faith and with whom he can discuss issues.
does not say the President can act only through Minister, it
nee
It may include outsiders.
Council of Ministers (As Centre and State level): Composition

r Assist Ministers with whom

ing
they are attached & perform
administrative duties

No Separate charge .ne


Collective Responsibility
to Lok Sabha at central
level and Legislative
D
EP
U T Y M I NIST

ERS O F ST
ER
S
t
I ST Head Important Ministries of
Assembly at State level N Central / State Government
A
I

TE
M

CABINET Attend Cabinet’s meetings


MINISTERS & decide policies

Individual Responsibility to (Independent Supreme Executive authority


President at central level &
Charge)
Governor at State level.
Independent charge of
Ministries / Departments.
Work under Supervision &
guidance of Cabinet ministers.
Not members of Cabinet & do
not attend Cabinet meeting.

Downloaded From : www.EasyEngineering.net


Downloaded From : www.EasyEngineering.net
Union Government P-51

Council of Ministers vs Cabinet to the parliamentary form of Prime Minister and other
Council of Ministers Cabinet government as developed in Ministers of Cabinet rank
It is a wider body consisting of It is a smaller body consisting Britain. It has however, got a appointed under Article 75”
60 to 70 ministers of 15 to 20 Ministers. legislative sanction. Thus, the and does not describes its
Salaries and Allowances Act powers and functions. In
It includes all the three It includes the cabinet
of 1952 defines a ‘Minister’ as other words, its role in our
categories of ministers, i.e. Ministers only. Thus, it
a “Member of the Council of politico-administrative system
Cabinet Ministers, Ministers is a part of the Council of
Minister”, by whatever name is based on the conventions
of State and Deputy Ministers. ministers. called and includes a Deputy of the parliamentary form of
It does not meet, as a body, to It meets as a body frequently Minister government as developed in
transact government business. and usually once in a week, to Britain.
It has no collective functions. deliberate and take decisions It is collectively responsible It enforces the collective
regarding the transaction of the to the Lower House of the responsibility of the Council
government business. Thus, it Parliament. of Ministers to the Lower
has collective functions. House of the Parliament.
It is vested with all powers, It exercises, in practice,
but in theory. the power of the Council of Attorney General of India

ww Ministers and thus acts for the


latter.
Its functions are determined It directs the Council of
Attorney General is the highest legal officer of the Union
Government and renders legal assistance to it. He is appointed
by the President and holds office during his pleasure. To be
by the Cabinet.

w.E Ministers by taking policy


decisions which are binding
on all ministers.
It implements the decisions It supervises the implementation
eligible for appointment as Attorney General of India, a person
must possess the qualifications prescribed for a judge of the
Supreme Court. He is entitled to such salary and allowances
taken by the Cabinet.
a of its decisions by the Council
of Ministers
syE
It is a constitutional body, The word ‘cabinet’ was
dealt in detail by the Articles inserted in the Article 352
as may be determined by the President. The Attorney General
is entitled to audience in all courts in the country and can take
part in the proceedings of the Parliament and its committees.
However, he is not given the right to vote.

74 and 75 of the Constitution. of the Constitution in 1978


Its size is determined by the by the 44th Constitutional
Prime Minister according to Amendment Act. Thus, it did
ngi Functions
He is the chief legal adviser of the Government of India and

the exigencies of the time, not find a place in the original


and the requirements to the text of the Constitution. Even,
situation. Its classification now Article 352 defines
nee
gives it advice on all such legal matters which may be referred
or assigned to him by the President. He also performs such
other legal duties as are assigned to him by the President

into a three-tier body, and the cabinet saying that, it is


is based on the conventions “the council consisting the r
from time to time. The Attorney General appears before the

ing
Supreme Court and various High Courts in cases involving the
Government of India.

The Parliament
UNION LEGISLATURE
•• .ne
The mode of election of the Lok Sabha is:

The Parliament is the Union Legislature of India. It consists of


the President and two houses the Lok Sabha (house of people)
and Rajya Sabha (council of states). Article 79 to 123 in Part-V
people of the state. t
ƒƒ State representatives are elected directly by the

ƒƒ Union territory representatives are elected in the


manner prescribed by parliament by law.
deals with the provisions of the Parliament.
ƒƒ Every citizen of India of 18 years and above and
Lok Sabha is not disqualified on the grounds of non-residence,
unsoundness mind, crime or corrupt or illegal
Lower House of the Parliament and also known as the first
practices is entitled to vote (Art 326).
Chamber.
•• Constitution 61st Amendment Act (1987) has reduced the
•• Members of Lok Sabha are directly elected by the people.
age of voting from 21 to 18 years.
•• Total membership is fixed at 552 by the Constitution. •• Term of Lok Sabha is normally 5 years but it can be
Their distribution among the States and Union Territories dissolved earlier by the President.
are:
•• Its term can be extended beyond 5 years by the Parliament.
ƒƒ 530 representatives from the states. This can be done during the proclamation of emergency
ƒƒ 20 members from the Union Territories. (Under Art 352). But this extension can not be done for
ƒƒ 2 Anglo Indian members nominated by the a period exceeding one year at a time and such extension
President if such community has not been adequately cannot continue beyond a period of 6 months after
represented. proclamation of emergency ceases to operate.

Downloaded From : www.EasyEngineering.net


Downloaded From : www.EasyEngineering.net
P-52 Union Government

Territorial Constituencies for Lok Sabha List of Speaker


•• For conduct of elections, each state is divided into Speakers Tenure
territorial constituencies. Art 81(2) provides for the Ganesh Vasudev Mavalankar (Died) 1952-1956
uniformity of representation in 2 respects (after 7th M Ananthasayanam Ayyangar 1956-1962
Amendment Act 1956) :
Hukam Singh 1962-1967
ƒƒ as between different states Neelam Sanjiva Reddy (Resigned) 1967-1969
ƒƒ as between different constituencies in same state Gurdial Singh Dhillon (Resigned) 1969-1975
•• Each state has been allotted a fixed number of seats Bali Ram Bhagat 1976-1977
keeping the ratio between the number of seats and the
Neelam Sanjiva Reddy (Resigned) 1977-1977
population.
KS Hegde 1977-1980
•• Each state has been divided into territorial constituencies
Bal Ram Jakhar 1980-1989
such that the ratio between the population of the state and
Rabi Ray 1989-1991
the number of seats allotted to it remains almost the same.
Shivraj V Patil 1991-1996
•• Art 82 stipulates delimitation of territorial constituencies
to the Lok Sabha. PA Sangma 1996-1998
•• Delimitation of Constituencies is done after each Census GMC Balayogi (Died) 1998-2002

ww
by a designated authority and in a manner as the Parliament
by law determines. 1971 census data is being used now.
The number of seats has been freezed till 2026 to maintain
Manohar Gajanan Joshi
Somnath Chatterjee
Ms Meira Kumar
2002-2004
2004-2009
2009-2014

••
declining. w.E
the share of states where the rate of population growth is

Constitution provides for proportional representation


Ms Sumitra Mahajan

Pro Tem Speaker


2014-till date

assemblies.
a
for Rajya Sabha and not for Lok Sabha and legislative

Offices of Speaker and Deputy Speaker of Lok syE


As provided by the Constitution, the Speaker of the last Lok
Sabha vacates his office immediately before the first meeting
of the newly-elected Lok Sabha. Therefore, the President
appoints a member of the Lok Sabha as the speaker Pro Tem.
Sabha
••
••
The Speaker presides over the Lok Sabha.
Speaker and Deputy Speaker are elected by the members
ngi Usually, the senior most member is selected for this. The
President himself administers oath to the speaker Pro Tem.
The speaker Pro Tem has all the powers of the speaker. He

••
of Lok Sabha.
Deputy Speaker performs the duties of the speaker if nee
presides over the first sitting of the newly-elected Lok Sabha.
His main duty is to administer oath to the new members. He
also enables the house to elect the new speaker.
the office of the speaker falls vacant. If the office of the
Deputy Speaker is also vacant, duties of the Speaker shall
be performed by a member of the house appointed by the
President.
r
When the new speaker is elected by the house, the office of

ing
the speaker Pro Tem ceases to exist. Hence, this office is a
temporary office, existing for a few days.

•• They vacate their office the moment they cease to be a


member of the house. The Speaker continues in his office
Rajya Sabha
.ne
The Rajya Sabha is the second chamber or Upper House of

••
even if Lok Sabha is dissolved. He/she holds the office till
the new Lok Sabha meets.
They can be removed by a resolution of the Lok Sabha
with a majority of all the then members of the house.
t
the Parliament. It consists of representatives of the states.
The maximum strength of the Rajya Sabha is 250. Of these,
238 represent the states and union territories and the rest are
nominated by the President. The nominees are persons who
However a 14 days’ notice is necessary to move such a have distinguished themselves in the field of literature, art,
resolution. science, social service and so on. Representatives of the
states are elected by members of state legislative assemblies
•• To ensure the independence of the Speaker, his/her salary
on the basis of proportional representation through a single
is paid from the Consolidated Fund of India and is not
transferable vote. It is noteworthy that in the Rajya Sabha, the
subject to the annual vote of the Parliament
states have been provided representation on the basis of their
•• He/ she cannot be removed from the office except by a population.
resolution passed by a special majority. •• Rajya Sabha is a permanent house and is not subject to
•• If the Speaker wants to resign, the letter of resignation dissolution.
should be addressed to the Deputy Speaker and vice-versa. •• Its members are elected for a period of 6 years but 1/3rd
•• Upto the 10th Lok Sabha, both the Speaker and Deputy of its members retire after every 2 years.
Speaker were usually from ruling party. Since the 11th As regards qualifications for membership of the Rajya Sabha,
Lok Sabha there has been a consensus that Speaker comes the candidate must –
from the ruling alliance and post of Deputy Speaker Goes •• be a citizen of India.
to the main opposition party. •• be 30 years of age or more.

Downloaded From : www.EasyEngineering.net


Downloaded From : www.EasyEngineering.net
Union Government P-53

•• be a parliamentary elector in the state in which he is Qualifications for the Membership of Parliament
seeking election. (Art 84)
•• possess such other qualifications as may be prescribed by •• The individual contesting should be a citizen of India.
the Parliament from time to time.
•• He should be at least 30 years of age for Rajya Sabha and
Chairman and Deputy Chairman of Rajya Sabha 25 years for Lok Sabha.
•• The Vice-President of India is the ex-officio chairman of •• He should possesses such other qualifications as prescribed
the Council of States. by the Parliament.
•• Deputy chairman is elected by the Rajya Sabha. He shall •• His name should be registered as a voter in any
be a member of Rajya Sabha. parliamentary constituency.
•• Office of Deputy Chairman terminates if he ceases to •• No minimum educational qualification has been prescribed.
become the member of the Council. •• He/she must not hold any office of profit under the Union
•• Deputy Chairman can also resign, submitting his or state government.
resignation to the Chairman in writing. He can also be Disqualification from Membership of Either
removed from his office by a resolution of the Rajya
House of Parliament
Sabha, passed by a majority of all the then members of
the Council. But such a resolution can only be moved by •• If the individual holds an office of profit under GOI or a

•• ww
giving at least 14 days notice in advance.
If the office of Chairman is vacant, Deputy Chairman
discharges his functions. But if the office of Deputy
••
state government. Some posts have been exempted from it.
If the individual is of unsound mind which has been
declared so by a competent court.

w.E
chairman is also vacant, the duties of his office shall be
discharged by such a member of the Rajya Sabha as the
President may appoint for the purpose.
••
••
If the person is undischarged or insolvent.
The individual is not a citizen of India and has voluntarily
acquired citizenship of a foreign country or has allegiance
••
a
The sitting of the house is presided over by the Chairman

syE
and in his absence, by the Deputy Chairman. But if both
of them are absent then such person as may be determined
by the rule of procedure of the Council shall preside over
••
to a foreign power.
If he is so disqualified by or under any law made by the
Parliament (Art 102).
ƒƒ President has to obtain the opinion of the Election
the sitting of the house.
Union Leg ngi Commission before disqualifying a member, (Art 103).
ƒƒ Representation of people’s act also provides
additional grounds for disqualifications.

President
(Parliament)

Rajya Sabha Lok Sabha


nee
ƒƒ A member can also be disqualified on the grounds
of defection.

Upper House Lower House


House of People
••

r
Conditions when a member of parliament shall vacate his
seat (Art 101)
ing
ƒƒ If he has obtained membership of both houses of the
Parliament, he needs to vacate one of the seats.
Also known as
council of states
Total members - 250
Maximum
Strength - 552 .ne
ƒƒ If elected to both to the Parliament and state legislature,
he needs to resign from the state legislature.

238
Indirectly Nominated
elected
12 530
(States)
20 2 (Nominated
(UTs) members from
Anglo-Indian
ƒƒ If he is disqualified under Art. 102.
ƒƒ If he resigns in a voluntary manner.
t
ƒƒ If he remains absent from all meetings of the house
for a period of 60 days without prior permission of
Permanent Body
Community) the house.
Term of individual
Members are
member - 6 years Powers, Privileges and Immunities of Parliament
directly elected
1/3 rd members and its Members
Term - 5 years
retires at expiration •• Both the houses of the Parliament and state legislature
of every 2nd year. Presiding Officers
have same privileges.
Presiding officers •• Art 105 (1) & (2) and Art 194 (1) & (2) state the privileges
Speaker Deputy
Speaker to both the houses of parliament and state legislature.
Chairman Deputy •• Supreme Court has held that if there is any conflict
Chairman Qualification
between the privileges of the Parliament and fundamental
Qualification rights of citizens, the former shall prevail.
Citizen Not less
of India than 25 •• Privileges can be classified into two categories:
Citizen Not less than ƒƒ Individual privileges
of India years.
30 years
ƒƒ Collective privileges of Lok Sabha and Rajya Sabha.

Downloaded From : www.EasyEngineering.net


Downloaded From : www.EasyEngineering.net
P-54 Union Government

Individual Privileges •• A Financial Bill, not containing solely the matters of


•• Freedom from arrest in civil cases. There would be arrest Article 110, also can be introduced only in the Lok
in criminal cases or under preventive detention. Sabha and not in the Rajya Sabha. But, with regard to
its passage, both have equal powers. The final power to
•• Freedom from attendance as a witness: a member cannot decide whether a particular bill is a Money Bill is vested
be summoned by a court to give evidence as a witness in the Speaker of the Lok Sabha. The Speaker of Lok
while the Parliament is in session. Sabha presides over the joint sitting of both the houses.
•• Freedom of speech: A member of the Parliament is not •• The Lok Sabha with a greater number wins the battle in
liable in any court for anything said in Parliament or any a joint sitting except when the combined strength of the
of its committees. ruling party in both houses is less than that of opposition
parties. Rajya Sabha can only discuss the budget, but
Collective Privileges of Each House cannot vote on the demands for grants. A resolution
•• Right to publish debates and proceedings. for the discontinuance of the national emergency can be
•• Right to restrain publications by others. passed only by the Lok Sabha and not by the Rajya Sabha.
•• Right to exclude others like withdrawal of strangers from •• The Rajya Sabha cannot remove the Council of Ministers
any part of the house. The Speaker and the Chairman by passing a No-Confidence Motion. This is because the
have the right to order such action. Council of Ministers is collectively responsible only to
••

•• ww
Right to regulate internal affairs of the house and to decide
matters within its walls.
Right to punish any parliamentary misbehavior.
the Lok Sabha.

Special power of Rajya Sabha with respect to


Lok Sabha
••
privilege.
w.E
Right to punish members and outsiders for breach of

Joint Session of the House


As a federal chamber, it can initiate Central intervention in the
State Legislative field. Article 249 of the Constitution provides
that the Rajya Sabha may pass a resolution, by a majority of
••
a
is sent to the other. The other house may:
ƒƒ Reject the bill altogether. syE
Art 108 provides that when a bill is passed by one house
not less than two-thirds of the members present and voting,
to the effect that it is necessary or expedient in the national
interest that Parliament should make laws with respect to any
matter enumerated in the State List. If such a resolution is
ƒƒ Disagrees on it and returns it with some amendments
which are not ultimately considered by the originating
house. ngi adopted, Parliament will be authorised, to make laws on the
subject specified in the resolution, for the whole or any part of
the territory of India.
ƒƒ Takes no action and more than 6 months time has
passed.
ƒƒ The President in such a case may summon a joint
nee
Such a resolution will remain in force for such period,
not exceeding 1 year, as may be specified therein, but this
period can be extended by 1 year at a time by passing further

••
sitting of both the houses.
At a joint sitting of two houses, the Speaker of the Lok
Sabha and in his absence, the Deputy Speaker, or if he
resolutions.

r
Sessions of the Parliament
••
ing
President has power to summon either of the house and
is also absent, Deputy Chairman of the Council of States
and if he is also absent, such person as may be determined
by the members present in the sitting presides. Lok Sabha .ne
has power to dissolve the Lok Sabha. President must
summon each house at such intervals that 6 months shall

••
••
by its numerical majority prevails over the joint sitting.
This provision does not apply to money bill.
There cannot be a joint sitting for Constitution Amendment
bills. Nor do such bills require previous sanction of the
••
t
not intervene between its last sitting in one session and the
first sitting of next session [Art 85(1)]. Hence it is necessary
that the parliament must meet at least twice a year.
Parliament normally meets in three sessions in a year:-
ƒƒ Budget Session: February - May
President. ƒƒ Monsoon Session: July - August
•• President cannot summon a joint sitting if the bill has ƒƒ Winter Session: November - December
lapsed by reason of a dissolution of Lok Sabha. •• Adjournments: During a session, there are daily sittings
separated by adjournments. These postpone the further
Special Powers of Lok Sabha with respect to consideration of business for a specified time which may
Rajya Sabha extend for hours, days and even weeks.
•• A Money Bill can be introduced only in the Lok Sabha •• Dissolution ends the life of the House. General elections
and not in the Rajya Sabha. Rajya Sabha cannot amend are held to elect a new Lok Sabha.
or reject a Money Bill. It should return the bill to the Lok •• Prorogation merely ends a session and not the life of the
Sabha within 14 days with or without recommendations. house. The house meets again after prorogation. Pending
The Lok Sabha can either accept or reject fall or any of notices, motions and resolutions lapse but the bills remain
the recommendations of the Rajya Sabha. In both cases, alive.
the Money Bill is deemed to have been passed by the two •• Recess is the period between the prorogation of the
houses. Parliament and its re-assembly in a new session.

Downloaded From : www.EasyEngineering.net


Downloaded From : www.EasyEngineering.net
Union Government P-55

Delimitation Commission •• And government, does not need to


resign immediately unlike in case of
Pursuant to the enactment of the Constitution (Eighty-
No-Confidence Motion.
fourth Amendment) Act 2001, the Delimitation Act, 2002
•• It can be moved against an individual
was enacted. The Delimitation Commission was constituted
minister for specific policies or actions.
on July 12, 2012 under the provisions of the Delimitation
This motion should state the reason for
Act, 2002 with justice Kuldip Singh, a retired judge of the its adoption. It is in practice since 1954.
Supreme Court as its chairperson and one of the election
commissioners of India and the state. Election commissioners Legislative Procedure in Parliament
as its exofficio members. The main task of the Commission •• The legislative procedure is identical in both the Houses
is to readjust the division of territorial constituencies of of Parliament. Every bill has to pass through the same
the seats in the House of the people allocated to each state stages in each House. A bill is a proposal for legislation
and readjust the division of territorial constituencies of the and it becomes an act or law when duly enacted.
total number of seats in the Legislative. Assembly of each •• Bills introduced in the Parliament are of two kinds: public
state. The Commission will also refix the seats reserved for bills and private bills (also known as government bills
the Scheduled Casts and the Scheduled Tribes. Earlier the and private members’ bills respectively). Though both are
census figures of 1991 were to be the basis, but after the governed by the same general procedure and pass through

ww
eighty seventh Amendment Act, the census figure of 2001
are to be the basis.

Control of the Parliament Over the Executive


the same stages in the house, they differ in various
respects. Public bill can introduced by a minister which
requires seven days notice. It reflects the policies of the
Government and its rejection by the House shows non
Question hour
w.E
•• First hour of every parliamentary sitting.
•• Starred questions are answered orally
and supplementary questions can follow.
confidence of ruling party in Parliament and may leads to
its resignation while private bill introduce by any member
of Parliament and requires one month notice. Its rejection

a
•• Unstarred questions are answered in
writing.
syE
•• Short notice questions are asked giving
less than 10 days notice. 1.
has no implication on parliamentary confidence.
•• The bills introduced in the Parliament can also be classified
into four categories:
Ordinary bills, which are concerned with any matter
Zero hour •• Starts immediately after the question
hour.
•• Any matter can be discussed during the
ngi
2.
other than financial subjects.
Money bills, which are concerned with the financial
matters like taxation, public expenditure, etc.

Half-an-hour
zero hour.
•• To clear fact on matters of public
3.

4. nee
Financial bills, which are also concerned with financial
matters (but are different from money bills).
Constitution amendment bills, which are concerned with
discussion

discussions
importance on which lot of debate has
occurred.
Short duration •• To discuss urgent matters.
•• Also known as two hour discussion.
r
the amendment of the provisions of the Constitution.

ing
The Constitution has laid down separate procedures for the
enactment of all the four types of bill.

Calling attention •• Moved to call the attention of a minister


motion to matters of public importance.
Ordinary Bill
•• .ne
This is a bill other than money bill and finance bill.
Adjournment
motion
•• To draw attention of parliament to a
matter of urgent public importance.
•• Motion needs the support of 50
members for admission.
••
Parliament.
First Reading
t
An ordinary bill may originate in either house of the

At this stage the title of the bill is read and a brief speech
•• Rajya Sabha cannot move this motion.
regarding the aims and objective of the bill is made. Opponents
No Confidence •• Moved to prove the confidence of Lok of the bill also make a brief speech at this stage and after a
motion Sabha in the Council of Ministers. formal vote, the bill is published in gazette.
•• If No Confidence motion is passed,
council of Ministers has to resign. Second Reading
•• No Confidence motion needs the At this stage the general principles of the bill as a whole are
support of 50 members to be admitted. discussed and decision regarding reference of the bill to the
•• Can be moved only in Lok Sabha. appropriate committee is taken. No amendments are possible
at this stage.
Censure Motion •• This motion seeks to censure the
government for its lapses. Committee Stage
•• If the Censure Motion is passed against After the second reading, the bill is referred to the appropriate
the government, it should pass a committee where its provisions are thoroughly discussed. The
Confidence motion as soon as possible committee can also make suitable suggestions for improvement
to regain the confidence of the house. of the bill and suggest necessary amendments.

Downloaded From : www.EasyEngineering.net


Downloaded From : www.EasyEngineering.net
P-56 Union Government

Report Stage Financial Bills


The committee submits its report to the House, where it is •• They are of 3 kinds-
thoroughly discussed. The members of the House hold a 1 Money bills
clause-by-clause discussion and vote thereon. At this stage, 2. Other financial bills
they can also propose fresh amendments, which are accepted 3. Bills involving expenditure
by majority vote.
•• A financial bill will deal with matters mentioned in Art
Third Reading 110 (1). A money bill deals with other matters also.
Therefore all money bills are financial bills but all
A general discussion on the bill takes place and formal financial bills are not money bills.
voting for the acceptance or rejection of the bill is held. No
•• All financial bills are introduced only in the Lok Sabha
amendments can be proposed at this stage.
after the recommendations of the President.
After a bill has been passed by one house it is transmitted to the •• A financial bill is passed like an ordinary bill.
other house, where it goes through all these stages once again.
•• Joint session can be held.
After the bill has been passed by the other house, it is sent to
the President for assent. However, if the other house proposes Constitutional Amendment Bill (Article 368)
certain amendments which are not acceptable to the originating
•• Certain provisions of the Constitution can be amended
house, it may lead to a deadlock. The deadlock is resolved by
by the Parliament by simple majority. These include
convening a joint-sitting of the two houses where the decision

ww
is taken by majority vote.
The President can either accord his assent or return the bill
for reconsideration of the Parliament. But if the Parliament ••
provisions relating to the creation of new states,
reconstitution of existing states, creation or abolition of
upper chambers in the state legislature, etc.
Some provisions can be amended by Parliament by a

w.E
repasses the bill, the President has to accord assent to it.

Money Bill (Article 110)


•• Whether a bill is a money bill or not is decided by the
two-third majority and also require the approval of
the legislatures of not less than one-half of the states,
(There is no time limit within which the states should give
their consent to the bill). Provisions that can be amended

•• a
speaker of the Lok Sabha.

syE
Art 109 says that a money bill can only be introduced in
Lok Sabha and not in Rajya Sabha and only with the prior
recommendation of the President.
this way include election of the President, powers of
the Union and state executive, Union judiciary, High
Courts, representation of states in Parliament, amendment
procedure, etc.
•• When a money bill is passed by the Lok Sabha, it is sent
to Rajya Sabha for its recommendations. Rajya Sabha
must return the bill with or without any recommendations, ngi •• But a major portion of the Constitution can be amended
by a two-third majority in Parliament. This must also
be the clear-cut majority of the total membership of each
within 14 days from the date of receipt of bill. It is the
discretion of the Lok Sabha whether to accept or reject
recommendations of Rajya Sabha. The bill now is deemed •• nee
house. The provisions which can be amended in this ways
are F. R., D.P.S.P, etc.
It may be noted that provisions which affect the federal

••
••
to be passed by the Lok Sabha and is sent to the President
for his/her assent.
President cannot withhold his/her to a money bill (Art 111).
There is no provision for a joint sitting in the case of a
•• r
character of the Constitution can be amended only with

ing
the approval of the states.
A notable feature of the amendment procedure in India
is that the initiative rests with the Centre and the states
money bills as the Lok Sabha has a final say in the matter. cannot initiate any amendments.
Comparisons among different bills .ne
Ordinary Bill

house of parliament.
Money Bill
Can be introduced in either Only in Lok Sabha.
Financial Bill
Only in Lok Sabha.

Does not need President Need President recommendation Need President recommendation Does
t
Constitutional Amendment Bill
In either house of parliament.

not need President


recommendation for introduction. for introduction. for introduction. recommendation for introduction.
Passed by simple majority. Passed by simple majority. Passed by simple majority. Passed by simple or special majority
(by both houses separately) and or
approval of legislatures of not less
than one-half of the states.
Equal legislative jurisdiction of RS has only recommendatory Equal legislative jurisdiction of Equal legislative jurisdiction of both
both houses of parliament. power (14 days) both houses of Parliament. houses of Parliament.
Joint session can be held. Joint session cannot be held. Joint session can be held. Joint session cannot be held because
if one house rejects the bill, it comes
to an end.
President has three options: President has choice of President has three options: President has to give assent to the
Absolute veto, suspensive veto, withholding or giving assent Absolute veto, suspensive veto, bill.
pocket veto. to the bill, but by convention pocket veto.
he can not withhold the assent.

Downloaded From : www.EasyEngineering.net


Downloaded From : www.EasyEngineering.net
Union Government P-57

Public and Private Bills •• Money Bill cannot be introduced in Rajya Sabha.
•• Rajya Sabha should send Money Bill within 14 days to
Public Bills
Lok Sabha.
A public bill is introduced by minister in the Parliament or state
•• No tax can be levied except by authority of law- (Article 265)
legislature which reflects the policies of the rulling party or
the government. A public bill generally has greater chance of Types of Budget
approval by the Parliament unlike a private bill. The rejection Budget Type Given By Relates To
of a public bill by the house amounts to the expression of want First Hoover
of parliamentary confidence in the government and may lead to Commission, USA.
its resignation. This bill is drafted by the concerned department Emphasis on
Performance Introduced in
in consultation with the law department and its introduction in India in 1968 on ‘purpose’ of
Budgeting
the house requires seven days’ notice. recommendations expenditure
of Administration
Private Bill Reforms Commission
A private bill is introduced by any member of Parliament other Every scheme
than a minister which reflects the stand of opposition party on Zero-Based critically reviewed
public matter. A private bill is rejection by the house has no Phyrr, USA
Budgeting & re-justified totally

ww
implication on the parliamentary confidence in the government
or the resignation. The introduction of a private bill requires
one month’s notice and has lesser chance of parliamentary
Line-Item
Developed in 18th
century. Traditional
from zero (or scratch)
Emphasis on items
of expenditure &

member concerned.
Youth Parliament w.E
approval. A private bill’s drafting is the responsibility of the
Budgeting system prevailed in
India.
not its purpose. Sole
objective is control
over expenditure.

a
On the recommendation of the fourth all India whips

syE
conference a scheme of youth parliament was started in
1960s with the objectives of
(a) acquainting the younger generation with parliamentary
Stages In Enactment of Budget
Presentation of budget

practices and procedures;


(b) imbibing the spirit of discipline and tolerance in the
minds of youth; and ngi General discussion
(on budget as a whole)
(c) in calculating the basic values of democracy in the
student community and enabling them to acquire a
proper perspective on the functioning of democratic
nee Voting on demands for grants
institutions. The youth parliament scheme was first
introduced in the schools in Delhi in 1966-67.

Annual Financial Statement – Budget


r
(detailed discussions, various cut motions moved,

ing
many matters disposed off without discussions due to
shortage of time- this is called Guillotine)

(Article 112)
•• Passing of appropriation bill.ne
t
This statement gives out the estimated income and
expenditure for that year, laid before the houses of (authorizes Parliament to withdraw money from Consolidated
Parliament. Fund,
•• It is the statement regarding annual financial policy of the includes grants voted by Lok Sabha and expenditure charged
government. Its estimated receipts and expenditure in a on Consolidated Fund,
financial years, which begins on 1 April and ends on 31 no amendment can be made to it)
March of the following year.
The Government of India has two budgets, namely, the Railway
Budget and the General Budget. While the former consists of Passing of finance bill
the estimates of receipts and expenditures of only the Ministry (last stage of budget enactment,
of Railways, the latter consists of the estimates of receipts and gives effect to financial proposals of government,
expenditure of all the ministries of the Government of India
amendments can be moved to it)
(except the railways).
•• President shall lay before each house of parliament, an Other Grants
annual financial statement (known as budget)- Article 112
•• No demand for a grant shall be made except on S u p p l e m e n t a r y It is granted when the amount
recommendation of President- (Article 113). grant authorized by the Parliament through the
•• Rajya Sabha has no power to vote on demand for grants. appropriation act for a particular service
for the current financial year is found to
•• No money shall be withdrawn from Consolidated Fund of
be insufficient of that year.
India except under appropriation made by law- (Article 114).

Downloaded From : www.EasyEngineering.net


Downloaded From : www.EasyEngineering.net
P-58 Union Government

Additional grant It is granted when a need has arisen during Adjournment Sine Die
the current financial year for additional •• Adjournment sine die means terminating a sitting of
expenditure upon some new service not Parliament for an indefinite period.
contemplated in the budget for that year. •• The power of adjournment as well as adjournment sine
Excess grant It is granted when money has been spent die lies with the presiding officer of the House. He can
on any service during a financial year also call a sitting of the House before the date or time to
in excess of the amount granted for that which it has been adjourned or at any time after the House
service in the budget for that year. It is has been adjourned sine die.
voted by the lok sabha after the financial
Prorogation
year.
The presiding officer (Speaker or Chairman) declares the
Vote of credit It is granted for meeting an unexpected House adjourned sine die, when the business of a session is
demand for the service, the demands completed. Within the next few days, the President issues
cannot be stated with the details in the a notification for prorogation of the session. However, the
budget. It is like a blank cheque given to President can also prorogue the House while in session.
the executive by the lok sabha.
Exceptional It is granted for a special purpose and Dissolution
grant
ww
Token grant
forms no part of the current service of
any financial year.
It is granted when funds to meet the
Rajya Sabha, being a permanent House, is not subject to
dissolution. Only the Lok Sabha is subject to dissolution.
Unlike a prorogation, a dissolution ends the very life of the

w.E
proposed expenditure on the new
service can be made available by re-
appropriation. A demand for the grant of
existing House, and a new House is constituted after general
elections are held. The dissolution of the Lok Sabha may take
place in either of two ways:

a
token sum of Re 1 is submitted to the vote

made available.
syE
of Lok Sabha and if assented, funds are

Various Cut Motions as Moved in Lok Sabha


1. Automatic dissolution, that is, on the expiry of its tenure
of five years or the terms as extended during a national
emergency; or
2. Whenever the President decides to dissolve the House,
Policy cut Disapproval of policy. It states that
amount of demand be reduced to ` 1. ngi which he is authorised to do. Once the Lok Sabha is
dissolved before the completion of its normal tenure, the
dissolution is irrevocable.
Economy cut
Token cut
Demand be reduced by a specified amount
Demand is reduced by ` 100.
nee
When the Lok Sabha is dissolved, all business including bills,
motions, resolutions, notices, petitions and so on pending before
it or its committees lapse. They (to be pursued further) must
Parliamentary Terms
Summoning r ing
be reintroduced in the newly-constituted Lok Sabha. However,
some pending bills and all pending assurances that are to be
examined by the Committee on Government Assurances do not
The president from time to time sumons each House of
Parliament to meet. But, the maximum gap between two
sessions of Parliament cannot be more than six months. In
respect to lapsing of bills is as follows:
.ne
lapse on the dissolution of the Lok Sabha. The position with

other words, the Parliament should meet at least twice a year.


There are usually three sessions in a year, viz,
1. The Budget Session (February to May);
2. The Monsoon Session (July to September); and
t
1. A bill pending in the Lok Sabha lapses (whether originating
in the Lok Sabha or transmitted to it by the Rajya Sabha).
2. A bill passed by the Lok Sabha but pending in the Rajya
Sabha lapses.
3. A bill not Passed by the two Houses due to disagreement
3. The Winter Session (November to December). and if the president has notified the holding of a joint
A ‘session’ of Parliament is the period spanning between the sitting before the dissolution of Lok Sabha, does not
first sitting of a House and its prorogation (or dissolution in lapse.
the case of the Lok Sabha). During a session, the House meets 4. A bill pending in the Rajya Sabha but not passed by the
everyday to transact business. The period spanning between Lok Sabha does not lapse.
the prorogation of a House and its reassembly in a new session 5. A bill passed by both Houses but pending assent of the
is called ‘recess. President does not lapse.
6. A bill passed by both Houses but returned by the President
Adjournment for reconsideration of Houses does not lapse.
A sitting of Parliament can be terminated by adjournment
or adjournment sine die or prorogation or dissolution (in the Quorum
case of the Lok Sabha). An adjournment suspends the work in Quorum is the minimum number of members required to be
a sitting for a specified time, which may be hours, day or present in the House before it can transact any business. It
weeks. is one-tenth of the total number of members in each House

Downloaded From : www.EasyEngineering.net


Downloaded From : www.EasyEngineering.net
Union Government P-59

including the presiding officer. It means that there must be at 1. Emoluments and allowances of the President and other
least 55 members present in the Lok Sabha and 25 members expenditure relating to his office.
present in the Rajya Sabha, if any business is to be conducted. 2. Salaries and allowances of the Chairman and the Deputy
If there is no quorum during a meeting of the House, it is the Chairman of the Rajya Sabha and the Speaker and the
duty of the presiding officer either to adjourn the House or to Deputy Speaker of the Lok Sabha.
suspend the meeting until there is a quorum. 3. Salaries, allowances and pensions of the judges of the
Supreme Court.
Funds
4. Pensions of the judges of high courts.
Indian Constitution provides three kinds of funds for Central 5. Salary, allowances and pension of the Comptroller and
Government Auditor General of India.
1. Consolidated Fund of India 6. Salaries, allowances and pension of the chairman and
2. Public Accounts of India members of the Union Public Service Commission.
3. Contingency Fund of India 7. Administrative expenses of the Supreme Court, the
office of the Comptroller and Auditor General of India
Consolidated Fund of India and the Union Public Service Commission including the
•• Article 266 provides the Parliament to have a ‘Consolidated salaries, allowances and pensions of the persons serving

ww
Fund of India’. It is a fund to which all receipts are in these offices.
credited and all payments are debited. In other words, 8. The debt charges for which the Government of India
▪▪ all revenues received by the Government of India. is liable, including interest, sinking fund charges and
redemption charges and other expenditure relating to

w.E
▪▪ all loans raised by the Government by the issue of
Treasury Bills, loans or ways and means of advances.
▪▪ all money received by the government in repayment of
the raising of loans and the service and redemption of
debt.
9. Any sum required to satisfy any judgement, decree or
••
a
loans for the Consolidated Fund of India.

syE
All the legally authorised payments on behalf of the
government are made out of this fund. No money out of
this fund, can be appropriated (issued or drawn) except in
award of any court or arbitral tribunal.
10. Any other expenditure declared by the Parliament to be
so charged.
accordance with a Parliamentary Law.

Public Account of India ngi


Parliamentary Committees
Parliamentary Committees are of two kinds–Adhoc Committees
All other public money received by Government of India on
or on behalf of it, shall be credited to the Public Accounts of
India. It includes departmental deposits, remittances, judicial nee
and the Standing Committees. Adhoc Committees are appointed
for a specific purpose and they cease to exist when they finish
the task assigned to them and submit a report. The principal
deposits, provident fund deposits, etc. Payments from this
account can be made by without parliamentary appropriation as
these payments are mostly in the nature of banking transactions. r
Adhoc Committees are the Select and Joint Committees on Bills.

ing
Apart from he Adhoc Committees, each House of Parliament
has Standing Committees like the Business Advisory
Committee, the Committee on Petitions, the Committee of
It is covered in Article 266 (1).

Contingency Fund of India


Privileges and the Rules Committee.
.ne
An other class of committees which act as Parliament’s Watch
The Constitution authorised the Parliament to establish a
‘Contingency Fund of India’ (Under Article 267), into which
amounts determined by law are paid from time-to-time.
Accordingly, the Parliament enacted the Contingency Fund
t
Dogs over the executive is of special importance. These are
the Committee on Subordinate Legislation, the Committee
on Government Assurances, the Committee on Estimates,
the Committee on Public Accounts and the Committee on
of India Act in 1950. Public Undertakings and the Departmentally Related Standing
This fund is placed at the disposal of the President and he can Committees (DRSCs). They play an important role in
make advances out of it to meet unforeseen expenditure pending exercising a check over Governmental Expenditure and Policy
its authorisation by the Parliament. In 2005, the amount of Formulation.
money in the Contingency Fund of India was increased from ` Estimates Committee
50 crore to ` 500 crore.
•• This Committee originated in 1921. This Committee
Charged Expenditure consists of 30 members by the Lok Sabha every year
The budget consists of two types of expenditure—the from amongst its members. A minister is not eligible for
expenditure ‘charged’ upon the Consolidated Fund of India election to this committee. The term of the committee is 1
and the expenditure ‘made’ from the Consolidated Fund year. The function of the Committee are as follows:
of India. The charged expenditure is non-votable by the ƒƒ To report what economies, improvements in
Parliament, that is, it can only be discussed by the Parliament, organisation, efficiency and administrative reform
which the other type has to be voted by the Parliament. The consistent with the policy underlying the estimates,
list of the charged expenditure is as follows: can be affected.

Downloaded From : www.EasyEngineering.net


Downloaded From : www.EasyEngineering.net
P-60 Union Government

ƒƒ To suggest alternative policies in order to bring about Presentation of States & Union territories in ‘Lok
efficiency and economy in administration. Sabha’ and ‘Rajya Sabha’
ƒƒ To examine whether the money is well laid out within
Lok Sabha Rajya Sabha
the limits of the policy implied in the estimates. States
(No. of Seats) (No. of Seats)
ƒƒ To suggest the form in which the estimates are to be
Andhra Pradesh 25 11
presented to the Parliament.
•• This committee was constituted in 1950 on the Assam 14 7
recommendation of John Mathai. Goa 2 1
The Principles of Proportional Representation by means of a Haryana 10 5
Single Transferable Vote is used in election of the members. Jammu & Kashmir 6 4
The Chairman of this Committee is appointed by the Speaker Kerala 20 9
from amongst its members and he is invariably from the ruling Maharashtra 48 19
party. Meghalaya 2 1
Committee on Public Undertakings Odisha 21 10
•• This committee was created in 1964 on the recommendation Rajasthan 25 10
of the Krishna Menon Committee. The Committee on Tamil Nadu 39 18

ww
Public Undertakings consists of 15 members elected by
the Lok Sabha; 7 members of the Rajya Sabha are also
associated with it. A minister is not eligible for election
West Bengal
Mijoram
42
1
16
1

w.E
to this committee. The term of the committee is 1 year.
The functions of the Committee on Public Undertakings
are as follows:
Jharkhand
Uttar Pradesh
Bihar
14
80
40
6
31
16

Undertakings;
a
ƒƒ To examine the reports and accounts of Public

syE
ƒƒ To examine the reports, if any, of the Comptroller
and Auditor General on the Public Undertakings.
Gujarat
Himachal Pradesh
Karnataka
Madhya Pradesh
26
4
28
29
11
3
12
11
ƒƒ To examine, in the context of the autonomy and
efficiency of the Public Undertakings, whether the
affairs of the Public Undertakings are being managed
ngi Manipur
Nagaland
2
1
1
1

in accordance with sound business principles and


prudent commercial practices.
Punjab
Sikkim
Tripura nee 13
1
2
7
1
1

r
ƒƒ The Chairman of the Committee is appointed by the
Arunachal Pradesh 2 1
Speaker, from amongst its members who are darwn
from the Lock Sabha only. Uttarakhand
Chattisgarh ing 5
11
3
5
Committee on Public Accounts
This committee was first set-up in 1921 under the provisions of
the Government of India Act, 1919. This committee consists
Telangana

Union Territories
17

Lok Sabha .ne


Rajya
7

Sabha
of 22 members (15 from Lok Sabha and 7 form Rajya Sabha).
A minister is not eligible for election to this committee.
The term of the committee is 1 year. The main duty of the
committee is to ascertain whether the money granted by
Delhi
Chandigarh
(No. of Seats)
7
1
t
(No. of Seats)
3
Nil
Dadra and Nagar
Parliament has been spent by government ‘within the Scope 1 Nil
Haveli
of the Demand.’
Daman and Diu 1 Nil
Rules Committee Andaman and
1 Nil
It considers matters of procedure and conduct of business in Nicobar
the house and recommends any amendments or additions to the Puducherry 1 1
Rules of Procedure and Conduct of Business in Lok Sabha that
Lakshadweep 1 Nil
are considered necessary.

Downloaded From : www.EasyEngineering.net


Downloaded From : www.EasyEngineering.net
Union Government P-61

Exercise - 1
1. The President takes an oath before taking office in the 9. _____ requires previous sanction of the President for
presence of Chief Justice of India. If the Chief Justice is introducing legislation
not available, he takes the oath in the presence of the (a) Money bill
(a) Vice-President (b) Bill affecting taxation in which States are interested
(b) Senior-most Judge of the Supreme Court (c) State bills imposing restrictions upon freedom of
(c) Attorney-General trades
(d) Election Commissioner (d) All of the above
2. _____ elects the Vice-President? 10. If the President returns a bill to the legislature for
(a) Electoral college which elects the President reconsideration:
(b) Members of the Rajya Sabha and Lok Sabha (a) a re-passage of the bill by two-thirds majority forces
(c) Electoral college consisting of members of
him to give his assent
Parliament
(b) a re-passage of the bill by a majority will make him

ww
(d) Members of Parliament in a joint meeting
3. Candidate for Vice-presidential election must possess
the qualifications prescribed for a Presidential candidate
give an assent
(c) joint sitting of the parliament is needed to pass the
bill.



w.E
except that he must be:
(a) a citizen of India
(b) over 35 years of age
(c) hold no office of profit under the Government
(d) legislature must accept the amendments proposed by
the President
11. If the Chairman of Rajya Sabha becomes acting President,


a
(d) Qualified to be a member of the Rajya Sabha

after 14 days’ notice signed by


syE
4. A resolution for impeaching the President can be moved

(a) not less than 50 members of the House




his duties as a Chairman are performed by
(a) Continues as Chairman
(b) A newly elected Chairman
(c) Deputy Chairman


(b) not less than one-third of the total number of
members of the House
(c) not less than one-fourth of the total number of ngi (d) Member of Rajya Sabha deputed by the Chairman
12. President may send the advice received from the Council
of Ministers back for reconsideration. This power can be


members of the House
(d) at least 100 members of Lok Sabha and 50 members
of Rajya Sabha
(a) nee
exercised by him
Once

r
(b) Twice

ing
5. If a resolution impeaching the President is passed, the (c) Thrice
President is considered to have been removed (d) Unlimited number of times
(a) from the date on which the resolution is passed
13. Military powers of the President EXCLUDES
(b) when the Chief Justice of India passes such an order


(c) notification in the Gazette of India
(d) once a new incumbent is elected


.ne
(a) Supreme command of the armed forces
(b) President’s power to declare war or peace is subject
6. _____ powers are exercised both by the President and the
Governors?
(a) Power to pardon a sentence by court martial
(b) Power to remit a sentence in an offence relating to


to control by the Parliament

t
(c) President needs the sanction of Parliament to approve
training and maintaining of armed forces
(d) President is independent of all legislative control
matters on the State List 14. President does NOT APPOINT the
(c) Power to commute a death sentence (a) Finance Commission
(d) Power to remit a sentence by court martial (b) Planning Commission
7. Legislative powers of the President EXCLUDES the power (c) Commission on Official Languages
to : (d) UPSC
(a) summon or prorogue the Houses of Parliament 15. Post of the Prime Minister of India:
(b) call a joint sitting of the Houses to resolve deadlocks (a) is based on conventions
(c) nominate 12 members to the Lok Sabha (b) has been created by the Parliament
(d) address either House at any time (c) has been created by the Constitution
8. An ordinance promulgated by the President : (d) is less powerful than that of the President
(a) has an indefinite life 16. In case of resignation or death of the Prime Minister
(b) effective only if the Lok Sabha is dissolved (a) Council of Ministers is dissolved
(c) must be laid before the Parliament when it meets (b) General elections are held
(d) is a parallel power of legislation available to the (c) Cabinet elects another leader
President even when Parliament is in session (d) Lok Sabha is dissolved

Downloaded From : www.EasyEngineering.net


Downloaded From : www.EasyEngineering.net
P-62 Union Government

17. Prime Minister holds his office during the pleasure of the 26. Members of Rajya Sabha are:
President but can continue if he enjoys the confidence of the (a) Elected indirectly
(a) Electorate (b) All are nominated
(b) Lok Sabha (c) Elected both directly and indirectly
(c) Party to which he belongs (d) Elected by members of State Legislative assemblies
(d) Parliament and Legislative Councils
18. In parliamentary democracy the 27. A dissolution does not affect:
(a) Executive controls the Legislature (a) A bill that originated in the Rajya Sabha and is with
Lok Sabha the President calls upon it to do so
(b) Executive and Legislature are separate
(b) Rajya Sabha passes a resolution by simple majority
(c) Judiciary controls both legislature and Executive
that it is necessary in national interest
(d) Legislature controls the Executive (c) Speaker certifies the need
19. According to the Constitution (d) There is a national emergency
(a) Prime minister allocates work amongst ministers 28. Parliament can legislate on a subject in the State List if:
based on specific guidelines (a) The President calls upon it to do so
(b) Number of ministers, including the prime minister (b) Rajya Sabha passes a resolution by simple majority
should not be more than 15% of total number of that it is necessary in national interest

ww members of Lok Sabha


(c) a member of either House of Parliament from any
political party who is disqualified under the, Anti-


(c) Speaker certifies the need
(d) There is a national emergency
29. A No-Confidence Motion:


w.E
Defection Act appointed a minister during that term
even if elected again
(d) all of the above are correct
(a) is expressed against an individual minister or council
of ministers
(b) no grounds are needed.



a
20. If an unqualified or disqualified person attends and votes
in either house of the Parliament
(a) he is prosecuted and jailed syE
(b) he is fined ` 500 per day of his sitting
(c) 100 members’ have to support it
(d) always against the Prime Minister
30. ______ ensures that no unauthorized money is spent out
of the Consolidated Fund of a State.
(c) he may be imprisoned
(d) Nothing can be done ngi
(a) Public Accounts Committee
(b) Comptroller and Auditor-General of India
21. Money Bill


(a) cannot be introduced in the Rajya Sabha
(b) has to be certified by the President


nee
(c) Finance Commission
(d) None of the above
31. If the Prime Minister of India Belonged to the Upper
(c) can be amended by the Council of States
(d) both (a) and (b)
22. Amendments cannot be put up in either House to

r
House of Parliament

ing
(a) he will not vote in his favour in the event of a no-
confidence motion
(b) he will not speak on the budget in the Rajya Sabha



(a) Annual Financial Statement
(b) Appropriation Bill
(c) Demand for grants
.ne
(c) he can make statements only in the Lok Sabha
(d) he has to become a member of the Lower House



(d) Any of the above
23. Privileges of the members of Parliament include:
(a) Freedom from arrest in all cases
(b) Freedom of attendance as witness if Parliament is in
Minister
t
within 6 months after being sworn as the Prime

32. Which duty is not performed by the Comptroller and


Auditor General of India?
(a) Expenditure from the Consolidated Fund of India
session
(b) Expenditure from the Contingency Funds and Public
(c) Total freedom of speech Accounts
(d) All of the above (c) Trading, manufacturing, profit and loss accounts
24. Parliament does not have the power to remove: (d) Receipt and issue of public money, and to ensure
(a) Comptroller and Auditor General that public revenue is lodged in the exchequer
(b) Supreme Court Judges 33. Parliament can legislate for the country or part of it for
(c) Chairman of UPSC implementation of international treaties. For this consent
(d) High Court Judges of all the:
25. _____ is not a Financial Committee of the Parliament (a) States is required
(a) Public Accounts Committee (b) State/States concerned needs to be taken
(b) Estimates Committee (c) Legislation has to be ratified by at least half of the
(c) Committee on Public Under-takings States of India
(d) Rules Committee (d) No consent is required

Downloaded From : www.EasyEngineering.net


Downloaded From : www.EasyEngineering.net
Union Government P-63

34. Among the following Presidents of India, who was also (c) by a resolution adopted by the two house meeting in
the Secretary General of Non-Aligned Movement for a joint session
some period ? (d) by following a formal process of impeachment
(a) Dr. Sarvepalli Radhakrishnan 42. According to Art. 81 of the Constitution, as amended
(b) Varahagiri Venkatagiri by the 87th Amendment, the population figures to
(c) Giani Zail Singh be considered for dividing each state into territorial
constituencies for Lok Sabha elections are of ____ census
(d) Dr. Shanker Dayal Sharma
(a) 1971 (b) 2001
35. Which one of the following devices calls the attention of
(c) 1991 (d) 2011
minister towards a matter of public importance?
43. The amendment of the Constitution can be initiated in
(a) Half-an-hour discussion
(a) the Lok Sabha only
(b) Calling attention notice
(b) the Rajya Sabha only
(c) Short duration discussion (c) the Legislative Assemblies of states
(d) Adjournment motion (d) either house of the Parliament
36. Which of the following is not a condition of admissibility 44. When a Money Bill is introduced in the Lok Sabha whose
of cut motions in the Parliament? recommendation is necessary?
(a) It should not make suggestions for the amendment of (a) Lok Sabha Speaker


ww existing laws.
(b) It should not relate to expenditure charged on the
Consolidated Fund of India.



(b) Union Finance Minister
(c) Leader of the Lok Sabha
(d) The President


w.E
(c) It should relate to more than one demand.
(d) It should not raise a question of privilege.
37. Which of the following statements about President’s
45. Who is empowered by the Constitution to dissolve the


Lok Sabha before the expiry of its term?
(a) The Prime Minister




a
ordinance-making power is not correct?

(b) Laid down in Article 123.


syE
(a) It is co-extensive with legislative power of Parliament.

(c) Shall cease to operate on expiry of six weeks from




(b) The President
(c) The President of India on the advice of the Prime
Minister
(d) The Chief Election Commissioner


the reassembly of the Parliament.
(d) Cannot be withdrawn at any time by the President.
38. The President of India is elected by an electoral college ngi 46. With reference to the passage of Money Bill in Indian
Parliament, which one of the following statements is
correct?



consisting of :
(a) Members of Parliament and state legislatures
(b) Elected members of Parliament and state legislatures

nee
(a) The decision of the Union Finance Minister is final
on the question whether a bill is Money Bill or not
(b) A Money Bill can be introduced only in the Lok


(c) Elected members of Parliament and state legislative
assemblies
(d) Elected members of Lok Sabha and members of

r
Sabha

ing
(c) After a money bill is passed by Lok Sabha and
transmitted to Rajya Sabha, it should be returned
Rajya Sabha and state Legislative Assemblies
39. Disqualification on grounds of defection for a member of
Parliament will not apply

within one month

.ne
(d) When Rajya Sabha returns the Money Bill with its
recommendations, the Lok Sabha has to accept such


(a) in case of a split in the original political party to which
he is elected
(b) if he has voluntarily given up his membership of a
political party

recommendations
t
47. In the absence of both the President of India and the Vice-
President, who shall act as the President of India?
(a) Prime Minister
(c) if the abstains from voting in the house contrary to
the direction of the political party (b) Speaker of the Lok Sabha
(d) if a nominated member of a house joins a political (c) Chief Justice of the Supreme Court
party after 6 months (d) Deputy Chairman of Rajya Sabha
40. The Rajya Sabha has exclusive jurisdiction in 48. Which one of the following statements is not correct ?
(a) approving a proclamation of emergency
(a) All the expenditure other than that which is charged
(b) the creation and abolition of states
on the Consolidated Fund of India is to be submitted
(c) the election of the Vice-President
(d) authorizing Parliament to legislate on a subject in the to the Lok Sabha in the form of demands for grants
state list (b) No demand for a grant is made except on the
41. The Vice-President of India can be removed from the office recommendation of the President of India
(a) by a resolution initiated in the council of states and (c) The Lok Sabha can refuse assent to any demand for
approved by the Lok Sabha grant
(b) by a resolution in the Lok Sabha agreed by the council (d) The Lok Sabha can suggest an increase in the
of states expenditure

Downloaded From : www.EasyEngineering.net


Downloaded From : www.EasyEngineering.net
P-64 Union Government

49. Article 75 of the Constitution of India provides that the 55. Which of the following statements is not correct?
Council of Ministers of the Union shall be collectively (a) The president shall not be answerable to any court
responsible to for the exercise and performance of the powers and
(a) the House of the People duties of his office
(b) both the Houses of Parliament (b) Parliament can initiate, in accordance with the
(c) the President only procedure laid down in the Constitution, impeachment
(d) the President and both the Houses of the Parliament proceedings against the President during the term of
50. The Union Executive of India consists of: his office
(a) The President; Vice-President, Lok Sabha Speaker (c) Civil proceedings can be instituted against the
and the Council of Ministers President, in respect of any act done in his personal
(b) The President, Deputy Chairman of the Rajya Sabha, capacity before he entered upon his office as
Lok Sabha Speaker and the Prime Minister President, during the term of his office
(c) The President and the Council of Ministers only (d) No criminal proceedings shall be instituted or
(d) The President, Vice-President and the Council of continued against the President in any Court during
Ministers only his term of office
51. When an ordinary Bill is referred to a joint sitting of both 56. The Comptroller and Auditor General of India


ww
the Houses of Indian Parliament, it has to be passed by a
(a) simple majority of the total number of members of
both the Houses present and voting
(a) Can be appointed as a member of U.P.S.C. after his
retirement

w.E
(b) two-third majority of the total number of members of
both the Houses
(c) simple majority of the total number of members of


(b) Can be appointed as Chairman of the State Public
Service Commission
(c) Shall not be eligible for any further office either under
the government of India or under the Government of

both the Houses

a syE
(d) two-third majority of the total number of members of
both the Houses present and voting
52. Which one of the following statements about a Money

any State after his retirement
(d) Can be appointed to any office after his retirement
57. Which one of the following is not a main feature of the


Bill is not correct?
(a) A Money Bill can be tabled in either House of
Parliament ngi
Cabinet system of government as it prevails in India?
(a) The members of the Council of Ministers are chosen
by the president and not by the Prime Minister


(b) The Speaker of Lok Sabha is the final authority to
decide whether a Bill is a Money Bill or not
(c) The Rajya Sabha must return a Money Bill passed by

nee
(b) The proceedings of the Cabinet are kept secret
(c) The Prime Minister presides over the Cabinet
meetings


the Lok Sabha and send it for consideration within 14
days
(d) The President cannot return a Money Bill to the Lok
Sabha for reconsideration

r
(d) All of the above

ing
58. Which one of the following is not a department in the
ministry of Human Resource Development?
53. Which one of the following duties is not performed by
Comptroller and Auditor general of India?

.ne
(a) Department of Elementary Education and Literacy
(b) Department of Secondary Education and Higher


(a) To audit and report on all expenditure from the
Consolidated Fund of India
(b) To audit and report on all expenditure from the
Contingency Funds and Public Accounts


Education
(c) Department of Technical Education
t
(d) Department of women and child development
59. The Comptroller and Auditor General is appointed by the
(c) To audit and report on all trading, manufacturing, profit president. He can be removed
and loss accounts (a) by the president
(d) To control the receipt and issue of public money, (b) on an address from both Houses of Parliament
and to ensure that the public revenue is lodged in the (c) on the recommendation of the president by the
exchequer supreme court
54. What is a ‘charged expenditure’? (d) by CJI
(a) An expenditure which has been incurred already and 60. Which one of the following expenditure is not charged on
for the bills which are pending for payment the consolidated fund of India?
(b) Expenditure on the essential aspects of adminstration (a) Salary and allowances of the president of India
(c) The expenditure that can be incurred by the president (b) Salary and allowances of the vice president of India
without the approval of the parliament. (c) Salary and allowances of the justice of the supreme
(d) The expenditure from the Consolidated Fund of court of India
India for which the approval of the Parliament is not (d) Salary and allowances of the speaker of the Lok
necessary, according to the Constitution. Sabha

Downloaded From : www.EasyEngineering.net


Downloaded From : www.EasyEngineering.net
Union Government P-65

61. Who has the right to seek advisory opinion of the supreme 70. The President of India enjoys emergency powers of
court of India, on any question of law? (a) Two types (b) Three types
(a) Prime Minister (c) Four types (d) Five types
(b) President 71. Who is the highest civil servant of the Union Government?
(c) Any of the high courts (a) Attorney General
(d) All of the above
(b) Cabinet Secretary
62. Recommendations to the President of India on the specific
(c) Home Secretary
Union state fiscal relation are made by the
(a) Finance Minister (d) Principal Secretary of the P.M.
(b) Reserve Bank of India 72. The speaker’s vote in the Lok Sabha is called
(c) Planning Commission (a) Casting vote (b) Sound vote
(c) Finance Commission (c) Direct vote (d) Indirect vote
63. Who amongst the following is not entitled to take part in 73. The Parliamentary Committee which scrutinizes the
the activities of Lok Sabha? report of the CAG of India is
(a) The Comptroller and Auditor General of India (a) Estimates Committee
(b) The Attorney General of India (b) Select Committee


ww
(c) The solicitor General
(d) The secretary to president of India
64. A bill presented in Parliament becomes an Act after


(c) Public Accounts Committee
(d) None of these
74. What is the period of appointment of the comptroller and




w.E
(a) It is passed by both the Houses
(b) The president has given his Assent
(c) The Prime minister has signed it
(d) The Supreme Court has declared it to be within the


Auditor General of India?
(a) 6 years
(b) upto 65 years of age

a
competence of the Union Parliament

syE
65. Which one of the following statements about the


parliament of India is not correct?
(a) The constitution provides for a parliamentary form


(c) 6 years or 65 years of age, whichever earlier
(d) upto 64 years of age
75. In order to be recognised as an official opposition group
in the parliament


of Government
ngi
(b) The foremost function of the parliament is to provide
a cabinet
(a) 1/3rd of the total strength
(b) 1/4th of the total strength


(c) The membership of the cabinet is restricted to the
lower house
(d) The cabinet has to enjoy the confidence of the
nee
(c) 1/6th of the total strength
(d) 1/10th of the total strength
76. Who among the following is the chairman of the National
majority in the popular chamber
66. The ‘Contingency Fund’ of the state is operated by
(a) The Governor of the state
r
Integration council?
(a) The President
(b) The Vice-president ing



(b) The Chief Minister of the state
(c) The State Finance Minister
(d) None of the above
(c) The Prime Minister
(d) The Chief Justice of India .ne
67. Who of the following Presidents of India was associated



with trade Union Movement?
(a) V.V. Giri
(b) N. Sanjiva Reddy



(a) Attorney General
(b) Chief justice of supreme court
(c) Chairman, Law Commission
t
77. Who advises the Government of India on legal matters?

(c) K.R. Narayanan (d) None of these


(d) Zakir Hussain 78. The Committee of parliament on official language
68. The office of the President does not characterize the comprises the members
combination of governmental systems of (a) 20 from Lok Sabha, 10 from Rajya Sabha
(a) Parliamentary and Federal (b) 10 from Lok Sabha, 20 from Rajya Sabha
(b) Republican and Parliamentary (c) 10 from Lok Sabha, 10 from Rajya Sabha
(c) Presidential and Republican
(d) 20 from Lok Sabha, 20 from Rajya Sabha
(d) Democratic and Republican
79. In case the President wishes to resign, to whom is he to
69. The system of Proportion of representation as an electoral
mechanism ensures address his resignation letter?
(a) Representation of minorities (a) Chief Justice of India
(b) Rule of majority (b) Secretary of Lok Sabha
(c) Stability in government (c) Vice President
(d) Common political thinking (d) Prime Minister

Downloaded From : www.EasyEngineering.net


Downloaded From : www.EasyEngineering.net
P-66 Union Government

80. Who among the following Indian Prime Ministers 88. A minister who is not a member of either house (Note:
resigned before facing a vote of no-confidence in the Lok A person can remain a minister for six months, without
Sabha? being a member of either house of parliament):
(a) Chandra Shekhar (a) Can participate in the proceedings of Lok Sabha
(b) Morarji Desai only.
(c) Chaudhary Charan Singh
(b) Can participate in the proceedings of Rajya Sabha
(d) V.P. Singh
only.
81. ‘Zero Hour’ in political Jargon refers to
(a) Suspended motion (c) Can participate in the proceedings of either house of
(b) Question hour parliament.
(c) Adjourned time (d) Cannot participate till he becomes a Member of
(d) Question-answer session either house of parliament.
82. Which of the following types of authorities are attributed 89. The first reading of the Bill in a House of Parliament
to the President of India? refers to :
(a) Real and Popular (a) The motion for leave to introduce a Bill in the House
(b) Titular and de jure (b) The general discussion on the Bill as whole where
(c) Constitutional and Nominal only the principle underlying the Bill is discussed

ww
(d) Both (b) and (c)
83. When the offices of both Speaker and Deputy Speaker
falls vacant –

and not the details of the bill.
(c) The general discussion on the Bill where the bill is
discussed in details.


Speaker.
w.E
(a) The members of Lok Sabha immediately elect a

(b) The senior most willing member of Lok Sabha


becomes the speaker.
(d) The state when the Bill is referred either to select
committee of the House or to the joint committee of
the two houses.


a
(c) The President appoints any member of Lok Sabha as
speaker.
syE
(d) The Deputy Chairman of Rajya Sabha presides over
till the next speaker is elected.
90. Department of official languages is subordinate office of



which ministry?
(a) Ministry of social justice and Empowerment
(b) Ministry of Home Affairs
84. With Regard to Constitutional Amendment Bill –

ngi
(a) The President can reject the bill but cannot return the
bill.


(c) Ministry of Rural Development
(d) Ministry of Culture


(b) The President cannot reject the bill but can return the
bill.
(c) The President can neither reject the bill nor return
nee
91. Which one of the following is part of the Electoral College
for the election of the president of India but does not form
part of the forum for his impeachment?


the bill.
(d) The President can either reject the bill or return the
bill.



r
(a) Lok Sabha
(b) Rajya Sabha
ing
(c) State Legislative Councils
85. The Vice President can be removed from office before


completion of his term in which of the following manner?
(a) She/he can be impeached in similar manner as
(d) State Legislative Assemblies
.ne
92. The speaker can ask a member of the House to stop


President.
(b) A Resolution of Rajya Sabha passed by special
majority and agreed to by the Lok Sabha.
(c) A Resolution of Rajya Sabha passed by simple


is known as?
(a) Decorum
(c) Interpolation
t
speaking and let another member speak. This phenomenon

(b) Crossing the floor


(d) Yielding the floor
majority and agreed to by the Lok Sabha.
93. Which one of the following Bills must be passed by each
(d) A Resolution of Rajya Sabha passed by an absolute
majority and agreed to by the Lok Sabha. House of the Indian parliament separately, by special
86. The office of the ‘Whip’ is mentioned in: majority.
(a) Constitution of India. (a) Ordinary Bill
(b) Rules of the house. (b) Money Bill
(c) In a separate Parliamentary Statute. (c) Finance Bill
(d) None (d) Constitution Amendment Bill
87. The office of the Leader of the opposition is mentioned 94. Department of Border management is a department of
in: which one of the following Union Ministers?
(a) Constitution of India (a) Ministry of Defense
(b) Rules of the house (b) Ministry of Home Affairs
(c) A separate Parliamentary Statute (c) Ministry of Shipping, Road Transport and Highways
(d) None of these (d) Ministry of Environment and Forest.

Downloaded From : www.EasyEngineering.net


Downloaded From : www.EasyEngineering.net
Union Government P-67

95. What will follow if a Money Bill is substantially amended (c) The Speaker of the Lok Sabha
by the Rajya Sabha? (d) Joint Parliamentary Committee
(a) The Lok Sabha may still proceed with the Bill, 104. The Speaker of the Lok Sabha can resign his office by
accepting or not accepting the recommendations of addressing his resignation to
the Rajya Sabha (a) the President
(b) The Lok Sabha cannot consider the bill further (b) the Prime Minister
(c) The Lok Sabha may send the Bill to the Rajya Sabha (c) the Deputy Speaker of the Lok Sabha
for reconsideration (d) the Chief Justice of India
(d) The President may call a joint sitting for passing the 105. Who can initiate impeachment proceedings against the
Bill President of India ?
(a) Only Lok Sabha
96. Which one of the following is the largest Committee
(b) Only Rajya Sabha
of the Parliament?
(c) Either House of the Parliament
(a) The Committee on Public Accounts
(d) Any Legislative Assembly
(b) The Committee on Estimates 106. The Government Bill means a bill introduced by a
(c) The Committee on Public Undertakings (a) Member of the Treasury bench in the Lok Sabha

ww
(d) The Committee on Petitions
97. Who among the following decides whether a bill is a
Money Bill or not ?



(b) Member of the Parliament who is not a Minister
(c) Minister in the Lok Sabha
(d) Minister in any House of the Parliament.



w.E
(a) Union Finance Minister
(b) Speaker of Lok Sabha
(c) Union Minister of Parliamentary Affairs
107. When an ordinary Bill is referred to a joint sitting of both


the Houses of Indian Parliament, it has to be passed by a
(a) simple majority of the total number of members of

a
(d) President of India

syE
98. While Proclamation of Emergency is in operation, the
term of the Lok Sabha can be extended for a period not
exceeding?


both the Houses present and voting
(b) two-third majority of the total number of members
of both the Houses
(c) simple majority of the total number of members of


(a) Six weeks
(c) Six months
(b) Three months
(d) One year
99. What is the term of a Member of the Rajya Sabha ? ngi
both the Houses
(d) two-third majority of the total number of members
of both the Houses present and voting


(a) Three years
(c) Five years
(b) Four years
(d) Six years
100. The Union Executive of India consists of:
nee
108. Which one among the following features of the
Constitution of India is indicative of the fact that the real


(a) The President; Vice-President, Lok Sabha Speaker
and the Council of Ministers
(b) The President, Deputy Chairman of the Rajya Sabha,
r
executive power is vested in the Council of Ministers

(a) Federalism

ing
headed by the Prime Minister?

(b) Representative legislature


Lok Sabha Speaker and the Prime Minister
(c) The President and the Council of Ministers only


(c) Universal adult franchise
(d) Parliamentary democracy .ne
(d) The President, Vice-President and the Council of
Ministers only
101. Who among the following chooses the Speaker of the
House of People?


(a) the Union List
(b) the Concurrent List
t
109. Normally the Parliament can legislate on the subjects
enumerated in

(a) The Prime Minister of India (c) the State List


(b) The Union Minister of Parliamentary Affairs (d) the Union as well as Concurrent List
(c) The Leader of the Opposition in the Lok Sabha 110. Vice-President of India is elected by an electoral college
(d) The House of People consisting of
102. In the Union Government, under whose charge is the (a) members of both Houses of Parliament
Cabinet Secretariat? (b) members of Rajya Sabha only
(a) The Minister of Parliamentary Affairs (c) elected members of both Houses of Parliament
(b) The President of India (d) elected members of Lok Sabha only
(c) The Prime Minister of India 111. Joint Parliamentary Sessions in India are chaired by the
(d) The Union Home Minister (a) President of India
103. If any question arises whether a Bill is a Money Bill or (b) Vice-President of India who is the Chairman of the
not, whose decision shall be final? Rajya Sabha
(a) The Supreme Court of India (c) Speaker of the Lok Sabha
(b) The President of India (d) Prime Minister of India

Downloaded From : www.EasyEngineering.net


Downloaded From : www.EasyEngineering.net
P-68 Union Government

112. Which of the following is not a Parliamentary Committee? 120. The function of a Protem Speaker is to
(a) Demands for Grants Committee (a) conduct the proceeding of the House in the absence
(b) Committee on Public Accounts of the Speaker
(c) Committee on Public Undertakings (b) officiate as Speaker when the Speaker is unlikely to
(d) Committee on Esti mates be elected
113. Who was the President of India at the time of proclamation (c) swear members and hold charge till a regular
of emergency in the year 1976? Speaker is elected
(a) V.V Giri (d) scrutinize the authenticity of the election certificates
(b) Giani Zail Singh of members
(c) Fakhr-ud-din Ali Ahmad
121. If the Prime Minister is a member of the Rajya Sabha
(d) Shankar Dayal Sharma
(a) He/she has to get elected to the Lok Sabha within 6
114. Who among the following was never a Deputy Prime
months
Minister of India?
(a) Devi Lal (b) GL Nanda (b) He/she can declare the government’s policies only in
(c) LK Advani (d) YB Chavan the Rajya Sabha
115. Who among the following determines the authority who (c) He/she cannot take part in the voting when a vote of
no confidence is under consideration

ww
shall readjust the allocation of seats in the Lok Sabha
to the states and division of each State into territorial
constituencies?
(d) He/she cannot take part in the budget deliberation in
the Lok Sabha




w.E
(a) The President of India
(b) The Parliament of India
(c) The Chief Election Commissioner of India
(d) The Lok Sabha alone
122. The impeachment of the President of India can be initiated


in
(a) either house of the Parliament


a
116. In India the Supreme Command of the Armed Forces is,

syE
vested in the President. This means that in the exercise of
this power
(a) he/she cannot be regulated by law



(b) a joint siting of both houses of the Parliament
(c) the Lok Sabha alone
(d) the Rajya Sabha alone
123. The President of India is elected by a proportional


(b) he/she shall be regulated by law
(c) during war, the President seeks advice only from the ngi
representation system through single transferable vote.
This implies that
(a) each elected MP or MLA has an equal number of
Chiefs of the Armed Forces
(d) during war the President can suspended the
Fundamental Rights of citizens nee
votes
(b) MPs and MLAs of a State have the same number of
117. The quorum for Joint Sitting of the Indian Parliament is


(a) One- twelveth of the total number of members of the
House
(b) One-sixth of the total numbers of members of the

r
votes

ing
(c) all MPs and MLAs have one vote each
(d) MPs and MLAs of different States have different
numbers of votes


House
(c) One-tenth of the total number of members of the .ne
124. With reference to the conduct of government business in
the Parliament of India, the term ‘closure’ refers to


House
(d) Two-third of the total number of members of the
House
118. Who among the following Indian Prime Ministers could


sitting of the Parliament t
(a) suspension of debate at the terminafan of a day’s

(b) a rule of legislative procedure under which further


debate on a motion can be hatted
not vote for himself during the ‘Vote of Confidence’ that
(c) the termination of a Parliamentary session
he was seeking from the Lok Sabha?
(d) refusal on the part of tie Government to have the
(a) VP Singh (b) PV Narasimha Rao
opposition look at important documents
(c) Chandra Sekhar (d) Manmohan Singh
119. Identify the correct sequence of passing a Budget in the 125. While a proclamation of emergency is in operation the
Parliament duration of the Lok Sabha can be extended for a period
(a) Vote on Account, Finance Bill, Appropriation Bill (a) not exceeding three months
Discussion on Budget (b) not exceedng nine-months
(b) Finance Bill, Appropriation Bill, Discussion on (c) of one year at a time
Budget, Vote on Accounts (d) of two years at a time
(c) Discussion on Budget, Vote on Account, Finance 126. With regard to the powers of the Rajya Sabha, which one
Bill, Appropriation Bill among the following statements is not correct?
(d) Discussion on Budget, Appropriation Bill, Finance (a) A money bill cannot be introduced in the Rajya
Bill, Vote on Account Sabha

Downloaded From : www.EasyEngineering.net


Downloaded From : www.EasyEngineering.net
Union Government P-69

(b) The Rajya Sabha has no power either to reject or 133. Which one of the following statements about the duties of
amend a money bill Prime Minister is correct?
(c) The Rajya Sabha cannot discuss the Annual Financial (a) Is free to choose his minister only from among
Statement members of either House of the Parliament
(d) The Rajya Sabha has no power to vote on the (b) Can choose his cabinet after consulting the President
Demands for Grants of India.
127. In the Rajya Sabha, the states have been given seats (c) Has full discretion in the choice of persons who are
(a) in accordance with their population to serve as ministers in his cabinet
(b) equally (d) Has only limited power in the choice of his cabinet
(c) on the basis of population and economic position colleagues because of the discretionary powers
(d) on the basis of present economic status vested the President of India
128. The Speaker of the Lok Sabha may be removed from 134. If the Prime Minister of India belonged to the Upper
office by House of Parliament:
(a) the majority party in the house adopting a no-
(a) he will not be able to vote in his favour in the event
confidence motion
of a no-confidence motion
(b) a resolution passed by not less than half of the total
(b) he will not be able to speak on the budget in the
membership of the house


ww
(c) a resolution passed by at least two-thirds of the total
membership of the house
(d) a resolution passed by a majority of all the members


Lower House
(c) he can make statements only in the Upper House
(d) he was to become a member of the Lower House
of the house
w.E
129. Which one among the following committees of the
Parliament of India has no members of the Rajya Sabha?
within six months after being sworn in
135. Which one of the following statements about a Money
Bill is not correct?




a
(a) Public Accounts Committee
(b) Estimates Committee
(c) Public Undertakings Committee
syE
(d) Departmentally Related Standing Committee on


(a) A Money Bill can be tabled in either House of
Parliament
(b) The Speaker of Lok Sabha is the final authority to
decide whether a Bill is a Money Bill or not

ngi
Finance (c) The Rajya Sabha must return a Money Bill passed by
130. Besides representation, the Parliament of India is also a the Lok Sabha and send it for consideration within
deliberative body with diverse functions. 14 days


Which one among the following is not a function of the
Parliament of India?
(a) Ventilating the grievances of the people

nee
(d) The President cannot return a Money Bill to the Lok
Sabha for reconsideration
136. The Parliament can make any law for whole or any part


(b) Executing major policy decisions
(c) Holding the government accountable for its actions
and expenditure
(d) Amending the Constitution


r
of India for implementing international treaties

ing
(a) with the consent of all the States
(b) with the consent of the majority of States
131. Which of the following statements is not correct?
(a) A Money Bill shall not be introduced in the Council

(d) without the consent of any State
137. The term of the Lok Sabha: .ne
(c) with the consent of the States concerned


of States
(b) The Council of States has no power to reject or
amend a Money Bill
(c) the Council of Ministers is responsible to the House
of the People and not to the Council of States



t
(a) cannot be extended under any circumstances
(b) can be extended by six months at a time
(c) can be extended by one year at a time during the
proclamation of emergency
(d) The House of the People has special powers with (d) can be extended for two years at a time during the
respect to the State List compared to the Council of proclamation of emergency
States 138. Under which Article of the Indian Constitution did the
132. Who among the following have the right to vote in the President give his asset to the ordinance on electoral
elections to both the Lok Sabha and the Rajya Sabha? reforms when it was sent back to him by the Union
Cabinet without making any changes (in the year 2002) ?
(a) Elected members of the Lower House of the
(a) Article 121 (b) Article 122
Parliament
(c) Article 123 (d) Article 124
(b) Elected members of the Upper House of the
139. Which one of the following Articles of the Indian
Parliament Constitution provides that ‘It shall be the duty of the
(c) Elected members of the Upper House of the State Union to protect every State against external aggression
Legislature and internal disturbance’?
(d) Elected members of the Lower House of the State (a) Article 215 (b) Article 275
Legislature (c) Article 325 (d) Article 355

Downloaded From : www.EasyEngineering.net


Downloaded From : www.EasyEngineering.net
P-70 Union Government

140. Which one of the following Bills must be passed by each (c) There is no constitutional bar for a nominated
House of the Indian Parliamentary separately by special member to be appointed as a Union minister
majority? (d) A nominated member can vote both in the Presidential
(a) Ordinary Bill and Vice Presidential elections
(b) Money Bill 146. Which one of the following statements is not correct?
(c) Finance Bill (a) In the Lok Sabha, a no-confidence motion has to set
(d) Constitution Amendment Bill out the grounds on which it is based
141. With reference to Indian Parliament, which one of the (b) In the case of a no-confidence motion in Lok Sabha,
following is not correct? no conditions of admissibility have been laid down
(a) The Appropriation Bill must be passed by both the in the Rules
Houses of Parliament before it can be enacted into (c) A motion of no-confidence once admitted, has to be
law taken up within ten days of the leave being granted
(b) No money shall be withdrawn from the Consolidated (d) Rajya Sabha is not empowered to entertain a motion
Fund of India except under the appropriation made of no-confidence
by the Appropriation Act 147. The resolution for removing the Vice-President of India
can be moved in the:
(c) Finance Bill is required for proposing new taxes
(a) Lok Sabha alone

ww but no additional Bill/Act is required for making


changes in the rates of taxes which are already under
operation.



(b) Either House of Parliament
(c) Joint Sitting of Parliament
(d) Rajya Sabha alone

w.E
(d) No Money Bill can be introduced except on the
recommendation of the President
142. Which of the following special powers have been
148. The authorization for the withdrawal of funds from the


Consolidated Fund of India must come from
(a) The President of India


India?
a
conferred on the Rajya Sabha by the Constitution of

syE
(a) To change the existing territory of a State and to
change the name of a State



(b) The Parliament of India
(c) The Prime Minister of India
(d) The Union Finance Minister
149. In the Parliament of India, the purpose of an adjournment
(b) To pass a resolution empowering the Parliament to
ngi
make laws in the State List and to create one or more

motion is
(a) to allow a discussion on a definite matter of urgent
public importance

All India Services
(c) To amend the election procedure of the President
and to determine the pension of the President after


nee
(b) to let opposition members collect information from
the ministers
(c) to allow a reduction of specific amount in demand
his/her retirement
(d) To determine the functions of the Election
Commission and to determine the number of Election

r
for grant

ing
(d) to postpone the proceedings to check the inappropriate
or violent behaviour on the part of some members
Commissioners
143. The Parliament can make any law for the whole or any
part India for implementing international treaties:
appointment
.ne
150. The Prime Minister of India, at the time of his/her

(a) need not necessarily be a member of one of the






(a) with the consent of all the State
(b) with the consent of the majority of States
(c) with the consent of the States concerned
(d) without the consent of any State

t
Houses of the Parliament but must become a member
of one of the Houses within six months
(b) need not necessarily be a member of one of the
Houses of the Parliament but must become a member
144. In what way does the Indian Parliament exercise control of the Lok Sabha within six months
over the administration? (c) must be a member of one of the Houses of the
(a) Through Parliamentary Committees Parliament
(b) Through Consultative Committees in various (d) must be a member of the Lok Sabha
ministries 151. What will follow if a Money Bill is substantially amended
by the Rajya Sabha?
(c) By making the administrators send periodic reports
(a) The Lok Sabha may still proceed with the Bill,
(d) By compelling the executive to issue writs
accepting or not accepting the recommendations of
145. Which one of the following statements is correct?
the Rajya Sabha
(a) Only the Rajya Sabha and not the Lok Sabha can
(b) The Lok Sabha cannot consider the bill further
have nominated members (c) The Lok Sabha may send the Bill to the Rajya Sabha
(b) There is a constitutional provision for nominating for reconsideration
two members belonging to the Anglo-Indian (d) The President may call a joint sitting for passing the
community to the Rajya Sabha Bill

Downloaded From : www.EasyEngineering.net


Downloaded From : www.EasyEngineering.net
Union Government P-71

152. Certain Bills can not be introduced or proceeded with (b) assent or reject the Bill as soon as possible
unless the recommendation of the President is received. (c) return the Bill as soon as possible after the Bill is
presented to him with a message requesting the
However, no recommendation is required in some
House to reconsider the Bill
other cases. In which one of the following cases such (d) with hold his assent even if the Bill is passed again
recommendation is not required? [CSAT–2014-I] by the Houses
(a) For introduction of Bills and for moving amendments 154. The Annual Financial Statement of the Government of
India in respect of each financial year shall be presented
relating to financial matters
to the House on such day as the  [CSAT–2014-I]
(b) For introduction of a Bill relating to formation of (a) Speaker may direct
new states or of alternation of areas of existing states (b) President of India may direct
(c) For moving of an amendment making provision for (c) Parliament may decide
the reduction or abolition of any tax (d) Finance Minister may decide
(d) For introduction of a Bill or moving of an amendment 155. When a bill is referred to a joint sitting of both the Houses
affecting taxation in which states are interested of the Parliament, it has to be passed by [CSAT 2015-I]
153. After a Bill has been passed by the Houses of the (a) a simple majority of members present and voting

ww
Parliament, it is presented to the President who may
either give assent to the Bill or with hold his assent. The
President may [CSAT–2014-I]


(b) three-fourths majority of members present and
voting
(c) two-thirds majority of the Houses

w.E
(a) assent within six months
(d) absolute majority of the Houses

a syE
ngi
nee
r ing
.ne
t

Downloaded From : www.EasyEngineering.net


Downloaded From : www.EasyEngineering.net
P-72 Union Government

Exercise - 2
Which of the above is/are correct?
Statement Based MCQ
(a) 3 and 4 (b) 2, 3 and 4
1. ______ take/takes part in the election of the President of (c) 1, 2 and 3 (d) 1 and 2
India? 7. Which factor can restrict the power of Parliament in
1. All Members of Parliament India?
1. A written Constitution prescribing the scope of
2. Members of State Legislative Assemblies
operation
3. Elected members of State Legislative assemblies
2. Supreme Court can strike down unconstitutional
4. Elected members of State Legislative Councils.
laws passed by Parliament
Which of the above is/are correct? 3. Parliament is limited by the Fundamental Rights
(a) 1 and 3 (b) 4 only provided by the Constitution
(c) 1 only (d) 3 only Which of the following statements is/are correct?
2. _______ can be declared by the President on his own?



ww
1. Emergency due to armed rebellion
2. Financial Emergency
3. President’s Rule in a State



(a) 1, 2 and 3
(b) 1 and 3
(c) 2 only




(a) 1 only w.E
Which of the above is/are correct?

(c) All of the Above


(b) 3 only
(d) None of the Above
(d) None, as Parliament is sovereign
8. Which of the following are the principles on the basis of
which the parliamentary system of government in India
operates ?




a
3. President appoints as Prime Minister:

syE
1. the leader of the majority party in the Lok Sabha
2. anyone he thinks fit
3. the person who can win the confidence of the




1. Nominal executive head
2. Vice-President as the chairman of the upper house
3. Real executive authority with the council of ministers
4. Executive responsibility to the lower house


majority in Lok Sabha
4. Leader of the party with a majority in either Lok
Sabha or Rajya Sabha
ngi


Which of the following statements is/are correct?
(a) 1, 2 and 4
(c) 1, 3 and 4
(b) 1, 2 and 4
(d) 2, 3 and 4


Which of the following statements is/are correct?
(a) 1 only (b) 1 or 3

nee
9. Consider the following statements :
1. The Chairman of the Committee on Public Accounts
is appointed by the Speaker of the Lok Sabha.
(c) 3 or 4 (d) 1, 3 or 4
4. Consider the following statements about the office of


Attorney-General of India?
1. He is a member of the Cabinet.

r
2. The Committee on Public Accounts comprises

ing
Members of Lok Sabha, Members of Rajya Sabha
and a few eminent persons of industry and trade.
Which of the statements given above is/are correct?


2. He can address either House of Parliament and vote.
3. He must have qualification of a judge of the Supreme


(a) 1 only
(c) Both 1 and 2
.ne
(b) 2 only
(d) Neither 1 nor 2




Court.
4. Salary of Attorney General is fixed by Parliament.
Which of the following statements is/are correct?
(a) 2 and 4 (b) 1, 2 and 4

following statements :
t
10. With reference to Union Government, consider the

1. The Ministries/Departments of the Government of


India are created by the Prime Minister on the advice
of the Cabinet Secretary.
(c) 3 only (d) 3 and 4 2. Each of the Ministries is assigned to a Minister by
5. Which statements about Financial Bill is CORRECT? the President of India on the advice of the Prime
1. It is same as a money bill Minister.
2. It can be introduced only in the Lok Sabha Which of the statements given above is/are correct ?
3. It can be amended by the Rajya Sabha. (a) 1 only (b) 2 only
(c) Both 1 and 2 (d) Neither 1 nor 2
4. It can only be introduced agreement of the President
11. The correct statements about calling attention notice are :
Which of the above is/are correct? 1. It is a device of calling the attention of a minister to
(a) 1 only (b) 2 and 4 a matter of urgent public importance.
(c) 2, 3 and 4 (d) 3 and 4 2. Its main purpose is to seek an authoritative statement
6. Both Houses of Parliament enjoy equal power in all from the minister.
spheres except: 3. It does not involve any censure against government.
1. financial matters 4. It is an Indian innovation in the parliamentary
2. responsibility of the Council of Ministers procedure since 1952.
3. amendment procedure 5. It is not mentioned in the Rules of Business and
4. presidential election Procedure.

Downloaded From : www.EasyEngineering.net


Downloaded From : www.EasyEngineering.net
Union Government P-73

Which of the following statements is/are correct? 3. He shall vacate office on attaining the age of 60
(a) 1, 2, 3 and 4 (b) 4 and 5 years.
(c) 1, 2, 3 and 5 (d) 1, 2 and 3 4. He can be removed by the President on his own.
12. Which of the following statements are true of Adjournment 5. He is responsible for maintaining the accounts of
Motion? Central and state governments.
1. It is an extraordinary procedure which sets aside the Which of the following statements is/are correct?
normal business of the House.
(a) 1,4 and 5 (b) 2,3 and 4
2. Its main object is to draw the attention of the House
(c) 1,2,3,4 and 5 (d) 3,4 and 5
to a recent matter of urgent public importance.
3. The Rajya Sabha can make use of this procedure. 17. Which of the following are the functions of the Public
4. It must be supported by not less than 50 members for Accounts Committee of Parliament?
introduction. 1. To examine, in the light of CAG’s report, the
5. It involves an element of censure against government. accounts showing the appropriation of sums granted
Which of the following statements is/are correct? by the Parliament.
(a) 1, 2, 4 and 5 (b) 2, 3 and 5 2. To examine, in the light of CAG’s report, the
(c) 2, 3 and 4 (d) 1, 2 and 4 statement of accounts of state corporations, trading
13. In parliamentary countries, like India, the legislative and manufacturing projects except of those as are

ww
control over administration is considerably reduced and
restricted in effectiveness due to which of the following
reasons?

allotted to the committee on public undertakings.
3. To examine the statement of accounts of autonomous
and semi-autonomous bodies, the audit of which is



w.E
1. The expansion in the volume and variety of
administrative work.
2. Frequent use of Guillitone.
3. The large size of the legislature

conducted by the CAG.
4. To examine if any money has been spent on any
service during a financial year in excess of the




a
4. The members of the legislature are laymen.

syE
5. The financial committees do post mortem work.
Which of the following statements is/are correct?
(a) 1, 2 and 5 (b) 2, 3 and 4



amount granted by house of people for that purpose.
Which of the following statements is/are correct?
(a) 1, 2 and 4
(c) 1, 2 and 3
(b) 1, 2, 3 and 4
(d) 1, 3 and 4

14.
(c) 2, 3, 4 and 5 (d) 1, 2, 3, 4 and 5
Which of the following documents are presented to the
legislature along with the budget? ngi 18.


The Indian President’s veto power is a combination of:
1. Pocket veto 2. Absolute veto
3. Suspensive veto 4. Qualified veto



1. An explanatory memorandum on the budget
2. A summary of demands for grants
3. An Appropriation Bill



nee
Which of the above is/are correct?
(a) 2 and 3
(c) 2,3 and 4
(b) 1, 3 and 4
(d) 1,2 and 3



4. A Finance Bill
5. The economic survey
Which of the above is/are correct?
19.


r ing
The correct statements regarding the difference between
the pardoning powers of President and Governor are:
1. The Governor can pardon sentences inflicted by


15.
(a) 1,3 and 5
(c) 2,3 and 5
(b) 1,2 and 3
(d) 1,2, 3 and 4
Which of the following statements are incorrect?

.ne
court martial while the President cannot.
2. The President can pardon death sentence while


1. Appropriation Bill cannot be amended while the
Finance Bill can be amended.
2. Finance Bill cannot be amended while Appropriation
Bill can be amended.


Governor cannot.

President cannot. t
3. The Governor can pardon death sentence while the

4. The President can pardon sentences inflicted by


3. Same procedure governs both the Appropriation Bill court martial while the Governor cannot.
and the Finance Bill. Which of the following statements is/are correct?
4. Appropriation Bill and the Finance Bill are governed (a) 1 and 2 (b) 2 and 4
by different procedures. (c) 1 and 3 (d) 3 and 4
5. Appropriation Bill cannot be rejected by the Rajya 20. Consider the following statements about the Attorney
Sabha while Finance Bill can be rejected by it. General of India:
Which of the following statements is/are correct? 1. He is appointed by the President of India
(a) 2 and 4 (b) 2, 4 and 5 2. He must have the same qualifications as required for
(c) 1 and 3 (d) 1, 3 and 5 a judge of the Supreme Court.
16. Which of the following are not correct about CAG of 3. He must be a member of either House of Parliament.
India?
4. He can be removed by impeachment by Parliament.
1. He is appointed by the President for a period of five
Which of these statements is correct?
years.
2. His salary and conditions of service are determined (a) 1 and 2 (b) 1 and 3
by President. (c) 2, 3 and 4 (d) 3 and 4

Downloaded From : www.EasyEngineering.net


Downloaded From : www.EasyEngineering.net
P-74 Union Government

21. Which of the following are included in Article 78 of Which of the statements given above is/are correct?
the Indian Constitution, defining the duties of Prime (a) 1, 3 and 4 (b) 2 only
Minister? (c) 2 and 3 (d) 1, 2 and 3
1. To communicate to the President all decisions of the 25. With reference to Indian public finance, consider the
council of ministers relating to the administration of following statements:
the affairs of the union and proposals for legislation.
1. Disbursements from Public Accounts of India are
2. To take prior presidential sanction for the budget
subject to the Vote of the Parliament
before submitting it in the Parliament
3. To furnish the information called for by the President 2. The Indian Constitution provides for the establishment
regarding administration of affairs of the union of a Consolidated Fund, a Public Account and a
4. If the President so requires, to submit for Contingency Fund for each State
consideration of the council of ministers a matter 3. Appropriations and disbursements under the Railway
on which a minister has taken a decision without Budget are subject to the same form of parliamentary
submitting the same for consideration by the council control as other appropriations and disbursements
beforehand. Which of the statements given above are correct?
Which of the following statements is/are correct? (a) 1 and 2 (b) 2 and 3
(a) 1 and 2 (b) 1, 3 and 4 (c) 1 and 3 (d) 1, 2 and 3

ww
(c) 2 and 4 (d) 1 and 3
22. Which of the following corporations, for the purpose of
auditing, are kept completely out of the purview of CAG
26. With reference to Indian Parliament, consider the


following statements :
1. A member of the Lok Sabha cannot be arrested by



of India ?

w.E
1. Industrial Finance Corporation
2. Food Corporation of India
3. Central Warehousing Corporation

police under any case when the Parliament is in
session.
2. Members of Indian Parliament have the privilege of




a
4. Life Insurance Corporation of India
5. Reserve Bank of India
syE
Which of the following statements is/are correct?
(a) 1, 2 and 5 (b) 1, 2, 4 and 5


exemption from attendance as witnesses in the law
courts.
Which of these statements is/are correct ?
(a) 1 only (b) 2 only
(c) 2, 3 and 4 (d) 2, 4 and 5
23. According to the Constitution of India, it is the duty ngi   (c) Both 1 and 2 (d) Neither 1 and 2
27. Consider the following statements:
1. The President of India cannot appoint a person as

nee
of the President of India to cause to be laid before the
Parliament which of the following? Prime Minister if he/she is not a member of either
1. The Recommendations of the Union Finance Lok Sabha or Rajya Sabha.
2. The candidate for the office of Prime Minster must


Commission
2. The Report of the Public Accounts Committee
3. The Report of the Comptroller and Auditor General
4. The Report of the National Commission for
r ing
have the support of the majority members of both
Lok Sabha and Rajya Sabha.
Which of the statements given above is/are correct ?



Scheduled Castes
Select the correct answer using the codes given below :
(a) 1 only (b) 2 and 4


(a) 1 only
(c) Both 1 and 2
.ne
(b) 2 only
(d) Neither 1 nor 2
28. Which among the following statements with respect to the
(c) 1, 3 and 4 (d) 1, 2, 3 and 4
24. In India, other than ensuring that public funds are
used efficiently and for intended purpose, what is the
importance of the office of the Comptroller and Auditor


t
Comptroller and Auditor General of India is/are correct?
1. The procedure and grounds for his removal from the
office are the same as of a Judge of Supreme Court.
2. He prescribes the form in which accounts of the
General (CAG)? Union and the States are to be kept.
1. CAG exercises exchequer control on behalf of the Select the correct answer using the code given below
Parliament when the President of India declares (a) 1 only (b) 2 only
national emergency/financial emergency. (c) Both 1 and 2 (d) Neither 1 nor 2
2. CAG reports on the execution of projects or 29. Which of the following statements about a parliamentary
programmes by the ministries are discussed by the committee in India are correct?
Public Accounts Committee. 1. It is appointed or elected by the house or appointed
3. Information from CAG reports can be used by by the speaker/chairman.
investigating agencies to press charges against those 2. It works under the direction of the speaker/chairman
who have violated the law while managing public of the house.
finances. 3. It presents its report to the president.
4. While dealing with the audit and accounting of Select the correct answer using the codes given below:
government companies, CAG has certain judicial (a) 1 and 2 only (b) 2 and 3 only
powers for prosecuting those who violate the law. (c) 1 and 3 only (d) 1, 2 and 3

Downloaded From : www.EasyEngineering.net


Downloaded From : www.EasyEngineering.net
Union Government P-75

30. Which of the following features of the Indian Government 35. Consider the following statements regarding ‘No
system are the essential features of the parliamentary confidence motion’
Government system? 1. Only a motion expressing want of confidence in the
1. Presence of nominal and real executives council of ministers as a whole is admitted and one
2. Membership of the ministers in the legislature expressing lack of confidence in an individual min-
3. Separation of powers between the Union and State ister is out of order
government 2. A no confidence motion needs to set out grounds on
4. Independent judiciary system which it is based.
Select the correct answer using the codes given below: 3. Any no confidence motion once moved can’t be
(a) 1 and 2 only withdrawn
(b) 1, 2 and 3 only 4. Rajya Sabha is not empowered to entertain a motion
(c) 2 and 3 only of no confidence
(d) 1, 2, 3 and 4 Which of the following given above are not correct?
31. Which of the following committee(s) is/are not exclusively (a) 1 and 2 only (b) 1, 2, and 3 only
the committee of the lower House of the Parliament of (c) 2 and 3 only (d) 1, 2, 3 and 4
India? 36. When the House of people clearly and conclusively deter-
1. Estimate committee mines that the government does not command its support,


ww
2. Committee on public undertaking
3. Committee on the welfare of Scheduled Castes and
STs
the government has to resign. By which of the ways
parliamentary confidence in the government may be ex-
pressed by the House of People?


w.E
4. Committee on Empowerment of women
Select the correct answer using the codes given below:
(a) 1 and 2 only (b) 2, 3 and 4 only



1. Defeating the government on a major issue of poli-
cy.
2. Passing an adjournment motion
3. Defeating the government on finance issues
(c) 1 only

a (d) 2 and 4

syE
32. Though the Rajya Sabha and Lok Sabha are constitutent
part of Parliament, on some subject they have unequal
powers which of the following matters depict the



4. Passing a motion of no confidence in the council of
ministers.
(a) 1 and 3 only
(c) 2 and 3 only
(b) 2, 3 and 4 only
(d) 1, 2, 3 and 4

ngi
difference of powers between these two houses?
1. No confidence motion 37. With reference to Union Government consider the
2. Power to vote on Demand for Grants following statements:

nee
3. Impeachment of judges of the High Court 1. The ministries/Departments of the government of
4. Passing of laws in the national interest on the subject India are created by the PM on the advice of the
enumerated in state list cabinet secretary.




5. Creation of all India services.
Select the correct answer by using the codes given below:
(a) 1, 3 and 4 only (b) 2, 3 and 5 only
(c) 1, 2, 4 and 5 only (d) 1, 2 and 5 only



r
2. Each of the ministries is assigned to a minister by the

ing
president of India on the advice of the PM.
Which of the statements given above is/are correct?
(a) 1 only (b) 2 only
33. Which among the following have the right to vote in the
elections to both the Lok Sabha and the Rajya Sabha?
(c) Both 1 and 2
.ne
(d) Neither 1 nor 2
38. Consider the following facts about Comptroller and
auditor general of India, and choose the correct answer:


1. Elected members of the Lower House of the
Parliament
2. Elected members of the Upper House of the
Parliament


1. CAG is a constitutional Body.
t
2. CAG has absolute power to audit accounts of all the
functionaries of Central and States’ governments as
well as of private corporate bodies.
3. Elected members of the Upper House of the State
Legislature. (a) Only 1 is correct
4. Elected members of the Lower House of the State (b) Only 2 is correct
Legislature. (c) 1 and 2 both are correct
(a) 1 and 2 only (b) 1, 2 and 3 only (d) Neither 1 nor 2 is correct
(c) 3 and 4 only (d) 4 only 39. Which of the following statements are correct about
34. Consider the following statements: Indian Government?
1. The chairman of the committee on public accounts is 1. Rajya Sabha represents the local interests of the
appointed by the speakers of the Lok sabha States.
2. The committee on public Accounts comprises 2. A member of Rajya Sabha must be a resident of the
members of Lok Sabha, members of Rajya sabha State from which he is elected.
and a few eminent persons of industry and trade 3. Number of seats allotted to a State has to be
Which of the statements given above is/are correct? proportionate to its population.
(a) 1 only (b) 2 only 4. The term of a member of Rajya Sabha is same as that
(c) Both 1 and 2 (d) Neither 1 nor 2 of Senator in the US.

Downloaded From : www.EasyEngineering.net


Downloaded From : www.EasyEngineering.net
P-76 Union Government

Codes: 3. Public account of States.


(a) 2, 3 and 4 (b) 1, 2 and 3 4. Receipts and expenditures of PPP.
(c) 1, 3 and 4 (d) 1 and 2 5. Local bodies.
40. Which of the following situation will bring about the (a) 1, 2 & 3 only (b) 1, 2 & 5 only
collapse of the council of ministers of a state. (c) 1, 3, 4 & 5 only (d) All
1. Resignation by Chief Minister. 47. Which of the following statement(s) is/are true with
2. Death of Chief Minister. regard to the Attorney General of India:
(a) 1 only (b) 2 only 1. He/she is the highest law officer of the country.
(c) Both (d) None 2. He/she can be removed by the President on same
41. Which of the following statements are correct: grounds and in the same manner as a judge of the
1. President can nominate 2 members from the Anglo SC.
– Indian community if not adequately represented in (a) 1 only (b) 2 only
LS. (c) Both (d) None
2. Governor can nominate 2 members from the Anglo 48. Which of the following statement/s is/are true with regard
– Indian community if not adequately represented in to the Attorney General of India:
LA. 1. He/she can be a member of a Parliamentary committee.
(a) 1 only (b) 2 only 2. He/she is debarred from private legal practice.

42.

ww
(c) Both. (d) None
Corrects statement/s with regard to UTs is/are:
1. The parliament can make laws on any subject of
(a) 1 only
(c) Both
(b) 2 only
(d) None
49. The correct statement/s with regard to Ordinance making

w.E
three lists for the UT except Delhi and Puducherry.
2. The Lt. Governor of Delhi is not empowered to
promulgate ordinances.

power of President is/are -
1. The President cannot promulgate an ordinance to
amend tax laws.


43.
(a) 1 only
(c) Both
a (b) 2 only
(d) None
syE
Which of the following statement/s is/are correct with
regard to Joint State Public Service Commission (JSPS(c):


2. The President cannot promulgate an ordinance to
amend the constitution.
(a) 1 only
(c) Both
(b) 2 only
(d) None


1. President can provide for a JSPSC on the request of
the state legislatures concerned.
2. JSPSC will be considered as a constitutional body ngi 50. The ‘Council of Ministers’ does not consist of:


1. Deputy Ministers.
2. Parliamentary Secretaries.



with all the powers of a SPSC.
(a) 1 only
(c) Both
(b) 2 only
(d) None



nee
3. Deputy Chairman – Planning Commission.
(a) 1, 2 & 3
(c) 3 only
(b) 2 only
(d) None of these
44.


Which of the following are quasi judicial bodies:
1. Finance Commission.
2. Central Vigilance Commission
r
are elected by:
ing
51. The Representatives of states & UT in the Rajya Sabha

1. The members of the State Legislative Assembly





3. National Human Rights Commission.
4. Central Information Commission.
5. Competition Commission of India.

only.

.ne
2. The elected members of the State Legislative
Assembly only.




6. Union Public Service Commission.
7. National Commission for SCs.
(a) All except 1, 2 & 6
(b) All except 2, 4 & 6



t
3. The system of proportional representation by single
transferrable vote.
4. The system of proportional representation by List.
(a) 1 & 3 (b) 1 & 4
(c) All except 6
(c) 2 & 3 (d) 2 & 4
(d) None of the above options are correct
52. Which of the following criteria is laid down by the
45. Which of the following official(s) is/are appointed by the
constitution for a person to be chosen a member of
President by Warrant under his hand and seal.
parliament:
1. CAG.
1. If a candidate is to contest a seat reserved for SC /
2. Chairperson of National Commission of SCs.
3. Attorney General of India. ST, he must be a member of a SC / ST in any state
4. CVC or Union Territory.
(a) 1, 2 & 4 only (b) 2 & 3 only 2. He/she must not have been punished for preaching
(c) 1 & 3 only (d) 1, 2 & 3 and practicing social crimes such as untouchability,
46. Which of the following funds/authorities/bodies can be dowry & sati.
audited by CAG: 3. He/she must not have any interest in government
1. Consolidated funds of India, States & UTs having contracts, works or services.
legislative assembly. (a) 1 only (b) 2 & 3 only
2. Contingency funds of India. (c) 1, 2, & 3 (d) None of these

Downloaded From : www.EasyEngineering.net


Downloaded From : www.EasyEngineering.net
Union Government P-77

53. Which of the following statements are correct. 2. Salaries, allowances and pensions of the Judges of
1. If a MLA is elected to be a MP, his seat in parliament the Supreme Court & high court.
becomes vacant if he does not resign his seat in the (a) 1 only (b) 2 only
state legislature within 14 days. (c) Both (d) None
2. If a person is elected to two seats in a house, he 61. Which of the following officials take the Oath that has
should exercise his option for one. Otherwise both the following lines: “To preserve protect and defend the
seats become vacant. constitution”
(a) 1 only (b) 2 only 1. President
(c) Both (d) None 2. Governor
54. When the Lok Sabha is Dissolved: 3. Chief Justice of India
1. A bill passed by Lok Sabha pending in Rajya Sabha (a) 1 only (b) 1 & 2 only
does not lapse. (c) 1, 2 & 3 (d) None of the above
2. A bill pending in Rajya Sabha & not passed by Lok 62. Consider the following statements : Attorney General of
Sabha does not lapse. India can
(a) 1 only (b) 2 only 1. take part in the proceedings of the Lok Sabha
(c) Both (d) None 2. be a member of a committee of the Lok Sabha
55. Consider the following statements with regard to Calling 3. speak in the Lok Sabha

ww
Attention motion and Zero Hour:
1. Both are Indian innovation in the parliamentary pro-
cedure.



4. vote in the Lok Sabha
Which of the statements given above is/are correct?
(a) 1 only (b) 2 and 4



w.E
2. Both are not mentioned in the rules of procedure.
3. Only Zero hour is Indian innovation.
4. Only Zero Hour is not mentioned in the rules of
(c) 1, 2 and 3 (d) 1 and 3 only
63. In the context of India, which of the following principles
is/are implied institutionally in the parliamentary




procedure.

(a) 1 & 2 only


(c) 2 & 3 only
a
The correct statements is/are:
syE
(b) 3 & 4 only
(d) 1 & 4 only


government?
1. Members of the Cabinet are Members of the
Parliament.
2. Ministers hold the office till they enjoy confidence in
56. Which of the following motion/s if passed leads to the


defeat of the government:
1. Censure Motion. ngi

the Parliament.
3. Cabinet is headed by the Head of the State.
Select the correct answer using the codes given below.


2. Cut Motion.
(a) 1 only
(c) Both
(b) 2 only
(d) None

nee
(a) 1 and 2 only
(c) 2 and 3 only
(b) 3 only
(d) 1, 2 and 3
64. Consider the following statements:
57. The decision whether a bill is money bill or not is decided


by the speaker, this decision cannot be questioned by:
1. Any court of law.


r ing
1. The Council of Ministers in the Centre shall be
collectively responsible to the Parliament.
2. The Union Ministers shall hold the office during the



2. Lok Sabha.
3. President of India.
(a) 1 & 2 only (b) 2 & 3 only

pleasure of the President of India.

.ne
3. The Prime Minister shall communicate to the
President about the proposals for legislation.


(c) 2 only (d) None of these
58. Which of the following condition may lead to the
resignation of the government:
1. Defeat of an ordinary bill introduced by a minister.



(a) 1 only
(c) 1 and 3 only
(b) 2 and 3 only
(d) 1, 2 and 3
65. Consider the following statements:
t
Which of the statements given above is/are correct?

2. Defeat of a money bill. 1. The Chairman and the Deputy Chairman of the
(a) 1 only (b) 2 only Rajya Sabha are not the members of that House.
(c) Both (d) None 2. While the nominated members of the two Houses of
59. Which of the following is/are correct: the Parliament have no voting right in the presidential
1. A Money bill can be introduced only in Lok Sabha. election, they have the right to vote in the election of
2. A Money bill is also a Financial Bill. the Vice President.
3. All Financial bills can be introduced only on the Which of the statements given above is/are correct?
recommendation of the President. (a) 1 only (b) 2 only
(a) 1 only (b) 1 & 2 only (c) Both 1 and 2 (d) Neither 1 nor 2
(c) 1, 2 & 3 (d) None of these 66. Consider the following statements :
60. Which of the following Expenditure is/are the expenditure 1. The President shall make rules for the more
‘Charged’ on the Consolidated fund of India: convenient transaction of the business of the
1. Emoluments and allowances of President & Vice Government of India, and for the allocation among
President. Ministers of the said business.

Downloaded From : www.EasyEngineering.net


Downloaded From : www.EasyEngineering.net
P-78 Union Government

2. All executive actions of the Government of India 72. Consider the following statements:
shall be expressed to be taken in the name of the 1. Salary and allowances of the Speaker of Lok Sabha
Prime Minister. are charged on the Consolidated Fund of India.
Which of the statements given above is/are correct? 2. In the Warrant of Precedence, the Speaker of Lok
(a) 1 only (b) 2 only Sabha ranks higher than all the Union Cabinet
(c) Both 1 and 2 (d) Neither 1 nor 2 Ministers other than Prime Minister.
67. Consider the following statements : Which of the statement given above is/are correct?
A Constitutional Government is one which (a) 1 only (b) 2 only
1. places effective restrictions on individual liberty in (c) Both 1 and 2 (d) Neither 1 nor 2
the interest of State Authority 73. Which one of the following statements is/are correct with
2. places effective restrictions on the Authority of the regard to the Vice-President of India?
State in the interest of individual liberty 1. He must be a member of Parliament.
Which of the statements given above is/are correct? 2. He is elected by proportional representation.
3. He is Ex-officio Chairman of the Rajya Sabha.
(a) 1 only (b) 2 only
Select the correct answer unsing the code given below:
(c) Both 1 and 2 (d) Neither 1 nor 2
(a) 1 only (b) 1 and 3
68. Which of the following is/are the function/functions of
(c) 2 and 3 (d) 1 and 2
the Cabinet Secretariat?



ww
1. Preparation of agenda for Cabinet Meetings
2. Secretarial assistance to Cabinet Committees
3. Allocation of financial resources to the Ministries
74. In which among the following cases, the joint session of


both the Houses of Parliament can be summoned?
1. To amend the Constitution.



(a) 1 only w.E
Select the correct answer using the code given below.

(c) 1 and 2 only


(b) 2 and 3 only
(d) 1, 2 and 3


2. When a Bill has been pending with one House for
more than six months after it was passed by the
other
3. When both the Houses disagree on the amendments


a
69. Consider the following statements:

syE
The parliamentary Committee on public accounts
1. consists of not more than 25 Members of the Lok
Sabha


to be made in a Bill.
4. When a bill is passed by one House and is rejected
by the other.
Select the correct answer using the codes given below.


2. scrutinizes appropriation and finance accounts of the
Government
3. examines the report of the Comptroller and Auditor ngi

(a) 1, 2 and 3
(c) 2 and 3 only
(b) 2, 3 and 4
(d) 1 and 4
75. Consider the following statements



General of India.
Which of the statements given above is/are correct?
(a) 1 only (b) 2 and 3 only


nee
1. A Money Bill cannot be introduced in the Council of
States.
2. The Council of States cannot reject a Money Bill nor
(c) 3 only (d) 1, 2 and 3
70. Consider the following statements:
1. The President of India cannot appoint a person as


r
amend it.

ing
Which of the statements given above is/are correct?
(a) Only 1 (b) Only 2


Prime Minister if he/she is not a member of either
Lok Sabha or Rajya Sabha.
2. The candidate for the office of Prime Minster must
(c) Both 1 and 2

.ne
(d) Neither 1 nor 2
76. Which one of the following statements is not correct?
(a) The Vice-President of India holds office for a period



have the support of the majority members of both
Lok Sabha and Rajya Sabha.
Which of the statements given above is/are correct ?
(a) 1 only (b) 2 only

of five years
t
(b) The Vice-President of India can be removed by a
simple majority of votes passed in the Rajya Sabha
only
(c) The Vice-President of India continues to be in office
(c) Both 1 and 2 (d) Neither 1 nor 2
even after the expiry of his term till his successor
71. Consider the following statements:
takes over
1. It is on the advice of the Speaker of Lok Sabha
(d) The Supreme Court of India has to look into all
that the President of India summons and prorogues
disputes with regard to the election of the Vice-
Parliament and dissolves Lok Sabha . President of India
2. The resignation of the Prime Minister means the 77. Consider the following statements:
resignation of the entire Council of Ministers. 1. The Annual Appropriation Bill is passed by the Lok
3. A vote of confidence against one Minister. in Lok Sabha in the same manner as any other Bill.
Sabha means the vote of confidence against the 2. An amendment to the Constitution of India can be
entire Council of Ministers. initiated by an introduction of a Bill in either Lok
Which of the following statements given above are Sabha or Rajya Sabha.
correct? Which of the statements given above is/are correct?
(a) 1 and 2 only (b) 2 and 3 only (a) Only 1 (b) Only 2
(c) 1 and 3 only (d) 1, 2 and 3 (c) Both 1 and 2 (d) Neither 1 nor 2

Downloaded From : www.EasyEngineering.net


Downloaded From : www.EasyEngineering.net
Union Government P-79

78. Consider the following statements: Act, 1985 as amended in 2003, a legislator attracts
1. When the Vice-President of India acts as the disqualification under the 10th Schedule if
President of India, he performs simultaneously the 1. he voluntarily gives up the membership of the party
functions of the Chairman of Rajya Sabha. on whose ticket he was elected.
2. The President, of India can promulgate ordinances 2. he votes or abstains from voting contrary to any
at any time except when both Houses of Parliament direction issued by his political party.
are in session. 3. as a result of split, less than one third of the members
Which of the statements given above is/are correct? formed a new group or party in the house.
(a) Only 1 (b) Only 2 4. a member who has been elected as a independent
(c) Both 1 and 2 (d) Neither 1 nor 2 member joins any political party.
79. Consider the following statements: Select the correct answer using the codes given below.
1. The Union Executive consists of the President and (a) 2 and 3 (b) 1, 2 and 4
the Council of Ministers with the Prime Minister as (c) 1 and 3 (d) All of these
the head. 85. Under which of the following conditions security deposits
2. The President may, by writing under his hand of a candidate contesting for a Lok Sabha seat is returned
addressed to the Vice-President, resign his office. to him/her?
3. Executive power of the Union is vested in the Prime 1. The nomination made by the candidate if found to be


ww Minister.
Which of the statements given above is/are correct?
(a) 1 and 3 (b) 2 and 3

invalid.
2. The candidate has withdrawn his/her nomination
even though it is found valid.
(c) 1, 2 and 3

w.E (d) Only 2


80. Consider the following statements:
1. Ministry of Parliamentary Affairs constitute
Consultative Committees of Members of both the



3. The candidate lost the polls but secured l/6th of the
total number of valid votes polled in that election.
Select the correct answer using the codes given below
(a) 1 and 2 (b) Only 3

a
Houses of Parliament.

syE
2. The main purpose of these Committees are to
provide a forum for formal discussions between the
Government and Members of Parliament on polices

86.

(c) 2 and 3 (d) All of these
Consider the following statements
1. The total elective membership of the Lok Sabha
is distributed among the States on the basis of the


and programmes of the Government.
Which of the statements given above is/are correct? ngi
population and the area of the State.
2. The 84th Amendment Act of the Constitution of India

nee
(a) Only 1 (b) Only 2 lifted the freeze on the delimitation of constituencies
(c) Both 1 and 2 (d) Neither 1 nor 2 imposed by the 42nd Amendment.
81. Consider the following statements on Parliamentary Which of the statements given above is/are correct?


Committees:
1. Members of the Rajya Sabha are not associated
with the Committees on Public Accounts and Public
Undertakings.


87.

r
(a) Only 1
(c) Both 1 and 2

ing
(b) Only 2
(d) Neither 1 nor 2
Which of the following statements is/are correct?
1. A registered voter in India can contest an election to


2. Members of the Committee on Estimates are drawn
from both the Lok Sabha and the Rajya Sabha.
Which of the statements given above islare correct?

.ne
Lok Sabha from any constituency in India.
2. As per the Representation of the People Act, 1951,
if a person is convicted of any offence and sentenced


(a) Only 1
(c) Both 1 and 2
(b) Only 2
(d) Neither 1 nor 2
82. Consider the following statements:
1. The Ministries Departments of the Union Government


disqualified to contest election. t
to an imprisonment of 2 years or more, he will be

Select the correct answer using the codes given below


(a) Only 1 (b) Only 2
are created by the Prime Minister. (c) Both 1 and 2 (d) Neither I nor 2
2. The Cabinet Secretary is the Ex-officio Chairman of 88. Which of the following Committees are the Committees
the Civil Services Board. of Parliament?
Which of the statement given above is/are correct? 1. Public Accounts Committee
(a) Only 1 (b) Only 2 2. Estimates Committee
(c) Both 1 and 2 (d) Neither 1 nor 2 3. Committee on Public Undertakings
83. Consider the following Vice-Presidents of India: Select the correct answer using the code given below
1. V.V Giri 2. M Hidayatullah (a) 1 and 2 (b) 1 and 3
3. BD Jatti 4. GS Pathak (c) 2 and 3 (d) 1, 2 and 3
Which one of the following is the correct chronology of 89. Consider the following statements about the powers of
their tenures? the President of India
(a) 1, 4, 3, 2 (b) 2, 1, 3, 4 1. The President can direct that any matter on which
(c) 3, 2, 1,4 (d) 4, 1,3, 2 decision has been taken by a Minister should be
84. According to the Constitution (Fifty Second Amendment placed before the Council of Ministers.

Downloaded From : www.EasyEngineering.net


Downloaded From : www.EasyEngineering.net
P-80 Union Government

2. The President can call all information relating to 93. Which of the following are/is stated in the Constitution of
proposals for legislation. India?
3. The President has the right to address and send 1. The President shall not be a member of either House
messages to either House of the Parliament. of Parliament
4. All decisions of the Council of Ministers relating 2. The Parliament shall consist of the President and two
to the administration of the Union must be Houses
communicated to the President. Choose the correct answer from the codes given below:
Which of the statements given above are correct? (a) Neither 1 nor 2 (b) Both 1 and 2
(a) 1,2 and 3 (b) 1 and 3 (c) Only 1 (d) Only 2
(c) 2 and 4 (d) 1,2, 3 and 4 94. Consider the following statements:
90. Consider the following statements relating to the 1. The joint sitting of the two houses of the Parliament
procedure of the election of the Speaker and the Deputy in India is sanctioned under Article 108 of the
Speaker of the Lok Sabha Constitution
1. The election of a Speaker shall be held on such date as 2. The first joint sitting of Lok Sabha and Rajya Sabha
the Prime Minister may fix and the Secretary General was held in the year 1961
shall send to every member. notice of this date. 3. The second joint sitting of the two Houses of Indian
2. The election of a Deputy Speaker shall be held on Parliament was held to pass the Banking Service

ww
such date as the Speaker may fix and the Secretary
General shall send to every member notice of this
date.


Commission (Repeal) Bill
Which of these statements is correct?
(a) 1 and 2 (b) 2 and 3

w.E
3. At anytime before noon on the day preceding the
date so fixed, any member may give notice in writing
of a motion that another member be chosen as the
Deputy Speaker of the House.
(c) 1 and 3

following statements:
(d) 1, 2 and 3
95. With reference to the Union Government, consider the




(a) 2 and 3
(c) 1 and 3
a (b) Only 2
(d) All of thesesyE
Which of the statement(s) given above is/are correct?

91. Which of the following pairs of constitutional authority



1. The Constitution of India provides that all Cabinet
Ministers shall be compulsorily be a sitting members
of Lok Sabha only.
2. The Union Cabinet Secretariat operates under the


and procedure of appointment is/are correctly matched?
1. President : Elected by an electoral college consisting ngi
direction of the Ministry of Parliamentary Affairs.
Which of the statements given above is/are correct?
(a) 1 only (b) 2 only

nee
of elected MLAs and MPs
2. Vice-President: Elected by an electoral college (c) Both 1 and 2 (d) Neither 1 nor 2
consisting of MLAs and MPs 96. According to the Constitution of India, it is the duty
of the President of India to cause to be laid before the
3. Speaker : The House of People chooses after its first
sitting
Select the correct answer using the codes given
below
r ing
Parliament which of the following?
1. The Recommendations of the Union Finance
Commission


92.
(a) 1, 2 and 3
(c) 1 and 3
(b) Only 1
(d) 2 and 3


.ne
2. The Report of the Public Accounts Committee
3. The Report of the Comptroller and Auditor General
4. The Report of the National Commission for
Council of states
Not more than 250
members
House of the People
Not more than 552 members



Scheduled Castes

(a) 1 only
(c) 1, 3 and 4 only
t
Select the correct answer using the codes given below :
(b) 2 and 4 only
(d) 1, 2, 3 and 4
“ Not more than 238 Not more than 530 97. Consider the following statements:
representatives of States representative of states plus not 1. While members of the Rajya Sabha are associated
and Union Territories” more than 2 nominated Anglo- with Committees on Public Accounts and Public
Indians plus X Undertakings, members of Committee on Estimates
are drawn entirely from the Lok Sabha.
Which one of the following will fit in the place marked ‘X’?
2. The Ministry of Parliamentary Affairs works under
(a) Ministers who are not members of Parliament but
the overall direction of Cabinet Committee on
who have to get themselves elected to either House
Parliamentary Affairs.
of Parliament within six months after assuming
3. The Minister of Parliamentary Affairs nominates
office
Members of Parliament on Committees, Councils,
(b) Not more than 20 nominated members
Board and Commissions etc. set up by the
(c) Not more than 20 representative of Union Territories
Government of India in the various ministries.
(d) The Attorney General who has the right to speak and
Which of these statements are correct?
take part in the proceedings of either House of the (a) 1 and 2 (b) 2 and 3
Parliament (c) 1 and 3 (d) 1, 2 and 3

Downloaded From : www.EasyEngineering.net


Downloaded From : www.EasyEngineering.net
Union Government P-81

98. Consider the following statements: 103. Regarding the office of the Lok Sabha Speaker, consider
1. The Rajya Sabha alone has the power to declare that the following statements:
it would be in national interest for the Parliament to 1. He/She holds the office during the pleasure of the
legislate with respect to a matter in the State List. President.
2. Resolutions approving the proclamation of 2. He/She need not be a member of the House at the
Emergency are passed only by the Lok Sabha. time of his/her election but has to become a member
Which of the statement(s) given above is/are correct?
of the House within six months from the date of his/
(a) 1 only (b) 2 only
her election.
(c) Both 1 and 2 (d) Neither 1 nor 2
99. Consider the following statements in respect of financial 3. If he/she intends to resign, the letter of his/her
emergency under Article 360 of the Constitution of India: resignation has to be addressed to the Deputy
1. A proclamation of financial emergency issued shall Speaker.
cease to operate at the expiration of two months, Which of the statements given above is /are correct?
unless before the expiration of that period it has (a) 1 and 2 only (b) 3 only
been approved by the resolutions of both Houses of (c) 1, 2 and 3 (d) None
Parliament. 104. A deadlock between the Lok Sabha and the Rajya Sabha
2. If any proclamation of financial emergency is in

ww operation, it is competent for the President of India


to issue directions for the reduction of salaries and
allowances of all or any class of persons serving

calls for a joint sitting of the Parliament during the
passage of
1. Ordinary Legislation

w.E
in connection with the affairs of the Union but
excluding the Judges of Supreme Court and the High
Courts.



2. Money Bill
3. Constitution Amendment Bill
Select the correct answer using the codes given below :



(a) 1 only
(c) Both 1 and 2 a
Which of the statements given above is/are correct?
(b) 2 only
syE
(d) Neither 1 nor 2
100. Consider the following statements:


(a) 1 only
(c) 1 and 3 only
(b) 2 and 3 only
(d) 1, 2 and 3
105. In the context of India, which of the following principles
is/are implied institutionally in the parliamentary


1. The Chairman of the Committee on Public Accounts
is appointed by the Speaker of the Lok Sabha.
2. The Committee on Public Accounts comprises ngi
government?
1. Members of the Cabinet are Members of the
Parliament.


Members of Lok Sabha, Members of Rajya Sabha
and few eminent persons of industry and trade.
Which of the statements given above is/are correct?

nee
2. Ministers hold the office till they enjoy confidence in
the Parliament.



(a) 1 only
(c) Both 1 and 2
(b) 2 only
(d) Neither 1 nor 2
101. Who among the following have held the office of the


r
3. Cabinet is headed by the Head of the State.

ing
Select the correct answer using the codes given below.
(a) 1 and 2 only (b) 3 only


Vice-President of India?
1. Mohammad Hidayatullah
(c) 2 and 3 only
106. Consider the following statements:
.ne
(d) 1, 2 and 3

1. The Council of Ministers in the Centre shall be






2. Fakhruddin Ali Ahmed
3. Neelam Sanjiva Reddy
4. Shankar Dayal Sharma
Select the correct answer using the code given below:


t
collectively responsible to the Parliament.
2. The Union Ministers shall hold the office during the
pleasure of the President of India.
3. The Prime Minister shall communicate to the
Codes:
President about the proposals for legislation.
(a) 1, 2, 3 and 4 (b) 1 and 4 only
Which of the statements given above is/are correct?
(c) 2 and 3 only (d) 3 and 4 only
(a) 1 only (b) 2 and 3 only
102. With reference to Union Government, consider the
(c) 1 and 3 only (d) 1, 2 and 3
following statements: 107. Consider the following statements:
1. The Ministries and Departments of the Government 1. The Chairman and the Deputy Chairman of the
of India are created by the Prime Minister on the Rajya Sabha are not the members of that House.
advice of the Cabinet Secretary. 2. While the nominated members of the two Houses of
2. Each of the ministries is assigned to a Minister by the Parliament have no voting right in the presidential
the President of India on the advice of the Prime election, they have the right to vote in the election of
Minister. the Vice President.
Which of the statements given above is/are correct? Which of the statements given above is/are correct?
(a) 1 only (b) 2 only (a) 1 only (b) 2 only
(c) Both 1 and 2 (d) Neither 1 nor 2 (c) Both 1 and 2 (d) Neither 1 nor 2

Downloaded From : www.EasyEngineering.net


Downloaded From : www.EasyEngineering.net
P-82 Union Government

108. Consider the following statements: 112. The principle of “collective responsibility’ under
In the electoral college for Presidential Election in India, parliamentary democracy implies that [CSAT–2014-I]
1. the value of the vote of an elected Member of 1. a motion of no-confidence can be moved in the
Legislative Assembly equals Council of Ministers as a whole as well as an
State Population individual minister.
×100
Number of Elected MLAs of the State 2. no person shall be nominated to the cabinet except
2. the value of the vote of an elected Member of on the advice of the Prime Minister.
Parliament equals to total value of the votes of all 3. no person shall be retained as a member of the
Cabinet if the Prime minister says that he shall be
elected MLA’s and total number of elected MP’s
dismissed.
3. there were more than 5000 members in the latest
Select the correct answer using the codes given
elections.
below
Which of these statements is/are correct?
(a) Only 1 (b) Only 2
(a) 1 and 2 (b) Only 2
(c) Only 3 (d) 2 and 3
(c) 1 and 3 (d) Only 3
113. The Committee on Public Accounts under the Constitution
109. Consider the following statements:
of India is meant fox [CSAT–2014-I]
1. The Speaker of Lok Sabha has the power to adjourn
1. the examination of accounts showing the

ww the House sine die but, on prorogation, it is only the


President who can summon the House
2. Unless sooner dissolved or there is an extension of

appropriation of sums granted by the House for the
expenditure of the Government of India
2. scrutinising the report of the Comptroller and

w.E
the term, there is an automatic dissolution of the Lok
Sabha by efflux of time, at the end of the period of
five years, even if no formal order of dissolution is

Auditor-General
3. suggesting the form in which estimates shall be
presented to the Parliament


a
issued by the President

syE
3. The Speaker of Lok Sabha continues in office
even after the dissolution of the House and until
immediately before the first meeting of the House
Select the correct answer using the codes given



below
(a) Only 1
(c) 1 and 2
(b) Only 2
(d) All of these



Which of the statements given above are correct?
(a) 1 and 2
(c) 1 and 3
(b) 2 and 3 .
(d) 1, 2 and 3 ngi 114. Which of the statements relating to the Deputy Speaker of


the Lok Sabha is/are correct? [CSAT–2014-I]
1. The office of the Deputy Speaker acquired a more
110. The functions of the committee on estimates, as
incorporated in the Constitution of India, shall be to
[CSAT–2014-I]
nee
prominent position after the enforcement of the
Constitution of India in 1950.
2. He/She is elected from amongst the members.
1. report what economies, improvements in
organisation, efficiency or administrative reform
may be effected.


r ing
3. He/She holds office until he/she ceases to be a
member of the House.
Select the correct answer using the codes given below


2. suggest alternative policies in order to bring about
efficiency and economy in administration.
3. examine whether the money is well laid out within


(a) Only 1
(c) 1, 2 and 3
.ne
(b) 1 and 2
(d) 2 and 3
115. Which of the following principles is/are taken into


the limits of the policy implied in the estimates.
4. examine the reports, if any, of the Comptroller and
Auditor General on the public undertakings.
Select the correct answer using the codes given

parliamentary party or group? t
consideration by the Speaker while recognising a
[CSAT–2014-I]
1. An association of members who have an organisation
both inside and outside the House
below 2. An association of members who shall have at least
(a) 1 and 2 (b) 2 and 3 one-third of the total number of members of the
(c) 1, 2 and 3 (d) 3 and 4 House
111. The subject matter of an adjournment motion in the 3. An association of members who have a distinct
Parliament[CSAT–2014-I] programme of parliamentary work
1. must be directly related to the conduct of the Union Select the correct answer using the codes given
Government. below
2. may involve failure of the Government of India (a) 1, 2 and 3 (b) Onfy 1
to perform its duties in accordance with the (c) 1 and 3 (d) 2 and 3
Constitution. 116. Which of the following statements in the context of
Select the correct answer using the codes given structure of the Parliament is/are correct?
below [CSAT–2014-I]
(a) Only 1 (b) Only 2 1. The Parliament of India consists of the President, the
(c) Both 1 and 2 (d) Neither 1 nor 2 Council of States and the House of the People.

Downloaded From : www.EasyEngineering.net


Downloaded From : www.EasyEngineering.net
Union Government P-83

2. The President of India is directly elected by an Which of the statements given above is / are correct?
electoral college consisting of the elected members (a) 1 only (b) 2 only
of both the Houses of the Parliament only. (c) Both 1 and 2 (d) Neither 1 nor 2
Select the correct answer using the codes given
below Matching Based MCQ
(a) Only 1 (b) Only 2
(c) Both 1 and 2 (d) Neither 1 nor 2 DIRECTIONS (Qs. 82 to 86) : Match List-I with List-II and
117. Which of the following statements are correct regarding select the correct answer using the codes given below the lists.
Joint Session of the Houses of the Parliament in India?
122. List-I List-II
1. It is an enabling provision, empowering the President
to take steps for resolving deadlock between the two (Form of government) (Essential features)
Houses. (A) Cabinet (1) Separation of powers
2. It is not obligatory upon the President to summon the government
Houses to meet in a join sitting. (B) Presidential (2) Collective responsibility
3. It is being notified by the President. government
4. It is frequently resorted to establish the supremacy (C) Federal (3) Concentration of powers
of the Lok Sabha. government

ww
Select the correct answer using the codes given below
[CSAT–2014-I]
(a) 1 and 2 (b) 1, 2 and 3



(D) Unitary
government
(4) Division of powers

(5) Administrative law


(c) 2 and 3

w.E (d) 3 and 4

118. Consider the following statements : 



[CSAT–2014-I]
1. The President shall make rules for the more



(a) A – 3 ; B – 4 ; C – 2 ; D – 1
(b) A – 2 ; B – 1 ; C – 4 ; D – 3
(c) A – 3 ; B – 4 ; C – 1 ; D – 2


a
convenient transaction of the business of the

Ministers of the said business.


syE
Government of India, and for the allocation among

2. All executive actions of the Government of India



(d) A – 4 ; B – 3 ; C – 2 ; D – 5
123. List-I
(a) The council of
List-II
(1) Article 74

ngi
shall be expressed to be taken in the name of the ministers shall be
Prime Minister. collectively responsible
Which of the statements given above is/are correct? to the House of People.

nee
(a) 1 only (b) 2 only
(c) Both 1 and 2 (d) Neither 1 nor 2 (B) Duties of the Prime (2) Article 77
119. With reference to the Union Government, consider the Minister towards the
following statements : [CSAT–2015-I] President.


1. The Department of Revenue is responsible for the
preparation of Union Budget that is presented to the
Parliament.
2. No amount can be withdrawn from the Consolidated



r ing
(C) Council of ministers to
aid and advise the President.
(D) All executive action of
(3) Article 76

(4) Article 75
Fund of India without the authorization from the
Parliament of India.


the Government of
India shall be taken in
.ne



3. All the disbursements made from Public Account
also need the authorization from the Parliament of
India.
Which of the statements given above is / are correct?
(a) 1 and 2 only (b) 2 and 3 only




the name of the President.
(a) A – 4 ; B – 5 ; C – 1 ; D – 3
(b) A – 3 ; B – 2 ; C – 4 ; D – 1
(c) A – 4 ; B – 5 ; C – 1 ; D – 2
t
(5) Article 78

(c) 2 only (d) 1, 2 and 3 (d) A – 3 ; B – 4 ; C – 1 ; D – 2


120. Consider the following statements: [CSAT–2015-I]
1. The Rajya Sabha has no power either to reject or to 124. List-I List-II
amend a Money Bill. (A) Chief Election (1) Elected by Rajya Sabha
2. The Rajya Sabha cannot vote on the Demands for
Grants. Commissioner
3. The Rajya Sabha cannot discuss the Annual Financial (B) Deputy Chairman (2) Elected by Lok Sabha
Statement.
Which of the statements given above is / are correct? of Rajya Sabha
(a) 1 only (b) 1 and 2 only (C) Chairman of (3) Appointed by Lok
(c) 2 and 3 only (d) 1, 2 and 3
121. Consider the following statements: Lok sabha Sabha Speaker
1. The Executive Power of the Union of India is vested (4) Appointed by the president
in the Prime Minister. (a) A-4, B-1, C-3 (b) A-2, B-3, C-4
2. The Prime Minister is the ex officio Chairman of the
Civil Services Board. (c) A-1, B-4, C-2 (d) A-4, B-1, C-2

Downloaded From : www.EasyEngineering.net


Downloaded From : www.EasyEngineering.net
P-84 Union Government

125. Match list I with list II and select the correct answer using 127. Match List I with List II and select the correct answer:
the codes given below the lists : List-I List-II
List I List II (Functionaries) (Oaths or affirmations)
(Functionaries) (Oaths or affirmations) A. President of India 1. Secrecy of information
A. President of India 1. Secrecy of B. Judges of Supreme 2. Faithful Discharge of
Information Court duties
B. Judges of Supreme 2. Faithful discharge C. Members of 3. Faith and Allegiance
Court of duties Parliament to the Constitution of
C. Members of 3. Faith and Allegiance India
Parliament to the constitution D. Minister for the 4. Upholding the
of India Union Constitution and the
D. Minister for 4. Upholding the law
the Union constitution and the Codes:
law
(a) A-3, B-4- C-1 D-2 (b) A-4, B-3, C-2, D-1
Codes:
(c) A-3, B-4, C-2, D-1 (d) A-4, B-3, C-1, D-2
A B C D
128. Match List-I with List-II and select the correct answer
(a) 3 4 1 2

ww
using the codes given below the lists:
(b) 4 3 2 1
(c) 3 4 2 1 List-I List-II
(d) 4 3 1 2 (Article of the (Content)
Constitution)
given below.
List - I List - II

w.E
126. Consider the List-I and List-II and choose the correct code

A. National Front 1. 1996-98



A. Article 54
President of India
B. Article 75
1. Election of the

2. Appointment of the
B. United Front
C. National
Democratic
Alliance
a 2. 1998-2004
3. 1989-90
syE


Prime Minister
C. Article 155

D. Article 164
3. Appointment of the
Governor of a State
4. Appointment of the
D. United Progressive 4. 2004-14
Alliance ngi
Chief Minister and

a State
Council of Ministers of
Codes:
A B C D
(a) 3 1 2 4

nee
`
Legislative Assemblies
5. Composition of

(b) 1 3 2 4
(c) 1
(d) 1 2 4 3
2 3 4
Codes:

r ing
(a) A-1, B-2, C-3, D-4 (b) A-1, B-2, C-4, D-5
(c) A-2, B-1, C-3, D-5 (d) A-2, B-1, C-4, D-3

.ne
t

Downloaded From : www.EasyEngineering.net


Downloaded From : www.EasyEngineering.net
Union Government P-85

EXERCISE-1 account of the government in many of the countries


1. (b) 2. (b) 3. (d) 4. (c) in the Commonwealth of Nations. All tax revenue is
5. (a) 6. (b) 7. (c) 8. (c) 9. (d) paid into the fund unless Parliament has specifically
10. (b) 11. (c) 12. (a) 13. (d) 14. (b) provided otherwise by law. Any money received
15. (c) 16. (a) 17. (b) 18. (d) by the government which is not taxation, and is
19. (b) 20. (b) 21. (a) 22. (b) 23. (b) not to be retained by the receiving department (for
24. (c) 25. (d) 26. (a) 27. (b) 28. (d) example, fines), is classed as a Consolidated Fund
29. (b) 30. (b) 31. (a) 32. (d) extra receipt (CFER). These are to be paid into the
33. (d) 34. (c) 35. (b) 36. (c) 37. (d) Consolidated Fund as soon as they are received.
38. (c) 39. (a) 40. (d) 41. (a) 42. (c) 61. (b)
43. (d) 62. (d) 63. (d) 64. (b) 65. (c) 66. (a)
44. (d) Money bill can be introduced by the prior permis- 67. (a) V.V. Giri, the president of India was associated
with trade Union movement. Varahagiri Venkata
sion of President only. The proposal is money bill
Giri, commonly known as V. V. Giri, was the

ww or not it is decided by Lok Sabha speaker.


45. (c) President can dissolve the Lok Sabha on the advice
of Prime Minister. The time duration of Lok Sabha is
fourth President of the Republic of India from 24
August 1969 to 23 August 1974. He served as
Acting President of India from 3 May 1969 to 20
5 years.

w.E
46. (b) Under Article 109, special procedure in respect of 68. (c)
July 1969, before getting elected.
Money Bill has been given, a Money Bill shall not 69. (a) 70. (b)
be introduced in the council of state, it can be in- 71. (b) Cabinet Secretary is the highest civil servant of

a
troduced in only Lok Sabha. After a money bill is

should be returned within 7 days.


syE
passed by Lok Sabha and send to Rajya Sabha, it

47. (c) In the absence of both the President of India and the
the Union Government. The Cabinet Secretary is
the senior-most civil servant in the Government of
India. The Cabinet Secretary is the ex-officio head
of the Civil Services Board. The Cabinet Secretary
absence, the senior-most judge of the supreme court
available) shall act as the president of India. ngi
vice president, the chief justice of India (or in his is under the direct charge of the Prime Minister.
Though there is no fixed tenure, the average tenure
of the Cabinet Secretary has been less than 3 years.
48. (d)

nee
His or her tenure however, can be extended.
49. (a) According to the Article 75 of Indian Constitution 72. (a) The speaker’s vote in the Lok Sabha is called casting
the Council of Ministers of the Union shall be col- vote. A casting vote is a vote given to the presiding
lectively responsible to the House of the People.
50. (c) The Union Executive of India, as in parliamentary
type of government, consist of president and the
council of ministers.
r
officer of a council or legislative body to resolve

ing
a deadlock and which can be exercised only when
such a deadlock exists.
73. (c) The parliamentary Committee which scrutinizes the
51. (a) 52. (a)
55. (c) 56. (c)
53. (d)
57. (a)
54. (d)

58. (d) Department of women and child development is not .ne


report of the CAG is public accounts committee.
The Comptroller and Auditor General (CAG) is
a department under Ministry of Human Resource
Development. Human Resources Development
(HRD) as a theory is a framework for the expansion
of human capital within an organization through
t
an authority, established by the Constitution of
India under Chapter V, which audits all receipts
and expenditure of the Government and the state
governments, including those bodies and authorities
substantially financed by the government.
the development of both the organization and the
74. (c) 75. (d)
individual to achieve performance improvement.
76. (c) The prime minister is the chairman of the National
59. (b) The comptroller and Auditor General can be removed
on an address from both Houses of Parliament. The Integration council. The National Integration
Comptroller and Auditor General (CAG) of India Council originated in a conference convened by
is an authority, established by the Constitution of Prime Minister Jawaharlal Nehru in September–
India under Chapter V, who audits all receipts and October 1961. The purpose was to find ways to
expenditure of the Government of India and the state counter problems that were dividing the country
governments, including those bodies and authorities including attachment to specific communities,
substantially financed by the government. The CAG castes, regions and languages.
is also the external auditor of government-owned 77. (a) 78. (a) 79. (c)
companies. 80. (c) Chaudhary Charan Singh was the Prime Minister
60. (d) Salary and allowances of the speakers of the Lok resigned before facing a vote of no-confidence in the
Sabha is not charged on the Consolidated fund Lok Sabha. Chaudhuri Charan Singh was the fifth
of India. Consolidated fund or the consolidated Prime Minister, serving from 28 July 1979 until 14
revenue fund is the term used for the main bank January 1980.

Downloaded From : www.EasyEngineering.net


Downloaded From : www.EasyEngineering.net
P-86 Union Government

81. (d) ‘Zero Hour’ in Political jargon refers to question 97. (b) Article 110 of Indian constitution defines a Money
answer session. Zero Hour in Parliament starts at Bill. The speakers decision as to whether a bill is a
12 noon during which members raise matters of Money Bill is final.
importance, especially those that cannot be delayed. 98. (d) While proclamation of Emergency is in operation
82. (d) 83. (c) 84. (c) be extended by parliament by law for a period not
85. (d) 86. (d) 87. (c) 88. (c) exceeding one year at a time and not extending in
89. (a) The first reading of the bill in a house of parliament any case beyond a period of six months after the
refers to motion for leave to introduce the bill in the proclamation has ceased to operate [Article 83 (2)]
House.If a motion for leave to introduce a Bill is 99. (d) The Rajya Sabha is a permanent house, not subject
opposed, the Speaker, after permitting, if he thinks to disolution, one third of its members retiring
fit, brief statements from the member who opposes after every two years. Thus every member enjoys a
the motion and the member who moved the motion, 6-year tenure.
may, without further debate, put the question: pro- 100. (c) The Union Executive of India, as in parliamentary
vided that where a motion is opposed on the ground type of government, consist of president and the
that the Bill initiates legislation outside the legisla- council of ministers.
tive competence of the House, the Speaker may per- 101. (d) The presiding officer of the Lok Sabha is the
mit a full discussion thereon. Provided further that Speaker who is elected by the member from amongst

ww the Speaker shall forthwith put to vote the motion


for leave to introduce a Finance Bill or an Appro-
priation Bill.
themselves.
102. (c) Cabinet Secretariat is under the charge of the Prime
Minister of India.

w.E
90. (b) Department of official languages is subordinate
office of the Ministry of Home affairs. It deals
with the implementation of the provisions of the
Constitution relating to official languages and the
103. (c) If any question arises whether a Bill is a money bill
or not, the decision of the Speaker of the Lok Sabha
shall be final under the article 110 of the constitution

a
provisions of the Official Languages Act, 1963.

syE
91. (d) State legislature assemblies, both houses of the par-
liament form part of the electoral college for the
election of the president. An electoral college is a
of India.
104. (c) The speaker of the Lok sabha can resign from his
office by addressing his resignation to the deputy
speaker of the Lok Sabha.
set of electors who are selected to elect a candidate
to a particular office. Often these represent different
organizations or entities, with each organization or ngi 105. (c) Either house of the parliament can initiate
impeachment proceedings against the president of
India.
entity represented by a particular number of electors
or with votes weighted in a particular way.
92. (d) Decorum = Parliamentary etiquette
106. (d)
nee
107. (a) Joint session is presided over by the speaker of Lok
Sabha or in his absence by the Deputy Speaker.






Crossing the floor = Changing the party
Interpolation = Seeking clarification through ruling
Yielding the floor = Respecting speaker’s order
93. (d) The Constitutional amendment bill must be passed
108. (d)
r ing
109. (b) The Concurrent List or List-III is a list of 52
items (previously 47 items) given in Part XI of
by each house of the Indian Parliament separately
by special majority.
94. (b) Department of border management is a department .ne
the Constitution of India, concerned with relations
between the Union and States. This part is divided
between legislative and administrative powers.
of ministry of Home affairs of the union minister.
Department of Border Management is dealing with
management of borders, including coastal borders.
95. (a) When a money bill returns to the Loksabha with
110. (a) Members of both houses of parliament.
111. (d) t
112. (a) Demands for Grants Committees are Departmentally
Related Standing Committees.After the General
amendments made by the Rajyasabha, it is open to Discussion on the Budget is over, the House is
Loksabha to accept or to reject any or all of the adjourned for a fixed period. During this period, the
recommendations. When the Loksabha chooses to Demands for Grants of the Ministries/ Departments
accept or decline the money bill with or without the are considered by the Committees. It is not a
recommendation, the money bill is deemed passed parliamentary committee.
in both houses. 113. (c) In India, “the Emergency” refers to a 21-month
96. (b) The largest Committee is Estimates, given its 30
period in 1975-77 when Prime Minister Indira
members
Gandhiunilaterally had a state of emergency declared
Committee on No. of members across the country.Fakhruddin Ali Ahmed was the
Public Accounts 22 President at that time.
114. (b) Gulzarilal Nanda became the Prime Minister of
Estimates 30
India for two short periods following the deaths of
Public Undertakings 22 Jawaharlal Nehru in 1964 and Lal Bahadur Shastri
Petitions LS(15) , RS(10) in 1966.

Downloaded From : www.EasyEngineering.net


Downloaded From : www.EasyEngineering.net
Union Government P-87

115. (b) According to article 82 of Indian Constitution, the of the Speaker or the Dy. Speaker shall be moved
parliament of India shall readjust the allocation of unless at least fourteen days notice has been given of
seats in the Lok Sabha to the states and division of the intention to move the resolution.
each state into territorial constituencies. 129. (b) The Estimates Committee is a Parliamentary
116. (b) According to Article 53(b) of the Indian Constitution Committee consisting of 30 Members, elected every
the supreme command of the Defence Forces of year by the Lok Sabha from amongst its Members.
the Union shall be vested in the President and the The Chairman of the Committee is appointed by the
exercise thereof shall be regulated by law. Speaker from amongst its members. The term of
117. (c) The quorum to constitute a joint sitting shall be one- office of the Committee is one year.
tenth of the total number of members of the Houses. 130. (b) Our Parliamentary system blends the legislative and
118. (b). P V Narasimha Rao could not vote for himself the executive organs of the State in as much as the
during vote of confidence. executive power is wielded by a group of Members
119. (d) The correct sequence is, Discussion on Budget, of the Legislature who command majority in the
Lok Sabha.
Appropriation Bill, Finance Bill, Vote on Account.
131. (d) The Constitution empowers Parliament of India to
120. (c) Protem Speaker performs the duties of the office
make laws on the matters reserved for States (States
of the Speaker from the commencement of the
List). However, this can only be done if Rajya Sabha
sitting of the new Lok Sabha till the election of the

ww Speaker. Protem speaker is mainly an operating and


temporary speaker.
121. (c) He/she cannot take part in the voting when a vote of
first passes a resolution by two-thirds supermajority
granting such a power to the Union Parliament. The
union government cannot make a law on a matter

w.E
no confidence is under consideration.
122. (a) Under Article 61, the President of India can
be removed from the office by a process of 132.
reserved for states without an authorisation from
Rajya Sabha.So the House of People does not have
special powers with respect to the state list.
(d) As Indian follows Universal Adult Suffrage

of Parliament.a
impeachment for the violation of the Constitution.

syE
The impeachment is to be initiated by either House

123. (c) Irrespective of the fact that a number of seats may


elected members of the Lower House of the State
Legislature i. e., Las have the right to vote in the
elections to both the Lok Sabha and Rajya Sabha.
The State Legislature besides making laws also has

each voter with the reservation that this single vote


ngi
have to be filled, this system postulates one vote for

is transferred to other candidates. This is the reason


one electoral power in electing the President of
India. Elected members of the Legislative Assembly
along with the elected members of Parliament are
why this system is known as “single transferable
vote system.”
124. (a) “Closure” is one of the means by which a debate
nee
involved in this process.
133. (c) PM has complete discretion to choose his ministers
in the Cabinet not necessarily from the two Houses
may be brought to a close by a majority decision
of the House, even though all members wishing to
speak have not done so.
r ing
of Parliament but can also choose any other person.
That person should become member of either house
within 6 months from the date he enters the office.
125. (c) While a proclamation of emergency is in operation,
this period may be extended by Parliament by law
for a period not exceeding one year at a time.Under .ne
134. (a) Because no-confidence motion can be moved only in
Lok Sabha (not in RS) by the opposition.
135. (a) A Money bill can be introduced only in LS (not in
Article 352 the president can declare a national
emergency when the security of India or part of it is
threatened by war or external aggression or armed
rebellion.
t
RS) that too on the recommendation of the President.
136. (d) Parliament has exclusive power to make law with
respect to any of the matters enumerated with the
Union List. According to entry no 14 in the Union
126. (c) A Money Bill cannot be introduced in Rajya Sabha. List it reads- ‘entering into treaties and agreements
with foreign contries and implementing of treaties,
Rajya Sabha has no power either to reject or amend
agreement and convention with foreign countries’.
a Money Bill. It can only make recommendations
137. (c) The term of the LS can be extended by not more
on the Money Bill. Whether a particular Bill is a
than one year at a time during the proclamation of
Money Bill or not is to be decided by the Speaker
national emergency under Article 352.
of Lok Sabha. Rajya Sabha may discuss the Annual
138. (c) President can issue Ordinance under Article 123.
Financial Statement. It has no power to vote on the 139. (d) According Article 355, it shall be the duty of
Demands for Grants. the Union to protect every State against external
127. (c) In the Rajya Sabha the states have been given seats aggression and internal disturbance and to ensure
on the basis of population and economic position. that the government of every State is carried on in
128. (d) The Speaker of Lok Sabha may be removed from accordance with the provisions of this Constitution.
his office by a resolution of the House of the People 140. (d) According to Article 368 an amendment of
passed by a majority of all the then members of the this Constitution may be initiated only by the
House. No resolution for the purpose of removal introduction of a Bill for the purpose in either House

Downloaded From : www.EasyEngineering.net


Downloaded From : www.EasyEngineering.net
P-88 Union Government

of Parliament, and when the Bill is passed in each government and Rajya Sabha is not permitted to
House by a majority of the total membership of that make use this device.
House present and voting, it shall be presented to (iv) The discussion an adjournment motion should last
the President who shall give his assent to the Bill for not less than two hours and thirty minutes.
and thereupon the Constitution shall stand amended 150. (a) Article 75(I) of the Indian Constitution provides
in accordance with the terms of the Bill. that the Prime Minister shall be appointed by the
141. (a) Appropriation Bill is a money bill. In case of money President. The Constitution permits a person to be
bill, RS has only recommendatory power and need appointed PM without his\her being a member of
not to be passed by RS. either House of the Parliament at the of appointment.
142. (b) The Constitution of India empowering the Parliament Before expiry of this time, he has to become a
to make laws in the State List and to create one or member of either the Rajya Sabha or the Lok Sabha.
more All India Services. 151. (a) When a money bill returns to the Loksabha with
143. (d) It is the sole prerogative of Parliament under Article amendments made by the Rajyasabha, it is open to
253 of the Constitution. Loksabha to accept or to reject any or all of the
144. (a) Parliamentary Committees are formed to dispose off recommendations. When the Loksabha chooses to
the large volume of work in time but with detailed accept or decline the money bill with or without the
scrutiny. Their appointment, terms of office as well recommendation, the money bill is deemed passed

ww as functions etc are regulated by provisions under


Article 118(1). These are of 2 kinds: Standing
Committees and Ad-hoc Committees.
in both houses.
152. (c) The recommendation of the president is required
for introduction of money bills or for moving

w.E
145. (c) In Rajya Sabha, 12 members are nominated by
the President from the persons who have special
knowledge in art, science, literature and social
service. In Lok Sabha, 2 members are nominated
amendments to acts relating to financial matters,
except those making provision for the reduction or
abolition of any tax. So, for option (c) president’s
recommendation is not required.

a
by the President from the Anglo-Indian community

Vice-Presidential elections. syE


(Art 331). A nominated member can vote only in the

146. (a) In case of a No-confidence motion, there is no need


153. (c) Article 111 of the Indian constitution stipulates
that the President shall give assent to a bill passed
by both houses of the parliament or return the bill
as soon as possible for reconsideration with his
to set out the grounds on which it is based. No-
Confidence motion is introduced only in the Lok
Sabha by the opposition and needs a support of not ngi recommendation.
154. (b) According to article 112,the President shall in

nee
respect of every financial year cause to be laid
less than 50 members of LS for its introduction. Rule before both the Houses of Parliament a statement
198 of the Lok Sabha specifies the procedure for a of the estimated receipts and expenditure of the
motion of no-confidence. Any member may give a
written notice; the speaker shall read the motion of
no-confidence in the House and ask all those persons
to rise who favours that the motion be taken up. If
r
Government of India for that year, in this Part

ing
referred to as the annual financial statement.
155. (a) In India, if a bill has been rejected by any house
of the parliament and if more than six months have
there are 50 MPs in favour, the speaker allots a date
for discussing the motion.
147. (d) Article 67(b) in the Constitution of India states, .ne
elapsed, the President may summon a joint session
for purpose of passing the bill. The bill is passed by
a simple majority of a joint sitting.
a Vice President may be removed from his office
by a resolution of the council of States passed by
a majority of all the then members of the council
and agreed to by the House of the People; but no 1. (d)
6. (d)
EXERCISE-2
2. (d) 3. (b) 4. (c) 5. (c)
7. (a) 8. (c) 9. (a) 10.
t (b)
resolution for the purpose of this clause shall be
moved unless at least fourteen days notice has been 11. (d) 12. (a) 13. (d) 14. (d) 15. (b)
given of the intention to move the resolution. 16. (c) 17. (b) 18. (d) 19. (b) 20. (a)
148. (b) Parliament shall have power to authorise by law the 21. (b) 22. (d) 23. (c) 24. (c) 25. (b)
withdrawal of moneys from the Consolidated Fund 26. (d) 27. (d) 28. (c)
of India for the purposes for which the said grants 29. (c) All the members of the Lok Sabha, except two
are made. members may be nominated by the President from
149. (a) Adjournment motion : Anglo-Indian Community.
(i) It is introduced in the Parliament to draw attention 30. (b) Parliamentary committee in India presents its report
of the house to a definite matter of urgent public to the speaker/Chairman of the house. The work
importance and needed to support of 50 members to done by the Parliament in modern times is not only
be admitted. varied and complex in nature, but also considerable
(ii) It is regarded as an extraordinary device, because it in volume. The time at its disposal is limited. It can-
interrupts the normal business of the house. not, therefore, give close consideration to all the
(iii) It involves an element of censure against the legislative and other matters that come up before it.

Downloaded From : www.EasyEngineering.net


Downloaded From : www.EasyEngineering.net
Union Government P-89

A good deal of its business is, therefore, transacted crisis. Thus, in practice, the Council of Ministers
in Committees of the House, known as Parliamen- cannot be dismissed as long as it commands the
tary Committees. support of a majority in the Lok Sabha.
31. (d) A no confidence motion does not need to set out 65. (b) The nominated members of the Rajya Sabha have
grounds on which it is based. The withdrawal of the the right to vote in the election of the Vice President
notice of no confidence motion by a member may be so far none from them has been inducted in the
made when he is called upon by the speaker to ask Council of Ministers.
for leave of the House. 66. (a) Clause (3) of Article 77 “Conduct of Business of
32. (c) Both the Houses have been enshrined with equal the Government of India” of the Constitution of
powers on the matter of impeachment of judges of India lays down as follows: “The President shall
the High Courts. Soumitra Sen is a retired judge of
make rules for the more convenient transaction of
the Calcutta High Court. He was the first judge in
the business of the Government of India, and for the
independent India to be impeached in India’s Rajya
Sabha for misappropriation of funds. allocation among Ministers of the said business.
33. (d) Article 80 states that the state legislature participates The Constitution of India mentions that All
in the election of the members of Rajya Sabha and executive action of the Government of India shall be
also of the president (Art.54) expressed to be taken in the name of the President.
34. (a) ‘Committee on public Accounts’ consists of 22 Therefore, only option (a) is correct.

ww members, 15 from the Lok Sabha and 7 from Rajya


Sabha. A Lok Sabha member from a major opposi-
tion party is appointed its chairman. The Public Ac-
67. (c) A constitutional Government needs to balance
between individual liberty viz a viz State Authority.
68. (c) The functions of the Cabinet Secretariat are

w.E
counts Committee (PAC) is a committee of selected
members of Parliament, constituted by the Parlia-
ment of India, for the auditing of the expenditure of
the Government of India.
preparation of agenda for Cabinet Meetings &
Secretarial assistance to Cabinet Committees.
However Allocation of financial resources to
the Ministries as per the provisions in budget is

a
35. (d) A no-confidence motion does not need to set out

syE
grounds on which it is based. The withdrawal of the
notice by a member may be made when he is called
upon by the speaker to ask for leave of the House.
prepared by finance ministry.
69. (b) The committee consists of 15 members of Lok
sabha not 25 members. The function of the
committee is to examine the accounts showing the
36. (d) The government may be expressed by the House of
People by all the statements given. So the all state-
ments are correct. ngi appropriation of the sums granted by Parliament
to meet the expenditure of the government of India

nee
37. (b) The government of India consists of a number of and such other accounts laid before the House as the
ministers/departments for its administration, each committee may think fit. Apart from the Reports of
ministry assigned to a minister who runs it with the the Comptroller and Auditor General of India on
assistance of a secretary in charge of the particular
ministry.
38. (a) 39. (c) 40.
42. (d) 43. (d) 44. (c)
(c) 41. (a) r
Appropriation Accounts of the Union Government,

ing
the Committee also examines the various Audit
Reports of the Comptroller and Auditor General on
revenue receipts, expenditure by various Ministries/
45. (a) 46. (d) 47 (a) 48. (a) 49. (b)
50. (d) 51. (c) 52. (d) 53. (c) 54. (b)
70. (d)
of autonomous bodies.
.ne
Departments of Government and accounts

55. (d) 56. (b) 57. (d) 58. (c) 59. (b)
60. (a) 61. (b)
62. (c) The Attorney General of India has a post parallel to
any minister in Parliament. He can take part in the
proceedings of either house. He can be a member
71. (b)
t
72. (c) Parliament may by law fix the salaries and allowances
of the officers of Parliament (Article 97). The salaries
are charged on the consolidated fund of India.
of any committee of Parliament. He has the right to 73. (c)
speak in the Parliament but he has no right to vote. 74. (c)
63. (a) Minister/ministers can be removed by issuing no 75. (c) Money Bills can be introduced only in Lok Sabha.
Money bills passed by the Lok Sabha are sent to the
confidence motion in the parliament. All cabinet
Rajya Sabha.Rajya Sabha(Council of States) cannot
members are mandated by the constitution to be the
reject or amend this bill. It can only recommend
member of either house of the parliament of India.
amendments.
Cabinet is headed by the cabinet secretary not by the 76. (b) According to Article 67, a Vice-President may be
Head of the State. removed from his office by a resolution of the Council
64. (c) The Council of Ministers shall be collectively of States passed by a majority of all the then members
responsible to the parliament; the Prime minister of the Council and agreed to by the House of the
shall communicate to the president about the People.
proposals for legislation but the union. If a President 77. (b) The Appropriation Bill is intended to give authority to
were to dismiss the Council of Ministers on his or Government to incur expenditure from and out of the
her own initiative, it might trigger a constitutional Consolidated Fund of India. The procedure for passing

Downloaded From : www.EasyEngineering.net


Downloaded From : www.EasyEngineering.net
P-90 Union Government

this Bill is the same as in the case of other money Bills. (ii) he dies before the commencement of the poll; or
An amendment of the Constitution can be initiated (iii) he is elected; or
only by the introduction of a Bill in either House (iv) he is not elected but gets more than 1/6th of
of Parliament. The procedure of amendment in the the total number of valid votes polled by all the
constitution is laid down in Part XX (Article 368) of candidates at the election.
the Constitution of India. 86. (c) The total elective membership of the Lok Sabha is
78. (b) Article 65 of the Indian Constitution says that while distributed among States in such a way that the ratio
acting as president or discharging the functions of between the number of seats allotted to each State
president, the Vice President does not perform the and population of the State is, as far as practicable,
duties of the office of the chairman of Rajya sabha. the same for all States. The 84th Amendment to
Article 123 of the Constitution grants the President the Constitution (which was numbered as the 91st
certain law making powers to promulgate Ordinances Amendment Bill before it was passed in Parliament)
when either of the two Houses of Parliament is not in lifted the freeze on the delimitation of constituencies,
session and hence it is not possible to enact laws in as stipulated by the 42nd Constitution amendment of
the Parliament. 1976, and allowed delimitation within States on the
79. (d) The Union executive consists of the President, the basis of the 1991 Census.
Vice-President, and the Council of Ministers with 87. (b) A registered voter in India can contest an election
the Prime Minister as the head to aid and advise the to Lok Sabha from any constituency in India except

ww President.He may, by writing under his hand addressed


to the Vice-President, resign his office (Article 61).
According to article 52,executive power of the Union
autonomous Districts of Assam, Lakshadweep and
Sikkim. According to Section 8 of Representation of
Peoples Act 1951,a person convicted of any offence

80. (c)
w.E
is vested in the President.
The Ministry of Parliamentary Affairs handles affairs
relating to the Parliament of India and works as a link
between the two chambers, the Lok Sabha and the
and sentenced to imprisonment for not less than two
years [other than any offence referred to in sub-section
(a) or sub-section (b)] shall be disqualified from
the date of such conviction and shall continue to be

a
Rajya Sabha. It constitutes consultative committees

syE
of members of Parliament and makes arrangements
for holding their meetings, both during and between
sessions.
88. (d)
disqualified for a further period of six years since his
release.
There are three Financial Committees in the Indian
Parliament mentioned below:
81. (d) Committee on Public Accounts and Committee on
Public Undertakings consist of 15 members elected
by the Lok Sabha and 7 members of the Rajya Sabha. ngi 1. Estimates Committee- This Committee consists of 30
members who are elected by the Lok Sabha every year
from amongst its members. The term of the Committee
Committee on Estimates consists of 30 members who
are elected by the Lok Sabha every year from amongst
its members.

nee
is one year.
2. Committee on Public Undertakings- The Committee
on Public Undertakings consists of 22 members(15
82. (b) The Government of India (Allocation of Business)
Rules, 1961 are made by the President of India under
Article 77 of the Constitution for the allocation of
business of the Government of India. The Ministries/
r
members from Lok Sabha and 7 members from Rajya

ing
Sabha) .The term of the Committee is one year.
3. P ublic Accounts Committee- This Committee
consists of 22 members (15 members from Lok Sabha
Departments of the Government are created by the
President on the advice of the Prime Minister under the Committee is one year. .ne
and 7 members from the Rajya Sabha.). The term of

83. (a)
these Rules. The Cabinet Secretary is the ex-officio
Chairman of the Civil Services Board of the Republic
of India.
Correct chronological order of the Vice-Presidents of
India is as follows:
89. (d)

(a)
Minister-
t
According to Article 78 it shall be the duty of the Prime

to communicate to the President all decisions of the


council of Ministers relating to the administration of
the affairs of the union and proposals for legislation;
1. V.V .Giri – 1967 (b) to furnish such information relating to the administration
2. G.S Pathak –1969 of the affairs of the Union and proposals for legislation
3. B.D Jatti –1974 as the President may call for;
4. M Hidayatullah – 1979 (c) if the President so requires, to submit for the
84. (d) The Tenth Schedule ( Anti-Defection Act) was included consideration of the Council of Ministers any matter
in the Constitution in 1985 by the Rajiv Gandhi on which a decision has been taken by a Minister;
ministry and sets the provisions for disqualification 90. (d) · Election of Speaker shall be held on such date as
of elected members on the grounds of defection to the President may fix, and the Secretary-General shall
another political party. send notice of this dateto every member.
85. (b) The deposit made by a candidate shall be returned if · The election of a Deputy Speaker shall be held on
the following conditions are satisfied:- such date as the Speaker may fix and the Secretary-
(i) the candidate is not shown in the list of General shall send notice of this dateto every member.
contesting candidates, that is to say, either his · At any time before noon on the day preceding the
nomination was rejected or after his nomination date so fixed, any member may give notice in writing,
was accepted, he withdrew his candidature; or addressed to the Secretary-General, of a motion that

Downloaded From : www.EasyEngineering.net


Downloaded From : www.EasyEngineering.net
Union Government P-91

another member be chosen as the Deputy Speaker of 100. (a) Statement 2 is incorrect as Public Accounts Committee
the House and the notice shall be seconded by a third consists of 22 members: 15 from Lok Sabha and 7
member and shall be accompanied by a statement by from Rajya Sabha.
the member whose name is proposed in the notice that 101. (b) Mohd. Hidayatullah (1979-84); Shankar Dayal Sharma
he is willing to serve as Deputy Speaker, if elected. (1987-92)
91. (c) The Vice-President is elected by an Electoral College, 102. (b) According to Article 70 the Prime Minister shall be
which consists of the members of the Lok Sabha and appointed by the President and the other Ministers
Rajya Sabha (both elected and nominated members). shall be appointed by the President on the advice of
92. (c) The maximum strength of the House envisaged by the Prime Minister.
the Constitution of India is 552, which is made up by 103. (b) If the Lok Sabha Speaker wants to resign, the letter of
election of up to 530 members to represent the states; his / her resignation has to be addressed to the Deputy
up to 20 members to represent the Union Territories Speaker.
and not more than two members of the Anglo-Indian 104. (a) Only 3rd and 4th are correct, thus the option (b) is
Community to be nominated by the President of India, right.
if, in his/her opinion, that community is not adequately 105. (a) Minister/ministers can be removed by issuing no
represented in the House. confidence motion in the parliament. All cabinet
93. (b) Statement 1 is correct as per provisions under Article members are mandated by the constitution to be the
59. Statement 2 is correct as according to Article 79, member of either house of the parliament of India.

ww
94. (d)
Parliament shall consist of the President and two
Houses.
So far three joint sittings have been held, First was held
Cabinet is headed by the cabinet secretary not by the
Head of the State.
106. (c) The Council of Ministers shall be collectively

w.E
on Dowry Prohibition Bill, 1961, Second was held on
Banking Service Commission (Repeal) Bill, 1978 and
Third was held on Prevention of Terrorism Bill, 2002.
responsible to the parliament; the Prime minister shall
communicate to the president about the proposals for
legislation but the union. If a President were to dismiss

95. (d)
Lok Sabha.
a
The presiding officer of joint sitting is Speaker of the

syE
Statement 1 is incorrect as members of RS can become
cabinet ministers. Statement 2 is incorrect as Cabinet
secretariat is under the direct charge of the PM.
the Council of Ministers on his or her own initiative, it
might trigger a constitutional crisis. Thus, in practice,
the Council of Ministers cannot be dismissed as long
as it commands the support of a majority in the Lok
Sabha.
96. (c) It is not the duty of the President of India to cause to
be laid report of public Accounts Committee before ngi 107. (b) The nominated members of the Rajya Sabha have the
right to vote in the election of the Vice President so

97. (a)
the Parliament.
The PAC is formed every year with a strength of not
more than 22 members of which 15 are from Lok nee
far none from them has been inducted in the Council
of Ministers.
108. (b) The value of a MP’s vote is calculated by dividing
Sabha, the lower house of the Parliament, and 7 from
Rajya Sabha, the upper house of the Parliament.
The term of office of the members is one year. The
Estimates Committee, constituted for the first time in
r
the total value of all MLAs’ votes by the number of
MPs.
Value of an MP vote =
ing
The sum of vote value of elected members of all the
1950, is a Parliamentary Committee consisting of 30
Members, elected every year by the Lok Sabha from
amongst its Members. The Ministry of Parliamentary
Legislative Assemblies
.ne
The sum of elected members of both the house of Parliament
Affairs renders secretarial assistance to the Cabinet
Committee on Parliamentary Affairs, which, inter-alia
recommends prorogation of both the Houses of the
Parliament, Govt’s stand on Private Members’ Bills
t
109. (b) Option 2 and 3 are correct. But as to option 1 there
are special provisions. The Speaker shall determine
the time when a sitting of the House shall be adjourned
sine die or to a particular day, or to an hour or part
and Resolutions. Option 3 is not correct. of the same day: provided that the Speaker may, if
98. (a) Statement 1 is correct as per provisions under Article he thinks fit, call a sitting of the House before the
249. Statement 2 is incorrect as resolutions approving date or time to which it has been adjourned or at any
the proclamation of Emergency are passed by both time after the House has been adjourned sine die. It
Houses of Parliament (not only LS). is not the President.
99. (a) Statement 1 is correct as under Article 360, any 110. (c) The Committee on Public Undertakings examines the
Proclamation of Financial Emergency issued shall reports of the Comptroller & Auditor General on public
cease to operate at the expiration of two months, undertakings.
unless before the expiration of that period it has 111. (b) The subject matter of the motion must have a direct or
been approved by the resolutions of both Houses indirect relation to the conduct or default on the part of
of Parliament. If approved by both Houses, then it the Union Government and must precisely pinpoint the
operates for 6 months. failure of the Government of India in the performance
Statement 2 is incorrect as it is excluding Judges of SC of its duties in accordance with the provisions of the
and High Courts; but under the provisions of effects Constitution and Law.
of article 360, Judges of SC and HCs are included. 112. (d) Cabinet collective responsibility is a tradition in

Downloaded From : www.EasyEngineering.net


Downloaded From : www.EasyEngineering.net
P-92 Union Government

parliamentary governments in which the prime parliament and if more than six months have elapsed,
minister is responsible for appointing the cabinet the President may summon a joint session for purpose
ministers. It is the Prime Minister who enforces of passing the bill. If at the joint sitting the Bill is
collective responsibility amongst the Ministers passed with or without amendments with a majority
through his ultimate power to dismiss a Minister. No of total number of members of the two Houses present
person shall be nominated to the cabinet except on the and voting, it shall be deemed to be passed by both
advice of the Prime Minister. Secondly, no person shall the Houses. It is not frequently restored to establish
be retained as a Member of the Cabinet if the Prime the supremacy of the Lok Sabha.
Minister says that he should be dismissed. In India, a 118. (a) Clause (3) of Article 77 “Conduct of Business of the
Motion of No Confidence can be introduced only in Government of India” of the Constitution of India lays
the Lok Sabha. down as follows: “The President shall make rules for
113. (c) The Public Accounts Committee (PAC) is a committee the more convenient transaction of the business of the
of selected members of Parliament, constituted by the Government of India, and for the allocation among
Parliament of India, for the auditing of the expenditure Ministers of the said business.
of the Government of India. Its chief function is to The Constitution of India mentions that, “All executive
examine the audit report of Comptroller and Auditor action of the Government of India shall be expressed
General (CAG) after it is laid in the Parliament. to be taken in the name of the President.” Therefore,

ww
114. (c) The Deputy Speaker of the Lok Sabha is the vice-
presiding officer of the Lok Sabha. He/She is elected
from amongst the members. It acquired a more
only option (a) is correct.
119. (c) All revenues received by the Government by way of
taxes like Income Tax, Central Excise, Customs and

115. (c)
w.E
prominent position after 1950. He holds office till
either he ceases to be a member of the Lok Sabha or
he himself resigns.
In recognizing a parliamentary party or group, the
other receipts flowing to the Government in connection
with the conduct of Government business i.e. Non-Tax
Revenues are credited into the Consolidated Fund
constituted under Article 266 (1) of the Constitution


principles:
a
speaker shall take into consideration the following

syE
An association of members who propose to form a
parliamentary party:
of India. No amount can be withdrawn from the Fund
without authorization from the Parliament.
120. (b) A Money Bill cannot be introduced in Rajya Sabha.
Rajya Sabha has no power either to reject or amend a


1. Shall have an association of members who have a
distinct programme of parliamentary work
2. Shall have an organization both inside and outside ngi Money Bill. It can only make recommendations on the
Money Bill. It has no power to vote on the Demands
for Grants.


the house
3. Shall have at least a strength equal to the quorum
fixed to constitute a sitting of the house i.e one tenth
nee
121. (d) The Executive powers of the Union of India is vested
in the President. The Cabinet Secretary (and not the
Prime Minister) is the ex-officio head of the Civil

116. (a)
of the total number of members of the house.
Article 79- There shall be a Parliament for the Union
which shall consist of the President and two Houses
r
Services Board.

ing
122. (b) 123. (c) 124. (c)
125. (b) President- Upholding the constitution and the law;


to be known respectively as the council of States and
the House of the People.
Article 54 - The President shall be elected by the

.ne
Judge of the supreme court- Faith and allegiance to
the constitution of India; Members of Parliament-
Faithful Discharge of Duties;

117. (b)
members of an electoral college consisting of the
elected members of both Houses of Parliament; and
the elected members of the Legislative Assemblies of
the States.
Article 108 of the Constitution empowers the President

126. (a) t
Minister for the Union-Secrecy of Information.

127. (c) Oath or affirmation by the President under article


60; Oath or affirmation by Judge of SC, Members of
to summon a joint session of both houses “for the Parliament, Ministers for the Union comes under Third
purpose of deliberating and voting on the Bill”. In Schedule of the Constitution.
India, if a bill has been rejected by any house of the 128. (a)

Downloaded From : www.EasyEngineering.net


Downloaded From : www.EasyEngineering.net

STATE 4
GOVERNMENT Chapter

ww
Introduction

w.E
The Constitution provides for a Federal Government having separate administrative systems for the Union and its units,
namely, the states. Provisions for the governance of both are contained in the Constitution. Articles 153 to 167 is Part VI of the
Constitution deal with the state executive. The pattern of Government at the State level is the same as that at the Union level - a

a
parliamentary system, in which the executive head is a constitutional ruler who is to act according to the advice of council of

syE
Ministers responsible to the State Legislature.

State

Executive ngi Legislature Judiciary Centre-state

Chief Council of Advocate


nee High Court
Relations

Governor
Minister Ministers General

STATE EXECUTIVE
Unicameral

r Bicameral

ing (Refer ch-6)

Governor ••
.ne
A President may transfer a Governor appointed to one

Article 153 states that there shall be a Governor for each state.
Same person can be appointed as Governor for 2 or more states
was added by 7th amendment act 1956.
Article 154 states that the executive power of the state is vested
••
in the same State or any other State.

Qualifications & Conditions for office (article


t
State to another State for rest of the term.
A Governor whose term has expired may be reappointed

in him and is exercised by him either directly or through 157 & 158) of Governor
officers subordinate to him. •• He must be –
Appointment & Tenure (Article 155 & 156) ƒƒ A citizen of India.
ƒƒ Has attained 35 years of age.
•• Governor is the executive Head/Nominal Head of the
ƒƒ Not a member of parliament or state legislature.
State.
ƒƒ Not hold any office of profit under the government.
•• Governor of a State is appointed by the President.
•• His emoluments, allowances and privileges are determined
•• Hold office during the pleasure of the President.
by parliament by law.
•• May resign by submitting his resignation to the President. •• The emoluments are charged on the Consolidated Fund of
Otherwise the normal term of his office is 5 years. India and cannot be diminished during his term of office.
•• Grounds for removal of the Governor are not mentioned •• If the same person acts as Governor of 2 or more states,
in the Constitution; however he must be involved in the the Constitution provides that President may decide about
gross deliquency like bribery, treason or violation of the the allocation of emoluments of Governor among states
Constitution for such an action. proportion wise (Article 158(3A)).

Downloaded From : www.EasyEngineering.net


Downloaded From : www.EasyEngineering.net
P-94 State Government

Oath •• Submission of reports from Auditor General, State Public


Service Commission, State Finance Commission, etc.
Article 159 says that the Governor and every person discharging
the functions of the Governor is to take an oath or affirmation before the Legislature.
before the Chief Justice of the High Court of that state, or in Discretionary Powers of Governor
his absence, the senior-most judge of that court available.
•• Discretion of the Governor is wider than that of the
Article 160 gives the president the power to make such
President. Article 163 (2) provides that as to the question
provision as he thinks fit for the discharge of the functions
of the Governor in any contingency not provided for in the of matter of discretion, it is the Governor himself who
Constitution. decides which matter falls in his discretion. And his action
based on such discretion shall not be called in question.
Executive Powers of Governor •• Though in most of the matters he has to act on the advice
•• Article 166 : All executive actions of the state are of Council of Ministers, but there are some matters in
to be taken in the name of the Governor. He acts as a which he can act according to his discretion.
representative of President in the state. He has power to •• He selects the CM if no party has clear-cut majority.
recommend President that the government of the state •• Dismissal of Ministry if he is convinced that it has lost
cannot be carried on in accordance with the provisions of majority support. But SC in S.R. Bommai Vs UOI (1994)
the Constitution. This leads to the imposition of President case directed that his opinion must not be subjective and

••
ww
rule in the State under Article 356.
All major appointments in the state are made by the Governor
– those of CM, Ministers, and Advocate-General (and decide
••
••
compulsory floor-testing must be done.
Dissolving the Legislative Assembly.
Submission of report to the President regarding failure of

w.E
his remuneration), Chairman & members of State Public
Service Commission (PSC), State Election Commissioner
and Finance Commission. Members and Chairman of State
••
constitutional machinery in the State.
Reservation of certain bills for the consideration of the
President (Article 200). He must reserve the bill that

••
a
PSC are however removed by President.

syE
He is the Chancellor of various universities in the state
and appoints their Vice-Chancellors.

Legislative Powers of Governor


endangers the position of high court. In addition, he can
also reserve the bills that are against the provisions of
Constitution, are against larger national interest or DPSP
and those which deal with compulsory acquisition of
•• He is an integral part of the state legislature, though not
a member of it, he discharges some important legislative ngi property under Article 31A.

Financial Powers of Governor


••
functions.
He summons the house(s) of the legislature of state to
meet at such a time and place as he thinks fit. However
••
nee
A money bill cannot be introduced in the Legislative
Assembly of the State without the recommendation of the
Governor.

••
6 months must not lapse between the last sitting in one
session and the first in the next session.
He may prorogue the house(s) and dissolve the legislative
assembly.
••

••
r
No demand of grants can be made except on the

ing
recommendation of the Governor.
The Governor is required to cause to be laid before
••

••
He has right to reserve certain bills for the assent of the
President [Article 200].
He nominates 1/6th of the members of Legislative •• .ne
the house or houses of the legislature “annual financial
statements”, that is “Budget” [Article 202].
He constitutes a finance commission after every five years

••
Council having special knowledge in literature, science,
arts, cooperative movement and social service.
Decides on the question of disqualification of a member
of State Legislature in consultation with Election
municipalities.

Judicial powers of governor


t
to review the financial position of the panchyats and the

Commission. •• Governor appoints judges of the courts below HC


•• His most important power is the ordinance making power •• He is consulted by the President before appointing judges
[Article 213]. of the HC.
•• But the Governor cannot issue an ordinance without the •• Under Article 161 he can grant pardons, reprieves, remission
previous instructions from the President in cases in which– of punishment to the persons convicted under state laws.
(a) Bill would have required his previous sanction. However he has no power to pardon a sentence of death
(b) Required to be reserved under the Constitution for or remit sentence by the court martial (military court).
the assent of the President.
•• The ordinances have to be approved by the state Other Power
legislature, in the same way as the Parliament does in The Governor receives the report of the Auditor General and
case of Presidential ordinances. places it before the State Legislature. He places the report of the
•• The scope of the ordinance making power of the Governor state Public Service Commission along with the observations
is co-extensive with the legislative powers of the State of the Council of Ministers before the State Legislature. As
Legislature and is confined to the subjects mentioned in chancellor of various universities within the jurisdiction of the
state List and Concurrent List. state, he appoints the vice-chancellors of the universities.

Downloaded From : www.EasyEngineering.net


Downloaded From : www.EasyEngineering.net
State Government P-95

Comparison between Powers and Position of that of the State Legislature i.e., five years. However, if the
the president and the Governor term of the State Legislature is extended, the tenure of the
Chief Minister is also extended.
President Governor The Chief Minister recommends to the Governor the names of
The President is not only Each state has its own laws and the persons to be appointed as members of the Council of Ministers,
the Head of the State Governor, who looks after internal and allocates portfolios among them. He can ask any minister
and the Government, he governance of every state. He is the to resign from the Council or drop him from the Council by
is also the Commander- person, who finalises the budget of reshuffling it. He coordinates the working of various ministers
in-Chief of the Armed the state and also has the power to and ensures that the Council works as a team.
Forces. appoint judges in the courts. The Chief Minister is the chief link between the Governor and
The President cannot Governor can exist without the the Council of Ministers and keeps the former informed of
function without the aid aid and advice of the Council of all decisions of the Council. The Chief Minister takes active
and advise of the Council Ministers. part in the deliberations of the State Legislature. He makes all
of Ministers. important policy announcements on the floor of the legislature
The President can grant Governor can suspend, remit or and defends the policies of his government in the house. He
pardon, reprieve, respite, commute a death sentence. The can recommend dissolution of the Legislative Assembly to the
suspension, remission of Governor does not enjoy power Governor even before the expiry of its term. Generally this

ww
commutation in respect to
punishment or sentence
by a Court Martial.
of pardoning a death sentence. advise is accepted by the Governor.

Council of Ministers (COM)

w.E
Every Ordinary Bill, after
it is passed by both the
houses of the Parliament
either single or at a joint
Every Ordinary bill after it
is passed by the Legislative
Assembly in case of a Unicameral
Legislature or both the Houses
•• Article 163 (1) provides that there should be COM with
Chief Minister as the head to aid and advise the Governor
in exercise of his functions.

President for his assent. a


sitting is presented to the

syE
in case a Bicameral Legislature
either in the first instance or in the
second instance is presented to the
Governor for his assent.
••

••
The real executive authority of the state is vested in the
COM, the Governor, however is not bound to act by
the advice of COM in all cases, as he can exercise his
discretionary powers.
Any person may be appointed as a minister, but he ceases
Every Money Bill after
it is passed by both the
Every Money Bill after it is
passed by the State Legislature is ngi to be a minister if he doesn’t become a member of the
state legislature within 6 months (Article 164).
House of the Parliament
is presented to the
President for his assent.
presented to the Governor for his
assent.
••

•• nee
Salaries and allowances of ministers are governed by laws
made by the state legislature.
Ministry which loses the confidence of the legislative
A President needs no Governor can make an ordinance
instructions for making without the instructions from
an ordinance the President only in three cases
i.e. if a bill containing the same
r
assembly should resign. The Governor may choose to

ing
dismiss the ministry if it does not resign and ask the leader
of opposition to constitute an alternative ministry, or if he
feels that no stable ministry can be formed, recommend
provisions would have required
the previous sanction of the ••
President’s rule in the State.
.ne
Like at the Centre, in the states too, the Council of
President for its introduction
into the State Legislature or if he
would have deemed it necessary to
reserve a Bill containing the same
provisions for the consideration
••

••
Ministers of State, & Deputy Minister.
t
Ministers consists of three categories-cabinet ministers,

Article 164 states that the COM is collectively responsible


to Legislative Assembly of state.
The Ministers hold office till the pleasure of the Governor
of the President or it an act of but, the Ministers can be removed only on the advice
the State Legislature containing of CM. Thus the CM can hold Ministers individually
the same provisions would have responsible by removing them in case of non-performance.
been invalid without receiving the
President’s assent. Functions
The Constitution does not assign any formal powers to the
Chief Minister Council of Ministers. However, in practice it has a wide range
of functions. It formulates the policy of the Government and
The Governor is assisted in the discharge of his functions by implements it. It assists the Governor in making all the important
a Council of Ministers headed by the Chief Minister. He is appointments. Most of the important bills are introduced in
the Real Executive Authority (de facto executive). The Chief the legislature by members of the Council of Ministers. It
Minister is appointed by the Governor. Generally, the leader formulates the State budget and submits it to the Legislature
of the majority party in the State Assembly is appointed Chief for its approval. It can present supplementary demands for
Minister, who holds position identical to that of the Prime grants whenever necessary. It also plays an important role in
Minister at the Centre. He enjoys a term that runs parallel to preparing and implementing State Plans.

Downloaded From : www.EasyEngineering.net


Downloaded From : www.EasyEngineering.net
P-96 State Government

Advocate General (Article 165) •• A person qualified to be a judge of a HC can be appointed


as an Advocate General.
•• He is an official of State, corresponding to the Attorney-
•• His remuneration is determined by the Governor.
General of India (Art 76).
•• He can speak and take part in the proceedings of
•• Provides advice to the government of state upon legal matters.
the Legislature of the State but he has no right to vote in
•• Appointed by the Governor, and holds office during his
it.
pleasure.

STATE LEGISLATURE (PART VI, ART. 168-212)


•• Legislature of every State consists of Governor and House •• Their number of members varies between 60 and 500.
(state legislative assembly) or Houses (state legislative However certain States like Sikkim, Goa, Mizoram and
assembly and council). Arunachal Pradesh have less than 60 members.
•• At present, only six states - Maharashtra, Karnataka, •• The Governor may nominate one Anglo-Indian to it.
Bihar, U.P, Andhra Pradesh, J&K have a bicameral •• The reservation of seats has been provided for SCs and
(consisting of 2 houses) Legislature. In the other states it STs on the basis of their population.
is unicameral (consisting of one house). •• According to Article 172, duration of assembly is normally
••

ww
But this list is subject to change as the Constitution provides
a simple procedure without constitutional amendment for
the abolition of the second chamber, i.e. the Legislative
••
5 years. But it may be dissolved earlier by the Governor.
Its term may also be extended by one year at a time
by parliament during national emergency, though this

w.E
Council, in a state where it exists as well as for the
creation if such a chamber in States where there is none.
The procedure involves a resolution at the Legislative
Assembly of the state concerned passed by a majority of
can in no case be extended beyond 6 months after the
proclamation has ceased to operate.

Legislative Council (Article 169)

a
the total membership of the Assembly not being less than

syE
two-thirds of the members actually present and voting,
followed by an Act of Parliament (Article 169).
State Legislative
••
••

••
It is the upper house.
Parliament may by law create or abolish Legislative
Council.

Bicameral Unicameral ngi ••


It can be created, if the Legislative Assembly of the state
passes a resolution to the effect by special majority.
It is not an amendment to the Constitution and therefore it

Legislative
Council
••
nee
can be passed like an ordinary piece of legislation.
Article 171 contains various categories of members.
According to this:

Leg. Council or Leg. Assembly or


r
ƒƒ 1/3rd of its members are elected by legislative
assembly.
ing
ƒƒ 1/3rd by local bodies.
Vidhan Parishad Vidhan Sabha

Lower House
ƒƒ 1/6th nominated by the Governor.
ƒƒ 1/12th are elected by teachers.
ƒƒ 1/5th by university graduates.
.ne
Upper House
Maximum
Strength - 1/3 rd
of total strength of
Maximum
Strength - 500
Minimum
Strength - 60
••
t
The maximum strength of Legislative Council can be
1/3rd of the total membership of Legislative Assembly,
but in no case less than 40.
Assembly. Members are •• Parliament has the final power to decide about its
Minimum Strength - 40 directly elected composition.
Members are by people •• It is not subject to dissolution. But 1/3rd of its members
indirectly elected Term - 5 years retire on the expiry of every 2nd year.
Permanent body
Qualifications
1/3rd of its members
•• Article 173 mentions the qualifications of members as:-
retire every second year.
ƒƒ A citizen of India.
Term - 6 years
ƒƒ Not less than 25 years of age for legislative assembly
and not less than 30 years of age for Legislative
Legislative Assembly (Vidhan Sabha) Council.
•• It is the lower and popular house of the State. Members ƒƒ Possesses such other qualifications as may be
are chosen by direct election on the basis of adult suffrage prescribed by the Parliament.
from territorial constituencies (Article 170). ƒƒ Not hold any office under the Union or state
government.

Downloaded From : www.EasyEngineering.net


Downloaded From : www.EasyEngineering.net
State Government P-97

•• Article 190: No person shall be a member of both houses Deputy Chairman or if he ceases to be a member of the Council.
of State Legislature and if anyone gets elected to both His powers and functions are comparable to the Speaker of the
Houses, he has to vacate one seat. Assembly with few exceptions. Such as the speaker has one
•• No person can become a member of legislature of 2 or special power which is not enjoyed by the Chairman. The speaker
more states. decides whether a bill is a Money Bill or not and his decision on
this question is final.
Presiding Officers of State Legislature Deputy Chairman of Legislative Council
Like the Chairman, the Deputy Chairman also elected by
The Speaker the Council itself from amongst its members. The Deputy
The Speaker is elected by the assembly itself from amongst its Chairman performs the duties of the Chairman’s office when
members and remains in office during the life of the assembly. it is vacant. He also acts as the Chairman, when the latter is
However, he may vacate his office by writing to the Deputy absent from the sitting of the Council. In both the cases, he has
Speaker or be removed by a resolution passed by a majority all the powers of the Chairman.
of all the then members of the assembly or he ceases to be a Comparison of Legislative Council and
member of the assembly. Such a resolution can be moved only
Legislative Assembly

ww
after giving 14 days prior notice.

Powers and Duties of Speaker


•• In the passage of an Ordinary Bill, both the houses enjoy
equal status but in case of disagreement the will of the
Assembly prevails over that of the Council and there is no
••

w.E
His functions are similar to those of the Speaker of the
Lok Sabha. He adjourns the assembly or suspends the
meeting in the absence of a quorum and maintains order ••
provision of joint sitting in this regard.
A Money Bill can be introduced only in Assembly not in
Council.

•• a
and decorum in the assembly for conducting its business
and regulating its proceedings.

syE
He can allow a secret sitting of the house at the request
of the leader of the house. He decides the questions of
••

••
The Council has no participation in the election of the
President.
It also has no effective say in the ratification of the
Constitutional Amendment Bill. Also, the existence of the
disqualification of a member of the assembly, arising on
the ground of defection under the provisions of the Tenth
Schedule of the Constitution. However, he also vacates ngi Council depends on the will of the assembly.

Legislative Procedure

his office earlier in any of the following three cases:
1. If he ceases to be a member of the assembly. nee
The legislative procedure in a State Legislature having two
houses is broadly similar to that in Parliament except for some


2. If he resigns by writing to the deputy speaker.
3. If he is removed by a resolution passed by a majority
of all the members of the assembly. Such resolution
aspects.

Money Bill r ing


••
can be moved only after giving 14 days advance
notice.
• He appoints the Chairman of all the committees of .ne
The position is the same at Union and State Levels : the
Bill can be introduced only in the Assembly; the will of the
Assembly prevails; and the Assembly is not bound to accept
the assembly and supervises their functioning. He himself
is the Chairman of the Business Advisory Committee, the
Rules Committee and the General Purpose Committee.
The speaker decides whether a bill is a Money Bill or not.
Ordinary Bill
t
any recommendation by the Council which may at the most
withhold the bill for 14 days from the date of its receipt.

The only power of the Council is to interpose some delay in


Deputy Speaker the passage of the Bill for a period of three months at the most.
Ultimately the will of the assembly prevails and when the bill
Like the speaker, the Deputy Speaker is also elected by the
comes to the Council a second time the Council can delay it for
assembly itself from amongst its members. He is elected
not more than a month.
after the election of the Speaker has taken place. The Deputy
Speaker performs the duties of the speaker’s office when it is Governor’s Assent
vacant. He also acts as the speaker when the latter is absent When a bill is presented to the Governor after its passage by
from the sitting of assembly. In both the cases, he has all the the houses of the legislature the Governor may (i) declare his
powers of the speaker. assent to the bill, in which case, it would become a law at once;
(ii) declare that he withholds his assent in which case the bill
Chairman of the Legislatuve Council fails to become a law; (iii) return the bill, if it is not a money
The Chairman is elected by the Council itself from amongst its bill with a message; (iv) reserve the bill for the consideration
members. He may vacate his office by resigning by writing to the of the President.

Downloaded From : www.EasyEngineering.net


Downloaded From : www.EasyEngineering.net
P-98 State Government

Comparing Legislative Procedure in the Parliament and State Legislature


Parliament State Legislature
A. With Regard to Ordinary Bills
1. It can be introduced in either house of the Parliament. 1. It can be introduced in either House of the state legislature.
2. It can be introduced either by a minister or by a private 2. It can be introduced either by a minister or by a private
member. member.
3. It passes through first reading, second reading and third 3. It passes through first reading, second reading and third
reading in the originating house. reading in the orginating house.
4. It is deemed to have been passed by the Parliament only 4. It is deemed to have been passed by the state legislature
when both the houses have agreed to it. either with or only when both the houses have agreed to it, either with or
without amendments. without amendments.
5. A deadlock between the two houses takes place when the 5. A deadlock between the two houses takes place when
second house, after receiving a bill passed by the first the Legislative Council, after receiving a bill passed by
House, rejects the bill or proposes amendments that are not the Legislative Assembly, rejects the bill or proposes
acceptable to the first House or does not pass the bill within amendments that are not acceptable to the Legislative

ww
six months. Assembly or does not pass the bill within three months.
6. The Constitution provides for the mechanism of joint sitting 6. The Constitution does not provide for the mechanism of
of two houses of the Parliament to resolve a deadlock joint sitting of two houses of the state legislature to resolve

w.E
between them over the passage of a bill.

the bill for the second time and vice versa. A joint sitting is
the only way to resolve a deadlock between the two houses.
a deadlock between them over the passage of a bill.
7. The Lok Sabha cannot override the Rajya Sabha by passing 7. The Legislative Assembly can override the Legislative
Council by passing the bill for the second time and vice versa.
When a bill is passed by the Assembly for the second time

a syE
and transmitted to the Legislative Council, if the Legislative
Council rejects the bill again or proposes amendments that
are not acceptable to the Legislative Assembly, or does not
pass the bill within one month, then the bill is deemed to

ngi have been passed by both the houses in the form in which it
was passed by the Legislative Assembly for the second time.
8. The mechanism of joint sitting for resolving a deadlock 8. The mechanism of passing the bill for the second time
applies to a bill whether originating in the Lok Sabha or
the Rajya Sabha. If a joint sitting is not summoned by the
President, the bill ends and becomes dead.
nee
to resolve a deadlock applies to a bill originating in
the Legislative Assembly only. When a bill, which has
originated in the Legislative Council and sent to the

r ing
Legislative Assembly, is rejected by the latter, the bill ends
and becomes dead.
B. With Regard to Money Bills

Rajya Sabha. in the Legislative Council.


.ne
1. It can be introduced only in the Lok Sabha and not in the 1. It can be introduced only in the Legislative assembly and not

2. It can be introduced only on the recommendation of the 2. It can be introduced only on the recommendation of the
President. governor.
t
3. It can be introduced only by a minister and not by a private 3. It can be introduced only by a minister and not by a private
member. member.
4. It cannot be rejected or amended by the Rajya Sabha. It 4. It cannot be rejected or amended by the Legislative Council.
should be returned to the Lok Sabha within 14 days, either It should be returned to the legislative Assembly within 14
with or without recommendations. days, either with or without amendments.
5. The Lok Sabha can either accept or reject all or any of the 5. The Legislative Assembly can either accept or rejedll all or
recommendations of the Rajya Sabha. any of the recommendations of the legislative council.
6. If the Lok Sabha accepts any recommendation, the bill is 6. If the legislative assembly accepts any recommendation, the
then deemed to have been passed by both the houses in the bill is then deemed to have been passed by both the houses
modified form. in the modified form.
7. If the Lok Sabha does not accept any recommendation, the 7. If the Legislative Assembly does not accept any
bill is then deemed to have been passed by both the houses recommendation, the bill is then deemed to have been
in the form originally passed by the Lok Sabha without any passed by both the houses in the form originally passed by
change. the legislative Assembly without any change.

Downloaded From : www.EasyEngineering.net


Downloaded From : www.EasyEngineering.net
State Government P-99

8. If the Rajya Sabha does not return the bill to the Lok Sabha 8. If the Legislative Council does not return the bill to the
within 14 days, the bill is deemed to have been passed by Legislative Assembly within 14 days, the bill is deemed to
both the houses at the expiration of the said period in the have been passed by both the houses at the expiration of the
form originally passed by the Lock Sabha. said period in the form originally passed by the Legislative
Assembly.
9. The Constitution does not provide for the resolution of any 9. The Constitution does not provide for the resolution of any
deadlock between the two houses. This is because, the will deadlock between two houses. This is because, the will of
of the Lok Sabha is made to prevail over that of the Rajya the Legislative Assembly is made to prevail over that of
Sabha, if the latter does not agree to the bill passed by the Legislative Council, if the latter does not agree to the bill
former. passed by the former.

The State of Jammu and Kashmir But, the residuary power belongs to the state legislature
except in few matters like prevention of activities involving
The State of Jammu and Kashmir holds a special position terrorist acts, questioning or disrupting the sovereignty
under the Constitution of India. Though it is one of the and territorial integrity of India. Further, the power to
states specified in the First Schedule and forms a part of the make laws of preventive detention in the state belongs
territory of India as defined in Article 1, all the provisions of to the state legislature. This means that the preventive

ww
the Constitution of India relating to the States do not apply to
Jammu and Kashmir. The state alone of all the states of the
Indian Union has its own Constitution. 4.
detention laws made by the Parliament are not applicable
to the state.
Part III (dealing with Fundamental Rights) is applicable

w.E
The Article 370 in Part XXI of the Constitution grants a special
status to state of Jammu and Kashmir. Accordingly, all the
provisions of the Constitution of India do not apply to it. It
is also the only state in the Indian Union which has its own
to the state with some exceptions and conditions. The
fundamental right to property is still guaranteed in the
state.

Kashmir.
a
separate state Constitution the Constitution, i.e. of Jammu and

syE
According the ‘Instrument of Accession of Jammu and
Kashmir to India’ was signed by Pandit Jawaharlal Nehru and
5.

6.
Part IV (dealing with Directive Principles of State Policy)
and Part IV A (dealing with Fundamental Duties) are not
applicable to the state.
A National Emergency declared on the ground of internal
Maharaja Hari Singh on 26 October 1947. Under this, the state
surrendered only three subjects (defence, external affairs and
communications) to the Dominion of India. (At that time, the
ngi 7.
disturbance will not have effect in the state except with the
concurrence of the state government.
The President has no power to declare a financial
Government of India made a commitment that ‘the people of
this state, through their own Constituent Assembly, would
determine the internal Constitution of this state and the nature
nee
emergency in relation to the state.

Constitution of Jammu and Kashmir


and extent of the jurisdiction of the Union of India over the
state, and until the decision of the Constitution of India could
only provide an interim arrangement regarding the state. In
r
The Constitution of Jammu and Kashmir was adopted on

ing
November 17, 1957 and given effect from January 26, 1957.
Some of its important provisions are considered here.
pursuance of this commitment, Article 370 was incorporated
in the Constitution of India. It clearly states that the provisions
with respect to the state of J & K are only temporary and not .ne
The Legislative Assembly consists of 100 numbers elected
directly from territorial constituencies of the state and two
women members nominated by the Governor. Twenty-four
permanent. It became operative on 17 November 1952, with
the following provisions:
In pursuance of the provisions of Article 370, an order issued
by the President the constitutional position of the state and its
t
seats are to remain vacant till filled by representatives of
people in Pakistan occupied Kashmir (PoK). The Legislative
Council consists of 36 member, 11 of whom are elected by
the Assembly from among the people of Kashmir provided
that of the members elected at least one shall be a resident
relationship with the Union. At present, is as follows:
of Tehsil Ladakh and at least one a resident of Tehsil Kargil,
1. Jammu and Kashmir is a constituent state of the Indian
and 11 elected by the Assembly from people of Jammu. The
Union and has its place in Part I and Schedule I of the
remaining 14 members are elected by various electorates,
Constitution of India (dealing with the Union and its
such as municipal councils and such other local bodies. The
Territory). But its name, area or boundary cannot be
High Court of the state consists of a Chief Justice and two
changed by the Union without the consent of its legislature.
or more other Judges. Every Judge is to be appointed by the
2. The state of J & K has its own Constitution. Hence, President after consultation with the Chief Justice of India and
Part VI of the Constitution of India (dealing with state the Governor.
government) is not applicable to this state. The very
The official language of the State is Urdu, but English will
definition of ‘state’ under this part does not include the
continue to be used for official purpose unless the State
state of J & K.
Legislature provides otherwise. And no amendment can be
3. Parliament can make laws in relation to the state on most made to change any provision regarding the relationship of the
of the subjects enumerated in the Union List and on a good state with the Union of India.
number of subjects enumerated in the Concurrent List.

Downloaded From : www.EasyEngineering.net


Downloaded From : www.EasyEngineering.net
P-100 State Government

Administration of Scheduled Areas Tribes Advisory Council


Article 244 in Part X of the Constitution envisages a special The Constitution of India provides for a Tribes Advisory
system of administration for certain areas designated as Council (TAC) in each State having Scheduled areas and if the
‘Scheduled areas’ and ‘tribal areas’. president so directs, also in any State having Scheduled Tribes
The Constitution has made special provision for the but not Scheduled areas. TAC shall consists of not more than
administration of Scheduled Areas in a state other than Assam, 20 members of whom three-fourths shall be the representatives
Meghalaya, Tripura and Mizoram. The right to declare any of the Scheduled Tribes in the Legislative Assembly of the
area as a Scheduled Area rests with the President and is subject State.
to legislation by the Parliament. The Constitution contains The Governor makes rules prescribing or regulating as the case
special provisions regarding the administration of Scheduled may be (i) the number of members of the Council, the mode
Areas, which are contained in the Fifth Schedule. The of their appointment and the appointment of the Chairman of
Union Government can also give directions to the respective the Council and of the officers and servants thereof; (ii) the
State regarding administration of the Scheduled Areas. The conduct of its meetings and its procedure in general; and (iii)
Governor of the state, where such areas are located, has to all other incidental matters.
submit reports to the President regarding the administration
of such areas annually or whenever required by the President. Tribal Areas

ww
To take care of the welfare and advancement of the Scheduled
Tribes in the State, a Tribes Advisory Council is constituted.
In addition, the Governor can also take certain steps to protect
the interests of the Scheduled Tribes. Thus, he can direct that a
These areas are located in the states of Assam, Meghalaya,
Mizoram and Tripura and have been specified in the Sixth
Schedule of the Constitution. Though these areas fall within
the executive authority of the state, provision has been made

w.E
particular Act of Parliament or state legislature shall not apply
to the Scheduled Area. He can make regulations prohibiting
or restricting transfer of land, allotment of land and money
for the creation of district councils and regional councils for
the exercise of certain legislative and judicial powers in these
areas. The district and regional councils are empowered to

approval of the President.


a
lending. However, all these actions of the governor need prior

syE
make laws in certain fields such as management of forests,
marriage and social customs, inheritance of property, etc.
These councils can also impose certain taxes and collect land
revenue.

ngi
CENTRE – STATE RELATIONS
•• The distribution of powers is an essential feature of
federalism. A federal Constitution establishes the dual
polity with the Union at the centre and the States at the
nee
be exercised in the field assigned to them respectively by
the Constitution. The one is not subordinate to the other
in its own field; the authority of one is co-ordinate with
periphery, each endowed with the sovereign powers to
r
the other.

Centre-State Relations ing


Commissions Legislative Relation Legislative Relation .ne Financial

Sarkaria Punchhi Union


List
State Concurrent
List List
Inter
State
Council
Inter
State
Commerce
Zonal
Council
tRelation

Distribution
of Taxes

•• In the Indian federal set-up the Constitution divides being included in the territory of India. It also possesses
powers between centre and states as:- the power of ‘Extra-Territorial Legislation’ which no
ƒƒ Legislative State Legislature possesses.
ƒƒ Administrative •• The State Legislature is competent only to make laws for
ƒƒ Financial the whole or any part of the state to which it belongs.
•• For certain UTs like Andaman & Nicobar and
Legislative Relations Lakshadweep, regulations made by the President have
the same force as the Acts of the Parliament and such
•• Article 245 to 255 in part XI of the Constitution deals with regulations may repeal or amend a law made by Parliament
the Legislative relation between the Centre & the States. in relation to such territory.
•• Article 245(1) of the Constitution of India provides that •• The application of Acts of parliament to any Scheduled
the Parliament has power to make laws for the whole or Area may be barred or modified by notifications made by
any part of territory of India. This includes not only the the Governor.
States but also the UTs or any other area, for the time

Downloaded From : www.EasyEngineering.net


Downloaded From : www.EasyEngineering.net
State Government P-101

•• In distributing the subject matters between Centre and Administrative Relations


states, our Constitution makers followed Canadian •• During the time of emergency Indian Constitution works
scheme. However they added one more list to it, like a unitary government.
Concurrent list. (GOI Act 1935 had 3 fold enumeration –
•• In normal times, there are constitutional provisions which
federal, provincial and concurrent)
ensure the control of the union over the states. Some of
1. Union List – 100 subjects the mechanisms are:
2. State List – 66 subjects •• Power to appoint and dismiss the Governor (articles 155-
3. Concurrent List –52 subjects 156), power to appoint judges of HC, members of state
•• Subjects mentioned in the Union List are of national PSC (articles 217, 317)
importance and only Parliament is competent to make •• Article 256 provides that the executive power of the state
laws on these subjects. shall be so exercised as to ensure compliance with the
•• For State List, only the States have exclusive power to laws made by the parliament and executive power of the
make laws. Union shall also extend to the giving of such directions to
•• Concurrent list is a “Twilight Zone”, where both the a state as it may deem essential for the purpose.
Union and the states are competent to make law, without •• Further article 257 provides that states must exercise their
any conflict. executive power in such a way so as not to impede or
••
ww
Residuary powers (Article 248): Parliament has exclusive
power to make laws on the subjects not enumerated in any
of the lists (Entry 97 of Union List).
prejudice the exercise of the executive power of the Union
in the state.
•• The powers of the Union Government also extend to
••

••
w.E
Wherever the conflict arises as to a subject matter, the
Union List predominates over the other lists.
To determine whether a particular enactment falls under
giving directions to a state in 2 specific matters:
1. Construction and maintenance of means of
communication which are declared to be of national

a
one entry or the other, it is the pith and substance of such

syE
enactment and not its legislative level that is taken account
of. Pith and substance here means the true object of the
legislation or statue, and the competence of legislature

or military importance.
2. Measures to be taken for the protection of the railway
within the states.
•• The Constitution prescribes and coercive sanction for the

••
which enacts it.
Colorable Legislation – The motives of the legislature
are, otherwise, irrelevant for determining whether it has
ngi enforcement of its directions through article 356.

Delegation of Union functions to the States


transgressed the constitutional limits of the legislative
power. This principle is based on the maxim that you can
not do indirectly what you can not do directly.
••

nee
Under article 258 (1), Parliament with the consent of the
state government can entrust to it any matter falling with
in the executive powers of the Union.

•• Under Art 249, in the national interest, Parliament has


the power to make laws w.r.t. any matter included in the
State List, for a temporary period, if Rajya Sabha passes
••
r
Under article 258(2) Parliament is empowered to use state

ing
machinery for the enforcement of Union laws. For such
purpose, it can confer powers or impose duties on state
a resolution supported by 2/3rd of the members present
and voting in that respect.
••
functionaries.

.ne
Therefore under clause (1) delegation of power is made
with the consent of the state, whereas consent of the state
•• Under the proclamation of emergency, Article 250
empowers the Parliament to make laws for the whole
or any part of the territory of India w.r.t. all matters in
the State List. Such a law however shall cease to effect
t
is not necessary under clause (2), and such delegation
is done by the parliament by law. This implies that
parliament can interfere in the state administration, even
without the consent of the State.
on the expiration of 6 months after the proclamation of •• State government has also the power to delegate its
emergency has ceased to operate. functions to the Union and its officers. Article 258 (A)
•• Under Article 252, if the legislatures of two or more provides that Governor of the state with the consent of
States pass a resolution to the effect that it is desirable GOI, entrust to the government, such functions to which
to have a law passed by Parliament on any matters in the the executive power of State extends.
state list common to these states, Parliament can make •• All India services are common to both the Union and the
laws in that respect. states. The officers of these services are appointed and
•• Under Article 253, Parliament has power to make any regulated by the Centre and are placed in various states.
law for the whole or any part of the territory of India for •• Grants in-aid (Article 275): Parliament has power
implementing treaties and international agreements and to make such grants as it may deem necessary to give
conventions. financial assistance to any state which is in need of such
•• Under Article 256, Parliament is empowered to make assistance (Article 275).
laws w.r.t. all matters in the state list when there is failure •• Under Article 263, President has power to establish Inter-
of constitutional machinery of the State under Article 356. State Councils. These Councils have duty of inquiring

Downloaded From : www.EasyEngineering.net


Downloaded From : www.EasyEngineering.net
P-102 State Government

into and advising upon disputes which arises between the 7. Taxes on the entry of
states. These councils also investigate and discuss the goods.
subjects of common interest between the union and the
8. Sales tax.
States or between two or more states.
9. Tolls.
ƒƒ The President has so far established Central Council
of health, a Central Council of local self-government 10. On matters in the State
and a Transport Development Council. List.
•• Parliament has power to constitute an Inter-State 11. On profession, trade, etc.
Commerce Commission (Article 307) and empower it to (not exceeding ` 2,500 per
execute such functions as it may deem fit. annum (List II).
•• Article 261 provides that full faith and credit shall be
given throughout the territory of India to public acts, IMPORTANT SUBJECTS IN VARIOUS LISTS
records and judicial proceedings of the Union and every Union List (List I)
state. 6. Atomic energy and mineral resources.
•• Article 261 (3) declares that final judgement or orders 18. Extradition.
delivered or passed by civil courts in any part of territory
of India can be executed anywhere in the country. 45. Banking.
47. Insurance.
••

ww
Article 262: Adjudication of disputes relating to waters of
inter-state rivers or river valleys. Article 262 (2) provides
that parliament may by law provide that neither the SC
48. Stock exchanges and futures markets.
49. Patents, inventions and designs; copyright; trade-marks

w.E
nor any other court shall exercise jurisdiction in respect
of any such dispute.

Financial Relations
and merchandise marks.
69. Census.
85. Corporation tax.
••

••
relations.
a
Articles 268 to 293 in part XII deal with the financial

syE
These relations are related to the distribution of taxes as
well as non-tax revenue and the power of borrowing.
97. Any other matter not enumerated in List II or List III
including any tax not mentioned in either of those lists.

1. Public order.
State List (List II)

••

••
Grant-in-aid forms an important part of centre-state
relations.
Parliament can levy taxes on the subjects enlisted in the ngi 5. Local government.
6. Public health and sanitation.
12. Libraries, museums and other similar institutions.

••
List - I (Union List) while the states can levy taxes on the
subjects mentioned in the State List. (List II).
There are no taxes on the subjects of the Concurrent List
nee
14. Agriculture.
21. Fisheries.

••
(List III).
Finance Commission (Article 280) recommends to the
President on the distribution of net proceeds of taxes
r
25. Gas and gas-works.
28. Markets and fairs.
61. Captivation taxes.
ing
Concurrent List (List III)
between the Centre and states.
Distribution of Taxes
1. Criminal law.
2. Criminal procedure. .ne
Taxes Belonging to Union
Exclusively
A. Customs
B. Corporation
C. Taxes on capital values of
Taxes Belonging to States
Exclusively
1. Land revenue.
2. Stamp duty except in
documents included in the
3. Preventive detention.
5. Marriage and divorce. t
6. Transfer of property other than agricultural land.
7. Contracts.
assets of Individuals and Union List. 13. Civil procedure.
Company. 3. Succession duty, estate 14. Contempt of court, but not including contempt of the
D. Surcharge on income duty and income tax on Supreme Court.
Tax. agricultural land. 17. Prevention of cruelty to animals.
E. Fees in respect of matters 4. Taxes on passengers and 20. Economic and social planning.
in Union List. goods carried on inland
waterways. 26. Legal, medical and other professions.
5. Taxes on lands and 38. Electricity.
buildings, mineral rights. 40. Archaeological sites.
6. Taxes on animals and
boats, road vehicles, on
Administration of Union Territories
advertisements, electricity, There is no uniform system of administration in the union
luxuries and amusements, territories. Parliament has been vested with the power to
etc. prescribe the structure of administration in the various union

Downloaded From : www.EasyEngineering.net


Downloaded From : www.EasyEngineering.net
State Government P-103

territories. The administrators of union territories are variously be disturbed except for some ex¬tremely compelling
known as Lieutenant Governors, Chief Commissioners or reasons.
Administrators. In Daman and Diu and Pondicherry, they 8. The surcharge on income tax should not be levied by the
are designated as Lieutenant Governors. In Andaman and Centre except for a specific purpose and for a strictly
Nicobar Islands and Chandigarh they are known as Chief limited period.
Commissioners and in Lakshadweep as Administrator. 9. When the president withholds his assent to the state
Similarly, some Union Territories possess Legislative bills, the reasons should to communicated to the state
Assemblies and Councils of Ministers such as Daman and government.
Diu, Pondicherry, and Delhi (National Capital Territory), 10. The Union Government should consult the Government
while others do not. It may be noted that in Union Territories of the States before enacting any law related to a subject
with Legislative Assemblies, the right to legislate on subjects included in the Concurrent List.
enumerated in the State List and Concurrent List vests with 11. As provided in Article 263, an Inter-State Council should
the Assembly. With respect to other union territories, the laws be constituted to resolve differences between the Union
are enacted by the Parliament. The administrators of the union and the States. The Chief Ministers of all the states should
territories enjoy the right to issue ordinance within certain be Ex-officio Members of the Council.
limitations. When the legislatures of the union territories are
12. The states should be given right to amend the “subjects
dissolved or suspended, responsibility for the peace, progress

ww
and good government of the territory falls on the President.

Commissions for the Improvement of Centre-


included in State List, this requires an amendment to
the Article 252 of the Constitution. An Expenditure
Commission should be set-up at the Union level. The
Corporation Tax should be distributed between Centre
State Relations
w.E
There have been several efforts for the improvement of
Centre-State relations. Central Government have set-up many
and States.

MM Punchhi Commission

a
commissions to review and examine the Federal Relations
from time to time, leading among them are

Sarkaria Commission syE


A Commission on Centre-State Relations was set-up by the
Government of India in April, 2007 under the Chairmanship
of Madan Mohan Punchhi, Former Chief Justice of India to
look into the issues of Centre-State Relations. Some of the
Sarkaria Commission was set-up in June, 1983 by the Central
Government of India. The Sarkaria Commission’s charter was
to examine the relationship and balance of power between ngi recommendations given in its various reports are as below
•• It recommended higher central transfers to backward
states to enable them to improve their physical and human
State and Central Governments in the country and suggest
changes within the framework of Constitution of India. The
commission was so named as it was headed by Justice Rajinder nee
infrastructure.
•• It recommended for the adoption of a multi-pronged
strategy in the backward regions of the country comprising
Singh Sarkaria, a retired Judge of the Supreme Court of
India and other two members of the committee were Shri B
Sivaraman and Dr SR Sen. r ing
public investment in infrastructure development,
proactive policies to attract private investment, higher
public expenditure on social sectors, such as health and
The Commission made 247 recommendations to improve
Centre-state relations. The important recommendations are
mentioned below:
agricultural production.
.ne
education and area specific strategy for the growth of

•• There should be much better coordination between the


1. A permanent Inter-State Council called the Inter-
Governmental Council should be set up under Article
263.
2. Article 356 (President’s Rule) should be used very
t
Finance Commission and the Planning Commission. The
synchronisation of the periods covered by the Finance
Commission and the Five Year Plan will considerably
improve such coordination. It recommended that another
sparingly, in extreme cases as a last resort when all the attempt be made to synchronise the periods.
available alternatives fail. •• There should be mandatory devolution of functions to
3. The institution of All-India Services should be further local government and it must be done by the year 2015.
strengthened and some more such services should be Priority should be given to items pertaining to basic
created. needs. Articles 243 G and 243 W should be amended to
4. The residuary powers of taxation should continue to mandate that devolution of functions as listed out in the
remain with the Parliament, while the other residuary Eleventh and Twelfth Schedules, together with the powers
powers should be placed in the Concurrent List. and authority to implement them, should be done by the
5. The National Development Council (NDC) should be year 2015.
renamed and reconstituted as the National Economic and •• The Collector is overburdened and hence, there should
Development Council (NEDC). be a separate administrative structure for the district
6. The Centre should consult the states before making a law Panchayat.
on a subject of the Concurrent List. •• The Constitution needs to be amended to provide a
7. The governor’s term of five years in a state should not specific entry in List 1 (Union List) of the Seventh

Downloaded From : www.EasyEngineering.net


Downloaded From : www.EasyEngineering.net
P-104 State Government

Schedule empowering the Union on matters concerning Inter-State Councils


environment, ecology and climate change.
•• The power of the President to set-up Inter-State Councils
•• The National Water Resources Council needs to play may be exercised not only for advising upon disputes,
a greater role in integrating policy and programmes on a but also for the purpose of investigating and discussing
continuous basis. subjects, in which some or all of the States or the Union
•• Government should ideally explore the possibilities of and one or more of the States or the Union have a common
setting up a single regulator for tariff regulation of power, interest.
coal and gas. •• In the exercise of this power the President has already
constituted the Central Council of Health, the Central
Inter-State Water Disputes
Council of Local Self-Government, the Central Council
•• Article 262 of the Constitution envisages the adjudication of Indian Medicine, the Central Council of Homeopathy
of inter-state water disputes which make two provisions the changing role of Inter-State Council.
1. Parliament may by law provide for the adjudication •• The Inter-State Council was set-up under Article 263 of
of any dispute or complaint with respect to the use, the Constitution of India on 28th May, 1990. If at any time
distribution and control of waters of any inter-state it appears to the President that the public would be served
river and river valley. by the establishment of a council charged with the duty of
2. Parliament may also provide that neither the Supreme

••
ww Court nor any other court is to exercise jurisdiction
in respect of any such dispute or complaint.
Under this provision, the Parliament has enacted two
ƒƒ inquiring into and advising upon disputes, which
may have arisen between states;
ƒƒ investigating and discussing subjects in which some

••
w.E
laws i.e. the River Boards Act of 1956 and the Inter-State
Water Disputes Act of 1956.
The River Board Acts, envisions the establishment of
or all of the States or the Union and one or more of
the states, have a common interest or
ƒƒ making recommendations upon any such subject
and in particular recommendations for the better

a
rivers boards for the regulation and development of

syE
inter-state river and river valleys. The Inter-State Water
Disputes Act empowers the federal government (i.e.
Central Government) to set-up an adhoc tribunal for the
••
coordination of policy and action with respect to that
subject;
It shall be lawful for the President by order to establish
such a council and to define the nature of the duties to be
adjudication of a dispute between two or more states
in relation to the waters of an inter-state river or river
valleys. ngi performed by it and its organisation and procedure.

Composition of Inter-State Council

Name
Inter-State Water Dispute Tribunals
Set-up in State Involved
nee
The Composition of the Inter-State Council includes the
Prime Minister, Chief Ministers of all States, Chief Ministers

Krishna Water Disputes


Tribunal
1969 Maharashtra,
Karnataka and Andhra
Pradesh
r
of Union Territories having Legislative Assemblies and

ing
Administrators of Union Territories not having Legislative
Assemblies, Governors of states under President Rule, six
Ministers of Cabinet rank in the Union Council of Ministers’
Godavari Water Disputes
Tribunal
1969 Maharashtra,
Karnataka, Andhra .ne
to be nominated by the Prime minister and two Ministers of
Cabinet rank in the Union Council of Ministers to be nominated

Narmada Water Disputes


Tribunal
1969
Pradesh, Madhya
Pradesh and Odisha
Rajasthan, Gujarat,
Madhya Pradesh and
by the Prime Minister permanent invites.

Duties of the Council


••
••
t
To investigate and discuss subjects of common interest.
Make recommendations for the better coordination of
Maharashtra
policy and actions on such subjects; and
Ravi and Beas Water 1986 Punjab and Haryana
•• Deliberate on such matters of general interest to the states
Disputes Tribunal
referred by the Chairman to the Council. It shall have its
Cauvery Water Disputes 1990 Karnataka, Kerala, own Secretariat.
Tribunal Tamil Nadu and
Puducherry INTER-STATE TRADE AND COMMERCE
Second Krishna Water 2004 Maharashtra, Articles 301 to 307 in Part XIII of the Constitution deal with
Disputes Tribunal Karnataka and Andhra the trade, commerce and intercourse within the territory of
Pradesh India.
Vansadhara Water 2010 Odisha and Andhra Article 301 declares that trade, commerce and intercourse
Disputes Tribunal Pradesh throughout the territory of India shall be free. The object of
Mahadayi Water Disputes 2010 Goa, Karnataka and this provision is to break down the border barriers between
Tribunal Maharashtra the states and to create one unit with a view to encourage the
free flow of trade, commerce and intercourse in the country.

Downloaded From : www.EasyEngineering.net


Downloaded From : www.EasyEngineering.net
State Government P-105

The freedom under this provision is not confined to inter-state bodies. They are established by an Act of the Parliament, that
trade, commerce and intercourse but also extends to intra-state is, States Reorganisation Act of 1956. The act divided the
trade, commerce and intercourse. Thus, Article 301 will be country into five zones (Northern, Central, Eastern, Western,
violated whether restrictions are imposed at the frontier of any Southern) and provided a zonal council for each zone.
state or at any prior or subsequent stage. Each zonal council consists of the follow members:
The freedom guaranteed by Article 301 is a freedom from all (a) Home Minister of Central government.
restrictions, except those which are provided for in the other (b) chief ministers of all the States in the zone.
provisions (Articles 302 to 305) of Part XIII of the Constitution (c) Two other ministers from each state in the zone.
itself. These are explained below:
(d) Administrator of each Union Territory in the zone.
(i) Parliament can impose restrictions on the freedom of
The Home Minister of Central Government is the common
trade, commerce and intercourse between the states or
chairman of the five zonal councils. Each Chief Minister acts
within a state in public interest. But, the Parliament cannot
as a vice-chairman of the council by rotation, holding office for
give preference to one state over another or discriminate
a period of one year at a time.
between the states except in the case of scarcity of goods
in any part of India. The Zonal Councils aim at promoting cooperation and
coordination between states, union territories and the Centre.
(ii) The legislature of a state can impose reason able restrictions
They discuss and make recommendations regarding matters
on the freedom of trade, commerce and intercourse with

ww
that state or within that state in public interest. But, a bill
for this purpose can be introduced in the legislature only
with the previous sanction of the president. Further, the
like economic and social planning, linguistic minorities,
border disputes, inter-state transport, and so on. They are only
deliberative and advisory bodies.

w.E
state legislature cannot give preference to one state over
another or discriminate between the states.
(iii) The legislature of a state can impose on goods imported
The objectives (or the functions) of the Zonal Councils, in
detail, are as follows:
•• To achieve an emotional integration of the country.

a
from other states or the union territories any tax to which

syE
similar goods manufactured in that state are subject. This
provision prohibits the imposition of discriminatory taxes
by the state.
•• To help in arresting the growth of acute state-
consciousness, regionalism, linguism and particularistic
trends.
•• To help in removing the after-effects separation in some
(iv) The freedom (under Article 301) is subject to the
nationalisation laws (i.e. laws providing for monopolies
in favour of the Centre or the states). Thus, the Parliament ngi cases so that the process of reorganisation, integration and
economic advancement may synchronise.
•• To enable the Centre and states to cooperate with each
or the state legislature can make laws for the carrying
on by the respective government of any trade, business,
industry or service, whether to the exclusion, complete or
nee
other in social and economic matters and exchange ideas
and experience in order to evolve uniform policies.
•• To cooperate with each other in the successful and speedy
partial, of citizens or otherwise.
The Parliament can appoint an appropriate authority for
carrying out the purposes of the above provisions relating to the
r
execution of major development projects.

ing
•• To secure some kind of political equilibrium between
different regions of the country.
freedom of trade, commerce and intercourse and restrictions
on it. The Parliament can also confer on that authority the
necessary powers and duties. But, no such authority has been
North-Eastern Council
.ne
In addition to the above Zonal Councils, a North-Eastern
appointed so far.

ZONAL COUNCILS
The Zonal Councils are the statutory (and not the constitutional)
t
Council was created by a separate Act of Parliament– the
North-Eastern Council Act of 1971. Its members include
Assam, Manipur, Mizoram, Arunachal Pradesh, Nagaland,
Meghalaya, Tripura and Sikkim.

Zonal Councils at a glance


Name Members Headquarters
1. Northern Zonal Council Jammu and Kashmir, Himachal Pradesh, Haryana, Punjab, Rajasthan, New Delhi
Delhi, and Chandigarh
2. Central Zonal Council Uttar Pradesh, Uttarakhand, Chhattisgarh, and Madhya Pradesh Allahabad
3. Eastern Zonal Council Bihar, Jharkhand. West Bengal and Orissa Kolkata
4. Western Zonal Council Gujarat. Maharastra, Goa, Dadra and Nagar Haveli and Daman and Mumbai
Diu
5. Southern Zonal Council Andhra Pradesh, Karnataka, Tamil Nadu, Kerala and Puducherry Chennai

Downloaded From : www.EasyEngineering.net


Downloaded From : www.EasyEngineering.net
P-106 State Government

Articles Related to Inter-State Relations at a Glance


Article No. Subject-matter
Mutual Recognition of Public Acts, etc.
261. Public acts, records and judicial proceedings
Disputes Relating to Waters
262. Adjudication of disputes relating to waters of Inter-State Rivers or river valleys
Co-ordination between States
263. Provisions with respect to an inter-state council
Inter-State Trade and Commerce
301. Freedom of trade, commerce and intercourse
302. Power of Parliament to impose restrictions on trade, commerce and intercourse
303. Restrictions on the legislative powers of the Union and of the states with regard to trade and commerce
304. Restrictions on trade, commerce and intercourse among states

ww
305.
306.
307.
Saving of existing laws and laws providing for state monopolies
Power of certain states in Part B of the First Schedule to impose restrictions on trade and commerce (Repealed)
Appointment of authority for carrying out the purposes of Articles 301 to 304

w.E
a syE
ngi
nee
r ing
.ne
t

Downloaded From : www.EasyEngineering.net


Downloaded From : www.EasyEngineering.net
State Government P-107

1.
Exercise -
A Governor holds office:
1 10. J & K Constitution was framed by:
(a) for five years (a) Constituent Assembly which framed India’s
(b) for a period specified by the Parliament Constitution
(c) during the pleasure of the President (b) Constituent Assembly set up by the Parliament
(d) till he has the confidence of the Parliament (c) Constituent Assembly set up by the State
2. In appointing a Governor, the President consults the (d) the State Legislature
Chief Minister of the State as this is: 11. An amendment to the Constitution of India extends to
(a) constitutionally imperative J&K:
(b) a convention (a) automatically
(c) as Parliament has legislated to the effect (b) only if ratified by State Legislature
(d) A duty of the President (c) by an order of the President under Article 370
(d) Can never be extended
3. Dual role of the Governor means:
12. If the State government fails to comply with the directions
(a) Constitutional and real executive


ww
(b) Head of a state and head of government under
certain circumstances
(c) Belonging both to Central and State executive


of the Centre in the exercise of administrative powers:
(a) Governor may be directed by the President to
dismiss the ministry
(b) President can declare a national emergency and

w.E
(d) Constitutional ruler and represents the Centre
4. Governor of which State has been vested with special
powers for scheduled tribes?
convert the federal structure into a unitary one
(c) Constitutional emergency can be declared and
the President can assume the powers of the State

(c) Maharashtra
a
(a) Arunachal Pradesh (b) Assam

syE
(d) West Bengal
5. ____ presides over meetings of Council of Ministers in a


State?
(a) Governor (b) Chief Minister


government
(d) Supreme Court may be asked to intervene
13. Privileges of the State Legislature are mentioned in
Article:
(a) 105 of the Constitution
(c) Senior ministers (d) Both (a) and (b)
6. To be a member of a State Council of Ministers, a person: ngi

(b) 194 of the Constitution
(c) chapter on Fundamental Rights under Article 19


(a) must belong to the Legislative Assembly
(b) must get membership of the State Legislature within
six months

nee
(d) nowhere in the Constitution as they have evolved as
part of parliamentary convention
14. Mizoram and Sikkim may have a maximum of :
(a) 7 ministers


(c) cannot be less than 35 years old
(d) an expert in some field if he is not a member of the
Legislature
7. Chairman of Legislative Council is:



r
(b) 12 ministers

ing
(c) 15% of their lower house members
(d) 5 % of the lower house members


(a) appointed by the Governor
(b) the Governor
.ne
15. If in an election of State Legislative Assembly, the
candidate who is not elected loses his deposit, it means
that


(c) elected by the members of the Legislative Council
from among themselves
(d) appointed by the Speaker of the Assembly
8. For enactment of a law, the State Legislative Council:
(a) has to pass the bill



(a) the polling was very poor
t
(b) the election was for a multi-member constituency
(c) the elected candidate’s victory over his nearest rival
was marginal
(d) he did not get the required number of minimum
(b) may delay it for a maximum of four months votes
(c) may disagree to, its provisions, if a joint sitting is 16. The Constitution says that the state council of ministers
called hold office during the pleasure of the Governor. The
(d) has nothing to do with the bill words “during the pleasure of the Governor” in reality
9. Limitations on authority of the State Legislature means :
EXCLUDES: (a) Pleasure of the President
(a) Parliament’s authority to make laws on subjects in (b) Pleasure of the Prime Minister
the State List during an Emergency (c) Pleasure of the Chief Minister
(b) Parliament’s authority to make laws on state subjects (d) Pleasure of the Legislative Assembly
if Rajya Sabha passes a reso-lution 17. The Chief Minister of a state is
(c) Governor’s discretionary power to dissolve the (a) elected by the State Legislature
legislature (b) appointed by the Governor
(d) Governors power to reserve bills for consideration (c) appointed by the President
of the President (d) None of the above

Downloaded From : www.EasyEngineering.net


Downloaded From : www.EasyEngineering.net
P-108 State Government

18. The oath of office is administered to the members of the 27. As per the Constitution of India, what is the limit
State Council of Ministers by the prescribed for the number of members in the
(a) Governor Legislative Assembly of a State?
(b) Chief Minister (a) 350 members (b) 400 members
(c) Chief Justice of the State High Court (c) 450 members (d) 500 members
(d) Speaker of Legislative Assembly
28. The Legislative Council in a State in India can be created
19. There is a constitutional requirement to have a minister is
charge of tribal welfare for the states of or abolished by the
(a) Assam, Nagaland and Manipur (a) Parliament on the recommendation of a Governor of
(b) Himachal Pradesh, Haryana and Rajasthan the state.
(c) Bihar, Madhya Pradesh and Odisha (b) Parliament alone
(d) Manipur, Tripura and Meghalaya (c) Parliament after the state assembly passes the
20. Oath of office is administered to the Governor by: resolution of that effect.
(a) Chief Justice of India (d) Governor of the state on the recommendation of the
(b) President Council of Ministers.
(c) Chief Justice of High Court
29. With respect to Article 371 A of the Constitution of
(d) Speaker of Legislative Assembly
21. As per the Constitution of India, what is the limit India, the Governor of which one of the following States
prescribed for the number of members in the Legislative has special responsibility with respect to law and order of


ww
Assembly of a State?
(a) 350 members
(c) 450 members
(b) 400 members
(d) 500 members
the State?
(a) Asom
(c) Nagaland
(b) Manipur
(d) Andhra Pradesh

w.E
22. The Legislative Council in a State in India can be created
or abolished by the
(a) Parliament on the recommendation of a Governor of
the state.
30. According to Article 164(1) of the Constitution of India,
in three States there shall be a Minister in charge of
tribal welfare who may in addition be in charge of the
welfare of the Scheduled Castes and Backward Classes.


a
(b) Parliament alone

resolution of that effect.


syE
(c) Parliament after the state assembly passes the

(d) Governor of the state on the recommendation of the


Council of Ministers.
Which one of the following States is not covered by the
Article?
(a) Jharkhand

(b) Punjab
(c) Madhya Pradesh (d) Odisha
23. The State Government’s responsibility for educational


planning is shared by the
(a) Ministery of Programme implementation ngi 31. Who among the following recommends to the Parliament
for the abolition of the Legislative Council in a State?

nee
(a) The President of India
(b) Ministry of Human Resource Development (b) The Governor of the concerned State
(c) Ministry of Planning
(c) The Legislative Council of the concerned State
(d) Ministry of Home Affairs
24. The Union Legislature cannot legislate on a subject in the



State List unless:
(a) The President call upon it to do so
(b) The Rajya Sabha passes a resolution that it is


r
(d) The Legislative Assembly of the concerned State

Parishad?
(a) Bihar ing
32. Which one of the following states does not have Vidhan

(b) Maharashtra



necessary in national interest to do so
(c) The Speaker certifies that it is necessary
(d) There is a national emergency
(c) Tamil Nadu
.ne
(d) Uttar Pradesh
33. The Constitution of India divided the states of India in
categories A, B, C and D in the year 1950.
25. If the States fail to carry out the directives of the


Central Government with regard to exercise of their
administrative powers:
(a) the President can impose President’s rule on the State
and assume all the powers of the State Government


t
In this context which of the following statements is correct
?
(a) The Chief Commissioner was the executive head
of category A states. The Rajpramukh was the
(b) the President can send reserve police to the State for executive head of category B states. The Governor
the implementation of these directives was the executive head of categories C and D states
(c) the President can dispatch army for their (b) The Rajpramukh was the executive head of category
implementation A states. The Chief Commissioner was the executive
(d) the President can direct the Governor to dismiss the head of categories B and C states. The Governor
State Council of Ministers was the executive head of the category D states
26. If in an election to a state Legislative Assembly the (c) The Governor was the executive head of category A
candidate who is declared elected losses his deposit, it states. The Rajpramukh was the executive head of
means that: category B states. The Chief Commissioner was the
(a) A very large number of candidate contested the executive head of categories C and D states
election (d) The Governor was the executive head of category A
(b) The elected candidate’s victory over his nearest states. The Chief Commissioner was the executive
rival was very marginal head of category B states. The Rajpramukh was the
(c) The election was for a multi-member constituency executive head of categories C and D states
(d) All of the above

Downloaded From : www.EasyEngineering.net


Downloaded From : www.EasyEngineering.net
State Government P-109

34. Which one among the following is not a recommendation 41. Article 154 states that the Governor can exercise his
of the Sarkaria Commission on the appointment of the executive authority either directly of through officers
Governor in a state? subordiante to him. The word subordinates includes :
(a) He/She must not have participated in active politics (a) All the ministers and the Chief Minister
at least for sometime before his/her appointment as (b) All the ministers except the Chief Minister
Governor (c) Only the Chief Minister and the Deputy Chief
(b) The Chief Justice of the Supreme Court may be Minister
consulted by the President in selecting a Governor (d) Only the Cabinet Minister
(c) The Governor’s term of office of five years should 42. Which of the following is not correctly matched?
not be disturbed except very rarely (a) Article 153 - Office of the Governor
(d) The Governor should not be the native of the state (b) Article 156 - Term of the Governor
35. The Governor may recommend the imposition of the (c) Article 154 - Executive authority of Governor
President’s rule in the state (d) Article 155 - Removal of Governor
(a) on the recommendation of the State Legislature 43. Who/Which of the following can abolish a State
(b) on the recommendation of the President Legislative Council.
(c) on the recommendation of the Chief Minister (a) Parliament (b) President
(c) Governor (d) State Assembly

ww
(d) if he is convinced thai the Government of the
State cannot be carried on in accordance with the
provisions of the Constitution of India
44. Union Territories are administered by the:
(a) Parliament
(b) Union Council of Ministers

w.E
36. Which one of the following States of India does not have
a Legislative Council even though the Constitution?
(Seventh Amendment) Act, 1956 provides for it?
(a) Maharashtra (b) Bihar


(c) President through administrators
(d) Prime Minister
45. A government is federal or unitary on the basis of relation
(c) Karnataka
a (d) Madhya Pradesh

syE
37. Which one of the following Articles of the Constitution
of India says that the executive power of every State shall
be so exercised as not to impede or prejudice the exercise



between the:
(a) three organs of the government
(b) Centre and the States
(c) Legislature and the Executive


of the executive power of the Union?
(a) Article 257 ngi (d) Constitution and the States
46. Grant in aid are provided every year to States in need of
assistance as recommended by the



(b) Article 258
(c) Article 355
(d) Article 358

nee
(a) President (b) Parliament
(c) Finance Commission (d) None of the above
47. Which statements regarding the levying, collection and
38. In which one of the following areas does the State


Government not have control over its local bodies?
(a) Citizens’ grievances
r ing
distribution of Income Tax is correct?
(a) Union levies, collects and distributes the collection
between itself and the states

.ne
(b) Financial matters
(b) Union levies, collects and keeps all the proceeds of
(c) Legislation
income tax
(d) Personnel matters
(c) Union levies and collects the tax but the proceeds
39. Which one of the following statements is correct?


(a) In India, the same person cannot be appointed as
Governor for two or more States at the same time
(b) The Judges of the High Court of the States in India

are allocated among the states
t
(d) Only the surcharge levied on income tax is shared
between the Union and the states
48. Which one of the following is not among the sources of
are appointed by the Governor of the State just as Union Revenues?
the Judges of Supreme Court are appointed by the (a) Taxes on income other than agricultural income
President (b) Land Revenue
(c) No procedure has been laid down in the Constitution (c) Custom duties including export duties
of India for the removal of a Governor from his/her (d) Duties of excise on tobacco and other goods
post manufactured or produced in India except alcoholic
(d) In the case of a Union Territory having a legislative liquors, opium, Indian hemp and other narcotic
setup, the Chief Minister is appointed by the Lt. drugs.
Governor on the basis of majority support 49. Which one is not the component of the ‘Pradhan Mantri
40. Which of the following constitutional Amendment Act Gramodaya Yojna’?
provided for the appointment of the same person as (a) Elementary Education
Governor for two or more states? (b) Primary health
(a) 4th Amnd. (b) 7th Amnd (c) Rural Road
(c) 11th Amnd (d) 24th Amnd (d) Nutrition

Downloaded From : www.EasyEngineering.net


Downloaded From : www.EasyEngineering.net
P-110 State Government

50. Which one of the following is charged to the Consolidated 54. The Sarkaria Commission Report deal with which one
Fund of India and can be spent without authorization by the following?
the Parliament? (a) Corruption in India (b) Centre-state relations
(a) Debit charges for which the Government of India is (c) local governance (d) Inter-river dispute
liable 55. Sarkaria Commission was established to study
(b) Salary and allowances of President (a) President and Governoers’ relations
(c) Salaries, allowances and pensions of the judges of (b) Centre-State relations
Supreme Court and High Courts (c) State and Panchayat body relations
(d) all the above (d) President and Prime Minister’ relations
51. The entry “Public health and Sanitation” is included in 56. Which one among the following pairs of level of
the Constitution of India in government and legislative power is not correctly matched ?
(a) Union List (b) State List (a) Central Government : Union List
(c) Concurrent List (d) None of these (b) Local Governments : Residuary powers
52. Which among the following is the exclusive jurisdiction (c) State Governments : State List
of the State Government? (d) Central and State Government : Concurrent List
(a) Corporation tax (b) Customs duty 57. With reference to the Constitution of India, which one of
(c) Sales tax (d) Income tax the following pairs is not correctly matched?

ww
53. The Parliament can legislate on the subjects in the State
List if the
(a) President issues an order authorizing it to do so




(a) Forests
(b) Stock Exchange
: Concurrent List
: Concurrent List
(c) Post Office Savings Bank : Union List
(d) Public Health : State List

w.E
(b) Supreme Court gives authority to the Parliament in
this regard
(c) Rajya Sabha passes a resolution by two-thirds of its
members present and voting, declaring it expedient
58. Which one of the following subjects is under the Union


List in the Seventh Schedule of the Constitution of India ?
(a) Regulation of labour and safety in mines and oilfields


a
to legislate on a State matter in the national interest

syE
(d) Prime Minister issues a special order
(b) Agriculture
(c) Fisheries
(d) Public Health

ngi
nee
r ing
.ne
t

Downloaded From : www.EasyEngineering.net


Downloaded From : www.EasyEngineering.net
State Government P-111

Exercise - 2 6. The correct statements about ordinance making power of


Statement Based MCQ the Governor are:
1. It is laid down in Article 213.
1. Which of the following is a discretionary powers of the 2. It can be issued by him after the advice of the
Governor? President or state council of ministers.
1. Selecting a chief minister if no single party has a 3. It is co-extensive with the legislative power of the
clear majority. state legislature.
2. Dismissing the ministry at any time. 4. It can be issued only during the recess of State
3. Reserving a bill for the President. Legislative Assembly and not the Legislative
Which of the following statements is/are correct? Council.
(a) 1 and 3 (b) 1 and 2 5. It can not be withdrawn by him anytime.
(c) 3 only (d) 1, 2 and 3 Which of the following statements is/are correct?
2. Legislative Council of a State: (a) 2,3 and 4 (b) 1,3 and 5
1. is not subject to dissolution (c) 1, 2 and 3 (d) 2, 4 and 5


ww
2. can be abolished by the State Legislative Assembly
3. can be abolished by the President on Governor’s
recommendation
7.

The Governor of state :
1. Possesses executive, legislative and judicial powers
analogous to the President.



(a) 1 only
(c) 1 and 2 w.E
Which of the following statements is/are correct?
(b) 2 only
(d) 3 only


2. Has to act with the aid and advice of the council of
ministers always.
3. Has the power to appoint and remove the members

a
3. Which of the following is/are required for a Legislative of State Public Service Commission.



1. Act of Parliament
syE
Council in a State to be created or abolished?

2. Resolution of the Legislative Assembly of the State


concerned



4. Has the power to allocate business of the government
among the various ministers.
Of the above, the correct statement are :
(a) 1 and 2 (b) 2, 3 and 4



3. Recommendation by the Governor of the State concerned
4. Constitutional Amendment requiring States’ ratification
Which of the following statements is/are correct?
ngi
8.
(c) 1 and 4 (d) 1, 3 and 4
Article 156 of the Constitution of India provides that a
Governor shall hold office for a term of 5 years from the


(a) 1 only
(c) 1, 2 and 3
(b) 1 and 2
(d) 2 and 4
4. R1 : The legislative Council is inferior to legislative Assembly
nee
date on which he enters upon his office. Which of the
following can be deducted from this?
1. No Governor can be removed from office till the
R2 : The very existence of the Council depends on the
will of the Assembly.
R2 : A bill originating in the Council can be put to an end

r ing
completion of his term.
2. No Governor can continue in office beyond a period
of five years.
by the Assembly rejecting it.
R3 : 1/6 of the Council’s members are nominated by the
Governor.



(a) 1 only
(c) Both 1 and 2 .ne
Which of the following statements is/are correct?
(b) 2 only
(d) Neither


3.
1. A and R1, R2 and R3 are correct and R1, R2 and, R3
explain A
2. A, Rl and R3 are correct and Rl and R3 explain A
A, Rl, R2 and R3 are correct but only Rl and R2 explain
9.


given to the Governor of a State?
t
Which of the following are the discretionary powers

1. Sending a report to the President of India for


imposing the President’s rule
2. Appointing the Ministers
A 3. Reserving certain bills passed by the State
4. A and R2 are wrong; Rl and R3, are correct. Legislature for consideration of the President of
Which of the following statements is/are correct? India
(a) 1 only (b) 2 and 3 4. Making the rules to conduct the business of the
(c) 1 and 4 (d) 2, 3 and 4 State Government
5. Which of the following statements are true about the Select the correct answer using the code given below.
Governor of a state? (a) 1 and 2 only (b) 1 and 3 only
(c) 2, 3 and 4 only (d) 1, 2, 3 and 4
1. The executive power of the state is vested in him.
2. He must have attained 35 years of age. 10. The correct statements about ordinance making power of
3. He holds office during the pleasure of the President. the Governor are :
4. The grounds for his removal are laid down in the 1. It is laid down in Article 213.
Constitution. 2. It can be issued by him after the advice of the
Which of the following statements is/are correct? president of state council of ministers.
(a) 1,2, and 4 (b) 1,2 and 3 3. It is co-extensive with the legislative power of the
(c) 1, 3 and 4 (d) 1, 2, 3 and 4 state legislature.

Downloaded From : www.EasyEngineering.net


Downloaded From : www.EasyEngineering.net
P-112 State Government

4. It can be issued only during the recess of State 2. The Governor can adjourn the sittings of the State
Legislative Assembly and not the Legislative Assembly.
Council. 3. The Governor addresses the first session of the
5. It can not be withdrawn by him anytime. Legislative Assembly after elections.
(a) 2, 3 and 4 (b) 1, 3 and 5 4. The Governor causes to lay the annual budget in the
(c) 1, 2 and 3 (d) 2, 4 and 5 State Assembly.
11. While appointing a Lokayuka, the Governor in most of Which of the statements given above are correct ?
the states consults : (a) 1 and 2 (b) 1, 3 and 4
1. President of India (c) 2 and 3 (d) 2 and 4
2. Speaker of the Legislative Assembly 16. Consider the following statements:
3. Leader of the opposition in the Legislative Assembly The Governor of a State has the power of appoint:
4. Chief justice of the State High Court. 1. Judges of the High Court
5. Leader of the Opposition in the Legislative 2. Members of the State Public Service Commission
Council. 3. Members of the State Finance Commission
(a) 1, 4 and 5 (b) 1, 2 and 4 4. The Accountant General
(c) 3, 4 and 5 (d) 3 and 4 Which of these statements are correct?
12. Under which of the following circumstances, the (a) 1 and 2 (b) 2 and 3
(c) 1, 3 and 4 (d) 1, 2, 3 and 4



ww
Governor can reserve a state bill for the consideration
of the President?
1. If it is ultra vires.
2. It it is opposed to the Directive Principles of State
17.


In which of the following bodies, does the Chief Minister
of a State hold membership?
1. National Integration Council



Policy.
w.E
3. If it endangers the position of the state High Court.
4. If it is dealing with the compulsory acquisition of



2. National Development Council
3. Inter-State Council
4. Zonal Council



(a) 1, 2 and 3
(c) 2, 3 and 4 a
property under Article 31 A.

syE
(b) 1, 2, 3 and 4
(d) 1, 3 and 4
13. The correct statements regarding the difference between



18.

Select the correct answer from the codes given below :
(a) 1, 3 and 4
(c) 1, 2, 3 and 4
(b) 2 and 3
(d) 2, 3 and 4
Consider the following statements :
1. The Governor cannot function without the State

the pardoning powers of President and Governor are :
1. The Governor can pardon sentences inficted by
court martial while the President cannot. ngi
Council of Ministers.
2. A person who is not a member of the State
Legislature cannot be appointed as a minister.


2. The President can pardon death sentence while
Governor cannot.
3. The Governor can pardon death sentence while the

nee
3. The State Council of Ministers can function for
sometime even after death or resignation of the
Chief Minister.


President cannot.
4. The President can pardon sentences inflicted by
court martial while the Governor cannot.
(a) 1 and 2 (b) 2 and 4

r ing
4. In the absence of the Chief Minister, only the Home
Minister can preside over emergency meetings of
the State Council of Ministers.
(c) 1 and 3 (d) 3 and 4
14. Select the constitutional duties of the Chief Minister from



Which of these is / are correct ?
(a) Only 1
(c) 1, 2 and 4
(b) 3 and 4
.ne
(d) 1, 2, 3 and 4


following by using codes given below
1. The Chief Minister communicates to the Governor
all decisions of the Council of Ministers related to
the administration of the affairs of the State.
2. The Chief Minister communicates to the Governor
19.


Consider the following statements:

t
1. No person is eligible for appointment as Governor
unless he has completed the age of thirty years.
2. The same person can be appointed as Governor for
three States.
the proposals for legislation. Which of the statements given above is/are correct?
3. The Chief Minister participates in the meetings of (a) 1 only (b) 2 only
National Development Council. (c) Both 1 and 2 (d) Neither 1 nor 2
4. The Chief Minister submits for the consideration of 20. Consider the following statements:
the Council of Ministers any matter on which decision 1. A bill pending in the Legislature of a State shall not
has been taken by a minister but which has not been lapse by reason of the prorogation of the House or
considered by the council as if the Governor requires. House thereof.
2. A bill pending in the Legislative Council of a
Codes:
State which has not been passed by the Legislative
(a) 1 and 2 (b) 1 and 4
Assembly shall not lapse on dissolution of the
(c) 1, 2 and 3 (d) 1, 2 and 4 Assembly.
15. Consider the following statements with respect to the Which of the statements given above is/are correct?
powers of the Governor of a State : (a) 1 only (b) 2 only
1. The governor can summon, prorogue and dissolve (c) Both 1 and 2 (d) Neither 1 nor 2
the State Assembly.

Downloaded From : www.EasyEngineering.net


Downloaded From : www.EasyEngineering.net
State Government P-113

21. Which of the statements given below is/are correct? Which of the statements given above is/are correct?
1. The Speaker immediately vacates his/her office (a) 1 only (b) 2 only
whenever the State Legislative Assembly is (c) Both 1 and 2 (d) Neither 1 nor 2
dissolved. 26. India’s is a federal system of government as:
2. No Member of a State Legislative Assembly shall 1. Union Legislature is bicameral
be liable to any proceeding in any court in respect 2. provision of single citizenship
of anything said or any vote given by him/her in the 3. Constitution is supreme
legislature. 4. there is an independent judiciary
Select the correct answer using the code given below : Which of the above is/are correct?
(a) 1, 3 and 4 (b) 1, 2 and 3
Code :
(c) 2 and 4 (d) 1, 2, 3 and 4
(a) 1 only (b) 2 only
27. Which of the following are matters on a which a
(c) Both 1 and 2 (d) Neither 1 nor 2
Constitutional Amendment is possible only with the
22. Which of the following statements is/are correct? ratification of the legislatures of not less than one-half of
Under Article 200 of the Constitution of India, the the states ?
Governor of a State may 1. Election of the President
1. withhold his assent to a bill passed by the State 2. Representation of states in Parliament


ww Legislature
2. reserve the bill passed by the State Legislature for
reconsideration of the President



3. Any of the lists in the seventh Schedule
4. Abolition of the Legislative Council of a state
Which of the above is/are correct?


w.E
3. return the bill, other than a money bill, for
reconsideration of the Legislature
Select the correct answer using the code given below.


(a) 1, 2 and 3
(c) 1, 3 and 4
(b) 1, 2 and 4
(d) 2, 3 and 4
28. Which of the following are the features of Indian federal


(a) 1 only
(c) 2 and 3 only
a (b) 1 and 2 only

syE
(d) 1, 2 and 3
23. Which of the following statements is/are correct?
Under the provisions of Article 200 of the Constitution of




system ?
1. Division of powers between the centre and the units
2. Residuary powers vested with the centre
3. Existence of the nominal and real executive
Which of the following statements is/are correct?

India the Governor of a state may
1. Withhold his assent to a Bill passed by the state
legislature. ngi

(a) 1, 2 and 3
(c) 1 and 3
(b) 1 and 2
(d) 2 and 3

2. Reserve the Bill passed by the state legislature for


consideration of the President.

nee
29. The Inter-State Council consists of
1. Prime Minister
2. Chief Ministers of all states

r
3. Return the Bill, other than a money Bill, for 3. Chief MInister of union territories with legislatures



reconsideration of the legislature.
Select the correct answer using the codes given below
(a) Only 1 (b) 1 and 2


(a) 1, 2, 3, 4 and 5 ing
4. Eight Union Cabinet Ministers
5. Administrators of union territories with legislatures
(b) 1, 2, 3 and 4
(c) 2 and 3 (d) All of the above
24. Article 156 of the Constitution of India provides that a
(c) 1, 2 and 3
.ne
(d) 1, 2, 3 and 5
30. The Prime Minister of India recently ruled out the decision


Governor shall hold office for a term of five year
from the date on which he enters upon his office.
Which of the following can be deduced from this?
1. No Governor can be removed from office till

t
of the Cauvery River Authority, in which Karnataka was
ordered to release water to Tamil Nadu. Which of the
following statement(s) is/are correct?
1. The Cauvery River Authority comprises of the
Chief Ministers of Puducherry, Kerala, Tamil Nadu
completion of his term
and Karnataka along with the PM as chairman.
2. No Governor can continue in office beyond five 2. Inter-state water disputes are excluded from the
years primary jurisdiction of the Supreme Court of India.
Codes: Select the correct answer using the codes given below:
(a) 1 only (b) 2 only (a) 1 only (b) 2 only
(c) Both 1 and 2 (d) Neither 1 nor 2 (c) Both 1 and 2 (d) Neither 1 nor 2
25. Consider the following statements: 31. Which of the following can be associated with
The Constitution of India provides that: MGNREGA programme?
1. the Legislative Assembly of each State shall consist 1. It provides wage employment to every household
of not more than 450 members chosen by direct whose adult members volunteer to do unskilled
election from territorial constituencies in the State manual work.
2. a person shall not be qualified to be chosen to fill a 2. It focuses on strengthening of natural resource
seat in the Legislative Assembly of a State if he/she management.
3. It encourages sustainable development.
is less than 25 years of age

Downloaded From : www.EasyEngineering.net


Downloaded From : www.EasyEngineering.net
P-114 State Government

Select the correct answer using the codes given below: 3. The State Election Commissioner cannot be removed
(a) 1 only (b) 1 and 2 only in any manner from his office until he demits himself
(c) 1 and 3 only (d) 1, 2 and 3 or completes his tenure.
32. Govt. decided to add four new tribes, Abuj Maria, Which of the above statements is/are correct?
Korba, Hill Korba and Kodaku into the list of scheduled (a) 1, 2 and 3 (b) 1 and 2 only
tribes. Which of the following is/are correct in regard to (c) 2 and 3 only (d) 1 only
granting the status of scheduled tribe to a tribe? 37. Consider the following statements : [CSAT 2015-I]
1. President has the authority to include or exclude a 1 The Legislative Council of a State in India can be
tribe from the list of schedule tribes. larger in size than half of the Legislative Assembly
2. The criterion for a community to be recognized as of that particular State
scheduled tribe is not spelled out in the constitution 2. The Governor of a State nominates the Chairman of
Select the correct answer using the codes given below: Legislative Council of that particular State.
(a) 1 only (b) 2 only Which of the statements given above is/are correct?
(c) Both 1 and 2 (d) Neither 1 nor 2 (a) 1 only (b) 2 only
33. Which of the following is among the main provisions of (c) Both 1 and 2 (d) Neither 1 nor 2
the ‘MGNREGA’ ?
1. There is no time bound limit to provide employment Matching Based MCQ

ww to the beneficiaries registered under the act


2. There is a provision for daily unemployment
allowance in cash to be paid.
DIRECTIONS (Qs. 38 to 43) : Match List-I with List-II
and select the correct answer using the codes given below



w.E
3. Employment is limited and confined within a limited
radius.
4. At least one-third beneficiaries have to be women
State the correct answer using the codes given below:
the lists.
38. Match List-I with List-II and select the correct answer
using the codes given below the lists:




(a) 1, 2 and 3 only
(b) 2, 3 and 4 only
(c) 1, 3 and 4 only
(d) 1, 2, 3 and 4
a syE




List-I
(Local bodies)
A. Zila Parishads at
the sub-divisional
List-II
(States as in 1999)
1. Andhra Pradesh

34. Which of the following statements with regard to Inter-


State Council is/are correct?
1. It was established under the provisions of the
ngi level


B. Mandal Praja Parishad 2. Assam
C. Tribal Councils 3. Mizoram



Constitution of India.
2. The Council is a recommendatory body.
3. There is a standing committee of the Council under

nee
D. Absence of Village 4. Meghalaya
Panchayats
Codes :


the Chairmanship of the Prime Minister of India to
process matters for consideration of the Council.
Select the correct answer using the codes given below




r
(a) A-2, B-1, C-4, D-3

ing
(b) A-1, B-2, C-4, D-3
(c) A-3, B-2, C-1, D-4
(d) A-2, B-1, C-3, D-4


(a) Both 1 and 3
(c) 1 and 2
(b) Only 2
(d) All of these
35. The legislative power of the Parliament includes making
39.

List-I
(A) Governor .ne List-II
(1) Article 167


laws
1. on matters not enumerated in the Concurrent List
and State List.
2. in respect of entries in the State List if two or more





(B) Council of ministers
(C) Duties of Chief Minister
(D) Legislative Council
(a) A – 1 ; B – 2 ; C – 3 ; D – 4
(b) A – 4 ; B – 3 ; C – 2 ; D – 1
t
(2) Article 169
(3) Article 155
(4) Article 163

State Legislatures consider it desirable (c) A – 3 ; B – 2 ; C – 4 ; D – 1


3. for implementing any treaty agreement or convention (d) A – 3 ; B – 4 ; C – 1 ; D – 2
with any country even if it falls in the State List. 40. List I (Article of the List II (Content)
Select the correct answer using the codes given Constitution)
below (A) Article 54 (1) Election of the President
(a) Only 2 (b) 1 and 2 of India
(c) 1 and 3 (d) All of these (B) Article 75 (2) Appointment of the Prime
36. Consider the following statements about State Election Minister and Council of
Commission ? Ministers
(C) Article 155 (3) Appointment of the
1. The State Election Commissioner shall be appointed
Governor of a state
by the Governor of the State. (D) Article 164 (4) Appointment of the Chief
2. The State Election Commission shall have the power Minister and Council of
of even preparing the electoral rolls besides the Ministers of a state
power of superintendence, direction and control of (5) Composition of Legislative
election to the panchayats. Assemblies

Downloaded From : www.EasyEngineering.net


Downloaded From : www.EasyEngineering.net
State Government P-115

(a) A – 1 ; B – 2 ; C – 3 ; D – 4 Codes :
(b) A – 1 ; B – 2 ; C – 4 ; D – 5 A B C D
(c) A – 2 ; B – 1 ; C – 3 ; D – 5 (a) 3 4 2 5
(d) A – 2 ; B – 1 ; C – 4 ; D – 3 (b) 2 1 4 3
41. List-I List-II (c) 4 3 1 2
(A) Union List (1) Banking (d) 5 3 2 4
(B) State List (2) Public order and police 43. List-I List -II
(C) Concurrent (3) Labour Wel-List fare (A) Article 156 (1) Executive authority of
(a) A – 2 ; B – 1 ; C – 3 Governor
(b) A – 1 ; B – 2 ; C – 3 (B) Article 154 (2) Tenure of Governor
(c) A – 1 ; B – 3 ; C – 2 (C) Article 153 (3) Appointment of Governor
(d) A – 3 ; B – 1 ; C – 2 (D) Article 155 (4) Office of Governor
42. List-I (States) List -II (Governor’s special (5) discretionary power of
responsibilities) Governor
(A) Madhya Pradesh (1) Law and order
(B) Gujarat (2) Administration of tribal Codes :


ww

(C) Nagaland

areas
(3) Development of backward
areas
A
(a) 3 4
(b) 2 1
B C D
2 5
4 3


(D) Assam

w.E (4) Minister for Tribal Welfare


(5) Hill Areas Committee
working
(c) 4 3
(d) 5 3 2 4
1 2

a syE
ngi
nee
r ing
.ne
t

Downloaded From : www.EasyEngineering.net


Downloaded From : www.EasyEngineering.net
P-116 State Government

EXERCISE-1 Governor instead of a Council of Ministers headed


by an elected Chief Minister accountable to the state
1. (c) 2. (b) 3. (d) 4. (a) 5. (b)
6. (b) 7. (c) 8. (b) 9. (c) 10. (c) legislature. Article 356 is invoked if there has been
11. (c) 12. (c) 13. (b) 14. (b) 15. (d) failure of the constitutional machinery in any states
16. (d) 17. (b) 18. (a) 19. (d) 20. (c) of India.
21. (d) The Legislative Assembly (the Vidhan Sabha) is 36. (d) There are only five states with bicameral legislature
the popular House of the State Legislative and its (Legislative assembly as well as Legislative Council)
member are chosen by direct election on the basis of - UP, Bihar, Maharashtra, Karnataka and Jammu &
adult suffrage from territorial constituencies (Article Kashmir
170).The number of members ranges between 60 37. (a) Article 257 in the Constitution states that the
and 500. executive power of every State shall be so exercised
22. (c) 23. (b) 24. (b) 25. (d) 26. (a) as not to impede or prejudice the exercise of the
27. (d) The Legislative Assembly (the Vidhan Sabha) is executive power of the Union, and the executive

ww the popular House of the State Legislative and its


member are chosen by direct election on the basis of
adult suffrage from territorial constituencies (Article
power of the Union shall extend to the giving of such
directions to a State as may appear to the Government
of India to be necessary for that purpose. Article

28. (c)
and 500.
w.E
170).The number of members ranges between 60

29. (c) Article 371A deals with the Special provision with
258: Power of the Union to confer powers on the
States in certain cases; Article 355: Duty of the
Union to protect States against external aggression
and internal disturbance; Article 358: Suspension of

a
respect to the State of Nagaland.

syE provisions of Article 19 during emergencies.


30. (b) According to Article 164(1) in the State of Bihar, 38. (a) The State government does not have control over its
Madhya Pradesh and Orissa, there shall be a Minister local bodies in matters of Citizens’ grievances.
in charge of tribal welfare who may in addition be 39. (d) A lieutenant Governor is in charge of a Union
in charge of the welfare of the Scheduled Castes and
backward classes or any other work. Punjab is not
covered by the Article. ngi Territory whereas a Governor is in charge of a
State. The rank of Lt.Governor is present only in

31. (d) The legislative assembly of the concerned state


recommends to the parliament for the abolition of
the legislative council in a state (Article 169).
nee
the states of Delhi, Andaman and Nicobar Islands
and Puducherry. So in the case of a Union Territory
specified where there is a legislative setup, the Chief

r
Minister is appointed by the Lieutenant Governor.

ing
32. (c) Up to 2014, seven (out of twenty-nine) states have 40. (b) 41. (a) 42. (d) 43. (a) 44. (c) 45. (b)
a Legislative Council: Andhra Pradesh, Bihar, 46. (b) 47. (a) 48. (b)
Jammu and Kashmir, Karnataka, Maharashtra, 49. (d) Nutrition is not the component of the Pradhan Mantri
Telangana and Uttar Pradesh.Tamilnadu does not
have Legislative Council.
33. (c) The constitution of 1950 distinguished between
Yojana’ aims at -
.ne
Gramodaya Yojna. Pradhan Mantri Gramodaya


three main types of states:
The Part A states were ruled by an elected governor
and state legislature.
The Part B states were governed by a rajpramukh.


3. generating employment in rural areas
t
1. meeting rural needs like primary education, health
care, drinking water, housing, rural roads
2. alleviating employment in rural areas

4. strengthening Panchayati Raj system in rural areas


The Part C states were governed by a chief 50. (d)
commissioner appointed by the President of India. 51. (b) The entry “public health and sanitation” is included
The Part D states were administered by a lieutenant in the state list of the constitution of India.
governor appointed by the central government. 52. (c) State government has exclusive right/jurisdiction
34. (b) Sarkaria Commission was set up in June 1983 by over the sales tax.
the central government of India. According to the 53. (c) State list consists of 61 items (previously 66 items).
commission, Chief Minister should be consulted Uniformity is desirable but not essential on items in
before appointing the Governor. this list: maintaining law and order, police forces,
35. (d) President’s rule refers to Article 356 of the healthcare, transport, land policies, electricity in
Constitution of India deals with the failure of the state, village administration, etc. The state legislature
constitutional machinery of an Indian state. In has exclusive power to make laws on these subjects.
the event that government in a state is not able to But in certain circumstances, the parliament can also
function as per the Constitution, the state comes make laws on subjects mentioned in the State list.
under the direct control of the central government, Then the parliament has to pass a resolution with
with executive authority exercised through the 2/3rd majority that it is expedient to legislate on this

Downloaded From : www.EasyEngineering.net


Downloaded From : www.EasyEngineering.net
State Government P-117

state list in the national interest. Though states have 16. (b) The Governor has the power to appoint the Council
exclusive powers to legislate with regards to items of Ministers including the Chief Minister of the state,
on the State list, articles 249, 250, 252, and 253 the Advocate General and the members of the State
state situations in which the federal government can Public Service Commission. However, the Governor
legislate on these items. cannot remove the members of the State Public Service
54. (b) Sarkaria Commission was set up in June 1983 to Commission as they can only be removed by an order
examine the relationship and balance of power of the President.
between state and central government. 17. (c)
55. (b) Sarkaria Commission was set up by the central 18. (a) The Constitution provides that there shall be a Council
government of India in June 1983 to examine the of Ministers with the Chief Minister at the head to aid
relationship and balance of power between state and advice the Governor in the exercise of his functions
and central governments in the country and suggest except in so far as he is by or under the Constitution
changes within the framework of Constitution of required to act in his discretion. The governor appoints
India. the Chief Minister and other Ministers on the advice of
56. (b) The State list contains 66 subjects of local or the Chief Minister.
state importance. The state governments have the 19. (b) 20. (c)
authority to make laws on these subjects. These 21. (b) members of state legislative assembly are not liable

ww subjects include police, local governments, trade,


commerce and agriculture. Parliament has exclusive
power to make any law with respect to any matter
to any proceeding in any court.
22. (d) When a Bill has been passed by the Legislative


w.E
not enumerated in the Concurrent List or State List.
Note: Residuary Power retained by a governmental
authority after certain powers have been delegated
Assembly of a State or, in the case of a State having
a Legislative Council, has been passed by both
Houses of the Legislature of the State, it shall be
presented to the Governor and the Governor shall

a
to other authorities.

syE
57. (b) Stock Exchanges are listed in the Seventh Schedule
(Article 246) List I-Union List, item no. 90 that
reads, taxes other than stamp duties on transactions
declare either that he assents to the Bill or that he
withholds assent there from or that he reserves the
Bill for the consideration of the President: Provided
that the Governor may, as soon as possible after the
in stock exchanges and futures markets. Forests-
Concurrent List, 17-A, Post Office Savings Bank
-Union List-3, Public health and sanitation; hospitalsngi presentation to him of the Bill for assent, return the
Bill if it is not a Money Bill.
and dispensaries - State List -6.
58. (a) Agriculture (Entry 14), Fisheries (Entry 21), Public
Health (Entry 6) are in the State List. Regulation of
nee
23. (d) All of the above statements are correct.
24. (d) According to Article 156, the Governor shall hold
office during the pleasure of the President, the
Labour and safety in mines and oil fields is in the
Union list under Entry 55 (Art 246). r
Governor may, by writing under his hand addressed

ing
to the President, resign his office. Subject to the
foregoing provisions of this article, a Governor shall

EXERCISE-2
1. (a) 2. (a) 3. (b) 4. (a) 5. (b) .ne
hold for a term of five years from the date on which
he enters upon his office. Provided that a Governor
shall, notwithstanding the expiration of his term,
6. (c) 7. (c) 8. (d) 9. (b)
11. (d) 12. (b) 13. (b)
10. (c)

14. (d) National Development Council is not constitutional


15. (b) The Governor is a part of the state legislative and can
upon his office. t
continue to hold office until his successor enters

25. (b) Statement 2 is correct as per provisions given under


Article 173. Statement 1 is incorrect as according
summon, adjourn or prorogue the state legislative.
The governor can even dissolve the Vidhan Sabha. to article 170, the legislative assembly of each state
At the commencement of the first session after each shall consist of not more than 500 and not less
general election to the Legislative Assembly and at than 60 members chosen by direct election from
the commencement of the first session of each year territorial constituencies in the state.
the Lieutenant Governor shall address the Legislative 26. (a) 27. (a) 28. (b) 29. (c)
Assembly and inform it of the causes of its summons. 30. (c) The Cauvery River authority comprises of the CMs
As per Article 202 of the Constitution of India the of puducherry, Kerala, Tamil Nadu and Karnataka.
Governor of a State shall, cause to be laid before the Inter-State water disputes are excluded from the
House or Houses of the Legislature of the State a primary jurisdiction of the supreme court of India.
Statement of the estimated receipts and expenditure of On 20 February 2013, based on the directions of the
the State for a financial year. This estimated statement Supreme Court, the Indian Government has notified
of receipt and expenditure for a financial year named in the final award of the Cauvery Water Disputes
the Constitution as the “Annual Financial Statement” is Tribunal (CWDT) . The Tribunal, in a unanimous
commonly known as “Budget”. decision in 2007, determined the total availability of

Downloaded From : www.EasyEngineering.net


Downloaded From : www.EasyEngineering.net
P-118 State Government

water in the Cauvery basin at 740 thousand million 35. (d) Article 248- Parliament has exclusive power to make
cubic (tmc) feet at the Lower Coleroon Anicut site, any law with respect to any matter not enumerated
including 14 tmcft for environmental protection in the Concurrent List or State List.
and seepage into the sea. The final award makes an Article 252- If it appears to the Legislatures of two
annual allocation of 419 tmcft to Tamil Nadu in the or more States to be desirable that any of the matters
entire Cauvery basin, 270 tmcft to Karnataka, 30 with respect to which Parliament has no power to
tmcft to Kerala and 7 tmcft to Puducherry. make laws for the States except as provided in
31. (d) MGNREGA Programme provides employment to Articles 249 and 250 should be regulated in such
every household adult members to do unskilled manual States by Parliament by law.
work, focuses on natural resources management and Article 253- Legislation for giving effect to
encourages sustainable development. The statute international agreements notwithstanding anything in
is hailed by the government as “the largest and the foregoing provisions of this Chapter, Parliament
most ambitious social security and public works has power to make any law for the whole or any part
programme in the world”. of the territory of India for implementing any treaty,
32. (b) According to article 342 of the constitution it is the agreement or convention with any other country or
parliament, which can decide on the inclusion and countries or any decision made at any international

ww exclusion of a tribe into the list of schedule tribe.


The President may with respect to any State or Union
conference, association or other body.
36. (b) Three Election Commissioners in the each State
shall be appointed by the Governor of the respective

w.E
territory, and where it is a State, after consultation
with the Governor thereof, by public notification,
specify the tribes or tribal communities or parts of
State from a penal of five names for each office
forwarded by the Election Commission of Bharatto
the provision of the Constitution (Seventy-third

a
or groups within tribes or tribal communities which

syE
shall for the purposes of this Constitution be deemed
to be Scheduled Tribes in relation to that State or
Union territory, as the case may be.
Amendment) Act, 1992 and the Constitution
(Seventy-fourth Amendment) Act, 1992, whereunder
the constitution of State Election Commissions and
appointment of State Election Commissioners are

employment to the beneficiaries registered under the


ngi
33. (b) Under this act, there is a time bound limit to provide contemplated to superintend, direct and control the
preparation of the electoral rolls for, and the conduct
of elections to Panchayats and Municipalities.
act.
34. (c) The Inter-State Council was established under
Article 263 of the Constitution of India through a nee
Election Commissioner of a State can be removed
by the Full Bench of State Judicial Commission
Presidential Order dated 28 May 1990. As the article
263 makes it clear that the Inter-State Council is not
a permanent constitutional body for coordination
r
on the basis of enquiry and investigation made by

ing
a judicial committee constituted for the purpose,
consisting of two Chief Justices and one Judge from
different High Courts.
between the States of the Union. It can be established
‘at any time’ if it appears to the President that the .ne
37. (d) The maximum strength of the legislative council
is fixed at one third of the total strength of the
public interests would be served by the establishment
of such a Council. The Council is a recommendatory
body. The Council shall consist of Prime Minister
(Chairman), Chief Ministers of all States and union
t
legislative assembly and the minimum strength is
fixed at 40. The chairman of the legislative council
is elected by the council itself from amongst its
members.
38. (a) Zilla parishads at the sub-divisional level-Assam
territories (Member), Administrators (UT) and Six Mandal Praja parishad-Andhra Pradesh, Tribal
Ministers of Cabinet rank to be nominated by the Councils-Meghalaya and Absence of village
Prime Minister (Member).This is not a standing panchayats-Mizoram
committee. 39. (d) 40. (a) 41. (b) 42. (c) 43. (b)

Downloaded From : www.EasyEngineering.net


Downloaded From : www.EasyEngineering.net

LOCAL 5
GOVERNMENT Chapter

ww
Introduction

w.E
The local self government includes both rural and urban government. It is the third level of government. There are 2 types
of local government in operation - Panchayats in rural areas and Municipalities in urban area. Lord Ripon is known as the
father of local self government.

a syE Local Government

Rural
ngi Urban Committees
+

Panchayati
Raj
73rd Amendment
Act
Structures
Composition,
74th
Amendment nee
Municipalities Structures
Composition,
Acts

Power &
Functions
Act

RURAL LOCAL GOVERNMENT


r Power &

ing
Functions

Panchayati Raj ••
.ne
Rajasthan was the first state to establish the institution of
Panchayati Raj in Nagaur District in 1959. Rajasthan was
The term Panchayti Raj in India signifies the system of rural
local self-government. It has been established in all the states of
India by the Acts of the State Legislatures to build democracy
at the grass root level. It is entrusted with rural development.
followed by Andhra Pradesh.

Balwant Rai Mehta Committee (1957)


••
t
It was set up to examine the working of the Community
It was constituationlised through the 73rd Constitutional Development Programme (1952) and the National
Amendment Act 1992. Extension Service (1953).
•• Recommendations:
Evolution of Panchayati Raj (i) Three-tier Panchayati Raj system: Gram Panchayat
•• The Narasimha Rao Government introduced the at the Village level, Panchayat Samiti at the block
Constitutional Amendment Bill in the Lok Sabha in level, Zila Parishad at the district level.
September, 1991. (ii) Village Panchayat is directly elected, while the
Panchayat Samiti and Zila Parishad constituted with
•• It was passed by the Lok Sabha on 22nd December, 1992
indirectly elected members.
and by the Rajya Sabha on 23rd December. Later it was
(iii) Panchayat Samiti is the executive body, while the
approved by the 17 State Assemblies and received the Zila Parishad is the advisory, coordinating and
assent of the President of India on 20th April, 1993. supervisory body.
•• Thus, it emerged as the 73rd Constitutional Amendment (iv) The District Collector should be the Chairman of the
Act, 1992 and came into force on 24th April, 1993. Zila Parishad.

Downloaded From : www.EasyEngineering.net


Downloaded From : www.EasyEngineering.net
P-120 Local Government

K Santhanam Committee levels should be assigned an important role with respect


to planning, implementation and monitoring of rural
One of the prime areas of concern in this long debate on
development programmes.
Panchayati Raj Institutions was fiscal decentralisation. The K
Santhanam Committee was appointed to look solely at the issue •• A post of District Development Commissioner should be
of PRI finance, in 1963. created.
•• Elections to the Panchavati Raj institutions should be held
Recommendations regularly.
•• Panchayats should have special powers to levy special tax
on land revenues, home taxes, etc. LM Singhvi Committee
•• All grants and subventions at the state level should be In 1986, Rajiv Gandhi Government appointed a committee
mobilised and sent in a consolidate form to various PRIs. on the “Revitalisation of the Panchayati Raj Institutions for
•• A Panchayati Raj finance corporation should be set-up democracy and development” under the Chairmanship of LM
to look into the financial resource of PRIs at all levels, Singhvi.
provide loans and financial assistance to these grassroots Recommendations
level governments and also provide non-financial
requirements of villages. •• The Panchayati Raj Institutions should be constitutionally
recognised, protected and preserved. It also suggested

ww
Ashok Mehta Committee
In December 1977, the Janata Government appointed
a Committee on Panchayati Raj Institutions under the ••
some constitutional provisions to ensure regular, free and
fair elections to the Panchayati Raj bodies.
Nyaya Panchayats should be established for a cluster of

w.E
Chairmanship of Ashok Mehta. It submitted its report in
August, 1978 and made recommendations to revive and
strengthen the declining Panchayati Raj System in the country.
••
villages.
The villages should be organised to make the Gram
Panchayats more viable.

Recommendations
•• a syE
The three-tier system of the Panchayati Raj should be
replaced by two-tier system, that is, the Zila Parishad
••

••
The Village Panchayats should have more financial
resources.
The judicial tribunals should be established in each state
to eradicate controversies about election to the Panchayati
at the district level and below it the Mandal Panchayat
consisting of a group of villages comprising a population
upto 20,000. ngi Raj Institutions, their dissolution and other matters related
to their functioning.

••

••
The Zila Parishad should be the executive body and be
made responsible for planning at the district level.
The Panchayati Raj Institutions should have Compulsory
nee
Constitutionalisation
Rajiv Gandhi Government

••
powers for taxation to mobilise their own financial
resources.
The Nyaya Panchayats should be kept as separate bodies
r ing
In July 1989, Rajiv Gandhi Government introduced the 64th
Constitutional Amendment Bill to constitutionalise PRIs, but it
was not approved by Rajya Sabha.

••
from that of development Panchayats.
A minister for the Panchayati Raj should be appointed in VP Singh Government
.ne
t
the State Council of Ministers to look after the affairs of In November 1989, Prime Ministers VP Singh proposed the
the Panchayati Raj Institutions. introduction of a fresh Constitutional Amendment Bill and the
•• Seats for the SCs and the STs should be reserved on the bill was introduced in Lok Sabha in September 1990, but it
basis of their population. lapsed due to the fall of government.

G V K Rao Committee Narsimha Rao Government


The Committee on Administrative Arrangement for Rural In September 1991, Prime Minister PV Narsimha Rao introduced
Development and Poverty Alleviation Programmes under the the modified proposal of Constitutional Amendment Bill, which
chairmanship of G.V.K. Rao was appointed by the Planning finally emerged as the 73rd Constitutional Amendment Act,
Commission in 1985. 1992 and came into force on 24th April, 1993.

Recommendations 73rd Amendment Act (1992)


The Committee made the following recommendations to
strengthen and revitalise the Panchayati Raj system: •• Added to Part-IX (Articles 243 to 243-O) and the Eleventh
Schedule to the Constitution.
•• “The district is the proper unit for planning and
development and the Zila Parishad should become the •• 11th Schedule contains 29 functional items and deals with
principal body for management of all development Article 243-G.
programmes which can be handled at that level”. •• The Act has given a practical shape to Article 40. The Act
•• The Panchayati Raj institutions at the district and lower has brought them under the purview of the justifiable part
of the Constitution.

Downloaded From : www.EasyEngineering.net


Downloaded From : www.EasyEngineering.net
Local Government P-121

•• State governments are under constitutional obligation to State Election Commission


adopt the new Panchayati raj system in accordance with
the provisions of the Act. However the Panchayati raj •• The superintendence, direction and control of the
system in each state is created through its own Act. preparation of lectoral rolls for, and the conduct of all
•• The main provisions of the Act are as follows: election to the Panchayats shall be vested in a State Election
Commission consisting of a State Election Commissioner
Three-Tier System to be appointed by the Governor.
•• Three-tier system with panchayats at the village, •• Subjects to the provisions of any law made by the
intermediate, and district levels. Panchayat means an legislature of a State, the conditions of service and tenure
institution of self-government for rural areas. of office of the State Election Commissions shall be such
as the Governor may by rule determine.
•• Act brings about uniformity in the structure of Panchayati
raj throughout the country. However a state having a •• The Governor of a State shall, when so requested by the
population not exceeding 20 lakhs may not constitute State Election Commission, make available to the State
panchayats at the intermediate level. Election Commission such staff as may be necessary
for the discharge of the functions conferred on the State
Gram Sabha (Article 243A) Election Communication by clause (1).
Gram sabha is the foundation of the Panchayati raj system. It
Reservation of Seats (Article 243D)
ww
shall exercise such powers & functions at the village level as
the legislature of a state determines. •• Reservation of seats for SCs & STs in every Panchayat
(at all levels) in proportion of their population to the total

•• w.E
Elections of Panchayats (Article 243C)
All the members of Panchayats at the village, intermediate
and district levels shall be elected directly by the people. ••
population in that area. State legislature shall provide for
the reservation of offices of chairpersons in the Panchayat
at the village or any other level for the SCs and STs.

••
a
Chairperson of Panchayat at the village level shall be

syE
elected in such manner as the state legislature determines.
However, the chairperson at the intermediate and district
levels shall be elected indirectly; by and from amongst the
Reservation of not less than one-third of the total number
of seats for women (including those reserved for women
of SCs and the STs). Not less than one-third of the total
number of offices of chairpersons in the Panchayats at
each level shall be, reserved for women.
elected members there of.

Qualifications ngi •• For any backward classes, state legislature is authorized


to make any provision for reservation.

•• A person seeking election to the panchayat must possess


the qualifications prescribed for a member of state •• nee
State Finance Commission (Article 243I)
Governor after every five years, constitute a Finance

r
legislature. Commission to review the financial position of the
•• Minimum age for contesting election to the panchayat is
21 year (as against 25 years for State Legislature). ••
Panchayats.
ing
State Legislature may provide for the composition of the
Commission, the required qualifications of its members
Disqualifications for Membership (Article 243F)
• A person shall be disqualified for being chosen as, and for ••
and the manner of their selection.
.ne
They review the financial position of the panchayats


being a member of a Panchayat-
(a) If he is so disqualified by or under any law for the
time being in force for the purposes of election, to
the Legislature of the State concerned.
t
and makes recommendations to the governor for the
distribution of the net proceeds of taxes between the states
and the panchayats. It also recommends grants-in-aid to
the panchayats, from the Consolidated Fund of the State.
(b) If he is so disqualified by or under any law made by •• Central Finance Commission shall also suggest the
the Legislature of the State. measures needed to augment the consolidated fund of a
• If any question arise as to whether a member of a Panchayat state to supplement the resources of the panchayats in the
has become subject to any of the disqualifications States.
mentioned in clause (1), the question shall be referred for
the decision of such authority and in such manner as the Audit of Accounts of Panchayats
legislature of a State may, by law, provide. The legislature of a State may, by law, make provisions with
• No person shall be disqualified on the ground that he is respect to the maintenance of accounts by the Panchayats and
less than twenty-five years, of age, if he has attained the the auditing of such accounts.
age of twenty-one years.
• The question regarding disqualification of the members Duration of Panchayats (Article 243E)
of panchayat are referred to such authority as may be •• Fixed tenure of Five-year
provided by the state legislature by law. •• In case the panchayat is dissolved before its term of 5
years, fresh elections must be held within six months. The
panchayat thus constituted are elected for the remaining

Downloaded From : www.EasyEngineering.net


Downloaded From : www.EasyEngineering.net
P-122 Local Government

period. It may be noted that if the remaining tenure of the PESA Act, 1996
dissolved panchayat is less than six months no elections
need be held. This act is called Provisions of the Panchayats (Extension to
the Scheduled Areas) Act, 1996. In this Act ‘Scheduled Areas’
Powers and Functions (Article 243G) means the Scheduled Areas as referred to in clause (1) of the
The State Legislature may endow the Panchayats, with such Article 244 of the Constitution. It meant to give Self-rule
powers and authority as may be necessary to enable them to authority to the Tribal people and govern themselves as well
function as institutions of self government. Such a scheme as their resources.
may contain provisions for the devolution of powers and
responsibilities upon Panchayats at the appropriate level with Bhuria Committee
respect to •• It was set-up in June 1994 to work out details as to how
•• Preparation of plants for economic development and structures similar to Panchayati Raj Institutions can take
social justice. shape in Tribal Areas and Scheduled Areas to define their
•• the implementation of schemes for the economic powers. It submitted it’s report in January 1995 and GOI
development and social justice as may be entrusted to made PESA, 1996.
them, including those in relation to the 29 matters listed •• According to this act, under Part IX of the Constitution,
in the Eleventh Schedule. the Legislature of a State shall not make any law under

ww
The following 29 functional items placed within the purview
of Panchayats are:
1. Agriculture, including agricultural extension.
that part, which is in consistent with the following features
ƒƒ A State Legislation on the Panchayats shall be
in consonance with the customary law, social and

w.E
2. Land improvement, implementation of land reforms, land
consolidation and soil conservation.
3. Minor irrigation, water management and watershed
religious practices and traditional management
practices of community resources.
ƒƒ Reservation for the Scheduled Tribes shall not
development.

a
4. Animal husbandry, dairying and poultry.

syE
5. Fisheries, social forestry and farm forestry.
6. Minor forest produce.
be less than one-half of the total number of seats.
Reservation shall be in proportion to the population
of the communities for whom reservation is sought.
All seats of Chairpersons at all levels shall be
reserved for the Scheduled Tribes.
7. Small-scale industries, including food processing
industries.
8. Khadi, village and cottage industries. ngi ƒƒ Gram Sabha shall be consulted before making
the acquisition of land in Scheduled Areas for
9. Rural housing.
10. Drinking water.
11. Fuel and fodder.
nee
development projects.
ƒƒ The recommendation of Gram Sabha shall be made
mandatory to grant of prospecting licence or mining
12. Roads, culverts, bridges, ferries, waterways and other
means of communication.
13. Rural electrification, including distribution of electricity.
r
lease for minor minerals in the Scheduled Areas.

ing
ƒƒ Powers of Panchayat in the Scheduled Areas to
regulate, sale and consumption of intoxicant,

.ne
ownership of minor forest produce, preventing
14. Non-conventional energy sources. alienation of land as per laws, manage village
15. Poverty alleviation programme. markets, exercise control over money lending,
16. Education, including primary and secondary schools.
17. Technical training and vocational education.
18. Adult and non-formal education.
19. Libraries.
sectors and local plans and resources.
t
control over institutions and functionaries in social

ƒƒ The act extended provisions of Panchayats to tribal


areas of 9 states that are included in Fifth Schedule
20. Cultural activities. these are - Andhra Pradesh, Himachal Pradesh,
21. Markets and fairs. Madhya Pradesh, Chhattisgarh, Jharkhand, Odisha,
22. Health and sanitation, including hospitals, primary health Gujarat, Maharashtra, and Rajasthan.
centres and dispensaries.
23. Family welfare. The Scheduled Tribes and other
24. Women and child development. Traditional Forest Dwellers
25. Social welfare, including welfare of the handicapped and (Recognition of Forest Rights) Act,
mentally retarded.
26. Welfare of the weaker sections and in particular, of the
2006
Scheduled Castes and the Scheduled Tribes. •• The Scheduled Tribes and other Traditional Forest
27. Public distribution system. Dwellers (Recognition of Forest Rights) Act, 2006 is a
28. Maintenance of community assets. result of the protracted struggle by the marginal and tribal
29. The act gives a constitutional status to the Panchayati Raj communities of our country to assert their rights over the
Institutions. forest land over which they were traditionally dependent.

Downloaded From : www.EasyEngineering.net


Downloaded From : www.EasyEngineering.net
Local Government P-123

ƒƒ This act is crucial to the rights of millions of tribals and of the Natural Resources and Conservation
other forest dwellers in different parts of our country Governance of India.
as it provides for the restitution of deprived forest
rights across India. Including both individual rights Significance of the Act
to cultivated land in forest land and community rights •• For the first time Forest Rights Act recognises and secures
over common property resources. The notification or ƒƒ Community Rights or rights over common property
rulers for the implementation of the Forest Right. resources of the communities in addition to their
Rights Act, 2006 on 1st January. 2008, has finally individual rights.
paved the way to undo the ‘historical injustice’ done ƒƒ Rights in and over disputed land rights of settlement
to the tribals and other forest dwellers. and conversion of all forest villages, old habitation,
ƒƒ The livelihood of perhaps 100 million poorest of the un-surveyed villages and other villages in forests into
poor (the Indian Forest Rights Act, 2006; Communing revenue villages.
Enclosures) stands to improve if implementation can ƒƒ Right to protect, regenerate or conserve or
succeed. The act is significant as it provides scope manage any community forest resource which the
and historic opportunity of integrating conservation communities have been traditionally protecting and
and livelihood rights of the people. conserving for sustainable use.
•• This act will prove a potential tool ƒƒ Right to intellectual property and traditional

ww
ƒƒ To empower and strengthen the local self-governance.
ƒƒ To address the livelihood security of the people,
leading to poverty alleviation and pro-poor growth.
knowledge related to biodiversity and cultural
diversity.
ƒƒ Rights of displaced communities.

w.E
ƒƒ To address the issues of conservation and management

URBAN LOCAL GOVERNMENT


ƒƒ Rights over developmental activities.

••
a syE
There are eight types of urban local governments in India
– Municipal Corporation, Municipality, Notified Area
Committee, Town Area Committee, Cantonment Board,
Municipal Corporation
It is created for the administration of big cities like Delhi,
Mumbai, Kolkata, Hyderabad, Bangalore and others. Established
township, port trust, special purpose agency. At the
Central level the subject of ‘urban local government’ is
dealt with by the following three Ministries. ngi in the States by the Acts of the concerned State Legislatures and
in the UTs by the Acts of the Parliament of India.


(i) Ministry of Urban Development created as a separate
ministry in 1985.
(ii) Ministry of Defense in the case of cantonment
nee
Municipal Council
Municipal Council is established for the administration of
towns and smaller cities. They are also set up in the States by


boards.
(iii) Ministry of Home Affairs in the case of Union
Territories.
r ing
the Acts of the concerned State Legislatures and in the UTs by
the Acts of the parliament. They are also known by various
other names like Municipal Committee, Municipal Board,

74th Amendment Act (1992)


Composition .ne
Borough municipality, City municipality and others.

••

••
74th Amendment Act pertaining to urban local government
was passed during the regime of P.V. Narsimha Rao’s
government in 1992. It came into force on 1st June, 1993.
Added Part IX -A and consists of provisions from articles
••

••
t
Members shall be elected directly. State legislature may
provide the manner of election of the chairperson of a
Municipality.
For this purpose, each Municipal area shall be divided
243-P to 243-ZG. into territorial constituencies to be known as Wards. It
•• Added 12th Schedule to the Constitution. It contains 18 may also provide for the representation of the following
functional items of Municipalities and deals with Article persons in a municipality.
243 W. 1. Persons having special knowledge or experience in
•• State governments are under constitutional obligation to municipal administration without the right to vote in
adopt the new system of Municipalities in accordance the meetings of municipality.
with the provisions of the Act. 2. The members of the Lok Sabha and the state
legislative assembly representing constituencies that
Municipalities comprise wholly or partly the municipal area.
There are three types of municipalities which are as follows: 3. The members of the Rajya Sabha and the state
(i) Nagar panchayat for a transitional area from a rural area Legislative Council registered as electors within the
to urban area. municipal area.
(ii) A municipal council for a smaller urban area 4. The chairpersons of committees (other than wards
(iii) A municipal corporation for a larger urban area. committees).

Downloaded From : www.EasyEngineering.net


Downloaded From : www.EasyEngineering.net
P-124 Local Government

Wards Committees (a) If he is so disqualified by as under any law for the


time being in force for the purposes of election to the
Wards committee, consisting of one or more wards, within
legislature of the state concerned provided that no
the territorial area of a municipality having population of three
person shall be disqualified on the ground that he is
lakhs or more.
less than twenty-five years of age, he has attained the
Reservation of Seats (Art. 243 T) age of twenty-one years;
•• For SCs & STs, in proportion of their population to the (b) If he is so disqualified by or under any law made by
total population in that area. For backward classes, state the legislature of the state.
legislature may make provisions. • If any question arises as to whether a member of
•• Reservation of not less than one-third of the total number a municipality has become subject to any of the
of seats for women (including those reserved for women disqualifications mentioned in clause (1), the question
belonging to the SCs and the STs) shall be referred for the decision of such authority and
in such manner as the legislature of a State may, by law,
•• State legislature may provide for the manner of reservation
provide.
of offices of chairpersons in the municipalities for the
SCs, the STs and the women. Bar to Interference by Courts in Electoral
Committee for District Planning (Art 243 ZD) Matters

ww
There shall be constituted in every state at the district
level a District Planning Committee to consolent the plans
Not with standing Naturestanding anything in this constitution-
• The validity of any law relating to the delimitation
of constituencies or the allotment of seats to such


district as a whole.w.E
prepared by the Panchayats and the municipalities in the
district and to prepare a draft development plan for the

The Legislature of a state may, by law, make provision,


constituencies made or purporting to be made under article
243ZA, shall not be called in question in any court.
• No election to any municipality shall be called in question



with respect to —
a syE
(a) The composition of the District Planning Committees;
(b) The manner in which the seats in such committees
shall be filled:
except by an election petition presented to such authority
and in such manner as is provided for by or under any law
made by the Legislature of a state.

Continuance of Existing Law and Municipality


Provided that not less than four-fifths of the total
number of members of such committee shall be ngi Nature standing anything in this part, any provision of any
law relating to municipalities in force in a state immediately
elected by and from amongst, the elected members
of the Panchayat at the district level and of the
municipalities in the district in proportion to the ratio nee
before the commencement of the constitution (seventy-
fourth amendment) Act, 1992, which is inconsistent with
the provisions of this part, shall continue to be in force until


between the population of the rural areas and of the
urban area in the district.
(c) The function relating to district planning which may
be assigned to such committees.
r ing
amended or replaced by a competent legislature or other
competent authority or until the expiration of are year from
such commencement, whichever is earlier.

(d) The manner in which the chairpersons of such


committees shall be chosen. .ne
Provided that all the municipalities existing immediately before
such commencement shall continue all the expiration of their
duration, unless sooner dissolved by a resolution passed to that



Every District Planning Committee shall, in preparing the
draft development plan,
(a) have regard to–
(i) matters of common interest between the panchayats
t
effect by the Legislative Assembly of that State or, in the case
of a state having a Legislative Council, by each House of the
Legislature of that state.

and the municipalities including spatial planning Duration of Municipalities


sharing of water and other physical and natural Five-year term. However, it can be dissolved before the
resources, the integrated development of completion of its term. Further, the fresh election to constitute
infrastructure and environmental conservation. a municipality shall be completed (i) before the expiry of its
(ii) the extent and type of available resources whether duration of five year; or (ii) in case of dissolution before the
financial or otherwise. expiry of a period of six months from the date of its dissolution.
(b) Consult such institutions and organizations as the
Governor may, by order specify. Powers and Functions
• The chairperson of every District Planning Committee Under Article 243(W), subject to the provisions of this
shall forward the development plan, as recommended by Constitution, the Legislature of a State may, by law, endow
such committee, to the Government of the State. •• The municipalities with such powers and authority as amy
be necessary to enable them to function as institutions of
Disqualifications for Membership self government and such law may contain provisions
• (A person shall be disqualified for being chosen as, and for for the devolution of powers and responsibilities upon
being a member of a municipality-

Downloaded From : www.EasyEngineering.net


Downloaded From : www.EasyEngineering.net
Local Government P-125

municipalities, subject to such conditions as may be Application to Union Territories


specified therein, with respect to
•• President of India may direct that the provisions of this
ƒƒ the preparation of plans for economic development act shall apply to any union territory subject to such
and social justice. exceptions and modifications as he may specify.
ƒƒ the performance of functions and the implementation
of schemes as my be entrusted to them including Areas Kept Out
those in relation to the matters listed in Twelfth •• Scheduled areas and tribal areas referred in Article 244
Schedule. of the Indian Constitution. It shall also not affect the
•• The committees with such powers and authority as may be functions and powers of the Darjeeling, Gorkha Hill
necessary to enable them to carry out the responsibilities Council of the West Bengal.
conferred upon them including those in relation to the
matters listed in the Twelfth Schedule. Urban-local Government Structures
in India
Twelfth Schedule
There are eight types of Urban Local bodies that are created
It contains 18 functional items which is covered in Article 243 in India for the administration of urban areas. These are as
(W). They are as follows follows:

ww
•• Urban planning including tow planning.
•• Regulation of land use and construction of buildings.
•• Planning for economic and social development.
1. Municipal Corporation
Municipal Corporation are for the big cities like Delhi,
•• Roads and bridges.
w.E
•• Water supply for domestic, industrial and commercial
purposes.

Mumbai, Kolkata, Hyderabad, Bengaluru and others.
These are established by the Acts of State Legislature and
the Parliament in case of Union Territories. A Municipal

management.
•• Fire services.
a
•• Public health, sanitation conservancy and solid waste

syE
Corporation has three authorities.

(i) The Council


It is a deliberative and legislative wing of the corporation.
It has councilors, directly elected by the people.
•• Urban forestry, protection of the environment and
promotion of ecological aspects.
•• Safeguarding the interests of weaker sections of society, ngi The council is headed by the Major and he/she is assisted
by a Deputy Major who is elected for 1 year with
including the handicapped and mentally retarded.
•• Slum improvement and upgradation.
•• Urban poverty alleviation.
nee
renewable term.

(ii) The Standing Committees

•• Provision of urban amenities and facilities such as parks,


gardens, and playgrounds.
•• Promotion of cultural, educational and aesthetic aspects.

r
lt is created to facilitate the working of the Council. These

ing
committees deal with public works, education, health,
taxation, finance, etc.

•• Burials and burial grounds; cremations, cremation


grounds and electric crematoriums.
(iii) The Municipal Commissioner

.ne
The Commissioner is responsible for the implementation
of the decisions taken by the Council and its standing
•• Cattle pounds; prevention of cruelty to animals.
•• Vital statistics including registration of births and deaths.
•• Public amenities including street lighting, parking lots,
bus stops and public conveniences.
t
committees. The Commissioner is appointed by the State
Government. So, that he is the chief executive authority of
the Municipal Corporation. Generally, the Commissioner
is a member of the IAS cadre.
•• Regulation of slaughter houses and tanneries. 2. Municipality
The Municipalities are established for the administration of
Finance Commission towns and small cities. Like the corporation, they are also
•• Consituted for every five years, review the financial established by the Act of State Legislature and the Parliament.
position of municipalities and make recommendation to Municipality has three authorities
the Governor. (i) The Council
•• The distribution of net proceeds of taxes, duties, tolls and (ii) The Standing Committees
fees between State Municipalities, which are levied by (iii) The Chief Executive Officer. All the authorities have
the State. same functions as the corporation has.
•• Central Finance Commission shall also suggest the 3. Notified Area Committee
measures needed to augment the Consolidated Fund of a
It is any land area earmarked by legal provision for future
State to supplement the resources of the Municipalities in
development. It is set-up by government notification and
the State.

Downloaded From : www.EasyEngineering.net


Downloaded From : www.EasyEngineering.net
P-126 Local Government

not a legislation. – Pollution control boards.


All its members and Chairman are appointed by the – Electricity supply boards.
State Government and not elected. It is set-up in area – City transport boards.
where municipality is not feasible, but potential for fast •• These bodies are created as statutory bodies by an Act
development is there. of State Legislature or as department by an executive
resolution.
4. Town Area Committee
•• They function as autonomous bodies and are not
It is set-up by an Act of State Legislature and can have both subordinate agencies of the local municipal bodies.
elected and nominated members. It is quasi-municipality
with limited number of Municipal functions like street/
lighting, sanitation, etc.
Co-operatives
In 1958, the National Development Council (NDC) had
5. Cantonment Board ‘recommended a national policy on co-operatives. Jawaharlal
They are autonomous bodies functioning under the overall Nehru had a strong faith in the co-operative movement. In
control of the Ministry of Defence. These boards comprise 2011, the cooperatives were given constitutional status by the
elected members besides ex-officio and nominated 97th Constitutional Amendment Act, 2011. Co-operatives
members with the station commander as the President of have been inserted in Part IXB covering Articles 243 ZH-ZT.

ww
the Board.
The resources of these boards are limited as the bulk of
the property is owned by government on, which no tax
Incorporation of co-operative societies (Article 243ZI) subject
to the provisions of this part, the Legislature of a State may
by law make provisions with respect to the incorporation,


can be levied.
w.E
Thus, the Central Government provides financial assistance
by way of grants-in-aid. The boards are responsible for
regulation and winding up of co-operative societies based on
the principles of voluntary formation, democratic control,
economic participation and autonomous functioning.

a
discharging the mandatory duties like provision of public

syE
health, sanitation, primary education and street lighting
etc.

6. Township
Audit of Accounts of Co-operative Societies
(Article 243 ZM)
The Legislature of a State may by law, make provisions with

The township is established to provide civic amenities to


its staff and workers who live in the housing colonies builtngi respect to the maintenance of accounts by the cooperative
societies and the auditing of such accounts atleast once in each
financial year.


near the plant by the large public enterprises.
The township is headed by the town administrator,
appointed by the enterprises, he/she is assisted by some nee
The accounts of every cooperative society shall be audited
within 6 months of the close of the financial year to which such
accounts relate. The audit report of the accounts of an apex
engineers and other technical and non-technical staff. It
shows the bureaucratic structure of the enterprises.

7. Port Trust
r
co-operative society as may be defined by the State Act shall

ing
be laid before the State Legislature in the manner, as may be
provided by the State Legislature by law.

The Port Trusts are established in the port areas such as


Mumbai, Kolkata, Chennai, etc. These are established for
Number and Term of Members of Board and its
Office Bearers (Article 243 ZJ) .ne



the following purposes
– To manage and protect the ports.
– To provide civic amenities.
The port trust is established by an Act of the Parliament. It
••
t
The board shall consist of such number of directors as
may be provided by the Legislature of a State by law. The
maximum number of directors of a cooperative society
shall not exceed twenty-one.
has both elected and nominated members. Its Chairman is •• The Legislature of a State shall by law provide for
an official and its civic functions are more or less similar reservation of one seat for the Scheduled Castes or the
to those of a municipality. Scheduled Tribes and two seats for women on board of
every cooperative society consisting of individuals as
Special Purpose Agency members and having members from such class or category
•• Above all seven area based urban bodies, the states have of persons.
set-up certain agencies to undertake designated activities •• The term of office of elected members of the board and
or specific functions. The special purpose agencies are its office bearers shall be 5 years from the date of election
established for single purpose. Some such bodies are as and the term of office bearers shall be continuous with the
follows : term of the board.
•• Town improvement trusts The Legislature of a State shall, by law, make provisions for
– Water supply and sewerage boards. persons to be members of the board having experience in the
– Urban development authorities. fields of banking, management, finance or specialisation in any
– Housing boards. other field relating to the objects and activities undertaken by

Downloaded From : www.EasyEngineering.net


Downloaded From : www.EasyEngineering.net
Local Government P-127

the cooperative society as members of the board. •• The Council is an advisory body. It consists of the Minister
for Urban Development in the Union and for local self-
Election of Members of Board (Article 243 ZK) government in the state. The Chairman of the Council is
The superintendence, direction and control of the preparation the Union Minister. The Council has following functions
of electoral rolls for and the conduct of all elections to a ƒƒ Considering and recommending the policy matters.
cooperative society shall vest in such an authority or body, as ƒƒ Making proposals for legislation.
may be provided by the Legislature of a State by law. ƒƒ Drawing up a common programme of action.
Central Council of Local Government ƒƒ Examining the possibility of cooperation between the
Centre and the States.
•• The Central Council of Local Government was set-
up in 1954. It was constituted under Article 263 of the ƒƒ Recommending Central Financial Assistance.
Constitution of India by an order of the President of India. ƒƒ Reviewing the work done by the local bodies with
the Central Financial Assistance.

LEVELS OF GOVERNMENT

ww Union Govt.

Administrationw.E
Responsible for
State Govt.
& Governance of
whole Country
a syE Responsible for
Administration &
Governance of
• District Level unit
• Examine the Budget ngi
State

• Organise meeting
& supervise work nee Local Govt.

of panchayat samiti

• Adopt measures for r


Panchayati Raj Institutions
(Rural) ingMunicipalities
(Urban)
completion of
activities
• Block level Zila Nagar .ne
• Executive Organ
of Gram Sabha
• Prepare Plans
Panchayat
Panchayat
Samiti
Panchayat
Municipal
Council
t
• Perform social, Gram Municipal
welfare &
Sabha Corporation
Developmental
activities
• Lowest Unit
• Directly Elected
Members

Downloaded From : www.EasyEngineering.net


Downloaded From : www.EasyEngineering.net
P-128 Local Government

Exercise - 1
1. ______ conducts elections to Panchayats and 9. “The state shall take steps to organise village Panchayats
Municipalities? and endow them with such powers as may be necessary
(a) State Government to enable them to function as units of self-government.”
(b) Central Government This provision is mentioned in:
(c) State Election Commission (a) Part I of the Constitution
(d) Central Election Commission (b) Part IV-A of the Constitution
2. Regarding reservation of seats on Panchayats and (c) Part III of the Constitution
Municipalities, it would INCORRECT to say: (d) Part IV of the Constitution
(a) Seats are reserved for SC/ST in proportion to their 10. Which of the following is not correct about a cantonment
number board?
(b) At least 1/3 of total number of seats are filled by (a) It is created by an executive resolution.
direct elections and are reserved for women (b) It works under the administrative control of the
(c) reservations for SC/ST are effective till 2010 Union Defence Ministry.

ww
(d) Unreserved seats cannot be contested by women
3. Direct elections to all tiers of the Panchayat were held
first after the 73rd Amendment came into force in ____?


(c) It is established for municipal administration for
civilian population in the cantonment area.
(d) It is a statutory body

(c) Karnataka w.E
(a) Andhra Pradesh (b) Rajasthan

4. 73rd and 74th Amendment Acts:


(d) Madhya Pradesh
11. A committee appointed in 1977 to review working of the


Panchayti Raj was chaired by
(a) Balwant Rai Mehta




a
(a) Came into force on being passed by the Parliament
(b) Both came into force in June, 1993
syE
(c) Had to be ratified by more than half the states
(d) Have not been ratified by the required number of



(b) Ashok Mehta
(c) K.N. Katju
(d) Jagjivan Ram
12. If a Panchayat is dissolved, elections are to be held
states
ngi
5. If the Governor of a State is appointed administrator of a


within:
(a) One month
(c) Six months
(b) Three months
(d) One year

nee
Union Territory, he exercises his functions 13. Which of the following committees is not concerned with
(a) on advice of his Council of Ministers Panchayati Raj?
(b) independently of Council of Ministers (a) Santhnam Committee


(c) according to Parliament’s directions
(d) on the directions of the State Legislature
6. _____ was not proposed by the 73rd constitutional
amendment for Panchayati raj?


r
(b) Ashok Mehta Committee

ing
(c) Balwant Rai Mehta Committee
(d) V.K.R.V. Rao Committee
14. Under 74th Amendment of the Constitution the local
(a) 30% seats in all elected rural local bodies will be
reserved for women .ne
body for a transitional area is known as
(a) Nyaya Panchayat
(b) States will constitute Finance Commissions to


allocate resources to Panchayati Raj bodies
(c) Elected functionaries would be disqualified to hold
offices if they have more than two children



(b) Municipal Panchayat
(c) Nagar Panchayat
(d) Gram Panchayat t
15. ‘Swaranajayanti Gram Swarozgar Yojna’ came into
(d) Elections will be held in six months’ time if being in:
Panchayati Raj bodies are superseded or dissolved. (a) April, 1995 (b) April, 1997
7. According to the Balwant Ray Mehta Committee, the (c) April, 1999 (d) July, 2001
District Collector should be: 16. The system of Panchayati Raj involves
(a) Kept out of the Zila Parishad (a) The village block and district levels
(b) A non-voting member of the Zila Parishad (b) The village and State levels
(c) A member of the Zila Parishad with the right to vote (c) The village State and Union levels
(d) The Chairman of the Zila Parishad (d) None of these
8. Panchayati Raj form of rural local government was 17. The primary aim of the Panchayati Raj administration is:
adopted first by (in the order): (a) to work for rural development
(a) Rajasthan and Madhya Pradesh (b) to ensure the upliftment of Harijans
(b) Andhra Pradesh and West Bengal (c) to arouse in the people continuous interest in the
(c) Rajasthan and Andhra Pradesh community development programmes
(d) Andhra Pradesh and Rajasthan (d) to increase agricultural production through the
involvement of the people in extension programmes

Downloaded From : www.EasyEngineering.net


Downloaded From : www.EasyEngineering.net
Local Government P-129

18. The lowest unit in the Panchayati Raj institutions is 30. In which part of the Constitution is the State enjoined to
(a) a village panchayat (b) the Panchayat Samiti establish Panchayati Raj institutions?
(c) Zilla Parishad (d) the Gram Sabha (a) Preamble (b) Directive Principles
19. The Committee on whose recommendation Panchayati (c) Fundamental Rights (d) Seventh Schedule
Raj was introduced in the country was headed by 31. Which of the following is not one of the sources of
(a) Jivraj Mehta (b) Ashok Mehta revenue of village Panchayats?
(c) Balwant Rai Mehta (d) None of these (a) Government Grants (b) House Tax
20. When was the Panchayati Raj introduced in India? (c) Income Tax (d) Local Taxes on land
(a) 1950 (b) 1959 32. Pick out the statement which is not correct.
(c) 1952 (d) 1962 (a) The Gram Panchayat is headed by the Sarpanch
21. One of the following is not a part of Panchayati (b) The Panchayat Samiti is headed by the Chairman
(a) District Board (c) The Zila Parishad is headed by the Chairman
(b) Town Area Committees (d) The Sarpanch and the Chairman are elected directly
by the people
(c) Village Block
33. A Panchayat Samiti at the Block level is:
(d) Panchayati Boards
(a) an advisory body
22. The original Scheme of Panchayati Raj introduced in
(b) an administrative authority
1959, operates at (in descending order of tiers):
(c) a consultant committee


ww
(a) Zilla Parishad. Samiti, Gram Sabha
(b) Panchayat Samiti, Gram Panchayat, Zilla Parishad
(c) Gram Sabha, Zilla Parishad, Village Panchayat
(d) a supervisory authority
34. Who has representation on the Zilla Parishad?
(a) Women

23.


India’? w.E
(d) Zilla Parishad, Panchayat Samiti, Village Panchayat
Who is known as the ‘Father of Local Government in

(a) Lord Curzon (b) Lord Wellesly





(b) Scheduled Castes and Scheduled Tribes
(c) Representatives of cooperative societies
(d) All of the above


24.


(c) Lord Mayo
a syE
(d) Lord Ripon
A person to be qualified for standing in a Panchayat
election must have attained the age of:
(a) 21 years (b) 18 years
35. Panchayati Raj is




(a) the functioning of village republics in a democracy
(b) the self-government of the villagers in India
(c) a complex system of rural local government
(d) a hierarchical set up for rural administration

25.
(c) 25 years (d) 30 years
One of the advantages of the Panchayati Raj is that ngi 36. Balwantrai Mehta team was set up in 1956 by the National


Development Council for the purpose of:
(a) reporting on the working of the village panchayats at

nee
(a) of providing a parallel government in rural areas
(b) of serving the rural people well that time
(c) of increasing the employment opportunities in rural (b) investigating the feasibility of setting up the new
panchayat machinery

26.
areas
(d) giving a sense of political awareness to the rural
masses
The Panchayati Raj institutions in India get their funds


r ing
(c) suggesting measures for democratic decentralization
(d) suggesting measures for better efficiency in the
implementation of the Community Development



mainly from
(a) voluntary contributions
(b) property tax

projects

.ne
37. Who are generally the members of Gram Sabha?
(a) All the heads of families of the respective villages


27.

(c) local taxes
(d) Government grants
What is main purpose of Panchayati Raj?
(a) To increase agricultural production



(c) All adult males in the villages
(d) Nominated members
38. Local government institutions:

t
(b) All the voters in the respective villages

(a) can levy taxes with prior approval of the State


(b) To create employment government
(c) To make people politically conscious (b) can levy all taxes
(d) To make people participate in developmental (c) propose taxes to the state governments
administration (d) can levy taxes
28. Nyaya Panchayats are expected to 39. In the three-tier Panchayati Raj structure the Block is:
(a) deal with all crimes committed in the villages (a) the lowest structure
(b) met out justice to villagers so that litigation is (b) the intermediary structure
minimum (c) the highest structure
(c) settle disputes through persuasion (d) None of these
(d) try petty civil suits and minor offences 40. The members of the Panchayat are:
29. Which of the following States was the first to establish (a) nominated by the District Officer
the new Panchayati Raj institutions? (b) elected by the people
(a) Rajasthan (b) Maharashtra (c) nominated by Local Self-Government and Ministers
(c) Bihar (d) Andhra Pradesh of the State
(d) nominated by the Block Development Organisation

Downloaded From : www.EasyEngineering.net


Downloaded From : www.EasyEngineering.net
P-130 Local Government

41. The Zilla Parishad is: 49. Which of the following is the main source of income for
(a) a co-ordinating and supervisory body the Municipal Committee?
(b) an implementing body (a) octroi duty
(c) a judicial body (b) income tax
(d) a combination of all the above (c) assistance from the centre
42. Which of the following States has no Panchayat Raj (d) excise duty
institution at all? 50. Which one of the following statements regarding local
(a) Assam (b) Nagaland government in India has been wrongly listed?
(c) Tripura (d) Kerala (a) The election to local bodies are determined by a
43. The Panchayati Raj institutions depend for funds mainly commission
on:
(b) 30 per cent of the seats in local bodies are reserved
(a) local taxes
for the women
(b) property tax
(c) Finances to local governments are provided by a
(c) government finances
(d) grants-in-aid from the Central Government commission
44. In the new Panchayati Raj Bill enacted in 1993, there are (d) None of the above
several fresh provisions deviating from the past. Which 51. What is the system of governance in the Panchayati Raj
one of the following is not such provision? set up?

ww
(a) A number of added responsibilities in the area of
agriculture, rural development, primary education
and social forestry among others


(a) Single tier structure of local self govt. at the village
level
(b) Two tier system of local self govt. at the village and


w.E
(b) Elections being made mandatory for all posts at the
time they are due
(c) A statutory representation for women in the


block levels.
(c) Three tier structure of local self govt. at the village,
block and district levels.
(d) Four tier system of local self govt. at the village,

45.
a
panchayats, upto a third of the strength

syE
(d) Regular remuneration to the Panchayat members, so
as to ensure their punctuality and accountability
For successful functioning, Panchayati Raj needs the
block, district and state levels.
52. Which one of the following was NOT proposed by the
73rd constitutional amendment in the area of Panchayati
Raj?


cooperation of:
(a) the state government
(b) the central government ngi (a) Thirty percent seats in all elected rural local bodies
will be reserved for women candidates at all levels.

nee
(c) the local people (b) The state will constitute their Finance Commissions
(d) the bureaucracy to allocate resources to Panchayati Raj Institutions.
46. What is the system of goverance in the Panchayati Raj set (c) The Panchayati Raj elected functionaries will be


up?
(a) Single tier structure of local self government at the
village level
(b) Two tier system of local self government at the
r
disqualified to hold their offices if they have more
than two children.

ing
(d) The elections will be held in six months time if
Panchayati Raj bodies are superseded or dissolved


village and block levels
(c) Three tier structure of local self government at the
by the state government.
.ne
53. The Balwant Rai Mehta Committee recommended which
one of the following Panchayati Raj structures?

47.


village, block and district levels
(d) None of these
Which one of the following is incorrect in respect of local
government in India?
(a) According to the Indian Constitution, local


t
(a) Gram Panchayat at the village level and Panchayat
Samiti at the block level only
(b) Panchayat Samiti at the block level and Zilla
Parishad at the district level only
government is not an independent tier in the federal (c) Gram Panchayat at the village level, Panchayat
system Samiti at the block level and Zilla Parishad at the
(b) 30% of the seats in local bodies are reserved for district level
women (d) Gram Panchayat at the village level and Zilla
(c) Local government finances are to be provided by a Parishad at the district level only
Commission 54. Which of the following is not a recommendation of the
(d) Elections to local bodies are to be determined by a Ashok Mehta Committee on Panchayati Raj ?
Commission (a) Open participation of political parties in Panchayati
Raj affairs
48. Which part of constitution directs the state to establish
(b) Creation of a three-tier system
Panchayati Raj institutions in the country?
(c) Reservation of seats for Scheduled Castes and
(a) The Preamble
Scheduled Tribes
(b) The Directive Principles of State Policy (d) Compulsory powers of taxation to Panchayati Raj
(c) The Fundamental Rights Institution.
(d) None of the above

Downloaded From : www.EasyEngineering.net


Downloaded From : www.EasyEngineering.net
Local Government P-131

55. The tenure of every Panchayat shall be for five years 60. What is the system of governance in the Panchayati Raj
from the date of set up?
(a) its first meeting (a) Single tier structure of local self government at the
(b) issue of notification for the conduct of elections to village level .
the Panchayat (b) Two tier system of local self government at the
(c) declaration of the election results village and block levels
(d) taking oath of office by the elected members (c) Three tier structure of local self government at the
56. The Parliament of India passed the Panchayats Extension village, block and district levels
to Scheduled Areas Law popularly known as PESA law. (d) Four tier system of local self government at the
Which one among the following statements regarding village block, district and in the state levels
PESA law is not correct?
61. Panchayat Raj was first introduced in India in October,
(a) PESA was meant to provide self-governance in the
1959 in:
scheduled areas
(a) Rajasthan (b) Tamil Nadu
(b) PESA disempowers Gram Sabhas
(c) Kerala (d) Karnataka
(c) PESA protects the interests of the tribals
62. In the new Panchayati Raj Bill enacted in 1993, there are
(d) PESA conducts public hearings to protect inheritance

ww rights of the tribals


57. Which one of the following was not proposed by the 73rd

several fresh provisions deviating from the past.
Which one of the following is not one such provisions?
(a) A number of added responsibilities in the area of

w.E
Constitutional Amendment in the area of Panchayati Raj?
(a) Thirty percent seats in all elected rural local bodies
will be reserved for women candidates at all level

agriculture, rural development, primary education
and social forestry among other
(b) Elections being made mandatory for all posts at the

a
(b) The States will constitute their Finance Commissions

syE
to allocate resources to Panchayati Raj institutions
(c) The Panchayati Raj functionaries will be disqualified
to hold their offices if they have more than two
time they are due
(c) A statutory representation for women in the
panchayats, upto a third of the strength


children
(d) The elections will be held in six months time if
Panchayati Raj bodies are superceded or dissolved ngi (d) Regular remuneration to the panchayat members, so
as to ensure their punctuality and accountability
63. If a Panchayat is dissolved, elections are to be held
by the State government
58. The 73rd Constitution Amendment Act, 1992 refers to nee
within:
(a) 1 month (b) 3 months


the:
(a) generation of gainful employment for the unemployed
and the under employed men and women in rural
areas

r
(c) 6 months

ing
(d) 1 year
64. In India, the first Municipal Corporation was set up in
which one among the following?

(b) generation of employment for the able bodied adults


who are in need and desirous of work during the
(a) Calcutta
(c) Bombay
.ne
(b) Madras
(d) Delhi
65. Under the Scheduled Tribes and Other Traditional


lean agricultural season
(c) laying the foundation for strong and vibrant
Panchayati Raj institutions in the country
(d) guarantee of right to life, liberty and security of
t
Forest Dwellers (Recognition of Forest Rights) Act,
2006 who shall be the authority to initiate the process
for determining the nature and extent of individual or
community forest rights or both?
person, equality before law and equal protection
without discrimination (a) State Forest Department
59. Which one of the following is incorrect in respect of (b) District Collector / Deputy Commissioner
Local Government in India? (c) Tahsildar / Block Development Officer / Mandal
(a) According to the Indian Constitution, local Revenue Officer
government is not an independent tier in the federal (d) Gram Sabha
system 66. The Government enacted the Panchayat Extension to
(b) 30% of the seats in local bodies are reserved for Scheduled Areas (PESA) Act in 1996. Which one of the
women following is not identified as its objective?
(c) Local government finances are to be provided by a (a) To provide self-governance
Commission (b) To recognize traditional rights
(d) Elections to local bodies are to be determined by a (c) To create autonomous regions in tribal areas
Commission (d) To free tribal people from exploitation

Downloaded From : www.EasyEngineering.net


Downloaded From : www.EasyEngineering.net
P-132 Local Government

Exercise - 2
5. Consider the following statements :
Statement Based MCQ
In India, a Metropolitan Planning Committee :
1. is constituted under the provisions of the Constitution
1. In municipalities,
of India.
1. there is 50% reservation for women in the scats to
be filled by direct elections 2. prepares the draft development plans for metropolitan
area.
2. grant-in-aid may be given to municipalities from the
Consolidated Fund 3. has the sole responsibility for implementing
Government sponsored schemes in the metropolitan
Which of the following statements is/are correct?
area.
(a) 1 only (b) 2 only
Which of the statements given above is/are correct?
(c) Both 1 and 2 (d) Neither 1 nor 2
(a) 1 and 2 (b) 2 only
2. The recommendations of Balwant Rai Mehta Committee

ww
includes:
1. Open participation of political parties in Panchayati
(c) 1 and 3 (d) 2 and 3
6. Which of the following provisions about the Panchayati
Raj in the Constitution of India is/are correct?

w.E
Raj affairs.
2. Genuine transfer of power and responsibility to the 1. All the members of Panchayats at the village,
Panchayati Raj institutions. intermediate and district levels are elected directly
by the voters.
3. Constitutional protection for Panchayati Raj.


Zila Parishad. a syE
4. District Collector should be the Chairman of the

5. Panchayat Samlti to be the executIve body



2. The Chairperson of Panchayats at the village,
intermediate and district levels is elected directly by
the voters.
Select the correct answer using the codes given below:


Which of the following statements is/are correct?
(a) 1, 2 and 5 (b) 2, 4 and 5
ngi

(a) 1 only
(c) Both 1 and 2
(b) 2 only
(d) Neither 1 nor 2

3.
(c) 2, 3 and 4 (d) 1, 3 and 4
Which of the following are the compulsory provisions of
the 73rd Amendment Act on Panchayati Raj? nee
7. Which one of the following is incorrect in respect of local
government in India?
(a) According to the Indian constitution, local


1. Indirect elections of the chairpersons of Panchayats
at the intermediate and district levels.
2. Fresh elections within six months in case of dissolution.
r
government is not an independent tier in the federal
system.
ing
(b) 30% of the seats in local bodies are reserved for


3. Provision for reservation of seats for back-ward
classes.
4. Giving representation to MPs and MLAs in Panchayats.

women.

.ne
(c) Local government finances are to be provided by a

t
commission.
Which of the following statements is/are correct? (d) Elections to local bodies are to be determined by a
(a) 1, 3 and 4 (b) 2 and 4 commission.
(c) 1 and 2 (d) 2,3 and 4 8. Which among the following are true about the 73rd
4. Consider the following statements constitutional Amendment in the area of Panchayati Raj?
1. Stamp duties : levied by and duties on the Union 1. Thirty per cent seats in all elected rural local bodies
medical and but collected toilet prepara and will be reserved for women candidates at all levels.
approtions priated by the States levied and collected 2. The States will constitute their Finance Commissions
by the Union but assigned to the States to allocate resources to Panchayati Raj institutions.
2. Duties on succession to property other than 3. The Panchayati Raj elected functionaries will be
agricultural land disqualified to hold their offices if they have more
3. Taxes on income other than corporation tax and tax than two children.
on agricultural income 4. The elections will be held in six months time if
4. Receipts from Broadcasting levied, collected and Panchayati Raj bodies are superceded or dissolved
appropriated by the Union Non-tax revenue of the by the State government.
Union Codes:
Which of the following statements is/are correct? (a) 2 and 4 (b) 1 and 4
(a) 1 and 3 (b) 1, 2 and 3 (c) 3 and 4 (d) 1, 2 and 4 (c) 1, 3 and 4 (d) 1, 2 and 4

Downloaded From : www.EasyEngineering.net


Downloaded From : www.EasyEngineering.net
Local Government P-133

9. Which of the following features of Panchayati Raj, Select the correct answer using the code given below.
envisaged under the 73rd Amendment of the Constitution,
(a) 1 only (b) 1 and 2 only
are correctly listed?
(c) 1, 2 and 3 (d) 2 and 3 only
1. the elections to the Panchayats will be held by the
Union Election Commission. 14. The 73rd Amendment of the Constitution provided
constitutional status to the Panchayati Raj Institutions.
2. there is mandatory reservation of seats for weaker
sections and women. Which of the following are the main features of this
3. the Panchayats shall have a fixed term of five years provision?
and, if dissolved before the expiry of this term, the 1. A three-tier system of Panchayati Raj for all states.
elections must be held within six months. 2. Panchayat election in every 5 years.
Codes: 3. Not less than 33% of seats are reserved for women.
(a) 1, 2 and 3 (b) 2 and 3 4. Constitution of district planning committies to
(c) 2 only (d) 1 and 2 prepare development plans.
10. Consider the following statements: Select the correct answer using the codes given below
1. Part IX of the Constitution of India contains (a) 1, 2 and 3 (b) 1, 3 and 4 (c) 1, 2 and 4 (d) 2, 3 and 4

ww
provisions for Panchayats and was inserted by the 15. Which among the following statements regarding Lord
Constitution (73rd Amendment) Act 1992. Ripon’s plan for local self-government in India is/are
2. Part IX A of the Constitution of India contains correct?

w.E
provisions for municipalities and the Article 243 Q 1. The district should be the maximum area served by
envisages two types of municipalities—a Municipal one Committee or Local Board.
Council and a Municipal Corporation for every
2. The Local Boards should consist of a large majority
State.
of nominated official members and be presided over



(a) 1 only
(c) Both 1 and 2
a
Which of the statements given above is/are correct?
(b) 2 only
syE
(d) Neither 1 nor 2


by an official member as Chairman.
Select the correct answer using the codes given below
(a) Only 1 (b) Only 2
11. Which of following are the features of 74th Amendment


Act on muncipalties?
1. Reservation of seats for SCs and Sts in Proportion ofngi (c) Both 1 and 2 (d) Neither 1 nor 2
16. How does participatory budgeting seek to make the
functioning of local governance institutions more



their population.
2. Mandatory periodic elections every 5 years.
3. The procedure for maintenance of accounts and
nee
transparent and accountable?
1. By allowing citizens to deliberate and negotiate over
the distribution of public resources.


audit would be decided by the state Governor.
4. Constitution of Nagar-Panchayats for smaller Urban
area:

r ing
2. By allowing citizens to play a direct role in deciding
how and where resources should be spent.
3. By allowing historically excluded citizens with


(a) 1, 2, and 3
(c) 3 and 4
(b) 2 and 3
(d) 1 and 2 .ne
access to important decision-making venues.
Select the correct answer using the codes given below:
12. Which one of the following is not a feature of part IX of


the Constitution of India?
(a) Five year tenure for panchayats
(b) Reservation of seats for Schedule Castes and


(a) 1 and 2
(c) Only 3
(b) 2 and 3
t
(d) 1, 2 and 3
17. Consider the following statements about local government
in India:
Schedule Tribes for Panchayat membership 1. Article 40 of Indian Constitution provides for the
(c) Indirect election for all panchayat seats (village/ State to organize village panchayats and endow them
intermediate level) with such powers and authority as may be necessary
(d) Reservation for not less than one-third of the seats to make them function as units of self-government.
for women 2. The 73rd and 74th Constitution Amendments inserted
13. Point out the difference between the local government in Part IX and IX A in the Constitution.
India before and after the Constitutional Amendments in 3. The provisions in Farts IX and IX A of Indian
1992: Constitution are more or less parallel and analogous.
1. It has become mandatory to hold regular elections to 4. The 73rd Constitution Amendment is applicable to
the local government bodies. all states irrespective of size of population.
2. 1/3rd positions are reserved forwomen. Which of the statements given above are correct?
3. Elected officials exercise supreme power in the (a) 1 and 2 only (b) 1, 2 and 3 only
government. (c) 3 and 4 only (d) 1, 2, 3 and 4

Downloaded From : www.EasyEngineering.net


Downloaded From : www.EasyEngineering.net
P-134 Local Government

18. Consider the following statements: 20. In the areas covered under the Panchayat (Extension to
1. Part IX of the Constitution of India provisions for the Scheduled Areas) Act, 1996, what is the role/power
Panchyats and was inserted by the Constitution of Gram Sabha?
(Amendment) Act, 1992. 1. Gram Sabha has the power to prevent alienation of
land in the Scheduled Areas.
2. Part IX A of the Constitution of India contains
provisions for Municipalities and the Article 243 Q 2. Gram Sabha has the ownership of minor forest
envisages two types of Municipalities a Municipal produce.
Council and a Municipal Corporation for every State. 3. Recommendation of Gram Sabha is required for
granting prospecting licence or mining lease for any
Which of the statements given above is/are correct? mineral in the Scheduled Areas.
(a) Only 1 (b) Only 2 Which of the statements given above is/are correct?
(c) Both 1 and 2 (d) Neither 1 nor 2 (a) 1 only (b) 1 and 2 only
19. The Constitution (Seventy-Third Amendment) Act, 1992, (c) 2 and 3 only (d) 1, 2 and 3
which aims at promoting the Panchayati Raj Institutions 21. The fundamental object of Panchayati Raj system is to
in the country, provides for which of the following ? ensure which among the following ?
1. Constitution of District Planning Committees. (1) People’s participation in development

ww
2. State Election Commissions to conduct all panchayat
elections.
3. Establishment of State Finance Commission.



(2) Political accountability
(3) Democratic decentralization
(4) Financial mobilization



(a) 1 only
w.E
Select the correct answer using the codes given below:

(c) 2 and 3 only


(b) 1 and 2 only
(d) 1, 2 and 3



Select the correct answer using the code given below.
(a) 1, 2 and 3 only
(c) 1 and 3 only
(b) 2 and 4 only
(d) 1, 2, 3 and 4

a syE
ngi
nee
r ing
.ne
t

Downloaded From : www.EasyEngineering.net


Downloaded From : www.EasyEngineering.net
Local Government P-135

Hints and Solutions


EXERCISE-1 53. (c) The Balwant Rai Mehta Committee was appointed
1. (c) 2. (d) 3. (d) 4. (c) 5. (b) by the Government of India in January 1957
to examine the working of the Community
6. (c)
7. (d) 8. (c) 9. (d) 10. (a)
Development Programme(1952).It recommended a
11. (b) The Committee appointed in 1977 to review 3-tier Panchayati Raj system-Gram Panchayat at the
working of the Panchayati Raj was chaired by village level, Panchayat Samiti at the block level,
Ashok Mehta. The committee submitted its report and Zila Parishad at the district level.
in August 1978 and made 132 recommendations to
54. (b) Creation of a three-tier system was not the
revive and strengthen the declining Panchayati Raj
recommendation of Ashok Mehta Committee.
system in the country. As a result of this report,
According to this committee,the 3-tier system of
the Indian states of Karnataka, Andhra Pradesh, and
Panchayati Raj should be replaced by the 2-tier
West Bengal passed new legislations. However, the

ww flow of politics at the state level did not allow the


institutions to develop their own political dynamics.
system. Ashoka Mehta committee was appointed
by the Janata Government under the chairmanship
of Ashoka Mehta.
12. (c) 13. (a)

w.E
14. (c) Under 74th Amendment of the constitution the local
body for a transitional area is known as Nagar
55. (a) According to Article 243(E), every Panchayat,
unless sooner dissolved under any law for the time
being in force, shall continue for five years from the

a
Panchayat. This article provides that there be a

syE
Nagar Panchayat for transitional areas i.e. an area
in transition from rural to urban, a municipality for
a smaller urban area and a municipal corporation for
date appointed for its first meeting and no longer.
56. (b) Panchayats Extension to Scheduled Areas Act(PESA)
is a law enacted by the Government of India to cover
a larger urban area.
15. (c) ‘Swaranajayanti Gram Swarojgar Yojna’ came ngi the “Scheduled areas” which are not covered in the
73rd amendment or Panchayati Raj Act of the Indian
Constitution. It was enacted on 24 December 1996
into being in April, 1999. Swarnajayanti Gram
Swarojgar Yojana (SGSY) is an initiative launched
by the Government of India to provide sustainable
nee
to enable Gram Sabhas to self-govern their natural
resources.

income to poor people living in rural areas of the


country. The scheme was launched on April 1,
1999.
r
57. (c) Statement given under option (c) is not mentioned

ing
in 73rd amendment act. But this norm is applied
in Haryana, Rajasthan, MP, Odisha and Andhra
The SGSY aims at providing self-employment to
villagers through the establishment of Self-help
Pradesh.

.ne
58. (c) 73rd Amendment Act added Eleventh Schedule to
groups.
16. (a) 17. (c) 18. (a) 19. (c) 20. (b)
21. (b) 22. (d) 23. (d) 24. (a) 25. (d)
Panchayati Raj System.
t
the Constitution and Part IX, which provides for the

59. (b) According to 73rd Amendment Act 1993, under


Article 243D, not less than 1/3rd i.e. 33% seats
26. (d) 27. (d) 28. (d) 29. (a) 30. (b)
should be reserved for women in local bodies.
31. (c) 32. (d) 33. (b) 34. (d) 35. (c)
60. (c) According to 73rd Amendment Act, three-tier
36. (d) 37. (b) 38. (d) 39. (b) 40. (b)
system of Panchayats exists: Village level, District
41. (a) 42. (b) 43. (c) 44. (d) 45. (c) Panchayat at the district level, the intermediate
46. (c) 47. (a) 48. (b) 49. (a) 50. (c) Panchayat which stands between the village
51. (c) Part IX of the constitution envisages a 3-tier and District Panchayats in the States where the
system of panchayats, namely (a) the village level population is above 20 Lakhs.
(b) the district panchayat (district-level) (c) The 61. (a) Panchayati Raj System was first introduced in
intermediate panchayat (block-level) Nagaur district of Rajasthan on October 2, 1959
52. (a) The act provides for the reservation of not less than followed by Andhra Pradesh in 1959.
one third(33%) of the total of seats for women at all 62. (d) Provision given in option (d) is not included under
levels. 73rd Amendment Act 1993.

Downloaded From : www.EasyEngineering.net


Downloaded From : www.EasyEngineering.net
P-136 Local Government

63. (c) Duration of Panchayats is five year. Fresh election 15. (d) Lord Ripon’s plan for local Self government in India
to constitute a Panchayat shall be completed before is as follows:
the expiry of its term; or in case of dissolution 1. The sub-division, not the district, should be the
before the expiry of a period of 6 months from the maximum area served by one committee or local
date of its dissolution.
board with primary boards under it serving very
64. (b) In 1688, the first Municipal Corporation of India small areas, so that each member of it might
was set up in Madras.
possess knowledge of and interest in its affairs.
65. (d) Section C of the Forest Dweller Act provides a
2. The local boards should consist of a large
transparent three step procedure for deciding on
who gets rights. majority of elected non-official members, and
they should be presided over by a non-official
Firstly, the Gram Sabha makes a recommendation- i.e,
member.
who has been cultivating land for how long, which minor
forest produce is collected; etc. The Gram Sabha plays 16. (d) Participatory Budgeting is a democratic process of
this role because it is a public body where all people deliberation by citizens, civic officials and elected
participate and hence is fully democratic and transparent. representatives on the issues that need attention and

ww
The Gram Sabha’s recommendation goes through two
stages of screening committees- the Taluka and the
District levels.
collectively arriving at decisions that would directly
be included in the budget of the government.
Participatory budgeting empowers the citizens to

w.E
66. (d) PESA Act does not identify the freedom of tribal
people from exploitation as its objectives, but it
automatically becomes a byproduct of its objectives.
present their demands and priorities for improvement
and influence through discussions and negotiations
the budget allocations made by their municipalities.

1. (b)
6.
a
EXERCISE-2
2. (b) 3. (c) 4. (d) 5. (a) syE
(a) The Chairperson of Panchayats at inter-mediate and
It is an opportunity in which the common citizens
can decide about the allocation and distribution of
public expenditure in their areas or regions.
17. (b) 73rd constitution amendment is applicable to all

7.
district levels are elected indirectly.
(b) Under Article 243, it has been mentioned that not ngi states except Jammu & Kashmir.
18. (a) Part IX and Eleventh Schedule were added by
less than 1/3rd (33%) seats in the local bodies shall
be reserved for women.
nee
73rd Constitutional Amendment Act, 1992 which
contain provisions for Panchayats, Part IX A.
8. (d) 9. (a) 10. (a) 11. (d)
13. (b) Not less than one-third (including the number
of seats reserved for women belonging to the
12. (c)

r
Twelfth Schedule were added by 74th Constitutional

ing
amendment act, 1992 which contain provisions for
Municipalities but Article 243 Q envisages three
Scheduled Castes and the Scheduled Tribes) of the
total number of seats to be filled by direct election in
every Municipality shall be reserved for women .ne
types of municipalities: Nagar Panchayats for a
transitional area, Municipal Council for smaller
urban areas and Municipal Corporation for larger
and such seats may be allotted by rotation to
different constituencies in a Municipality.
14. (a) The salient features of the 73rd Amendment Act,
urban areas.
t
19. (c) District planning committee comes under 74th
Amendment not in 73rd Amendment.
1992 are: -
20. (d) According to Panchayat Extension to the Scheduled
• To provide 3-tier system of Panchayati Raj for all Areas) Act 1996, Gram Sabha has the power to
States having population of over 20 lakhs. prevent alienation of land in the Scheduled Areas,
• To hold Panchayat elections regularly every 5 has the ownership of minor forest produce and the
years. recommendation of Gram Sabha is required for
• To provide reservation of seats for Scheduled granting prospecting licence or mining lease for any
Castes, Scheduled Tribes and women (not less mineral in the Scheduled Areas.
than 33%) 21. (a) The fundamental object to Panchayati Raj system
• To appoint State Finance Commission to make is to ensure people’s participation in development,
recommendations as regards the financial powers political accountability and democratic
of the Panchayats. decentralization.

Downloaded From : www.EasyEngineering.net


Downloaded From : www.EasyEngineering.net

6
JUDICIARY
Chapter

ww
Introduction
w.E
The Indian Judicial System is one of the Oldest Legal systems is the world today. Inspite of India adopting the features of
a federal system of government, the Constitution has provided for the setting up of a single integrated system of courts to
administer both Union and State Laws.

a syE
ngi
nee
r ing
.ne
t

Downloaded From : www.EasyEngineering.net


Downloaded From : www.EasyEngineering.net
P-138 Judiciary

The above simple Flowchart illustrates sub-ordinate courts- Appointment and Removal of Judges
both Civil and Criminal courts - at (i) Metropolitan and (ii)
Qualifications to be appointed as a judge of Supreme Court:
District levels. Besides, it illustrates the hierarchy in terms
of (i) Labor Courts and (ii) Family Courts together with •• He must be a citizen of India.
specialized tribunals. •• He must either be a distinguished jurist, or one who has
However, if required, the hierarchy can be illustrated in a different been a High Court judge for at least 5 years or an advocate
manner for those who want to have the structure by (i) Civil of a High Court (or 2 or more such courts in succession)
Courts at (a) Metropolitan and (b) District levels and (ii) Criminal for at least 10 years (Article 124).
Courts at (a) Metropolitan and (b) District levels. i.e. •• No minimum age is fixed for the appointment of a judge.
Supreme Court •• The Chief Justice of India is appointed by the
President. In this matter, the President shall consult such
High Court judges of the Supreme Court and the high courts as he
may deem necessary. A 9 judge bench of the SC has laid
Civil Criminal down that the senior most judge of Supreme Court should
Courts Courts be appointed as Chief Justice of India.
•• In the appointment of other judges, the President shall always

ww
Metropolitan
area
District
level
Metropolitan
area
District
level
consult the Chief Justice of India. He ‘may’ consult other
judges of SC and high courts as he may deem necessary
[Article 124(2)].
Union Judiciary
w.E
Supreme Court (Articles 124-147)
•• Power of appointment is exercised by the President on the
advice of Council of Ministers.
•• There is no fixed period of office for Supreme Court
••

••
a
The Supreme court of India was inaugurated on

syE
January 28, 1950. It succeeded the Federal Court of India,
established under the Government of India Act of 1935.
Article 124 to 147 in part V of the Constitution deal
judges. Once appointed, they hold office till the age of
65 years. He can quit office earlier by submitting his
resignation to the President.
•• He can be removed by an order of President only on the
with the organisation, independence, jurisdiction, powers,
procedures and so on of the Supreme Court. The Parliament
is also authorised to regulate them. ngi grounds of proved misbehavior or incapacity. The order
of President in this regard can only be passed after it has
been addressed to both houses of parliament in the same
•• Supreme Court is the final interpreter and guardian of our
Constitution. It is also the guardian of Fundamental rights
of the people. nee
session, by special majority (majority of the membership
of house and majority of not less than 2/3rd of members of
that house present and voting). [Article 124 (4)].
•• It decides the disputes between Centre and States regarding
encroachment of power, thus maintains the supremacy of
the Constitution.
r ing
•• A Supreme Court judge can become Chief Justice of India
but cannot practice before any other court or act as a Judge
before any other authority. But there is one exception.
••
••
It is the highest court of appeal in India.
Originally the total number of judges was 7, but in 1977,
their number was increased to 18. In 1986, it was further .ne
This is regarding the retired SC judge appointed as a
judge of the Supreme Court for a temporary period by
the Chief Justice of India with the previous consent of

••
raised to 26 (including CJI). Presently there are 31
judges in Supreme court.
Constitution does not provide for minimum no. of
judges who will constitute a bench for hearing cases.
President [Article 128].
t
•• Salaries of Judges are determined by the Parliament by
law. These cannot be varied to their disadvantage during
their term (except during financial emergency). Their
Largest bench constituted so far has been of 13 judges salaries and expenses are charged on the Consolidated
in Keshavanand Bharati vs. Union of India case in 1973. Fund of India. Salary of Chief Justice - `1,00,000/month
INTEGRATED JUDICIARY Salary of Judges - ` 90,000/month
•• Seat of the Supreme Court is in New Delhi. However it
President can be shifted elsewhere in India or more benches of SC
can be established in India by Chief Justice of India in
Supreme Court consultation with the President.
Governor 1. Chief Justice Parliament •• According to Article 129, Supreme Court is a “Court of
2. Judges Record”. It means:-
Court records are admitted to be of evidentiary value.
High Courts It can punish for contempt of the court-
1. Chief Justice •• Contempt is of 2 type: Criminal and Civil.
2. Judges •• Judges can be liable for the contempt of their own court.

Downloaded From : www.EasyEngineering.net


Downloaded From : www.EasyEngineering.net
Judiciary P-139

What is the Collegium system? Jurisdiction of the Supreme Court


The Collegium system is one where the Chief Justice of •• It extends to the cases originating in Supreme Court alone.
India and a forum of four senior-most judges of the Supreme No other court has power to try such cases.
Court recommend appointments and transfers of judges. •• Therefore Supreme Court is a federal court. These are
However, it has no place in the Indian Constitution. The between:
system was evolved through Supreme Court judgments in (i) GOI on one side and one or more states on the other.
the Three Judges Cases (October 28, 1998) (ii) GOI and one or more States on one side and other states
Why is Collegium system being criticised? on the other
The Central government has criticised it saying it has created •• However such jurisdiction does not apply to the disputes
an imperium in imperio (empire within an empire) within arising out of a treaty or agreement which is in operation
the Supreme Court. The Supreme Court Bar Association has or wherein provided for such exclusion. These matters
blamed it for creating a “give-and-take” culture, creating are:-
a rift between the haves and have-nots.“While politicians (i) Exclusion of Jurisdiction of Supreme Court by Parliament
and actors get instant relief from courts, the common man in case of use, distribution or control of water of any
struggles for years for justice.” inter-state river valley (Article 262).
How and when was the NJAC established? (ii) Financial matters between Centre and states (Article 280).

ww
The National Judicial Appointment Commission (NJAC)
was established by amending the Constitution [Constitution
(Ninety-Ninth Amendment) Act, 2014] passed by the Lok
(iii) Adjustment of expenses between Centre and states (Article
290).
Appointment of acting Chief Justice

w.E
Sabha on August 13, 2014 and by the Rajya Sabha on
August 14 2014. Alongside, the Parliament also passed the
National Judicial Appointments Commission Act, 2014, to
When the office of Chief Justice of India is vacant or when the
Chief Justice is by reason of absence or otherwise unable to
perform the duties of his office, the duties of the office shall be

a
regulate the NJAC’s functions. Both Bills were ratified by 16

syE
of the State legislatures and the President gave his assent on
December 31, 2014. The NJAC Act and the Constitutional
Amendment Act came into force from April 13, 2015.
performed by such one of the other judges of the court as the
President may appoint for the purpose.

Attendance of retired Judges at sittings of the


Who will be in the NJAC?
It will consist of six people- the Chief Justice of India, the two ngi
Supreme Court
Notwithstanding anything in this chapter the Chief Justice

nee
most senior judges of the Supreme Court, the Law Minister, of India may at any time, with the previous consent of the
and two ‘eminent persons’. These eminent persons are to be President, request any person who has held the office of a
nominated for a three-year term by a committee consisting judge of the Supreme Court or of the Federal Court or who was
of the Chief Justice, the Prime Minister, and the Leader of
the Opposition in the Lok Sabha, and are not eligible for
re-nomination.
If politicians are involved, what about judicial independence?
r
held the office of a judge of a High Court and is duly qualified

ing
for appointment as a Judge of the Supreme Court to sit and act
as a Judge of the Supreme Court to fit and act as a Judge of
the Supreme Court and every such person so requested shall,
The judiciary representatives in the NJAC -- the Chief Justice
and two senior-most judges – can veto any name proposed .ne
while so sitting and acting be entitled to such allowances as the
President may by order determine and have all the jurisdiction ,
for appointment to a judicial post if they do not approve of it.
Once a proposal is vetoed, it cannot be revived. At the same
time, the judges require the
support of other members of the Commission to get a name
be a judge of that court.
t
power and privileges of, but shall not otherwise be deemed to

Provided that nothing in this article shall be deemed to require


any such person as aforesaid to sit and act as a Judge of that
Court unless he consents so to do.
through.
Appellate Jurisdiction
Adhoc and Acting Judges
•• Appeal lies with the Supreme Court against the High
•• Article 127 says that if there is no Quorum of the Supreme Courts in the following 4 categories of cases:
Court Judges to hold or continue any session of the court,
(a) Constitutional matters (civil, criminal or others)
the Chief Justice of Indian (CJI), with the previous consent
– Article 132
of the President and in consultation with the Chief Justice
(b) Civil matters (except Constitutional) – Article 133
of the High Court concerned, can request in writing a
(c) Criminal matter (except Constitutional) – Article 134
Judge of the High Court, who is qualified to be a Judge of
the Supreme Court, to function as an adhoc Judge of the (d) Special leave to appeal – Article 136
Supreme Court. •• Special leave to appeal is issued by Supreme Court in its
discretion. It cannot be issued in case of judgment passed
•• While so attending as the Judge of the Supreme Court,
by a court or tribunal of armed forces.
he shall have all the Jurisdiction, powers and privileges
and shall discharge the duties of a Judge of the Supreme •• It can be granted in any judgement whether final or
Court. interlocutory.

Downloaded From : www.EasyEngineering.net


Downloaded From : www.EasyEngineering.net
P-140 Judiciary

•• It may be related to any matter —constitutional, civil, The constitutional validity of a legislative enactment or an
criminal, income-tax, labour, revenue, advocates, etc. executive order can be challenged in the Supreme Court on the
•• High Court can certify a case involving substantial following three grounds:
question of law as to the interpretation of the Constitution (a) it infringes the Fundamental Rights (Part III),
and thus refer it to Supreme Court. (b) it is outside the competence of the authority which has
framed it, and
Advisory Jurisdiction (c) it is repugnant to the constitutional provisions.
•• Article 143 of the Constitution provides that if it appears From the above, it is clear that the scope of judicial
to the President that: review in India is narrower than that of what exists in USA,
1. A question of law or fact has arisen or is likely to though the American Constitution does not explicitly mention
arise. the concept of judicial review in any of its provisions. This is
because, the American Constitution provides for ‘due process
2. A question is of a fact of public importance.
of law’ against that of ‘procedure established by law’ which is
He may refer such question for the advisory opinion of the contained in the Indian Constitution. The difference between
Court and the Court may after such hearing as it thinks fit, the two is: ‘The due process of law gives wide scope to the
report to the President its opinion thereon. Supreme Court to grant protection to the rights of its citizens.
•• Supreme Court is not bound to give advisory opinion on It can declare laws violative of these rights void not only on
the matters of political significance and may refuse to do substantive grounds of being unlawful, but also on procedural
so.
ww
•• The Court, however, is bound to give its advisory
grounds of being unreasonable.
Our Supreme Court, while determining the constitutionality
of a law, however examines only the substantive question

w.E
jurisdiction on the matters relating to disputes arising
out of a treaty or agreement entered into before the i.e., whether the law is within the powers of the authority
commencement of the Constitution. concerned or not. It is not expected go into the question of its
•• The advice is not binding on the President and he may reasonableness, suitability or policy implications.’
accept or reject it.
a syE
•• Law declared by the Supreme Court is binding on all the
courts in India (Article 141). But Supreme Court itself is
not bound by its own decisions. Article 137 empowers
The exercise of wide power of judicial review by the
American Supreme Court in the name of due process of
law’ clause has made the critics to describe it as a ‘third
chamber’ of the Legislature, a super-legislature, the arbiter
Supreme Court to review its own judgment.
•• Under Article l39A (inserted by 44th amendment Act
ngi of social policy and so on. This American principle of judicial
supremacy is also recognised in our constitutional system, but
to a limited extent. Nor do we fully follow the British Principle

nee
1978) Supreme Court may transfer to itself cases from of parliamentary supremacy. There are many limitations on
one or more high courts if these involve substantial the sovereignty of Parliament in our country, like the written
question of law or that great significance. Supreme Court character of the Constitution, the federalism with division

r
may transfer cases from one High Court to another in the of powers the Fundamental Rights and the judicial review.
interest of justice.

Power of Judicial Review ing


In effect, what exists in India is a synthesis of both that is,
the American principle of judicial supremacy and the British
principle of parliamentary supremacy.
Judicial review is the power of the Supreme Court to examine the
constitutionality of legislative enactments and executive orders
of both the Central and state governments. On examination, if
Other Powers
•• .ne
Besides the above, the Supreme Court has numerous other
they are found to be violative of the Constitution (ultra-vires),
they can be declared as illegal, unconstitutional and invalid
(null and void) by the Supreme Court. Consequently, they
cannot be enforced by the Government.
••
powers:
t
It decides the disputes regarding the election of the
President and the Vice-President. In this regard, it has the
original, exclusive and final authority.
Judicial review is needed for the following reasons: •• It enquires into the conduct and behaviour of the chairman
(a) To uphold the principle of the supremacy of the and members of the Union Public Service Commission on
Constitution. a reference made by the President. If it finds them guilty
of misbehaviour, it can recommend to the President for
(b) To maintain federal equilibrium (balance between Centre
their removal. The advice tendered by the Supreme Court
and states).
in this regard is binding on the President.
(c) To protect the fundamental rights of the citizens.
•• It is authorised to withdraw the cases pending before
The Supreme Court used the power of judicial review in various the High Courts and dispose them by itself. It can also
cases, as for example, the Go laknath case (1967), the Bank transfer a case or appeal pending before one High Court
Nationalisation case (1970), the Privy Purses Abolition case to another High Court.
(1971), the Kesavananda Bharati case (1973), the Minerva •• The Supreme Court’s jurisdiction and powers with
Mills case (1980) and so on. respect to matters in the Union List can be enlarged by
Though the phrase ‘Judicial Review’ has nowhere been used in the Parliament. Further, its jurisdiction and powers with
the Constitution, the provisions of several articles’ explicitly respect to other matters can be enlarged by a special
confer the power of judicial review on the Supreme Court. agreement of the Centre and the states.

Downloaded From : www.EasyEngineering.net


Downloaded From : www.EasyEngineering.net
Judiciary P-141

Chief Justice of India (Since, 1951) 33. VN Khare 19th Dec, 2002 2nd May, 2004
S. Name Took Office Left Office 34. S Rajendra Babu 2nd May, 2004 1st June, 2004
No. 35. RC Lahoti 1st June, 2004 1st Nov, 2005
1. HJ Kania 15th August, 1947 16th Nov, 1951 36. YK Sabharwal 1st Nov. 2005 14th Jan, 2007
2. MP Sastri 16th Nov, 1951 3rd Jan, 1954 37. KG 14th Jan, 2007 11th May, 2010
3. Mehr Chand Balakrishnan
Mahajan 3rd Jan, 1954 22nd Dec, 1954 38. SH Kapadia 12th May, 2010 28th Sep, 2012
4. BK Mukherjee 22nd Dec, 1954 31st Jan, 1956 39. Altamas Kabir 29th Sep, 2012 18th July, 2013
5. Sudhi Ranjan 31st Jan, 1956 30th Sep, 1959 40. P Sathasivam 19th July, 2013 26th April, 2014
Das 41. Rajendra Mai 27th April, 2014 27th Sep, 2014
6. Bhuvaneshwar 30th Sep, 1959 31st Jan, 1964 Lodha
Prasad Sinha 42. HL Dattu 28th Sep, 2014 02nd Dec, 2015
7. PB 31st Jan, 1964 15th Mar, 1966
43. T.S.Thakur 03rd Dec. 2015 Till date
Gajendragadkar

ww
8. AK Sarkar
9. K Subba Rao
10. Kailash Nath
16th Mar,1966
30th June, 1966
12th April, 1967
29th June, 1966
11th April, 1967
24th Feb, 1968
Public Interest Litigation (PIL)
In principle, all citizens of India can access the courts in the
country. But in reality access to courts has always been difficult
Wanchoo
11. M Hidayatullah
12. Jayantilal
w.E
25th Feb, 1968
17th Dec, 1970
16th Dec, 1970
21st Jan, 1971
for a vast majority of the Poor in India, as legal procedures
involve a lost of money a paperwork as well as take up a lot of
time. In response to this, the Supreme Court in the early 1980s
Chhotalal
Shah
13. SM Sikri
14. AN Ray
a
22nd Jan, 1971
25th April, 1973
syE
25th April, 1973
28th Jan, 1977
devised a mechanism of PIL.
Through PIL, the judiciary has also shown readiness to take
into consideration rights of those sections who cannot easily
approach the courts. For this purpose, the judiciary allowed

15. Mirza
Hameedullah
29th Jan, 1977 21st Feb, 1978
ngi
public spirited citizens, social organisations and lawyers to
file petitions on behalf of the needy and the deprived. Justice
Krishna Iyer and Justice Bhagwati were champions of the
Beg
16. YV
Chandrachud
22nd Feb, 1978 11th July, 1985
nee
concept of PIL in India.

Judicial Activism
17. PN Bhagwati
18. RS Pathak
19. ES
12th July, 1985
21st Dec, 1986
19th June, 1989
20th Dec, 1986
6th June, 1989
17th Dec, 1989
r
It means the assertive role played by the judiciary to force

ing
other organs of the state namely the Executive and Legislature
to discharge their duties properly, as assigned to them by

Venkataramiah
20. S Mukherjee 18th Dec, 1989 25th Sep, 1990 .ne
the Constitution. The Judiciary has played an activist role in
the recent 2G Scam Case, CVC Case, Jharkhand Legislative
Assembly Case, Cancellation of Coal Blocks Case.
21. Ranganath
Mishra
22. Kamal Narain
Singh
25th Sep, 1990

25th Nov, 1991


24th Nov, 1991

12th Dec, 1991


Landmark Judgements of the Supreme
Court
t
23. MH Kania 13rd Dec, 1991 17th Nov, 1992 AK Gopalan Case, 1950
24. Lalit Mohan 18th Nov, 1992 11th Feb, 1993 The case corresponds to the charges of violation of Fundamental
Sharma Rights to freedom under the Preventive Detention Act. The
25. MN 12th Feb, 1993 24th Oct, 1994 court was approached over the validity of the act.
Venkatachaliah •• The Supreme Court held that the constitutional validity of
a law cannot be verified by the judiciary and the judiciary
26. AM Ahmadi 25th Oct, 1994 24th Mar, 1997
has only the capacity to verify whether the procedure
27. JS Verma 25th Mar, 1997 18th Jan, 1998 according to the law has been followed.
28. MM Punchhi 18th Jan, 1998 9th Oct, 1998
29. AS Anand 10th Oct, 1998 1st Nov, 2001
Champakam Dorairajan Case, 1951
•• The case challenged the reservations given to backward
30. SP Bharucha 2nd Nov, 2001 6th May, 2002
classes in educational institutions in Tamil Nadu.
31. BN Kirpal 6th May, 2002 11th Nov, 2002
•• First Amendment Act was inserted as Article 15(4) in the
32. GB Pattanaik 11th Nov, 2002 19th Dec, 2002 Constitution.

Downloaded From : www.EasyEngineering.net


Downloaded From : www.EasyEngineering.net
P-142 Judiciary

Shankari Prasad Case, 1952 Black Money, 2012


The First Constitutional Amendment Act, providing for The government refused to disclose details of about 18 Indians
reservations, was challenged that it violated Fundamental holding accounts in LGT Bank, Liechtenstein evoking a sharp
Rights. The court held that Parliament has the power to amend response from a bench comprising Justice B Sudershan Reddy
the Constitution. and SS Nijjar. The court ordered the SIT to probe the matter.

Berubari Case, I960 Right to Reject, 2013


While ceding a part of Indian Territory to an alien state, the The Supreme Court directed the Election Commission to
court in an advisory opinion held that such process cannot take introduce a ‘None of the Above’ (NOTA) button on electronic
place unless a Constitutional Amendment to that effect is made. voting machines and ballot papers which can be used by the
voters to reject all the candidates contesting elections in a
Golaknath Case, 1967 constituency.
The Supreme Court held that Constitutional Amendment
Re-Opening Dance Bars, 2013
cannot be extended to infringement of Fundamental Rights.
Eight years after the Maharashtra Government banned dance
Keshvananda Bharati Case, 1973 bars in Mumbai, the Supreme Court on 16th July gave its go-
The Supreme Court propounded the Basic Structure Doctrine ahead to their re-opening in the city and elsewhere in the state.

ww
and held that certain basic features of the Constitution cannot
be amended while others can be done, so without having a
sweeping change in the Constitution.
Supreme Court Recognises Transgenders as ‘Third Gender,
2014
In a Landmark judgement the Supreme Court recognised the

Minerva Mills Case, 1980


w.E
The Supreme Court held that Fundamental Rights and Directive
Principles of State Policy are complementary to each other and
transgender persons as third gender.

State Judiciary High Court


•• Article 214 provides that there shall be a High Court for

a
curb the Fundamental Rights.

Shah Bano Case, 1985


syE
any law enacted to implement the Directive Principle could not each state. However under Article 231 (1) Parliament can
establish by law, a common High Court for two or more
States or for two or more States and a UT. There are 24
High Courts in India. Out of them three are common High
The Supreme Court held that Muslim women also have right to
get maintenance from their husbands when they are divorced.
ngi ••
Courts.
Calcutta High Court Madras High Court Bombay High
Court and Allahabad High Court are the oldest four High
Sto Stephen’s College Case, 1992
The Supreme Court held that atleast 50% of seats in minority
institutions should be reserved for non- minority students. ••
nee
Courts in India Among the four, the Calcutta High Court
is the oldest, established on 2nd July 1862.
Parliament may by law constitute a High Court for UT
Indira Sawhney Case, 1993
In this case, the Supreme Court declared that reservation ••
r
(Article 241).
ing
or declare any court in any such UT to be a High Court

Guwahati High Court is the largest High Court in India;


cannot exceed 50% and introduced the ‘creamy layer’.

SR Bommai Case, 1994


••
North East.
.ne
its territorial jurisdiction extends to seven states of the

Kolkata High Court has territorial jurisdiction covers


The Supreme Court held that Federalism is a part of Basic
Structure and State Governments cannot be arbitrarily
dismissed by a Governor. The case laid down the guidelines in
proving a majority under Article 356.
••
Andaman and Nicobar.
t
Delhi has a separate high court but the other UTs come
under the jurisdiction of various High Courts.

Appointment of Judges of High Court


Chandra Kumar Case, 1997
•• Article 217 provides that every judge of a high court
The Supreme Court upheld that judicial review is a part of shall be appointed by the President.
basic structure of the Constitution. •• President appoints Chief Justice of High Court after
consultation with Chief Justice of India and the Governor
TMA Pai Case, 2002
of the state concerned. In case of appointment of others
The Supreme Court held that the right to administer minority judges of the High Court he may consult the Chief Justice
educational institution is not absolute and the state can regulate of High Court concerned.
the institutional affairs in the interest of educational standards. •• The strength of the judges of the High Courts is not the same.
•• In Re-Presidential Reference Case (popularly known as
2G Spectrum Scam, 2008
Appointment and Transfer of Judges Case), Supreme
The Supreme Court declared allotment of spectrum as Court held that the Chief Justice of India should consult
‘unconstitutional and arbitrary’ and quashed all the 122 “a collegium of two senior most judges of the Supreme
licenses issued in 2008 during tenure of A Raja, the main Court” for the appointment of a judge of Supreme Court
official accused in the 2G scam case. or High Court.

Downloaded From : www.EasyEngineering.net


Downloaded From : www.EasyEngineering.net
Judiciary P-143

•• Further in case of transfer of High Court judges, in (f) The four high courts (i.e., Calcutta, Bombay, Madras
addition to the collegium of 4 judges of Supreme Court, and Delhi High Courts) have original civil jurisdiction in
the Chief Justice of India is required to consult Chief cases of higher value.
Justice of both the High Courts (one from where the judge
is being transferred and the other, receiving him).
Writ Jurisdiction
•• Article 222 empowers the President after consultation Article 226 of the Constitution empowers High Court to
with Chief Justice of India to transfer a judge from one issue writs including habeas corpus. mandamus, certiorari,
High Court to another High Court. prohibition and quo-warranto for enforcement of the
fundamental rights of the citizens and for any other purpose.
Qualifications of a Judge of High Court The phrase ‘for other purpose’ refers to the enforcement of
1. Citizen of India, an ordinary legal right. The High Court can issue writs to any
2. Have held a judicial office for at least 10 years or person, authority and government not only within its territorial
3. Have been an advocate of one High Court or two or more jurisdiction but also outside its territorial jurisdiction if the
High Courts in succession for at least 10 years. cause of action arises within its territorial jurisdiction.
The writ jurisdiction of the High Court (under Article 226)
Term of Adhoc Judge is not exclusive but concurrent with the writ jurisdiction of
••
••
••
ww
Until he attains the age of 62 years.
He may resign by writing to the President.
He may removed by the President on the grounds of
the Supreme Court (under Article 32). It means, when the
fundamental rights of a citizen are violated, the aggrieved party
has the option of moving either the High Court or the Supreme

w.E
proved misbehavior or incapacity on an address by both
houses of parliament supported by the vote of 23rd of
members present and voting in each house.
Court directly. However, the writ jurisdiction of the High
Court is wider than that of the Supreme Court. This is because,
the Supreme Court can issue writs only for the enforcement of
••
as a judge of SC.

Emoluments
a
Thus a judge of the HC can be removed in the same way

syE
fundamental rights and not for any other purpose, that is, it
does not extend to a case where the breach of an ordinary legal
right is alleged.”

•• Besides other facilities, Chief Justice and other Judges


of High Court get a salary of ` 90,000 and `80,000 per
ngi Appellate Jurisdiction
A high court is primarily a court of appeal. It hears appeals

••
month, respectively.
The salaries and allowances of the judges are charged
on the Consolidated Fund of the State. These cannot be nee
against the judgements of subordinate courts functioning in its
territorial jurisdiction. It has appellate jurisdiction in both civil
and criminal matters. Hence, the appellate jurisdiction of a
varied by the parliament to their disadvantage after their
appointment (except under financial emergency). The
pensions of the judges are charged on the Consolidated r
High Court is wider than its original jurisdiction.

Supervisory Jurisdiction
ing
A high court has the power of superintendence over ‘all courts
••
Fund of India.
After retirement, a permanent judge of HC cannot plead
or act in a court of India except SC or HC other than the military courts or tribunals). Thus, it may— .ne
and tribunals functioning in its territorial jurisdiction (except

one in which he has held office.

Jurisdiction of high court


(a) call for returns from them;
t
(b) make and issue, general rules and prescribe forms for
regulating the practice and proceedings of them;
(c) prescribe forms in which books, entries and accounts are
Original Jurisdiction to be kept by them; and
It means the power of a High Court to hear disputes in the (d) settle the fees payable to the sheriff, clerks, officers and
first instance, not by way of appeal. It extends to the legal practitioners of them.
following: This power of superintendence of a High Court is very broad
(a) Matters of admirality, will, marriage, divorce, company because,
laws and contempt of court.
(i) it extends to all courts and tribunals whether they are
(b) Disputes relating to the election of members of Parliament
subject to the appellate jurisdiction of the High Court or
and state legislatures.
not;
(c) Regarding revenue matter or an act ordered or done in
(ii) it covers not only administrative superintendence but also
revenue collection.
judicial superintendence;
(d) Enforcement of fundamental rights of citizens.
(e) Cases ordered to be transferred from a subordinate court (iii) it is a revisional jurisdiction and
involving the interpretation of the Constitution to its own (iv) it can be suo-motu (on its own) and not necessarily on the
file. application of a party.

Downloaded From : www.EasyEngineering.net


Downloaded From : www.EasyEngineering.net
P-144 Judiciary

However, this power does not vest the High Court with any Comparison between Supreme Court
unlimited authority over the subordinate courts and tribunals.
It is an extraordinary power and hence has to be used most
and High Court
sparingly and only in appropriate cases. Usually, it is limited to, Supreme Court High Court
(i) excess of jurisdiction, 1. This is the union court and the 1. The High Court is
(ii) gross violation of natural justice apex institution of the united constituted in every State
(iii) error of law, court system. for a group of states.
(iv) disregard to the law of superior courts, 2. All the Judges of the Supreme 2. The Judge of the High
(v) perverse findings, and Court, retire on attaining the Court retires after
(vi) manitest injustce. age of 65 years. attaining the age of 62
years.
Control over Subordinate Courts
3. The Judges of the Supreme 3. The Judge of the High
In addition to its appellate jurisdiction and supervisory
Court cannot do their practice Court cannot do his legal
jurisdiction over the subordinate courts as mentioned above, after retirement. These are practice during his tenure
a High Court has an administrative control and other powers also restricted during their but we can do this after
over them. These include the following: tenure. his tenure in any High

ww
(a) It is consulted by the Governor in the matters appointment,
posting and promotion of district judges and in the
appointments of persons in the judicial service of the state
Court or Supreme Court.
He cannot do his legal
practices in courts below

w.E
(other district judges).
(b) It deals with the matters of posting, promotion grant of
leave, transfers and discipline of members of the judicial
High Court.
4. The Judges of the Supreme 4. The Judges of the High
Court cannot be transferred Courts are transferrable

a
service of the state (other than district judges).

syE
(c) It can withdraw a case pending in a subordinate court if
it involves a substantial question of law that require the
interpretation of Constitution. It can then either dispose
and cannot be promoted. to the other high courts.
They can be promoted
upto Judge of the
Supreme Court.
of case itself or determine the question of law and return
the case to the subordinate court with its judgement.
(d) Its law is binding on all subordinate courts functioning
ngi 5. The Supreme Court is not 5. The High Courts are
bounded to obey the decisions
of the High Courts or any
bounded to obey the
decision of Supreme
within its territorial jurisdiction in same sense as the law
declared by the Supreme Court is binding on all courts in nee
other courts. Court.
6. The Supreme Court only has 6. The High Court has no
India.
Power of Judicial Review
Judicial review is the power of a High Court to examine the
r
the power to take decisions
regarding constitutions.

ing
power to take decisions
regarding constitution.
7. The Chief Justice of the 7. The chief Justice of High
constitutionality of legislative enactments and executive orders
of both the Central and state governments. On examination, if
Supreme Court draws a salary
of 100,000 Rupees per month
while other Judges draw .ne
Court draws a salary of
` 90,000 while other
Judges draw ` 80,000
they are found to be violative of the Constitution (ultra-vires),
they can be declared as illegal, unconstitutional and invalid
(null and void) by the High Court.
Though the phrase ‘judicial review’ has no where been used
` 90,000 per month.

interpretation of the
Constitution are decided only
p.m.
t
8. The cases involving the 8. The cases involving the
interpretation of the
Constitution are not
in the Constitution, the provisions of Articles 13 and 226 by the Supreme Court. decided by the High
explicitly confer the power of judicial review on a High Court.
Court. The constitutional validity of a legislative enactment or 9. The Supreme Court can issue 9. High Court can issue
an executive order can be challenged in a High Court on the writs only for the enforcement writs not only for
following three grounds: of fundamental Rights the enforcement of
(a) It infringes the fundamental rights (Part III), fundamental Rights
(b) It is outside the competence of the authority which has but also for any other
framed it, and purpose.
(c) It is repugant to the constitutional provisions. 10. A remedy under Article 32 in 10. A remedy under Article
The 42nd Amendment Act of 1976 curtailed the judicial in Itself a Fundamental Right 226 is discretionary and
review power of High Court. It debarred the High Courts and hence, the Supreme Court hence, a High Court may
from considering the constitutional validity of any central may not refuse to exercise its refuse to exercise its writ
law. However, the 43rd Amendment Act of 1977 restored the writ jurisdiction. jurisdiction.
original position.

Downloaded From : www.EasyEngineering.net


Downloaded From : www.EasyEngineering.net
Judiciary P-145

The High Courts : Seats and year. Thus cases built up during this interim period are judged
when the circuit court is in session.
Jurisdiction The Calcutta High Court is the oldest The Madras High Court in Chennai, Bombay High Court in
High Court in the country, established on 2 July 1862. High
Mumbai, Calcutta High Court in Kolkata and Allahabad High
Courts that handle a large number of cases of a particular
Court in Allahabad are the oldest four High Courts in India.
region have permanent benches established there. Benches are
also present in states which come under the jurisdiction of a The following are the 24 High Courts of India sorted by
court outside its territorial limits. Smaller states with few cases name, year established, Act by which it was established,
may have circuit benches established. Circuit benches (known jurisdiction, headquarters, benches, the maximum number of
as circuit courts in some parts of the world) are temporary judges sanctioned and the presiding Chief Justice of the High
courts which hold proceedings for a few selected months in a Court

Act
Court name Established Jurisdiction Seat Benches Judges Chief Justice
established
Allahabad 11 June 1866 Indian High Uttar Pradesh Allahabad Lucknow 95 Hon’ble Justice
High Court Courts Act Dhananjaya
1861 Y. Chandrachud
Bombay 14 August Indian High Maharashtra,  Mumbai Aurangabad, 60 Hon’ble Justice 

ww
High Court 1862 Courts Act
1861
Goa, Dadra
and Nagar
Haveli,
Nagpur,Panaji Mohit Shantilal
Shah

Calcutta
High Court w.E
2 July 1862
Indian High
Courts Act
1861
Daman and Diu
West Bengal,  Kolkata
Andaman and
Nicobar Islands
Port Blair 63 Hon’ble Jus-
tice Manjula
Chellur
Chhattisgarh 1 November
High Court 2000
a
Madhya
Pradesh Reor-
ganisation Act,
2000
syE
Chhattisgarh Bilaspur   12 Hon’ble Jus-
tice Navin
Sinha (Acting)

Delhi High 31 October


Court 1966
Delhi High
Court Act,
1966
Territory of
Delhi ngi
National Capital New Delhi   43 Hon’ble Justice G.
Rohini

Gauhati
High Court
1 March 1948 Government of
India Act, 1935
ArunachalPrade Guwahati
sh,Assam, Nag
aland,Mizoram nee Aizwal, Itanagar, 27
Kohima
Hon’ble Jus-
tice K. Sreedhar
Rao(Acting)
Gujarat
High Court

High Court
1 May 1960

5 July 1954
Bombay Re-
organsisation
Act, 1960
Andhra State
Gujarat Ahmedabad

Andhra Pradesh Hyderabad


 
42

ing
49
Hon’ble Jus-
tice Jayant .M.
Patel(Acting)
Hon’ble Jus-
of Judicature
at Hyder-
Act, 1953 Telangana
.ne
tice Dilip Babasa-
heb Bhosale (Act-
abad
Himachal
Pradesh
High Court
Jammu and
1971

28 August
State of Him-
achal Pradesh
Act, 1970
Letters Patent
Himachal
Pradesh

Jammu and
Shimla

Srinagar/
 

 
09

14
ing)

t
Hon’ble Jus-
tice Mansoor
Ahmad Mir
Hon’ble Justice N.
Kashmir 1943 issued by then Kashmir Jammu[6] Paul Vasanthaku-
High Court Maharaja of mar
Kashmir
Jharkhand 15 November Bihar Reor- Jharkhand Ranchi   12 Hon’ble Justice Vi-
High Court 2000 ganisation Act, render Singh
2000
Karnataka 1884 Mysore High Karnataka Bengaluru Dharwad, 40 Hon’ble Justice 
High Court Court Act, Gulbarga Subhro Kamal
1884 Mukhergee 
(Acting)
Kerala High 1956 States Reor- Kerala, Lak- Kochi   40 Hon’bl Justice
Court ganisation Act, shadweep Ashok
1956 Bhushan(Acting)

Downloaded From : www.EasyEngineering.net


Downloaded From : www.EasyEngineering.net
P-146 Judiciary

Madhya 2 January Government of Madhya Jabalpur Gwalior,Indore 42 Hon’ble Justice 


Pradesh 1936 India Act, 1935 Pradesh Ajay M. Khan-
High Court wilkar
Madras 15 August Indian High Tamil Chennai Madurai 42 Hon’ble Justice 
High Court 1862 Courts Act Nadu, Puduch- Sanjay Kishan
1861 erry Kaul
Manipur 25 March North-Eastern Manipur, Imphal 03 Hon’ble Justice L.
High Court 2013 Areas (Reor- K. Mohapatra
ganisation) and
Other Related
Laws (Amend-
ment) Act,
2012
Meghalaya 25 March North-Eastern Meghalaya, Shillong 03 Hon’ble Jus-
High Court 2013 Areas (Reor- tice Uma Nath
ganisation) and Singh
Other Related

ww
Orissa High 3 April 1948
Laws (Amend-
ment) Act,
2012
Orissa High Odisha Cuttack   27 Hon’ble Justice D.
Court

Patna High
w.E
2 September
Court Order,
1948
Government of Bihar Patna   43
H. Waghela

Hon’ble Justice 
Court

Punjab and
Haryana
High Court
1916

21 March
1919
a India Act, 1915

High Court
syE
(Punjab) Order,
1947
Punjab,
Haryana,
Chandigarh
Chandigarh   53
Iqbal Ahmed
Ansari
Hon’ble Justice 
Shiavax Jal
Vazifdar(Acting)
Rajasthan
High Court
21 June 1949 Rajasthan High
Court Ordi-
Rajasthan
ngi
Jodhpur Jaipur 40 Hon’ble Justice 
Ajit Singh (Acting)

Sikkim High 16 May 1975


Court
nance, 1949
The 36th
Amendment
Sikkim Gangtok
nee
  03 Hon’ble Justice 
Sunil Kumar Sinha

r
to the Indian

Tripura
High Court
26 March
2013
Constitution
North-Eastern
Areas (Reor-
Tripura Agartala  
ing 04 Hon’ble Justice 
Deepak Gupta
ganisation) and
Other Related
Laws (Amend- .ne
Uttarakhand 9 November
High Court 2000
ment) Act,
2012
Uttar Pradesh
Reorganisation
Uttarakhand Nainital   09
t
Hon’ble Justice K.
M. Joseph
Act, 2000

H. Court by State/ U. Principal Seat/(Bench having jurisdic-


Court
Territory tion of the State)
Andaman and Nicobar
Calcutta High Court Kolkata (Bench at Port Blair)
Islands
Arunachal Pradesh Gauhati High Court Guwahati (Bench at Itanagar)
Andhra Pradesh High Court of Judicature at Hyderabad Hyderabad
Assam Gauhati High Court Guwahati
Bihar Patna High Court Patna
Chhattisgarh Chhattisgarh High Court Bilaspur
Chandigarh Punjab and Haryana High Court Chandigarh
Dadra and Nagar Haveli Bombay High Court Mumbai

Downloaded From : www.EasyEngineering.net


Downloaded From : www.EasyEngineering.net
Judiciary P-147

Daman and Diu Bombay High Court Mumbai


National Capital Territory
Delhi High Court New Delhi
of Delhi
Goa Bombay High Court Mumbai (Bench at Panaji)
Gujarat Gujarat High Court Ahmedabad
Haryana Punjab and Haryana High Court Chandigarh
Himachal Pradesh Himachal Pradesh High Court Shimla
Jammu and Kashmir Jammu and Kashmir High Court Srinagar/Jammu
Jharkhand Jharkhand High Court Ranchi
Bengaluru (Bench at Dharwad and Gulbar-
Karnataka Karnataka High Court
ga)
Kerala Kerala High Court Kochi
Lakshadweep Kerala High Court Kochi
Madhya Pradesh Madhya Pradesh High Court Jabalpur (Bench at Gwalior and Indore)
Mumbai (Bench at Aurangabad and Nag-

ww
Maharashtra
Manipur
Bombay High Court
Manipur High Court
pur)
Imphal
Meghalaya
Mizoram
Nagaland
w.E Meghalaya High Court
Gauhati High Court
Gauhati High Court
Shillong
Guwahati (Bench at Aizawl)
Guwahati (Bench at Kohima)
Odisha
Puducherry
Punjab
Rajasthan
a Orissa High Court

syE
Madras High Court
Punjab and Haryana High Court
Rajasthan High Court
Cuttack
Chennai
Chandigarh
Jodhpur (Bench at Jaipur)
Sikkim
Tamil Nadu
Sikkim High Court
Madras High Court ngi Gangtok
Chennai (Bench at Madurai)
Telangana
Tripura
High Court of Judicature at Hyderabad
Tripura High Court nee Hyderabad
Agartala
Uttarakhand
Uttar Pradesh
West Bengal
Uttarakhand High Court
Allahabad High Court
Calcutta High Court
r Nainital

ing
Allahabad (Bench at Lucknow)
Kolkata

Courts under High Court


• District Courts of India
Appointment of District Judges
.ne



District Munsiff Court
Courts of Judicial Magistrate of First Class
Courts of Judicial Magistrate of Second Class
t
The appointment, posting and promotion of a District Judge
is done under the Governor of the State in consultation with
the High Court. The necessary qualifications for a person to be
appointed as a District Judge are as follows:
Subordinate Courts (part VI, Articles •• Article 233 strictly says that a person to be appointed as
233 to 237) District Judge must not be in the service of the Central or
the State Government.
Under the High Court there are three types of courts in the •• He should have been an advocate or a pleader for 7 years.
districts. They are the Civil Courts, the Criminal Courts and
the Revenue Courts. The highest Civil Court in a district is that • He should be recommended by the High Court for
of the district judge. They have the power to try civil cases and appointment as a District Judge.
to hear appeals. They have additional civil judges to help them.
The less important cases are decided by sub-judges and munsifs. Other Local Courts
The highest District Court to try criminal cases is that of the In addition to the three type of courts mentioned above, there
Sessions Judge. The criminal cases are heard by the Magistrates are the Panchayati Adalats or Nyaya Panchayats which are
too. The district judge also acts as the Sessions Judge in a also under the District Judge in some states. Four or Five Gram
district. Appeal cases against the lower courts are heard by the Sabha have one such Panchayati Court. They are established to
District Courts and appeals against the decisions taken by the try small cases of all kinds. Under this system the cases can be
District Court can be made to the State High Court. decided fast and need not in value much expenditure.

Downloaded From : www.EasyEngineering.net


Downloaded From : www.EasyEngineering.net
P-148 Judiciary

The Nyaya Panchayats function in ruler areas a similar concept Lok Adalat accepts the cases which could be settled by
introduced in some urban area this is called the ‘Lok Adalat’ conciliation and in which, compromise was pending in the
The first Lok Adalat was held in Delhi in 1985, in which one Regular Courts within their jurisdiction. The Lok Adalat is
hundred and fifty cases were decided in a single day. presided over by a sitting or Retired Judicial Officer or an
other person of respect and legal knowledge as the Chairman,
National Legal Services Authority with two other members, usually a lawyer and a social worker.
In 1987 the Legal Services Authorities Act (LSAA) was enacted The first Lok Adalat was held on March 14, 1982 at Junagarh
by the parliament, which came into force on November 9, in Gujarat.
1995 to establish a nationwide uniform network for providing
free and Competent Legal Services to the weaker section of the Alternative Dispute Resolution (ADR)
society on the basis of equal opportunity. The National Legal It encompasses arrangement of means to resolve conflict
Service Authority (NALSA) has been constituted under the without formal litigation. It seeks to reduce cost and delay and
Legal Services Authority Act 1987 to monitor and evalulate avoid the adversarial n ature of litigation. ADR have following
implementation of legal aid programmes and lay down policies techniques:
and principles for making legal services available under the • Conciliation is an informal process designed to create
Act. an environment where negotiations can take place. If the
In every state, a State Legal Services Authority and in every parties fail to reach an agreement, the case is referred to

ww
High Court a High Court legal services committee has been
constituted. District legal services authorities and Taluka Legal
Service Committees have been constituted in the district and
mediation.
• Mediation is voluntary and confidential process where
a neutral third party assists negotiations. The parties are

w.E
most of the Talukas in order to give effect to the policies and
directions of the NALSA and to provide free legal services to
the people and conduct Lok Adalat in the states.
responsible for reaching an agreement and the negotiator
cannot impose settlement. If the mediation fails to reach
agreement, the case is referred to arbitration.

a
NALSA issues guidelines for the State Legal Services

syE
Authorities to implement, the legal aid programmes and
schemes through out the country. Primarily, the state legal
services authorities, district legal service authority, Taluka
Legal service committee, etc. have been assigned the task of
• Arbitration is form of private adjudication where a
mutually acceptable third party hears arguments from either
side in a dispute and renders a judgement. The judgement
known as an Award, is confidential and binding.

discharging the following two main functions on regular basis:


(i) To provide free legal services to the eligible persons; andngi • Community-driven resolution mechanism (Lok Adalat)
literally means ‘Peoples Court’. It is an alternative dispute
settlement mechanism, which settles disputes. It helps
(ii) To organise Lok Adalats for amicable settlement of
disputes.

Mobile Courts
nee
in quick disposal of cases and the process is simple and
carries no fees. Lok Adalats are statutory forums since the
enactment of the Legal Serivces Authorities Act, 1987.
Mobile Court means a court set-up in a vehicle, which can
move from one place to another, according to a well-prepared
plan and schedule. Mobile Courts will be of great relief to
r ing
Difference between IPC and CrPC.
Indian Penal Code, 1860 is one of the 3 major criminal statutes
the rural people. It would create greater awareness about the
judicial system among rural masses, cut costs for them and
.ne
in India which forms the base of criminal justice system in
India. IPC was passed into law on October 6, 1860 and came
into operation on January 1, 1862.
render justice at their doorstep. These courts should see to that
hearings are not unnecessarily postponed.
The Mobile Court is equipped to receive complaints, civil
and criminal applications, grant bail and remand accused
to custody, issue summons, receive police reports, record
t
The other two major statutes are Indian Evidence Act, 1872
and Code of Criminal Procedures, 1973.
The Indian Penal Code (IPC) is the factual law defining various
crimes and prescribing the punishment which would visit
evidence, pronounce and execute decrees and judgements, those who would commit those crimes. The Code of Criminal
pass sentences and can send convicts to prison. It also delivers Procedure(Cr PC) is essentially a procedural law prescribing
certified copies of its orders and judgements. The Country’s the procedure for investigation and trial of an offence to be
first mobile court was launched at Mewat district in Haryana. followed by the Courts for a criminal case. So, hierarchy
of criminal courts, the stages of a criminal suit, Powers of
Lok Adalat magistrates, different types of criminal cases, the powers of
It is a system of alternative dispute resolution developed in the police to arrest, etc. is laid down in the CrPC. .Moreover,
India. It roughly means People’s court. They are governed the basic difference is that the former is a substantive law and
by Legal Services Authorities Act of 1987. The Award of the the latter is a procedural law.
Lok Adalat is binding upon all the parties. Lok Adalats are The Indian Penal Code applies to the whole of India except
given certain powers of the Civil Courts. The Lok Adalats for the state of Jammu & Kashmir. It contains 23 Chapters
and 511 Sections. Before the Indian Penal Code came into
have wide jurisdiction that means any matter falling within the
effect, the Mohammedan Criminal Law was applied to both
jurisdiction of Civil, Criminal, Revenue Courts or Tribunals
Mohammedans and Hindus in India.
are dealt by them.

Downloaded From : www.EasyEngineering.net


Downloaded From : www.EasyEngineering.net
Judiciary P-149

Broad classification of crimes under the Indian Penal Code (IPC) category of persons trafficked.  Employment of a trafficked
1. Crimes Against Body: Murder, Its attempt, Culpable person will attract penal provision as well.
Homicide not amounting to Murder, Kidnapping & The most important change that has been made is the change
Abduction, Hurt, Causing Death by Negligence; in definition of  rape under IPC. Although the Ordinance
2. Crimes Against Property: Dacoity, its preparation & sought to change the word rape to sexual assault, in the Act
assembly, Robbery, Burglary, Theft; the word ‘rape’ has been retained in Section 375, and was
3. Crimes Against Public order: Riots, Arson; extended to include acts in addition to vaginal penetration.
4. Economic Crimes: Criminal Breach of Trust, Cheating, The definition is broadly worded with acts like penetration
Counterfeiting; of  penis, or any object or any part of body to any extent,
5. Crimes Against Women: Rape, Dowry Death, Cruelty by into the vagina, mouth, urethra or anus of another person or
Husband and Relatives, Molestation, Sexual harassment making another person do so, apply of mouth or touching
and Importation of Girls; private parts constitutes the offence of sexual assault. The
6. Crimes Against Children: Child Rape, Kidnapping & section has also clarified that penetration means “penetration
Abduction of Children, Procreation of minor girls, Selling/ to any extent”, and lack of physical resistance is immaterial for
Buying of girls for Prostitution, Abetment to Suicide, constituting an offence. Except in certain aggravated situations
Exposure and Abandonment, Infanticide, Foeticide; the punishment will be imprisonment not less than seven years
7. Other IPC crimes. but which may extend to imprisonment for life, and shall also

ww
Criminal Law (Amendment) Act, 2013
Criminal Law (Amendment) Act, 2013 Popularly, known as
be liable to fine. In aggravated situations, punishment will be
rigorous imprisonment for a term which shall not be less than
ten years but which may extend to imprisonment for life, and

w.E
the Anti-rape Act, amends the Indian Penal Code (IPC), 1860,
the Code of Criminal Procedure (CCP), 1973, the Indian
Evidence Act (IEA), 1872 and the Protection of Children from
shall also be liable to fine.
A new section, 376A has been added which states that if a
person committing the offence of sexual assault, “inflicts an

Introduction a
Sexual Offences Act, (PCSO), 2012.

syE
Against the backdrop of the nation-wide outrage over the tragic
injury which causes the death of the person or causes the
person to be in a persistent vegetative state, shall be punished
with rigorous imprisonment for a term which shall not be less
than twenty years, but which may extend to imprisonment for
life, which shall mean the remainder of that person’s natural
Delhi gang-rape, Nirbhaya (Fearless) incident of December
16, 2012, propelled the Government of India (GOI) to drive
the issue of violence against women (VAW) to the centre-stage ngi life, or with death. In case of “gang rape”, persons involved
regardless of their gender shall be punished with rigorous
of political discourse. Consequently, on December 22, 2012,
GOI appointed a three-member judicial committee headed by
the former Chief Justice of India, Justice J.S. Verma. nee
imprisonment for a term which shall not be less than twenty
years, but which may extend to life and shall pay compensation
to the victim which shall be reasonable to meet the medical
Based on some of the recommendations of the Justice
Verma Committee (JVC) report, an antirape Ordinance was
enacted and signed by the Honourable President of India,
Mr Pranab Mukherjee on February 03, 2013. The Criminal
r
expenses and rehabilitation of the victim. The age of consent in

ing
India has been increased to 18 years, which means any sexual
activity irrespective of presence of consent with a woman
below the age of 18 will constitute statutory rape.
Law (Amendment) Bill, 2013, passed in the Parliament (Lok
Sabha1 and Rajya Sabha2 respectively on March 19 and 21, .ne
Certain changes has been introduced in the CrPC and Evidence
Act, like the process of recording the statement of the victim
2013) replaced the promulgated Ordinance, which lapsed on
April 04, 2013. However, as per the Gazette Notification
New Delhi, Tuesday, April2, 2013,3 the word ‘bill’ has been
replaced by the word ‘Act’.
critical changes are:
t
has been made more victim friendly and easy but the two

1. the ‘character of the victim’ is now rendered totally


irrelevant, and
2. there is now a presumption of ‘no consent’ in a case where
Changes in law sexual intercourse is proved and the victim states in the
Section 370 of Indian Penal Code (IPC) has been substituted court that she did not consent.
with new sections, 370 and 370A which deals with trafficking
of person for exploitation. If a person : Criticisms
(a) recruits, The Criminal Law (Amendment) Ordinance, 2013 has been
(b) transports, strongly criticised by several human rights and women’s
(c) harbours, rights organisations for not including certain suggestions
(d) transfers, or recommended by the Verma Committee Report like,  marital
(e) receives, a person, by using threats, or force, or coercion, rape, reduction of age of consent, amending Armed Forces
or abduction, or fraud, or deception, or by abuse of power, (Special Powers) Act so that no sanction is needed for
or inducement for exploitation including prostitution, slavery, prosecuting an armed force personnel accused of a crime
forced organ removal, etc. will be punished with imprisonment against woman. The Government of India, replied that it has
ranging from at least 7 years to imprisonment for the remainder not rejected the suggestions fully, but changes can be made
of that person’s natural life depending on the number or after proper discussion.

Downloaded From : www.EasyEngineering.net


Downloaded From : www.EasyEngineering.net
P-150 Judiciary

The changes made in the Act incomparison with the Ordinance is listed as follows:
Offence Changes
Fine shall be just and reasonable to meet medical expenses for treatment of victim, while in the
Acid attack
Ordinance it was fine up to Rupees 10 lakhs.
“Clause (v) any other unwelcome physical, verbal or non-verbal conduct of sexual nature” has
been removed. Punishment for offence under clause (i) and (ii) has been reduced from five years of
Sexual harassment
imprisonment to three years. The offence is no longer gender-neutral, only a man can commit the
offence on a woman.
Voyeurism The offence is no longer gender-neutral, only a man can commit the offence on a woman.
The offence is no longer gender-neutral, only a man can commit the offence on a woman. The
definition has been reworded and broken down into clauses, The exclusion clause and the following
sentence has been removed “or watches or spies on a person in a manner that results in a fear of
violence or serious alarm or distress in the mind of such person, or interferes with the mental peace
Stalking
of such person, commits the offence of stalking”. Punishment for the offence has been changed; A
man committing the offence of stalking would be liable for imprisonment up to three years for the

ww
Trafficking of person
first offence, and shall also be liable to fine and for any subsequent conviction would be liable for
imprisonment up to five years and with fine.
“Prostitution” has been removed from the explanation clause.
FEW KNOWN SECTIONS OF IPC
Section 120 w.E
Concealing design to commit offence
punishable with imprisonment.
Section 397: Robbery, or dacoity, with attempt to cause
death or grievous hurt.
LEGAL TERMS

and B
Section 141
a
Section 120-A Definition of criminal conspiracy and

syE
Punishment of criminal conspiracy.
Unlawful assembly.
Affidavit This is a sworn statement made by a party,
in writing, made in the presence of an oath
commissioner or a notary public which is
used either in support of applications to the

ngi
Section 146 Rioting and Punishment for rioting. Court or as evidence in court proceedings.
and 147
Alimony The maintenance given by a husband to his
Section 169 Public servant unlawfully buying or bidding divorced wife.
for property.
Section 171-B Bribery.
Section 279 Rash driving or riding on a public way
nee
Amicus curiae Translated from the Latin as ’friend of
the Court’. An advocate appears in this
capacity when asked to help with the case
Section 295

Section 298
Injuring or defiling place of worship with
intent to insult the religion of any class
Uttering, words, etc., with deliberate Arbitration
r the Court.
ing
by the Court or on volunteering services to

Settling disputes by referring them to


intent to hurt the religious feelings of any
person.
.ne
independent third parties as an alternative
to court proceedings.
Section 300

Section 307
Section 317
Murder.
Section 304-B Dowry death.
Attempt to murder.
Exposure and abandonment of child under Bequeath
side’ or ’hear both sides’. t
Audi alteram This is a rule of natural justice which
partem translates from the Latin as ‘hear the other

To dispose of personal property by Will.


twelve years, by parent or person having Caveat Where it is apprehended that an opposite
care of it. party may file a case, a party may file a
Section 369 Kidnapping or abducting child under ten document requesting the court that no
order be made in the case without hearing
years with intent to steal from its person. the caveator.
Section 372 Selling minor for purposes of prostitution, C o g n i z a b l e An offence in which arrest can be made
etc. offence without a warrant.
Section 373 Buying minor for purposes of prostitution,
etc. Dasti Notice Dasti is a persian word, which means
‘by hand’. Dast Notice means service
Section 378 Theft of the notice by the Petitioner on the
Section 375 Rape Respondent(s) in person, and not by the
Section 376 Punishment against rape. Registry through post.
Section 383 Extortion Decree The formal expression of an adjudication
which, so far as regards the Court
Section 390 Robbery expressing it.

Downloaded From : www.EasyEngineering.net


Downloaded From : www.EasyEngineering.net
Judiciary P-151

Estoppel A legal principle that bars a party from Pleadings A collective noun for all the petitions,
denying or alleging a certain fact owing to affidavits, replies, rejoinders drafted by or
that party’s previous conduct, allegation, on behalf of the parties to a case.
or denial. Prima facie At first sight; on the face of it.
Habeas A writ requiring a person under arrest to Pro bono Translated from the Latin as ’for the public
Corpus be brought before a judge or into court, publico good’. In PIL, this refers to a petitioner
especially to secure the person’s release acting bonafide in the public interest.
unless lawful grounds are shown for their
detention. Respondent A party against whom a petition is filed.
A proforma respondent is a party against
In pari delicto When both the parties are equally in fault. whom no relief is sought.
Interim Order Any order by a court before a final order Stare decisis The principle that decisions of Courts
is made. in previous cases must be followed in
Interlocutory Petition seeking a relief even while the subsequent cases of similar nature.
Application main petition remains in the Court. Statute A codified law that is enacted by the
Intervenor A person who is not a party to the Parliament or a State Legislature.
proceedings may, with the permission of Stay Order A party filing a petition may require
the court, intervene if it is shown that the

ww
Judgment-
outcome of the case will affect such person
in some way.
Any person against whom a decree Suo Motu
some immediate relief, even before the
respondents can be heard or a final decision
given.
The Court may take action on its own when
debtor

Judicial
w.E
has been passed or an order capable of
execution has been made;
A term that describes the function of the
facts requiring legal intervention reach its
notice. The Court is then said to be acting
suo moto.
Review

a
judiciary being able to examine and correct

syE
the actions of all the organs of State—the
executive, the legislature and the judiciary
itself.
Void

Vox populi
One that law regards as never having taken
place.
Translated from the Latin as ’the voice of
the people’.
Justiciable A matter is justiciable if it lends itself to
adjudication by a court. This is determined
by criteria laid down in law. ngi
Writ A writ is a direction that the Court issues,
which is to be obeyed by the authority/
Litigation

Locus Standi
The totality of the legal proceedings in any
dispute.
Translated from Latin as ’place of
nee
Writ Petition
person to whom it is issued.
A petition seeking issuance of a writ is a
writ petition.
standing’, locus standi gives the right to
pursue a litigation. Under this rule, only a
person or group of persons affected by the
issue may petition the Court.
r
Juvenile Justice Act 2015
ing
Juvenile Justice (Care and Protection of Children)Act, 2015

Ordinance A codified law made, as a temporary


measure, by the President of India or the .ne
of Ministry of Women and child Development came into
force on 15th January 2016. This is an Act to consolidate
and amend the Law relating to children in need of care

Perjury
Governor of a State when the Parliament
or legislature of a state is not in session.
This occurs when a person gives false
evidence or false affidavit in a case.
t
and protection, by catering to their basic needs through
development, treatment, and social re-integration, by
adopting a child-friendly approach. This Act enhances the
time and punishment for sale of tobacco products to minors.
Petition A written document filed in a court The Law for underage offenders also would Rave the way
asserting a claim or a right and seeking for trying those between 16 and 18 years of age, accused
relief on legal grounds. of heinous crimes as adults, punishable with a jail terms of
upto 7 years.

Downloaded From : www.EasyEngineering.net


Downloaded From : www.EasyEngineering.net
P-152 Judiciary

1.
Exercise - 1
Judges of the Supreme Court take an oath or affirmation 10. If any question arises as to the age of a judge of a High
before entering upon office conducted by Court, the question shall be decided by the President
(a) Chief Justice of India after consultation with:
(b) President or Vice-President (a) The Chief Justice of the concerned High Court
(c) President (b) The Governor of the concerned state
(d) None of the above (c) The Attorney-General of India
2. A judge of a High Court can be removed before the (d) The Chief Justice of India
expiry of his time by: 11. The Chief Justice of the High Court is appointed by
(a) President on the recommendation of the Supreme (a) the Governor of the state
Court (b) the President of India
(b) Governor on the recommendation of the State (c) the Chief Minister of the state
legislature (d) the Chief Justice of India
(c) President on the recommendation of the State 12. The Supreme Court of India tenders advice to the

ww legislature
(d) President on the recommensdation of the Parliament
3. A judge of a High Court wanting to resign addresses his


President on a matter of law or fact
(a) on its own initiative
(b) only if he seeks such advice



(a) the President
w.E
letter or resignation to:

(b) the Chief Justice of his High Court



(c) only if the matter relates to the fundamental righ of
citizens
(d) only if the issue poses a threat to the unity and


a
(c) the Chief Justice of India
(d) Governor of the State
4. Which is not a power of a High Court?
syE
(a) Supervision over all courts under its jurisdiction
integrity of the country
13. Which one of the following is directly related to the


appellate jurisdiction of the Supreme Court of India ?
(a) Appeals made in civil, criminal and constitutional
(b) Jurisdiction over revenue matters
(c) Supervision over tribunals constituted by law
relating to armed forces ngi

cases
(b) Appeals made in constitutional cases
(c) Adjudication of disputes between the union and the
(d) Issue writs for enforcing fundamental rights or for
any other purpose
5. Oath to a High Court judge is administered by:

nee
states
(d) Adjudication of disputes between the states
14. Which of the following statements is correct?




(a) Chief Justice of India
(b) Chief Justice of that High Court
(c) Governor of the State
(d) President of India

r
(a) The President of India is the custodian of the

ing
Constitution of India
(b) The Supreme Court of India can declare a law passed
by any State/Union Legislature null and void if it
6. Public interest litigation applies to cases of public injury
arising from: .ne
encroaches upon the Fundamental Rights guaranteed
by the Constitution of India




(a) Breach of public duty
(b) Violation of a constitutional provision
(c) Violation of law
(d) all of the above
7. In Minerva Mills cases, the Supreme Court has further


the State concerned t
(c) The number of judges in a High Court is to be
determined from time to time by the Governor of

(d) The Chief Justice of a High Court is appointed


by the Governor of the State concerned on the
reaffirmed its decision in recommendation of the Chief Justice of India
(a) Golakh Nath Case (b) Keshavanand Bharati case 15. Which of the following is covered under the original
(c) Sajjan Singh Case (d) None of the above jurisdiction of the Supreme Court:
8. District Judges are appointed by: (a) Dispute relating to civil matters
(a) The Chief Justice of High Court (b) Dispute relating to criminal cases involving
(b) The State Public Service Commission murder
(c) The Chief Minister of state (c) Disputes between two states of the Indian Union
(d) Disputes between two citizens from two different
(d) The Governor of state
states
9. The District and sessions Judge works directly under the 16. Who has the right to seek advisory opinion of the Supreme
control of: Court of India, on any question of Law?
(a) District Collector (a) Prime Minister
(b) Governor of the state (b) President
(c) Law Minister of the state (c) Any judge of the high court
(d) High Court of the state (d) All of the above

Downloaded From : www.EasyEngineering.net


Downloaded From : www.EasyEngineering.net
Judiciary P-153

17. The power of the Supreme Court of India to decide 27. Which of the following writs literally means ‘we
disputes between the Centre and the State falls under its command’?
_________ (a) Habeas Corpus (b) Mandamus
(a) Advisory jurisdiction (c) Prohibition (d) Quo-Warranto
(b) Appellate Jurisdiction 28. In which of the following cases Supreme Court of India
(c) constitutional Jurisdiction enunciated the doctrine of basic structure?
(d) Original Jurisdiction (a) Keshwanand Bharti case
18. Sovereignty of Indian Parliament is restricted by: (b) Golaknath case
(a) Powers of the President of India
(c) Minnerva Mills case
(b) Judicial review
(d) Gopalan case
(c) Powers of the Prime Minister of India
29. In Indian constitution the power to issue a writ of ‘Habeas
(d) Leader of the opposition
19. Who of the following Chief Justice of India acted as the corpus’ is vested only in-
President of India also? (a) The Supreme Court
(a) Justice M. Hidayatullah (b) The High Court
(b) Justice P.N. Bhagwati (c) The Supreme Court and the High Court
(c) Justice Mehar Chand Mahajan (d) Lower Courts

ww
(d) Justice B.K. Mukherjee
20. The Indian constitution provides for the appointment of
Ad hoc judges in:
30. The Provisions concerning the powers of the Union


judiciary in the Constitution can be amended by
(a) Simple majority of the parliament



(b) High Court
w.E
(a) Supreme Court

(c) District and session Court




(b) Two-third majority of the parliament
(c) Two-third majority of the parliament and the
majority of states
(d) All of these

a syE
21. Which of the following State/Union territories have a


common High Court?
(a) Uttar Pradesh and Bihar
(d) None of the above
31. The Supreme Court of India declares by issuing a writ
that “respondent was not entitled to an office he was
holding or a privilege he was exercising”. which writ is



(b) Punjab, Haryana and Chandigarh
(c) Punjab and Jammu and Kashmir
(d) Assam and Bengal ngi
that?
(a) Habeas Corpus
(c) Prohibition
(b) Quo Warranto
(d) Certiorari
22. Which of the following High Courts has the largest


number of Benches?
(a) Kolkata High Court
nee
32. What is the objective advocated for appointment of the
National judicial commission?
(a) Training of the judges



(b) Madhya Pradesh High Court
(c) Bombay High Court
(d) Guwahati High Court

r ing
(b) Reforms in legal system
(c) Bringing about transparency and impartiality in the
appointment of judges of the highest level.

.ne
23. The first High/Supreme Court judge, who voluntarily (d) To examine the working period of the judges.
made his assets public is 33. How can the number of judges in the Supreme Court in
(a) justice D.V.S. Kumar India be increased ?



(b) justice K. Chandra
(c) justice K. Kannan
(d) justice V.C. srivastava
24. Which one of the following High Courts has the territorial




(b) By amendment of the constitution
(c) By a Parliamentary Act
(d) by Presidential notification
t
(a) Representation from the Supreme Court

jurisdiction over Andaman and Nicobar Islands? 34.  Which of the following cases cannot be filed directly in
(a) Andhra Pradesh (b) Calcutta the Supreme Court ?
(b) Madras (d) Odisha (a) Cases against encroachment on Fundamental Rights
25. In the verdict of which case, Supreme Court has nullified (b) Both (a) and (b) above.
Parliament, effort to establish preference of all the (c) If one’s property is forcefully occupied by the other
Directive Principles of state policy over Fundamental (d) Disputes between two or more States
rights? 35. Which is not an eligibility criterion for appointment as a
(a) Keshvanand Bharti Vs State of Kerala Case Judge of the High Court ?
(b) Francis Coralie Mullin Vs Union Territory of Delhi (a) Must have been an advocate of a High Court for not
(c) Minerva Mills Vs. Union of India case less than 10 years
(d) Indira Sawhani (b) Must be, in the opinion of the President, a
26. Original jurisdiction of Supreme Court is mentioned in distinguished jurist.
which of the following articles of Indian constitution? (c) Must have attained the age of 55 years
(a) Article 131 (b) Article 132 (d) Must have been a High Court Judge for at least 5
(c) Article 143 (d) Article 148 years

Downloaded From : www.EasyEngineering.net


Downloaded From : www.EasyEngineering.net
P-154 Judiciary

36. Judicial Review function of the Supreme Court means the 46. Congnizable offence refers to an offence where
power to (a) Arrests can be made without warrant
(a) Review the functioning of judiciary in the country (b) Police can register a case without formal complaints
(b) Undertake periodic review of the Constitution. (c) Arrests can be made with warrant
(c) Examine the constitutional validity of the laws (d) It is under the jurisdiction of a court
(d) Review its own judgement 47. Under the writ of Mandamus, the Court can
37. The High Courts in India were first started at (a) Ask the person to be produced
(a) Bombay, Delhi, Madras (b) Order to transfer the case from one court
(b) Madras and Bombay (c) Ask to let a person free for a temporary period
(c) Bombay, Madras, Calcutta (d) Direct the Government to do or not to do a thing
(d) Delhi and Calcutta 48. Which of the following writs is a bulwark of personal
38. Besides its permanent seal at Delhi, the Supreme Court freedom ?
can also meet at (a) Certiorari (b) Habeas Corpus
(a) Any other Union Territory (c) Mandamus (d) Quo Warranto
(b) Any other place as decided by the Chief Justice of 49. Appointment of officers and servants of a High Court are
India in consultation with the President made by the
(c) Any other metropolitan city (a) None of these

ww
(d) Any other major city
39. What is meant by a Court of Record?
(a) The court that maintains records of all lower courts.



(b) Chief Justice of the High Court
(c) President
(d) Governor


w.E
(b) The court that is competent to give directions and
issue writs.
(c) The court that can punish for its contempt.
50.



Salaries of the Judges of the Supreme Court are drawn
from the
(a) Grants-in-aid (b) Public Accounts
(c) Contingency Fund (d) Consolidated Fund

a
(d) The court that preserves all its records.

(a) Chief Justice of the High Court


(b) President syE
40. Judges of the High Court are appointed by the 51.



Which of the following High Courts covers more than
one State/ Union Territories ?
(a) Allahabad
(c) Guwahati
(b) None of these
(d) Delhi
(c) Governor
(d) Chief Justice of India
41. After retirement, a Judge of a High Court can undertake ngi 52. Which of the following writs may be issued to enforce a
Fundamental Right ?

nee
practice in (a) Certiorari (b) Habeas Corpus
(a) Any other court except the same court (c) Mandamus (d) Prohibition
(b) Wherever he intends to practice. 53. The Judges of the Supreme Court can be removed from


(c) The same court
(d) Lower courts only
42. Separation of the Judiciary from the Executive is enjoined



r
office by the

ing
(a) President on request of Parliament
(b) Chief Justice of India
(c) President

.ne
by
(d) Prime Minister
(a) VII Schedule to the Constitution
54. The authority competent to suspend the operation of
(b) Judicial decision
Fundamental Rights guaranteed under the Constitution of
(c) Directive Principles
(d) Preamble
43. The Chief Justice and other Judges of the High Court are
appointed by the


55.
India is
(a) Supreme Court
(c) Parliament
t
(b) Prime Minister
(d) President
Which is the highest and final judicial tribunal in respect
(a) Chief Justice of the Supreme Court of the Constitution of India ?
(b) Chief Minister of the concerned state (a) President (b) Union Cabinet
(c) Governor of the concerned state (c) Supreme Court (d) Parliament
(d) President 56. Which of the following is an extensive original jurisdiction
44. Judicial Review signifies that the Supreme Court given by the Constitution of India to the Supreme Court ?
(a) Can impeach the President (a) Enforcement of Fundamental Rights
(b) Can declare a state law as unconstitutional (b) Advising the Chief Executive in legal matters
(c) Can review cases decided by the High Courts. (c) Hearing revenue cases of appeal
(d) Has final authority over all cases (d) Hearing criminal cases of appeal
45. Which one of the following comes under the jurisdiction 57. The High Court of West Bengal (Calcutta) has got the
of both the High Court and the Supreme Court ? additional jurisdiction to hear cases from
(a) Disputes between two States (a) Arunachal Pradesh
(b) Protection against the violation of the Constitution (b) Mizoram
(c) Protection of the Fundamental Rights (c) Tripura
(d) Disputes between the Centre and the States (d) Andaman and Nicobar islands

Downloaded From : www.EasyEngineering.net


Downloaded From : www.EasyEngineering.net
Judiciary P-155

58. Which of the following is enforceable in a court of law ? 68. A Judge of a Supreme court may resign his office by
(a) Fundamental Rights writing under his hand addressed to the
(b) Fundamental Duties (a) Chief Justice of India
(c) Directive Principles (b) Senior most judge of the supreme court
(d) Preamble (c) Prime Minister
59. In whom are the powers of Judicial Review vested in India ? (d) President
(a) All the courts 69. The Appellate Jurisdiction of the Supreme Court dose not
involve
(b) Supreme Court and all the High Courts
(a) Criminal Cases
(c) President
(b) Cases involving interpretation of the Constitution
(d) Parliament
(c) Civil Cases
60. The lowest court of revenue is that of a (d) Disputes arising out of pre-Constitution treaties and
(a) Naib Tehsildar agreements
(b) Sub-judge 70. When the chief justice of a High Court acts in an
(c) Third class magistrate administrative capacity, he is subject to.
(d) Munsif (a) The writ jurisdiction of any of the other judges of
61. To ensure impartiality, the retired Chief Justice and the High Court.

ww
other Judges of the Supreme Court are debarred from
practising law
(a) In any court other than State High Courts


(b) Special control exercised by the chief justice of
India.
(c) Discretionary power of the Governor of the State


w.E
(b) In any Criminal Court
(c) In any court of India
(d) In any court other than the Supreme Court

71.
(d) Special powers provided to the Chief Minister in
this regard
Article 136 of Indian Constitution authorizes the Supreme
Court to grant special leave to appeal. Which of the

(b) Parliament
(c) President
a
62. Who decides the number of Judges in a High Court ?
(a) Governor of the State

syE

following statement is not correct with respect to ‘Appeal
by Special Leave’?
(a) It is a discretionary power of the Supreme Court
(b) It can be granted against any court or tribunal


(d) State Government
63. Who is appointed as an adhoc judge of the Supreme Court ?
(a) A sitting judge of a High Court duly qualified for ngi

including the military court
(c) It can be related to any matter
(d) None of these


appointment as a Supreme Court Judge
(b) A person fully qualified for appointment as a Judge
of the Supreme Court
72.
nee
The power of the Supreme Court of India to decide
disputes between the Centre and the States falls under
its


(c) A retired judge of Supreme Court
(d) An acting judge of the Supreme Court
64. Which of the following is covered under the original


73.
r ing
(a) advisory jurisdiction (b) appellate jurisdiction
(c) original jurisdiction (d) writ jurisdiction
The power to increase the number of judges in the


jurisdiction of the supreme court ?
(a) Dispute relating to civil matters

Supreme Court of India is vested in
(a) the President of India
(b) the Parliament .ne



(b) Dispute between two citizens from two different
states
(c) Dispute relating to criminal cases involving murder
(d) Disputes between two states of the Indian Union
65. Which High Court in India, has held that the


74.
(c) the Chief Justice of India
(d) the Law Commission t
Who among the following appoints the Judges of a High
Court?
(a) The President of India
pronouncement of Talaq thrice in one go is illegal ?
(b) The Chief Justice of India
(a) Bombay High Court
(c) The Governor of the State concerned
(b) None of these
(d) The Union Minister of Law
(c) Calcutta High Court 75. What is the number of Judges (including Chief Justice) in
(d) Allahabad High Court the Supreme Court of India as provided in the Constitution
66. A common High Court for two or more states and Union of India?
Territory may be established by (a) 20 (b) 24
(a) Parliament by Law (b) Chief Justice of India (c) 26 (d) 28
(c) President (d) Governer of the state 76. Public Interest Litigation (PIL) may be linked with
67. Under a single, integrated, hierarchical judicial system, (a) judicial review
the High Court in the states are directly under the (b) judicial activism
(a) President (b) Union Parliament (c) judicial intervention
(c) Governor of the state (d) Supreme Court (d) judicial sanctity

Downloaded From : www.EasyEngineering.net


Downloaded From : www.EasyEngineering.net
P-156 Judiciary

77. The writ of certiorari is issued by a superior court to 84. With reference to Lok Adalats, which one among the
(a) an inferior court to stop further proceedings in a following statements is correct?
particular case (a) Lok Adalats have the jurisdiction to settle the matters
(b) an inferior court to transfer the record of proceedings at pre-litigative state and not those matters pending
in a case for review before any court
(c) an officer to show his/her right to hold a particular (b) Lok Adalats can deal with matters which are civil
office and not criminal in nature
(d) a public authority to produce a person detained by it (c) Lok Adalats has not been given any statutory status
so far
before the court within 24 hours
(d) No appeal lies in a civil court against the order of
78. The Bombay High Court does not have a bench at which
the Lok Adalat
one of the following places?
85. Power of the Supreme Court of India to decide the dispute
(a) Nagpur (b) Panaji
between centre and state falls under
(c) Pune (d) Aurangabad
(a) advisory jurisdiction
79. Which one of the following jurisdictions of the Indian
(b) original jurisdiction
judiciary covers Public Interest Litigation?
(c) appellate jurisdiction
(a) Original Jurisdiction
(d) constitutional jurisdiction


ww
(b) Appellate Jurisdiction
(c) Epistolary Jurisdiction
(d) Advisory Jurisdiction
80. For which one of the following Judgements of the
86. The Judge of the High Courts in India is administered


oath of office by
(a) the Chief Justice of the High Court


w.E
Supreme Court of India, the Kesavananda Bharati vs
State of India case is considered a landmark?
(a) The religion cannot be mobilised for political ends



(b) the Governor of the State
(c) the President of India
(d) the Chief Justice of India
87. In the SR Bommai vs Union of India case, which one


a
(b) Abolishing untouchability from the country

syE
(c) The basic structures of the Constitution, as defined
in the preamble, cannot be changed
(d) Right to life and liberty cannot be suspended under
among the following features of the Constitution of India
was upheld by the Supreme Court as a basic structure?
(a) Liberalism
(b) Secularism
any circumstance
81. The original jurisdiction of the Supreme Court of India
extends to ngi

(c) Dignity of the human person
(d) Freedom of religion

nee
88. Which of I the following statements about Indian
(a) treaties and agreements signed by the Government Judiciary is not correct ?
of India (a) The Constitution of India has not provided for
(b) disputes between the Government of India and one


or more States
(c) disputes relating to implementation of the Directive
Principles of State Policy


r
double system of courts as in the United States

ing
(b) The organization of the subordinate judiciary in
India varies slightly from State to State
(c) Every State in India has separate High Court
(d) a bill passed by the Parliament which is violative of
the Constitution
82. According to the Administrative Tribunal Act, 1985. the

.ne
(d) The Supreme Court has issued direction to constitute
an All India Judicial Service to bring about
uniformity in designation of officers in criminal and


Central Administrative Tribunal adjudicates disputes and
complaints with respect to the service of persons who
are
(a) appointed to pubic services and posts in connection
civil side
t
89. In which of the following cases did the Supreme Court
rule that Constitutional - Amendments were also laws
under Article 13 of the Constitution of India, which could
with the affairs of the Union except members of the be declared void for being inconsistent with Fundamental
Defence services Rights ?
(b) official and servants of the Supreme Court or any (a) Keshavanand Bharati Case
High Courts (b) Golaknath Case
(c) members of the Secretarial staff of the Parliament or (c) Minerva Mills Case
any state legislatures (d) Maneka Gandhi Case
(d) members of the Defence services 90. The concept of public Interest litigation originated in:
83. In which one of the following cases the Supreme Court (a) United Kingdom (b) Australia
of India gave verdicts which have a direct bearing on the (c) USA (d) Canada
Centre-State relations? 91. With reference to the Consumer Disputes Redressal
(a) Keshavananda Bharati case at district level in India, which one of the following
(b) Vishakha case statements is not correct ?
(c) S R Bommai case (a) A State Government can establish more than one
(d) Indira Sawhney case District Forum in a district if it deems fit.

Downloaded From : www.EasyEngineering.net


Downloaded From : www.EasyEngineering.net
Judiciary P-157

(b) One of the members of the District Forum shall be a 99. Which one of the following is the correct sequence in
woman the descending order of precedence in the warrant of
(c) The District Forum entertains the complaints where precedence?
the value of goods or services does not exceed (a) Attorney General of India–Judges of the Supreme
rupees fifty lakhs. Court–Members the of Parliament–Deputy
(d) A complaint in relation to any goods sold or any Chairman of Rajya Sabha
service provided may be filed with a District Forum (b) Judges of the Supreme Court–Deputy Chairman of
by the State Government as a representative of the Rajya Sabha–Attorney General of India–Members
interests of the consumers in general.
of the Parliament
92. The power of the Supreme Court of India to decide
(c) Attorney General of India–Deputy Chairman
disputes between the Centre and the State falls under
of Rajya Sabha–Judges of the Supreme Court–
its:
(a) advisory jurisdiction Members of Parliament
(b) appellate jurisdiction (d) Judges of the Supreme Court–Attorney General of
(c) original jurisdiction India–Deputy Chairman of Rajya Sabha–Members
(d) constitutional jurisdiction of Parliament
93. When the Chief Justice of a High Court acts in an 100. Who was the Chief Justice of India when Public Interest


ww
administrative capacity, he is subject to :
(a) the writ jurisdiction of any other judges of the High
Court
Litigation (PIL) was introduced to the Indian judicial
system?
(a) M. Hidayatullah (b) A. M. Ahmadi



India
w.E
(b) special control exercised by the Chief Justice of

(c) discretionary powers of the Governor of the state


(d) special powers provided to the Chief Minister
(c) A. S. Anand (d) P. N. Bhagwati
101. How many High Courts in India have jurisdiction over
more than one State (Union Territories not included)?



a
judge’ shall not include:
(a) chief presidency magistrate
(b) sessions judges
syE
94. According to the Constitution of India the term ‘district
(a) 2
(c) 4
(b) 3
(d) 5
102. With reference to Lok Adalats, which of the following
statements is correct ?


(c) tribunal judge
(d) chief judge of a small cause court ngi (a) Lok Adalats have the jurisdiction to settle matters
at pre-litigating stage and not those matters pending
before any court

nee
95. The Supreme Court of India tenders advice to the
President on a matter of law or fact: (b) Lok Adalats can deal with matters which are civil
(a) on its own initiative and not criminal in nature.
(c) Every Lok Adalat consists of either serving or retired


(b) only if he seeks such advice
(c) only if the matter relates to the Fundamental Rights
of citizens
(d) only if the issue poses a threat to the unity and
r ing
judicial officers only and not any other person.
(d) None of the statements given above is correct.
103. The power of the Supreme Court of India to decide
integrity of the country
96. The salaries and allowances of the Judges of the High
Court are charged to the:
its
.ne
disputes between the Centre and the States falls under
(CSAT 2014-I)





(a) Consolidated Fund of India
(b) Consolidated Fund of the State
(c) Contingency Fund of India
(d) Contingency Fund of the State




(a) advisory jurisdiction
(b) appellate jurisdiction
(c) original jurisdiction
(d) writ jurisdiction
t
104. The power to increase the number of judges in the
97. The power to enlarge the jurisdiction of the Supreme
Supreme Court of India is vested in (CSAT 2014-I)
Court of India with respect to any matter included in the
(a) the President of India
Union List of Legislative Powers rests with:
(b) the Parliament
(a) The President of India
(b) The Chief Justice of India (c) the Chief Justice of India
(c) The Parliament (d) the Law Commission
(d) The Union Ministry of Law, Justice and Company 105. Who/Which of the following is the custodian of the
Affairs Constitution of India? (CSAT 2015-I)
98. Which one of the following High Courts has the Territorial (a) The President of India
Jurisdiction over Andaman and Nicobar Islands? (b) The Prime Minister of India
(a) Andhra Pradesh (b) Kolkata (c) The Lok Sabha Secretariat
(c) Chennai (d) Orissa (d) The Supreme court of India

Downloaded From : www.EasyEngineering.net


Downloaded From : www.EasyEngineering.net
P-158 Judiciary

Exercise - 2
6. Consider the following statements :
Statement Based MCQ 1. A person who has held office as a permanent Judge
of a High Court cannot plead or act in any court or
1. Regarding advisory jurisdiction of the Supreme Court, before any authority in India except the Supreme
which statements is/ are Not Correct? Court.
1. It is obligatory for the Supreme Court to give its 2. A person is not qualified for appointment as a Judge
opinion if it is sought. of a High Court in India unless he has for at least five
2. The advice is not binding on the President. years held a judicial office in the territory of India.
3. President may ask the Court’s opinion on treaties Which of the statements given above is/are correct?
and agreements made before the Constitution was (a) 1 only (b) 2 only
framed. (c) Both 1 and 2 (d) Neither 1 nor 2
(a) 1 and 2 (b) 1, 2 and 3 7. Which of the following statements are incorrect about the

ww
(c) 1 and 3 (d) 2 and 3 difference between the writ jurisdiction of the Supreme
2. Supreme Court is a court of record. This implies that: Court and high courts in India ?
1. It can punish for its contempt 1. The Supreme Court can issue writs not only for the
2. its decisions’ are admitted as evidence and cannot be purpose of enforcement of Fundamental Rights but

w.E
questioned by any court
3. it has to keep a record of all the important cases in
India
also for any other purpose, whereas high courts can
issue writs only for the purpose of enforcement of
Fundamental Rights.




(a) 1, 2 and 3
(c) 1, 3 and 4
a
4. its decisions, once taken, are binding upon it

(b) 1 and 2 syE


Which of the following statements is/are correct?

(d) 1, 2, 3 and 4


2. High courts can issue the writ of Injunction,
whereas the Supreme Court cannot issue the writ of
Injunction.
3. The Supreme Court can issue writs only in the case
3.


________ is/are the qualifications for a High Court judge?
1. Citizenship of India
ngi
2. Must have held a judicial office for at least ten years
of appeal, whereas high courts can issue writs only
when the party directly approaches it.
4. High courts can issue writs not only for the purpose


3. Must not be over 62 years of age
4. Must have been an advocate of a High Court for at nee
of enforcement of Fundamental Rights but also for
any other purpose, whereas the Supreme Court can
issue writs only for the purpose of enforcement of



least ten years or be an eminent jurist
Which of the following statements is/are correct?
(a) 1, 2 and 3
(c) 1, 3 and 4
(b) 1 and 2
(d) 1, 2, 3 and 4


r
Fundamental Rights.

(a) 1 and 2 ing


Which of the following statements is/are correct?
(b) 1, 2 and 3
4. Chief Justice of a High Court is appointed by the President
after consultation with
(c) 2 and 3 (d) 4 only

.ne
8. The Chief Justice of a High Court is appointed by the
President after consultation with





1. Chief Justice of India
2. Governor of the State
3. Chief Minister of the State
Which of the above is/are correct?
(a) 1 only (b) 1 and 2




1. the Chief Justice of India
2. the Governor of the state
3. the Chief Minister of the state
Which of the above is/are correct?
t
(a) 1 only (b) 1 and 2
(c) 3 only (d) 1, 2 and 3 (c) 3 only (d) 1, 2 and 3
5. In a criminal case, an appeal lies to the Supreme Court if 9. Consider the following statements regarding the advisory
the High Court: jurisdiction of the Supreme Court:
1. has convicted the accused and awarded him a death 1. The reference for advice may be made to the
sentence Supreme Court on a question of law of fact by the
2. has on appeal reversed an order for acquittal of an President of India
accused and sentenced him to imprisonment of ten 2. Disputes arising out of pre-constitution treaties and
years or more agreements excluded from the original jurisdiction
3. has withdrawn for trial before itself any case from a of the Supreme Court may also be referred to it.
subordinate court and has convicted the accused and 3. The advice given by the Supreme Court is binding
sentenced him to death on the government
Which of the following statements is/are correct? 4. One of the cases referred to the Supreme Court
(a) 1 and 3 (b) 2 and 3 for its advice was the constitutionally of the Kerala
(c) 3 only (d) 1, 2 and 3 education bill.

Downloaded From : www.EasyEngineering.net


Downloaded From : www.EasyEngineering.net
Judiciary P-159

Which of these are correct? 15. With reference to National Legal Services Authority,
(a) 1, 2 and 4 (b) 2 and 3 consider the following statements:
(c) 1 and 2 (d) 3 and 4 1. Its objective is to provide free and competent legal
10. In which of the following categories of cases the Supreme services to the weaker sections of the society on the
Court of India has the power to decide basis of equal opportunity.
1. Reference made by the President on a question of 2. It issues guidelines for the State Legal Services
law or fact Authorities to implement the legal programmes and
2. A case involving interpretation of the constitution schemes throughout the country.
3. A case involving substantial question of law of Which of the statements given above is/are correct?
general importance (a) 1 only (b) 2 only
4. A case where the constitutionally of any law has (c) Both 1 and 2 (d) Neither 1 nor 2
been challenged 16. Consider the following statements :
Which of the following statements is/are correct? 1. The Chief Justice of a High Court is appointed by
(a) 1, 2 and 3 (b) 1, 3 and 4 the Governor of the state.
(c) 1, 2 and 4 (d) 2, 3 and 4 2. Every Judge of a High Court including the Chief
11. Which of the following characteristics are essential to Justice holds office until he/she attains the age of 65
federal government ? years.


ww
1. A supreme and written constitution
2. Separation of powers and the system of checks and
balances



Which of the statements given above is/are correct ?
(a) Only 1
(c) Both 1 and 2
(b) Only 2
(d) Neither 1 nor 2




w.E
3. Distribution of powers between the centre and states
4. Fundamental Rights guaranteed to citizens
Which of the following statements is/are correct?
(a) 1 and 2 (b) 1 and 3
17. On which of the following grounds can a Judge of the



Supreme Court or a High Court be impeached?
1. Violation of the Constitution
2. Proved misbehaviour

12.


(c) 2 and 4

Courts in India:
a syE
(d) 2, 3 and 4
Consider the following statements regarding the High

1. There are eighteen High Courts in the Country.


3. Incapacity



Select the correct answer using the codes given below:
(a) 1 only
(c) 1, 2 and 3
(b) 2 only
(d) 2 and 3
2. Three of them have jurisdiction over more than one
state. ngi 18. Which of the following statements with respect to the
judiciary in India is/are correct?

nee
3. No Union territory has a High Court of its own. 1. Unlike in the United States, India has not provided
4. Judges of the High court hold office till the age of for a double system of courts.
62. 2. Under the Constitution of India, there is a single


13.
(a) 1, 2 and 4
(c) 1 and 4
(b) 2 and 4
(d) 4 only
The Supreme Court of India issued certain guidelines to
put a halt to eve-teasing. In this regard consider the
r
integrated system of courts for the Union as well as
the states.

ing
3. The organisation of the subordinate judiciary varies
slightly from state to state.


following statements:
1. women cops, in civil uniform, should be deputed at
the public places



(a) Only 1
(c) 2 and 3 .ne
Select the correct answer using the codes given below
(b) 1 and 2
(d) All of these


2. The court also ordered the states and the UTs to
form a uniform law
3. The eve-teasing can lead to violation of the
fundamental rights.

t
19. Consider the following statements regarding e-courts,
launched recently in India
1. They will facilitate hearing of cases via video
conferencing.
Which of the following statement given above are correct? 2. They will follow the same procedures that are laid
(a) 1and 2 only (b) 1 and 3 only out for the bench for hearing appeals in an open
(c) 2 and 3 only (d) 1, 2 and 3 court.
14. Consider the following statements about the judicial Which of the statement(s) given above is/are correct?
system introduced by the British in India: (a) Only 1 (b) Only 2
1. It judicially unified India. (c) Both 1 and 2 (d) Neither 1 nor 2
2. The British established a new system of law through 20. Consider the following statements :
the process of enactment and relevant interpretation The Supreme Court of India tenders advice to the
of customary laws. President of India on matters of law or fact:
3. In general the British tended to avoid the customary 1. on its own initiative (on any matter of larger public
laws of India. interest).
Which of the Statements given above are correct? 2. if he seeks such an advice.
(a) 1 and 2 only (b) 2 and 3 only 3. only if the matters relate to the Fundamental Rights
(c) 1 and 3 only (d) 1, 2 and 3 of the citizens.

Downloaded From : www.EasyEngineering.net


Downloaded From : www.EasyEngineering.net
P-160 Judiciary

Which of the statements given above is/are correct ? 26. Consider the following statements:
(a) 1 only (b) 2 only 1. The mode of removal of a Judge of a High Court in
(c) 3 only (d) 1and 2 India is same as that of removal of a Judge of the
21. Consider the following statements regarding the High Supreme Court.
Courts in India: 2. After retirement from the office, a permanent judge
1. There are eighteen High Courts in the country of a High Court cannot plead or act in any court or
2. Three of them have jurisdiction over more than one before any authority in India.
state Which of the statements given above is/are correct?
3. No Union Territory has a High Court of its own (a) 1 only (b) 2 only
4. Judges of the High Court hold office till the age of (c) Both 1 and 2 (d) Neither 1 nor 2
62 27. With reference to Lok Adalats, consider the following
Which of these statements is/are correct? statements:
(a) 1, 2 and 4 (b) 2 and 3 1. An award made by a Lok Adalat is deemed to be
(c) 1 and 4 (d) 4 only a decree of a civil court and no appeal lies against
22. Consider the following statements: there to any court.
1. The highest criminal court of the district is the Court 2. Matrimonial/Family disputes are not covered under
of District and Session Judge Lok Adalat.

ww
2. The District Judge are appointed by the Governor in
consultation with the High Courts
3. A person to be eligible for appointment as a District
Judge should be an advocate or a pleader of seven



Which of the statements given above is/are correct?
(a) 1 only
(c) Both 1 and 2
(b) 2 only
(d) Neither 1 nor 2


w.E
years’ standing or more, or an officer in judicial
service of the Union or the State
4. When the sessions judge awards a death sentence,
28. With reference to the Delimitation Commission, consider


the following statements :
1. The orders of the Delimitation Commission cannot
be challenged in a Court of Law.



carried out
a
it must be confirmed by the High Court before it is

syE
Which of the statements given above are correct?
(a) 1 and 2 (b) 2, 3 and 4
2. When the orders of the Delimitation Commission
are laid before the Lok Sabha or State Legislative
Assembly, they cannot effect any modifications in
the orders.

23.

(c) 3 and 4 (d) 1, 2, 3 and 4
Consider the following statements:
1. There are 25 High Courts in India. ngi

Which of the statements given above is/are correct?
(a) 1 only (b) 2 only

nee
2. Punjab, Haryana and the Union Territory of (c) Both 1 and 2 (d) Neither 1 nor 2
Chandigarh have a common High Court. 29. What is the provision to safeguard the autonomy of the
3. National Capital Territory of Delhi has a High Court Supreme Court of India?




of its own.
Which of the statements given above is/are correct?
(a) 2 and 3
(c) 1, 2 and 3
(b) 1 and 2
(d) 3 only


r
1. While appointing the Supreme Court Judges, the

India.
ing
President of India has to consult the Chief Justice of

2. The Supreme Court Judges can be removed by the


24.

Consider the following statements:
1. The Parliament cannot enlarge the jurisdiction of the
Chief Justice of India only.
.ne
3. The salaries of the Judges are charged on the
Consolidated Fund of India to which the legislature

t
Supreme Court of India as its jurisdiction is limited
to that conferred by the Constitution. does not have to vote.
2. The officers and servants of the Supreme Court and 4. All appointments of officers and staffs of the Supreme
High Courts are appointed by the concerned Chief Court of India are made by the Government only
Justice and the administrative expenses are charged after consulting the Chief Justice of India.
on the Consolidated fund of India. Which of the statements given above is/are correct?
Which of the statements given above is/are correct? (a) 1 and 3 (b) 3 and 4
(a) 1 only (b) 2 only (c) 4 only (d) 1, 2, 3 and 4
(c) Both 1 and 2 (d) Neither 1 nor 2 30. With reference to National Legal Services Authority,
25. Consider the following statements: consider the following statements:
1. A person who has held office as a permanent Judge 1. Its objective is to provide free and competent legal
of a High Court cannot plead or act in any court or services to the weaker sections of the society on the
before any authority in India except of the Supreme basis of equal opportunity.
Court. 2. It issues guidelines for the State Legal Services
2. A person is not qualified for appointment as a Judge Authorities to implement the legal programmes and
of a High Court in India unless he has for at least five schemes throughout the country.
years held a judicial office in the territory of India. Which of the statements given above is/are correct?
Which of the statement(s) given above is/are correct? (a) 1 only (b) 2 only
(a) 1 only (b) 2 only (c) Both 1 and 2 (d) Neither 1 nor 2
(c) Both 1 and 2 (d) Neither 1 nor 2

Downloaded From : www.EasyEngineering.net


Downloaded From : www.EasyEngineering.net
Judiciary P-161

EXERCISE-1 The principal seat of the Gauhati High Court is


at Guwahati in Assam. The court has 3 outlying
1. (c) 2. (d) 3. (a) 4. (c) 5. (c)
benches. These are:
6. (d) 7. (b) 8. (d) 9. (d) 10. (d)
The Kohima bench for Nagaland state.
11. (b) 12. (b) 13. (a) 14. (b) 15. (c)
The Aizawl bench for Mizoram state.
16. (b)
The Itanagar bench for Arunachal Pradesh state.
17. (d) The power of the Supreme Court of India to
Former benches, now full fledged high courts:
decide disputes between the centre and the states
The Imphal bench (established on 21 January 1972)
falls under its original jurisdiction. The original
(Converted to a High Court in March 2013)
jurisdiction of a court is the power to hear a case for
The Agartala bench established on 24 January 1972)
the first time, as opposed to appellate jurisdiction,
(Converted to a High Court in March 2013)
when a court has the power to review a lower court’s
The Shillong bench established on 4 September
decision. In India, the Supreme Court has exclusive
1974)(Converted to a High Court in March 2013)

ww original jurisdiction on all cases between the


Government and the States or between Government
and states on side and one or more states on other
23. (c)
24. (b) Calcutta High court has the territorial jurisdiction
over Andaman and Nicobar island. The Calcutta

w.E
side or cases between different states. In addition,
Article 32 of the Constitution of India grants original
jurisdiction to the Supreme Court on all cases
involving the enforcement of fundamental rights of
High Court is the oldest High Court in India. It was
established as the High Court of Judicature at Fort
William on 1 July 1862 under the High Courts Act, 1861.
citizens.
a syE
18. (b) Sovereignty of Indian parliament is restricted by
judicial review. Parliamentary sovereignty (also
called parliamentary supremacy or legislative
25. (c) In the verdict of Minerva Mills Vs. Union of India

26. (a)
case, Supreme Court has nullified parliament’s effort
to establish preference of all the directive principles
of state policy over Fundamental Rights.
supremacy) is a concept in the constitutional law
of some parliamentary democracies. It holds that
the legislative body has absolute sovereignty, and ngi 27. (b) The writ mandamus literally means ‘we command’.
Mandamus is a judicial remedy in the form of an
is supreme over all other government institutions,
including executive or judicial bodies. The concept
also holds that the legislative body may change
nee
order from a superior court, to any government
subordinate court, corporation, or public authority—
to do (or forbear from doing) some specific act which
or repeal any previous legislation, and so that it
is not bound by written law (in some cases, even
a constitution) or by precedent. Parliamentary
r
that body is obliged under law to do (or refrain from

ing
doing)—and which is in the nature of public duty,
and in certain cases one of a statutory duty. It cannot
be issued to compel an authority to do something
sovereignty may be contrasted with the doctrines of
separation of powers, which limits the legislature’s
scope often to general law-making, and judicial
against statutory provision.
.ne
28. (a) In Keshvanand Bharti case the Supreme Court of

19. (a)
review, where laws passed by the legislature may be
declared invalid in certain circumstances.

20. (d) The Indian constitution provides for the appointment


t
India enunciated the doctrine of basic structure.
The case originated in February 1970 when Swami
HH Sri Kesavananda Bharati, Senior Pontiff and
head of “Edneer Mutt” - a Hindu Mutt situated in
Edneer, a village in Kasaragod District of Kerala,
of ad hoc judge in supreme court, high court and
challenged the Kerala government’s attempts, under
district and session court. If at any time there should
two state land reform acts, to impose restrictions
not be a quorum of the Judges of the Supreme Court
on the management of its property. Although the
available to hold or continue any session of the Court,
state invoked its authority under Article 31, a noted
the Chief Justice of India may, with the previous
Indian jurist, Nanabhoy Palkhivala, convinced the
consent of the President and after consultation with
Swami into fighting his petition under Article 26
the Chief Justice of the High Court concerned,
(not Article 29), concerning the right to manage
request in writing the attendance at the sittings of
religiously owned property without government
the Court, as an ad hoc Judge, for such period as
interference. Even though the hearings consumed
may be necessary, of a Judge of a High Court duly
five months, the outcome would profoundly affect
qualified for appointment as a Judge of the Supreme
India’s democratic processes.
Court to be designated by the Chief Justice of India.
29. (c) In Indian constitution, the power to issue a writ
21. (b)
of ‘Habeas Corpus’ is vested only in the supreme
22. (d) Guwahati high court has the largest number of
court and the High court. A writ of habeas corpus
Benches.

Downloaded From : www.EasyEngineering.net


Downloaded From : www.EasyEngineering.net
P-162 Judiciary

is a writ (court order) that requires a person under 77. (b) Literally ‘certiorary’ means ‘to be certified’. It can
arrest to be brought before a judge or into court. The be issued by the Supreme Court or the High Court
principle of habeas corpus ensures that a prisoner for quashing the order already passed by an inferior
can be released from unlawful detention—that is, court, tribunal or quasi-judicial authority.
detention lacking sufficient cause or evidence. The 78. (c) The Bombay High Court has benches in Nagpur,
remedy can be sought by the prisoner or by another Aurangabad and Panaji.
person coming to the prisoner’s aid. This right 79. (b) PIL (Public Interest Litigation) writ petition can
originated in the English legal system, and is now be filed in Supreme Court under Article 32 only
if a question concerning the enforcement of a
available in many nations. It has historically been an
fundamental right is involved. Under Article 226, a
important legal instrument safeguarding individual
writ petition can be filed in a High court whether or
freedom against arbitrary state action.
not a Fundamental Right is involved. Thus, it comes
30. (c) under appellate jurisdiction.
31. (b) The above provision came under a writ quo 80. (c) Kesavananda Bharati vs State of Kerala(1973)is a
warranto issued by the Supreme Court of India. Quo landmark decision of the Supreme Court of India
warranto (Medieval Latin for “by what warrant?”) that outlined the Basic Structure doctrine of the
is a prerogative writ requiring the person to whom Constitution.In the case, the Supreme Court ruled

ww it is directed to show what authority they have for


exercising some right or power (or “franchise”) they
claim to hold.
that all provisions of the constitution, including
Fundamental Rights can be amended. However, the
Parliament cannot alter the basic structure of the

w.E
32. (c) The main objective advocated for appointment of
the National judicial commission is bringing about
transparency and impartiality in the appointment of
constitution like secularism, democracy, federalism,
separation of powers.
81. (b) The original jurisdiction of supreme court of India

a
judges of the highest level.
33. (b) 34. (d) 35. (a) 36. (c) 37. (c)
38. (b) 39. (d) 40. (b) 41. (a) 42. (c)
43. (d) 44. (c) 45. (c) 46. (b) 47. (d)
syE
extends to all cases between the Government of
India and the States of India or between Government
of India and states on side and one or more states on
other side or cases between different states.
82. (a) The Central Administrative Tribunal has been
48. (b) 49. (b) 50. (d) 51. (c) 52. (b)
53. (a) 54. (d) 55. (c) 56. (a) 57. (d)
58. (a) 59. (b) 60. (a) 61. (c) 62. (d)
ngi established for adjudication of disputes with respect
to recruitment and conditions of service of persons

63. (a) 64. (d) 65. (d) 66. (a) 67. (d)
68. (d) 69. (d) nee
appointed to public services and posts in connection
with the affairs of the Union or other local authorities
within the territory of India.
70. (c) According to Article 227(b), the Chief Justice of
High Court when acts in an administrative capacity,
any rule made by him shall not be inconsistent with
the provision of any law in force and requires the
r
83. (c) S. R. Bommai Vs. Union of India was a landmark

ing
judgment of the Supreme Court of India regarding
provisions of Article 356 of the Constitution of India
and related issues. This case had huge impact on
previous approval of the governor.
71. (b) According to Article 136, ‘Appeal by special leave’ .ne
Centre-State Relations. The misuse of Article 356
was stopped after this judgment. Article 356 deals
can be granted against any court or tribunal including
the military court.
72. (c) It is under original jurisdiction the supreme court
decides the disputes between centre and one or more
India.
t
with imposition of President’s Rule over a State of

84. (d) Award has the same effect as of a Civil Court


decree. The Supreme Court has held that award of
states. the Lok Adalat is as good as the decree of a Court.
The award of the Lok Adalat is fictionally deemed
73. (b) It is the Parliament which has the power to increase
to be decrees of Court and therefore the courts have
the number of judges in the Supreme Court of India.
all the powers in relation thereto as it has in relation
Parliament increased the number of judges from the
to a decree passed by itself. It was the legal services
original eight in 1950 to eleven in 1956, fourteen authority act 1987, which gave statutory status to
in 1960, eighteen in 1978, twenty-six in 1986 and Lok Adalat.
thirty-one in 2008. 85. (b) Original jurisdiction of the Supreme Court(Article
74. (a) 131): Supreme court has power to decide disputes
75. (c) The Supreme Court originally consistsd of a Chief- • between the Government of India and one or more
Justice and seven other judges. In 1985, the strength States
was increased. It comprises the chief justice and not • between the Government of India and any State or
more than 25 other judge. States on one side and one or more other States on
76. (b) Public Interest litigation (PIL) may be linked with the other
judicial activism in India. • between two or more States.

Downloaded From : www.EasyEngineering.net


Downloaded From : www.EasyEngineering.net
Judiciary P-163

86. (b) According to Article 219 of Indian Constitution 98. (b) The U.T. of Andaman and Nicobar Islands comes
(Oath or affirmation by Judges of High Courts) under the jurisdiction of high court of Calcutta under
every person appointed to be a Judge of a High its extended jurisdiction Act 1953.
Court shall, before he enters upon his office, make 99. (b) President comes first, Vice-President second, Prime
and subscribe before the Governor of the State, or Minister third and Governors of states with in their
some person appointed in that behalf by him, an respective State comes fourth in the Warrant of
oath or affirmation according to the form set out for Precedence.
the purpose in the Third Schedule. According to Indian order of precedence,
87. (b) The case of S.R.Bommai vs Union of India is Judges of the Supreme Court – Rank 9
a landmark case in the purview of the Indian Deputy Chairman of Rajya Sabha – Rank 10
Attorney General of India – Rank 11
Constitutional history relating to the proclamation
Members of Parliament – Rank 21
of emergency under Article 356 of the Constitution.
100. (d) PN Bhagwati was CJI during July 1985–Dec 1986.
The case mainly came up with the issue of the
During his tenure as CJI, PIL was introduced to the
power of the President to issue proclamation under
Indian judicial system.
Article 356 of the Constitution including the power 101. (b) Bombay HC (Maharashtra & Goa); Guwahati
to dissolve State Legislative Assemblies and also (Assam, Manipur, Meghalaya, Nagaland, Tripura,

ww issues relating to federalism and secularism as a part


of basic structure.
88. (c) Every state in India does not have a separate High
Mizoram and Arunachal Pradesh); Punjab and
Haryana HC (Punjab, Haryana)
102. (d) Cases that are pending in regular courts can be

w.E
Court .The constitution provides that parliament
may by law establish a common High Court for two
or more states and a Union Territory.
89. (a) Kesavananda Bharati. vs State of Kerala is a
transferred to a Lok Adalat if both the parties agree.
These are usually presided over by retired judges,
social Activists, or other members of the legal
profession. Lok Adalats can deal with any matter

a
landmark decision of the Supreme Court of India

syE
that outlined the Basic Structure doctrine of the
Constitution Upholding the validity of clause (4) of
article 13 and a corresponding provision in article
falling within the jurisdiction of civil, criminal etc.
103. (c) It is under original jurisdiction the supreme court
decides the disputes between centre and one or more
states.
368(3) inserted by the 24th Amendment. The Court
settled in favour of the view that Parliament has the ngi 104. (b) It is the Parliament which has the power to increase
the number of judges in the Supreme Court of India.

nee
power to amend the Fundamental Rights also. Parliament increased the number of judges from the
90. (c) PIL originated in USA. It seeks to protect and original eight in 1950 to eleven in 1956, fourteen
promote interest of the public at large. in 1960, eighteen in 1978, twenty-six in 1986 and
91. (c) The District Forum entertains the complaints where
the value of goods or services does not exceed
rupees twenty lakhs. r
thirty-one in 2008.

ing
105. (d) The Supreme Court of India is the apex court in
India. As stated by the Indian Constitution, the
function of the Supreme Court of India is that of

.ne
92. (c) According to Article 131, The SC has original
a custodian of the Constitution, a court established
jurisdiction in any dispute - (a) between the
by the authority of a federal government, and the
Government of India & one or more States; or (b)
uppermost court of appeal.
between the Government of India and any State or
States on one side and one or more other States on
the other; or (c) between two or more States
93. (a) It his administrative capacity he is subject to writ
1. (d)
6. (d)
EXERCISE-2
2. (b) 3. (a) 4. (b) 5. (a)
7. (b) 8. (b) 9. (a) 10.
t (a)
judiciary in (Article 1720–224). 11. (b)
94. (c) Under article 236 of the Constitution, The term 12. (b) There are 21 High Courts in the country, three
“District Judge” includes judge of a city civil having jurisdiction over more than one state. The
court, additional district judge, joint district judge, Judge in the High Court holds office up to 62 years
assistant district judge, chief judge of a small cause of age.
court, chief presidency magistrate, additional chief 13. (d) The Supreme Court of India ordered the state
presidency magistrate, sessions judge and assistant governments and the UTs to depute female police
sessions judge and additional sessions judge. officers in plain clothes at the public places to check
95. (b) As per provisions under Article 143 the incidents of eve-teasing.
96. (b) The salaries and allowances of the Judges of the HC 14. (a) In general the British observed customary laws of
are charged to the Consolidated Fund of the state but India.
their pensions are payable as Charged Expenditure / 15. (c) The National Legal Services Authority (NALSA)
Art 112(3). has been constituted under the Legal Services
97. (c) Such is the prerogative of the Parliament. Authorities Act, 1987 to provide free Legal Services

Downloaded From : www.EasyEngineering.net


Downloaded From : www.EasyEngineering.net
P-164 Judiciary

to the weaker sections of the society and to organize 23. (a) There were 21 High Courts in India with three new
Lok Adalats for amicable settlement of disputes. states created in 2000, having their own High Courts
In every state, State Legal Services Authority has (Chattisgarh at Bilaspur, Uttarakhand at Nainital
been constituted to give effect to the policies and and Jharkhand at Ranchi). Punjab, Haryana and
directions of the NALSA and to give free legal Chandigarh have a common HC at Chandigarh.
services to the people and conduct Lok Adalats in In the year 2013, three new High Courts in the
the State. northeast - Meghalaya, Manipur and Tripura were
16. (d) The Chief Justice of the High Court is appointed by created taking the total number of High Courts in the
the President in consultaion with the chief justice of country from 21 to 24.
India and Governor of the state concerned (Article National Capital Territory of Delhi has a High Court
217). Every judge of a High Court including the of its own which was established in the year 1966.
Chief Justice holds office until he/she attain the age 24. (b) The statement (1) is not correct as according to
of 62 years. Article 138(1) of the Constitution, Parliament can
17. (d) The Judge of the Supreme Court or a High Court can enlarge the jurisdiction and powers of the SC w.r.t.
be impeached on the basis of proved misbehaviour to any of the matters in the Union List. Whereas
and incapacity. SC’s jurisdiction w.r.t. to any other matter can be

ww
18. (d) The U.S. court system is divided into two
administratively separate systems, the federal
and the state, each of which is independent of the
enlarged by a special agreement between Government
of India and government of the concerned State.

w.E
executive and legislative branches of government.
One of the unique features of the Indian Constitution
is that, notwithstanding the adoption of a federal
25. (d) Statement 1 is incorrect because after retirement a
permanent judge of High Court shall not plead or act
in a Court or before any authority in India, except
the SC and a HC other than the HC in which he had

a
system and existence of Central Acts and State

syE
Acts in their respective spheres, it has generally
provided for a single integrated system of Courts to
administer both Union and State laws. At the apex of

held his office (Art 220).
Statement 2 is incorrect as according to Article 217,
a person is not qualified for appointment as a judge

the entire judicial system, exists the Supreme Court


of India below which are the High Courts in each ngi of a High Court in India unless he has for at least ten
years held a judicial office in the territory of India.
26. (a) Statement 2 is incorrect because after retirement a
State or group of States. Below the High Courts, lies
a hierarchy of Subordinate Courts.
19. (c) The E-courts project was established in 2005. nee
permanent judge of High Court shall not plead or act
in a Court or before any authority in India, except
the SC and a HC other than the HC in which he had
According to the project, all the courts including
taluk courts will get computerized. As per the project
in 2008, all the District courts were initialised under
r ing
held his office (Art 220).
27. (a) When statutory recognition had been given to Lok
Adalat, it was specifically provided that the award
the project. In 2010, all the District courts were
computerized. The project also includes producing
witnesses through video conferencing. The judicial .ne
passed by the Lok Adalat formulating the terms of
compromise will have the force of decree of a court

service centres are available in all court campuses.


The Public as well as the advocates can walk in
directly and ask for the case status, stage and next
hearing dates.
t
which can be executed as a civil court decree.
28. (c) Both are correct. Hence the option (c) is right.
29. (a) 1st and 3rd are correct statements thus option (a) is
right.
30. (c) The National Legal Services Authority (NALSA)
20. (b) According to Article 143 (Power of President to
has been constituted under the Legal Services
consult Supreme Court).
Authorities Act, 1987 to provide free Legal Services
21. (d) There are 21 High Courts in India. Six (Bombay,
to the weaker sections of the society and to organize
Calcutta, Guwahati, Kerala, Madras and Punjab and
Lok Adalats for amicable settlement of disputes.
Haryana High Court) of them have jurisdiction over
In every state, State Legal Services Authority has
more than one state. National Capital Territory of
been constituted to give effect to the policies and
Delhi has High Court of its own.
directions of the NALSA and to give free legal
22. (d) These provisions are given under Article 233-235 in
services to the people and conduct Lok Adalats in
the chapter of Subordinate Courts in the Constitution
the State.
of India.

Downloaded From : www.EasyEngineering.net


Downloaded From : www.EasyEngineering.net

MISCELLANEOUS 7
TOPICS Chapter

ww
Introduction

w.E
Miscellaneous chapter consists many different kinds of topics that are not connected and do not easily form a group and are of
different nature. In Likewise, different topics for examples planning commission/NITI Aayog, Finance Commission, Election
Commission, National Commission, UPSC, and many more given in the chart below are covered in this chapter.

a syE Important Commissions/Institutions

Planning
Commission, NITI
Aayog and NDC
Finance Election
Commission Commission Commission
ngi
National Union Public
Service
Staff Law
Selection Commission
Commission Commission
Lokpal
and
National
Green
Lokayukta Tribunal

SCs STs Women


nee
Composition Functional
and Role
Difference
r ing Miscellaneous

16th Lok Sabha


Elections
Political
Parties
Attorney
General
CAG
.ne
Anti
Defection
Civil
Serving

Planning Commission
•• Constituted in March 1950 by a resolution of the
••
and Pressure
Groups
Law

t
Deputy chairman is the de-facto executive (full time
functional head). Responsible for the formulation and
submission of the draft Five-year plans to the Central
Government of India on the recommendation of the cabinet. He is appointed by the Central cabinet for a fixed
Advisory Planning Board in 1946 under the chairmanship tenure and enjoys a rank of a cabinet minister. Though he
of K.C. Neogi. is not a member of cabinet, he is invited to attend all its
•• Planning Commission is neither a constitutional body meetings (without a right to vote).
nor a statutory body. It is a non-constitutional or extra-
•• Finance Minister and Planning Minister are the ex-officio
constitutional or a non-statutory body.
(by virtue of) members of the Commission.
•• Objective is to formulate 5 year plans for economic and
•• Four to seven full time expert members. The full time
social development and to advice Central government in
members enjoy the rank of a Minister of State.
this regard.
•• The Commission has a member-secretary, usually a
Composition senior member of IAS.
•• The Prime Minister of India has been the chairman of •• The state governments are not represented in the
the Commission. He presides over the meetings of the Commission in any way. Thus, the Planning Commission
Commission. is wholly a Centre-constituted body.

Downloaded From : www.EasyEngineering.net


Downloaded From : www.EasyEngineering.net
P-166 Miscellaneous Topics

Functions and Role Planning Commission is only a staff agency, an advisory body
and has no executive responsibility. Not responsible for taking
•• The planing Commission was constituted to perform
and implementing decisions. This responsibility rests with the
various functions:
Central and State governments.
•• Assessment of material, capital and human resources.
•• Assessment of material, capital and human resources of
the Country and investigate the possibilities of augmenting
NITI Aayog
them. The Government of India has replaced Planning Commission
•• Formulate a plan for effective and balanced utilization of with a new institution named NITI Aayog (National Institution
resources. for Transforming India).
•• Determine priorities and to define stages in which plan The institution will serve as ‘Think Tank’ of the Government -
should be carried out. a directional and policy dynamo.
•• Indicate factors which retard economic development. NITI Aayog will provide Governments at the Central and State
•• Determine the nature of the machinery required for Levels with relevant strategic and technical advice across the
successful implementation of 5 years plan. spectrum of key elements of policy, which includes matters of
•• To appraise the progress achieved. national and international importance on the economic front,
dissemination of best practices from within the country as well
•• Make recommendations for facilitating the discharge of

ww
its duties.
Allocation of Business Rules have assigned the following
matters:
as from other nations, the infusion of new policy ideas and
specific issue-based support.

Composition

•• Public co-operationw.E
•• Perspective planning

•• Unique Identification Authority of India (UIDAI)


NITI Aayog will have Prime Minister as its chairman, one
Vice-Chairman cum chief-executive officer, 3 full time
members and 2 part time members, apart from 4 Central

Chairperson Vice
a
constituted in January 2009 as an attached office.

Full-Time syE
Government ministers.
Present members of NITI Aayog

Ex-officio Members Special Invites


Chief Executive
Chairperson Members

ngi Officer
Shri Narendra Shri Arvind Shri Bibek Debroy. Shri Rajnath Singh, Minister Shri Nitin Gadkari, Minister Shri Amitabh
Modi, Hon'ble Panagariya Shri V.K. Saraswat. of Home Affairs. of Road Transport and Kant
Prime Minister
nee
Prof. Ramesh Chand. Shri Arun Jaitley, Minister of Highways; and Minister of
Finance; Minister of Corporate Shipping.
Affairs; and Minister of Shri Thawar Chand

r ing
Information and Broadcasting. Gehlot,Minister of Social
Shri Suresh Prabhu, Minister Justice and Empowerment.
of Railways. Smt. Smriti Zubin Irani,
Shri Radha Mohan Singh, Minister of Human Resource
Minister of Agriculture. Development.
.ne
1. Financial clout
Difference Between Planning Commission and NITI Aayog
S.No Points of Difference NITI Aayog
To be an advisory body, or a think-tank. The
powers to allocate fund vested in the finance
Planning Commission t
Enjoyed the powers to allocate funds to ministries
and state governments
ministry.
2. Full-time members Three full-time members. Had eight full-time members
3. States' role Includes the Chief Ministers of all States and the States' role was limited to the National
Lieutenant Governors of all Union territories in Development Council and annual interaction
its Governing Council, devolving more power to during plan meetings
the States of the Union.
4. Member secretary To be known as the CEO and to be appointed by Secretaries or member secretaries were appointed
the Prime Minister through the usual process
5. Part-time members To have a number of part time member, Full Planning Commission had no provision for
depending on the need from time to time part time members
6. Constitution Governing Council has state Chief Ministers and The Commission reported to National
lieutenant governors. Development Council that had State Chief
Ministers and lieutenant governors.

Downloaded From : www.EasyEngineering.net


Downloaded From : www.EasyEngineering.net
Miscellaneous Topics P-167

7. Organization New posts of CEO, of secretary rank, and Had deputy chairperson, a member secretary and
vice Chairperson. Will also have two full-time 8 full time members.
members and part-time members as per need.
Four cabinet ministers will serve as ex-officio
members.
8. Participation Consulting states while making policy and Policy was formed by the Commission and states
deciding on funds allocation. Final policy would were then consulted about allocation of funds.
be a result of that.
9. Allocations No power to allocate funds Had power to decide allocation of government
funds for various programs at national and state
levels.
10. Nature NITI is a think-tank and does not have the power Imposed policies on states and tied allocation of
to impose policies. funds with projects it approved.

National Development Council (NDC) STAGES IN MAKING OF A PLAN


Drafting of Five-Year Plan
•• It was established in August 1952 by an executive resolution

ww
of the Government of India on the recommendation of the
First Year Plan.
Prepared by the Planning Commission

Submitted to Union Cabinet for approval
••
••
w.E
It is neither a constitutional body nor a statutory body.
Sarkaria Commission recommended for its constitutional
status under article 263 of the Constitution and should
be renamed as ‘National Economic and Development

Placed before the NDC for its acceptance

Council’.

Composition
••
a syE
The NDC is composed of the following members.
Presented to the Parliament

Emerges as the official plan

ƒƒ Prime Minister of India as its chairman /head.
ƒƒ All Union cabinet ministers (since 1967). ngi ••
Published in the official gazette
NDC is the highest body, below the Parliament. However
it (the NDC) is listed as an advisory body to the Planning
ƒƒ Chief ministers of all states.
ƒƒ Administrators of all Union territories.
ƒƒ Members of the Planning Commission.
•• nee
Commission and its recommendations are not binding.
It should meet at least twice every year.

•• Secretary of the Planning Commission acts as the secretary


to the NDC. •• r
Finance Commission
ing
Article 280 provides Finance Commission as a quasi-
judicial body constituted by the President every fifth year
Objectives
•• Chief objective –To secure cooperation of states in the •• .ne
or at such earlier time as he considers necessary.
Recommendations made by the Finance Commission

••

••
execution of the plan.
To strengthen and mobilize the efforts and resources of
the nation in support of the plan.
To promote common economic policies.
government.

Composition
t
are only of advisory nature and not binding upon the

•• To ensure balanced and rapid development of all parts of •• A chairman and four other members appointed by the
the country. President.
•• They hold office for such period as specified by the
Functions President in his order. Eligible for reappointment.
•• Prescribe guidelines for preparation of the national plan. •• The Constitution authorises Parliament to determine the
•• Consider the national plan as prepared by Planning qualifications of the members and the manner in which
they should be selected. Accordingly, the Parliament
Commission.
enacted Finance Commission Act of 1951, specifying the
•• Make an assessment of the resources required for qualifications of the chairman and its members.
implementing the plan.
•• Chairman should be a person having experience in public
•• Consider important questions of social and economic affairs.
policy affecting national development. •• The four other members should be selected from the
•• Review the working of the national plan from time to time. following :
•• Recommend measures for achievement of the aims and ƒƒ Judge of high court or one to be qualified to be
targets set out in plan. appointed as one.

Downloaded From : www.EasyEngineering.net


Downloaded From : www.EasyEngineering.net
P-168 Miscellaneous Topics

ƒƒ A person having specialized knowledge of finance Composition (Article 324)


and accounts of the government.
•• Election Commission shall consist of Chief Election
ƒƒ Wide experience in financial matters and in Commissioner and such number of other Election
administration. Commissioners, as the President may from time to time
ƒƒ Special knowledge of economics. fix.
Functions •• Appointment made by the President.
•• When any other Election Commissioner is appointed the
•• The Finance Commission is required to make
Chief Election Commissioner shall act as the Chairman of
recommendations to the President of India on the
the Election Commission.
following matters :
•• President may also appoint after consultation with the
ƒƒ Distribution of the net proceeds of taxes to be shared
Election Commission, regional commissioners to assist
between the Centre and the States.
the Election Commission.
ƒƒ Principles that should govern the grants-in-aid to
the states by the Centre (i.e. out of the Consolidated •• Conditions of service and tenure of office shall be
Fund of India). determined by the President.
ƒƒ The measures needed to augment the Consolidated •• Till 1989, single member body. In 1989, two more
Fund of a State to supplement the resources of the members were added to cope with the increased work on

ww
Panchayats and the Municipalities in the state on
the basis of the recommendations made by the State
Finance Commission. This function is added by the ••
account of lowering of the voting age from 21 to 18 years
by the 61st Amendment 1989.
Chief Election Commissioner and other Election

w.E
73rd and 74th Constitutional Amendment 1992.
ƒƒ Any other matter referred to it by the President in the
interests of sound finance. ••
Commissioners have equal powers and receive equal
salary, allowances and other prerequisites.
All entitled to the same salary and other facilities as a
judge of the Supreme Court.
••

a
Commission also suggests the amounts to be paid to Assam,

syE
Bihar, Orissa and West Bengal in lieu of assignment of
any share of the net proceeds in each year of export duty
on jute and jute products (Article 273).
ƒƒ Commission submits its report to the President.
•• Term is six years or until they attain the age of 65 years,
whichever is earlier.

Independence
He lays it before both the Houses of Parliament
along with an explanatory memorandum as to the ngi •• Article 324 of the Constitution safeguard –
(i) Security of tenure i.e. Chief Election Commissioner
action taken on its recommendations (Article 281).
Finance
Commission
Chairman Operational
Duration nee
is removed in same manner and on the same grounds
as a judge of the Supreme Court. Parliament with
special majority, either on the ground of proved
First
Second
Third
K. C. Neogy
K. Santhanam
A. K. Chanda
1952–57
1957–62
1962–66
r
misbehaviour or incapacity. Thus, he does not hold

ing
his office till the pleasure of the President, though
appointed by the President.
(ii) Service conditions cannot be varied to his
Fourth
Fifth
P. V. Rajamannar
Mahaveer Tyagi
1966–69
1969–74

.ne
disadvantage after his appointment.
(iii) Other election commissioner or a regional commissioner

Sixth
Seventh
Eighth
K. Brahmananda Reddy
J. M. Shelat
Y. B. Chavan
1974–79
1979–84
1984–89
Election Commissioner.
t
removed only on the recommendation of the chief

•• Constitution has not prescribed the qualifications (legal,


educational, administrative or judicial), term and has not
debarred the retiring Election Commissioners from any
Ninth N. K. P. Salve 1989–95
further appointment by the government.
Tenth K. C. Pant 1995–2000
Eleventh A. M. Khusro 2000–2005 Powers and Functions
Twelfth C. Rangarajan 2005–2010 •• The powers and function of the Election Commission with
regard to elections to the Parliament, State Legislatures
Thirteenth Dr. Vijay L. Kelkar 2010–2015
and offices of President and Vice-President can be
Fourteenth Dr. Y. V Reddy 2015–2020 classified into three categories, :
ƒƒ Administrative
Election Commission ƒƒ Advisory
•• It is a permanent and an independent body established by ƒƒ Quasi-Judicial
the Constitution of India directly to ensure free and fair In details, these powers and functions are :
elections in the country (Art. 324). •• To determine the territorial areas of the electoral
•• Elections to Parliament, State legislatures, President and constituencies throughout the country on the basis of the
Vice-President are vested in it. Delimitation Commission Act of Parliament.

Downloaded From : www.EasyEngineering.net


Downloaded From : www.EasyEngineering.net
Miscellaneous Topics P-169

•• Decide delimitation of constituencies, allocation of seats of emergency after one year.


in Parliament and state legislatures. •• To register political parties for the purpose of elections
•• To constitute administrative machinery for conducting and grant them the status of national or state parties on the
elections, election disputes, by-elections, etc. basis of their poll performance.
•• Not concerned with the elections to panchayats and List of Chief Election Commissioners
municipalities in the states. For this, the Constitution of
Name Took Office Left Office
India (Art 243K and 243 AZ) provides for a separate state
election commission. 1 Sukumar Sen 21 March 1950 19 December 1958
•• To prepare and periodically revise electoral rolls and to 2 Kalyan Sundaram 20 December 1958 30 September 1967
register all eligible voters. 3 S. P. Sen Verma 1 October 1967 30 September 1972
•• To notify the dates and schedules of elections and to 4 Nagendra Singh 1 October 1972 6 February 1973
scrutinise nomination papers.
5 T. Swaminathan 7 February 1973 17 June 1977
•• To grant recognition to political parties and allot election
symbols to them. 6 S.L. Shakdhar 18 June 1977 17 June 1982
•• To act as a court for settling disputes related to granting 7 R.K. Trivedi 18 June 1982 31 December 1985
of recognition to political parties and allotment of election 8 R.V.S. Peri 1 January 1986 25 November 1990

••
ww
symbols to them.
To appoint officers for inquiring into disputes relating to
electoral arrangements.
9
10
Sastri
V.S. Ramadevi 26 November 1990 11 December 1990
T.N. Seshan 12 December 1990 11 December 1996
••

•• w.E
To determine the code of conduct to be observed by the
parties and the candidates at the time of elections.
To advise the president on matters relating to the
11
12
13
M.S. Gill
J.M. Lyngdoh
T.S.
12 December 1996 13 June 2001
14 June 2001 7 February 2004
8 February 2004 15 May 2005

a
disqualifications of the members of Parliament.
Krishnamurthy
••

•• syE
To advise the governor on matters relating to the
disqualifications of the members of State Legislature.
To cancel polls in the event of rigging, booth capturing,
14
15
B.B. Tandon 16 May 2005
N. Gopalaswami 30 June 2006
29 June 2006
20 April 2009

••
violence and other irregularities.
To request the President for requisitioning the staff
necessary for conducting elections. ngi 16
17
Navin Chawla
S.Y. Quraishi
21 April 2009
30 July 2010
29 July 2010
10 June 2012
••

••
To supervise the machinery of elections throughout the
country to ensure free and fair elections.
To advise the President whether elections can be held in a
18
19
20
nee
V.S. Sampath
H.S. Brahma
Nasim Zaidi
11 June 2012 15 January 2015
16 January 2015 18 April 2015
19 April 2015 Incumbent
state under President's rule in order to extend the period
Elections r ing
3 Types Procedure Basis
.ne
General
Election
By
Election
Mid term
Election
Steps
Adult
Suffrage
Geographical
Representation
t
Single Member
Territorial
1. Nomination Constituencies
Elections Election Election of Candidates
of Lok to till due to 2. Declaration of final
Sabha vacant Dissolvement list of candidates
& State seat of of Lok Sabha or
a member 3. Allotment of
Assemblies Vidhan Sabha symbols to political
of Lok before its term
Sabha or parties and Independent
is over Candidates
State
Legislative Election Manifesto
Assembly & campaign
Polling
Counting of votes
Declaration of Result

Downloaded From : www.EasyEngineering.net


Downloaded From : www.EasyEngineering.net
P-170 Miscellaneous Topics

16th LOK SABHA ELECTION, 2014 New Happenging in this Election


Major Facts– (1) 
NOTA – NOTA (None of the above) was a category
•• The elections were conducted in 9 phases from 7th
April, introduced this time for voters who could raise their voice
2014 to 12th may 2014. not to choose any representative if they dislike any of the
•• The BJP (of the NDA) achieved an absolute majority with above.
282 seats out of 543. (2) Spread Out – This was the longest tenured election that
•• Its PM condidate, Narendra Modi, took office on the 26th lasted in India.
of May, 2014 as the 15th prime minister of independent
India. (3) Aam Aadmi Party – A new born party become a major
challenge to BJP and Congress reaching a National
•• The India National Congress (of the UPA) could only
manage 44 seats. Support base for the first time.
•• The All India Anna Dravide Munnetra Kazhagam (4) BJP won majority (282 seats) of the total seats declaring
(AIADMK) party from Tamil Nadu Came a close 3rd with a clear single party majority win with a lowest vote share
37 seats. of 31% of total valid vote casted. But this win marked as
•• Name of states and No. of seats acquired: a least percentage win as compared to the Previous years.

1.
2.
ww
Bihar
States

NCT of Delhi
NDA
31
7
UPA
7
0
Other
2
0
The previous lowest vote share for a single party majority
was in 1967, when congress won 283 out of 520 seats
with 40.8% of total valid votes polled.

3.
4.
Haryana
Himachal Pradeshw.E 7
4
1
0
2
0
••

••
There was an approximate turn out at 66.4% among the
Indians.
Govt. spent 131% more on organising elections more than
5.
6.
7.
Jammu and Kashmir
Madhya Pradesh
Punjab
a 3
27
6 syE
0
2
3
3
0
4
••
••
what was spent in the 2009 election.
The total expenditure in this election was Rs. 3426 crore.
Cash and 2.25 crore litres of illicit liquor was seized by
8.
9.
Utter Pradesh
Uttarakhand
73
5
2
0
5
0
ngi the pool panel.
Voters Turn out
••

nee
10. Arunachal Pradesh 1 0 0 India recorded the highest number of voters this time
during Election 2014.
11. Assam 7 3 4
•• The voting percentage was recorded at 66.38% with 551
12. Chattisgarh 10 1 0
13. Jharkhand
14. Manipur
13
0
1
2
0
0
•• r
million people casting their ballot.

ing
Voter turnout in Varanasi was 55.56%, which is a massive
improvement from 44% in 2009.
15. Meghalaya
16. Mizoram
1
0
1
1
0
0
••

Turn out Numbers voters:
East India – 75% .ne
17. Nagaland
18. Odisha
19. Sikkim
1
1
0
0
0
0
0
20
1



South India – 72%
North India – 60%
West India – 62%
t
20. Tripura 0 0 2 List of member of the 16th Lok Sabha.
21. West Bengal 2 4 36 •• Speaker – Sumitra Mahajan.
22. Andhra Pradesh 19 2 21 •• Deputy Speaker – Thambinduri
23. Karnataka 17 9 2 •• Leader of the House – Narendra Modi
24. Kerala 0 10 10 •• Leader of the Opposition – Mallikarjun Kharge
25. Tamil Nadu 02 00 37 Leader of the House in the Rajya Sabha – Arun Jaitley
26. Goa 02 00 00 •• Secretary General – P. Sreedharan.
27. Gujarat 26 00 00
28. Maharashtra 42 06 00
29. Rajasthan 25 00 00

Downloaded From : www.EasyEngineering.net


Downloaded From : www.EasyEngineering.net
Miscellaneous Topics P-171

About NOTA •• Seek support of party leaders, legislators and bureaucracy


in vigorous pursuit of their goals.
The Election Commission of India asked the Supreme
•• Exert pressure on government in order to obtain laws
Court that to offer the voter a 'NOTA' option at the ballot
and administrative measures in favour of their specific
as its would give voters the freedom of not selecting any
interests.
undeserving candidate. The Government was not in favour
of such an idea. 'The people's Union for civil Liberties' •• Termed as a “Anonymous Empire”.
which is an NGO, filed a PIL to favour NOTA. Finally on Types of Pressure Group in India–
27th September 2013, the right to register a 'NOTA. Finally
on 27th September 2013, the right to register a 'NOTA'
vote in elections was applied by the supreme court of
India, which then ordered the election commission that then
ordered the election Commission that all voting machines
should be provided with a NOTA button so as to give voters
the option to choose 'None of the above'. The Symbol for
NOTA, a ballot paper with a black cross, is designed by
National Institute of Design, Ahmedabad. In the Indian

ww
general election, 2014, NOTA polled 1.1% of the votes,
counting to over 6 million.

Political Parties and Pressure Group


Attorney-General of India (Article 76)

Political Parties w.E •• The President shall appoint a person who is qualified to be
appointed a judge of the Supreme Court to be Attorney-
General of India. He is the highest law officer in the
Country.

a
It is group of persons who agree on some ideology and seek

syE
to capture the power and form the government on the basis of
collective leadership.
Type of Party System in India - Multi Party System
•• In other words, he must be a citizen of India and he must
have been a judge of some high court for five years or an
advocate of some high court for ten years or an eminent
Jurist, un the opinion of the president.
Functions-
(i) Recruitment of leaders.
(ii) To contest election ngi •• The term of office of the AG is not fixed by the
Constitution. Further, the Constitution does not contain
(iii) Formation of government
(iv) Formulation of laws when in power.
(v) Role of oppositions
nee
the procedure and grounds for his removal. He holds
office during the pleasure of the President. This means
that he may be removed by the President at any time.

(vi) Shaping public opinion.


(vi) Provide politely stability.
Category of Political Party in India:
r
He may also quit his office by submitting his resignation

ing
to the President. Conventionally, he resigns when the
Government (Council of Ministers) resigns or is replaced,
as he is appointed on its advice.
Category ••
.ne
It shall be the duty of the Attorney General to give advice
to the Government of India upon such legal matters, and

National Party State Party


t
to perform such other duties of a legal character, as may
from time to time be referred or assigned to him by the
President and to discharge the functions conferred on him
by or under this Constitution or any law for the time being
in force.
•• The President has assigned the following duties to the AG :
6% of valid 2% seats in lok 6% of valid 3% oftotal ƒƒ To appear on behalf of the Government of India in all
votes in 4 states sabha & members votes in state no. of seats
or more OR are elected from or 3 seats
cases in the Supreme Court in which the Government
+ + OR
3 different states 2 seats in in assembly, of India is concerned.
4 seats in lok whicheever is
sabha from any assembly ƒƒ To represent the Government of India in any reference
more
state or states made by the President to the Supreme Court under
Article 143 of the Constitution.
Pressure Groups ƒƒ To appear (when required by the Government of India)
•• Represents socio-economic and political interests of a in any high court in any case in which the Government
particular section in political system. For examples. of India is concerned.)
farmers, industrial workers, etc. •• In the performance of his duties the Attorney General
•• Organised on the basis of common goals and share similar shall have right of audience in all courts in the territory
values. of India.

Downloaded From : www.EasyEngineering.net


Downloaded From : www.EasyEngineering.net
P-172 Miscellaneous Topics

•• He has the right to speak and to take part in the proceedings Government of India or of any state, after he ceases
of both the Houses of Parliament of their joint sitting and to hold his office.
any committee of the Parliament of which he may be ƒƒ His salary and other service conditions are determined
named a member, but without a right to vote. He enjoys by the Parliament. His salary is equal to that of a
all the privileges and immunities that are available to a judge of the Supreme Court.
member of parliament.
ƒƒ Neither his salary nor his rights in respect of leave of
•• The Attorney-General shall hold office during the pleasure absence, pension or age of retirement can be altered
of the President, and shall receive such remuneration as
to his disadvantage after his appointment.
the President may determine.
ƒƒ The conditions of service of persons serving in the
•• He does not fall in the category of government servants.
Indian Audit and Accounts Department and the
Further, he is not debarred from private legal practice.
administrative powers of the CAG are prescribed by
Official Language of the Union (Article 343) the President after consultation with the CAG.
The official language of the Union shall be Hindi in ƒƒ The administrative expenses of the office of the
Devanagri script. The form of numerals to be used for the CAG, including all salaries, allowances and pensions
Official purposes of the Union shall be the international form of persons serving in that office are charged upon
of Indian numerals. the Consolidated Fund of India. Thus, they are not

ww
Notwithstanding anything in clause(1), for a period of
fifteen years from the commencement of this Constitution,
the English language shall continue to be used for all the
••
subject to the vote of Parliament.
The duties and functions of the CAG as laid down by the
Parliament and the Constitution are:

w.E
official purposes of the Union for which it was being used
immediately before such commencement:
Provided that the president may, during the said period, by
ƒƒ He audits the accounts related to all expenditure from
the Consolidated Fund of India, consolidated fund
of each state and consolidated fund of each union

a
order authorise the use of the Hindi language in addition to

syE
the English language and of the Devanagri form of numerals
in addition to the international form of Indian numerals for
any of the official purposes of the Union.
territory having a Legislative Assembly.
ƒƒ He audits all expenditure from the Contingency Fund
of India and the Public Account of India as well as
the contingency fund of each state and the public
Notwithstanding anything in this Article, Parliament may by
law provide for the use, after the said period of fifteen years, ngi account of each state.
ƒƒ He audits all trading, manufacturing, profit and
of:
(a) the English language or,
(b) the Devanagri form of numerals for such purposes as
nee
loss accounts, balance sheets and other subsidiary
accounts kept by any department of the Central
Government and state governments.
may be specified in law.

Comptroller and Auditor-General of India r


ƒƒ He audits the receipts and expenditure of the Centre

ing
and each state to satisfy himself that the rules and
procedures in that behalf are designed to secure an
•• The Constitution of India (Article 148) provides for
an independent office of the Comptroller and Auditor
General of India (CAG). He is the head of the Indian
proper allocation of revenue.
.ne
effective check on the assessment, collection and

ƒƒ He audits the receipts and expenditure of the following :

••
Audit and Accounts Department. He is the guardian of
the public purse and controls the entire financial system
of the country at both the levels, the Centre and the state.
The CAG is appointed by the President of India by a


the Central or state revenues
(b) Government companies; and
t
(a) All bodies and authorities substantially financed from

warrant under his hand and seal. He holds office for a (c) Other corporations and bodies, when so required by
period of six years or up to the age of 65 years, whichever related laws.
is earlier. He can resign any time from his office by ƒƒ He audits all transactions of the Central and
addressing the resignation letter to the President. He can governments related to debt, sinking funds, deposits,
also be removed by the President on same grounds and in advances, suspense accounts and remittance business.
the same manner as a judge of the Supreme Court. He also audits receipts, stock accounts and others,
•• The Constitution has made the following provisions to with approval of the President, or when required by
safeguard and ensure the independence of CAG: the President.
ƒƒ He is provided with the security of tenure. He can ƒƒ He audits the accounts of any other authority
be removed by the President only in accordance with when requested by the President or Governor. For
the procedure mentioned in the Constitution. Thus, example, the audit of local bodies.
he does not hold his office till the pleasure of the ƒƒ He advises the President with regard to prescription
President, though he is appointed by him. of the form in which the accounts of the Centre and
ƒƒ He is not eligible for further office, either under the the states shall be kept (Article 150).

Downloaded From : www.EasyEngineering.net


Downloaded From : www.EasyEngineering.net
Miscellaneous Topics P-173

ƒƒ He submits his audit reports relating to the accounts Commission for Scheduled Castes (under Article 338) and
of the Centre to President, who shall in turn, place National Commission for Scheduled Tribes (under Article
them before both the Houses of Parliament (Article 338-A).
151). •• The separate National Commission for SCs came into
ƒƒ He submits his audit reports relating to the accounts existence in 2004. It consists of a chairperson, a vice-
of a state to governor, who shall, in turn, place them chairperson and three other members. They are appointed
before the state legislature (Article 151). by the President by warrant under his hand and seal.
ƒƒ He ascertains and certifies the net proceeds any tax Their conditions of service and tenure of office are also
or duty (Article 279). His certificate is final. The determined by the President.
‘net proceeds' means the proceeds of a tax or a duty •• The functions of the Commission are :
minus the cost of collection. ƒƒ To investigate and monitor all matters relating to the
ƒƒ He acts as a guide, friend and philosopher of Public constitutional and other legal safeguards for the SCs
Accounts Committee of the Parliament. and to evaluate their working;
ƒƒ He compiles and maintains the accounts of the ƒƒ To inquire into specific complaints with respect to
state governments. In 1976, he was relieved of his the deprivation of rights and safeguards of the SCs;
responsibilities with regard to the compilation and ƒƒ To participate and advise on the planning process

ww
maintenance of accounts of the Central Government
due to the separation of accounts from audit, that is,
departmentalisation accounts.
of socio-economic development of the SCs and to
evaluate the progress of their development under the
Union or a state;

w.E
The CAG submits three audit reports to the president—audit
report on appropriation accounts, audit report on finance
accounts, and audit report before on public undertakings.
ƒƒ To present to the President, annually and at such
other times as it may deem fit, reports upon the
working of those safeguards;

a
The President lays these reports before both the Houses of

syE
Parliament. After this, the Public Accounts Committee
examines them and reports its findings to the Parliament.

National Commission for Scheduled


ƒƒ To make recommendations as to the measures that
should be taken by the Union or a state for the
effective implementation of those safeguards and
other measures for the protection, welfare and socio-
Castes and Scheduled Tribes
ngi economic development of the SCs; and
ƒƒ To discharge such other functions in relation
(I) The National Commission for Scheduled Castes (SCs)
It is a constitutional body in the sense that it is directly
established by Article 338 of the Constitution. On the other nee
to the protection, welfare and development and
advancement of the SCs as the president may specify.
•• The commission presents an annual report to the President.
hand, the other national commissions like the National
Commission for Women (1992), the National Commission
for Minorities (1993), the national Commission for
Backward Classes (1993), the National Human Rights

r ing
The president places all such reports before the Parliament.
The President also forwards any report of the Commission
pertaining to a state government to the state governor.
Commission (1993) and the National Commission for
protection of Child Rights (2007) are statutory bodies in .ne
The governor places it before the state legislature.
(II) National Commission for Scheduled Tribes like the
the sense that they are established by acts of the Parliament.
•• Originally, Article 338 of the Constitution provided for
the appointment of a Special Officer for Scheduled Castes
(SCs) and Scheduled Tribes (STs) to investigate all matters
t
National Commission for Schedules Castes (SCs), the
National Commission for Scheduled Tribes (STs) is
also a constitutional body in the sense that it is directly
established by Article 338-A of the Constitution.
relating to the constitutional safeguards for the SCs and •• Geographically and culturally, the STs are different from
STs and to report to the President on their working. He the SCs and their problems are also different from those of
was designated as the Commissioner for SCs and STs and SCs. In 1999, a new Ministry of Tribal Affairs was created
assigned the said duty. to provide a sharp focus to the welfare and development
•• Later the 65th Constitutional Amendment Act of 1990 of the STs. It was felt necessary that the Ministry of
provided for the establishment of a high level multi- Tribal Affairs should coordinate all activities relating to
member National Commission for SCs and STs in the the STs as it would not be administratively feasible for the
place of a single Special Officer for SCs and STs. This Ministry of Social Justice and Empowerment to perform
constitutional body replaced the Commissioner for SCs this role.
and STs as well as the Commission set up under the •• Hence, in order to safeguard the interests of the STs more
Resolution of 1987. effectively, it was proposed to set up a separate National
Again, the 89th Constitutional Amendment Act of 20036 Commission for STs by bifurcating the existing combined
bifurcated the combined National Commission for SCs National Commission for SCs and STs. This was done by
and STs into two separate bodies, namely. National passing the 89th Constitutional Amendment Act of 2003.

Downloaded From : www.EasyEngineering.net


Downloaded From : www.EasyEngineering.net
P-174 Miscellaneous Topics

This Act further amended Article 338 and inserted a new or has been an officer of the Central Government in the rank of
Article 338-A in the Constitution. a Secretary to the Government of India.
•• The separate National Commission for STs came into Recently, the NCBC has demanded that the government
existence in 2004. It consists of a chairperson, a vice- introduce quotas for OBCs in private sector.
chairperson and three other members. They are appointed
by the President by warrant under his, hand and seal. Current members
Their conditions of service and tenure of office are also •• Chairperson — Justice V.Eswaraiah
determined by the President. •• Secretary — Shri A. K. Mangotra
•• The Functions of the Commission are: •• Member — S.K.Kharventhan
ƒƒ To investigate and monitor all matters relating to the •• Member — A. K. Saini
constitutional and other legal safeguards for the STs •• Member — Shakeel-uz-Zaman Ansari
and to evaluate their working;
ƒƒ To inquire into specific complaints with respect to National Commission for Minorities
the deprivation of rights and safe guards of the STs. The Union Government set up the National Commission
ƒƒ To participate and advise on the planning process for Minorities (NCM) under the National Commission for
of socio-economic development of the STs and to Minorities Act, 1992. Five religious communities, viz;

wwevaluate the progress of their development under the


Union or a state;
ƒƒ To present to the President, annually and at such
Muslims, Christians, Sikhs, Buddhists and Zoroastrians
(Parsis) have been notified as minority communities by
the Union Government. Further vide notification detail 27th

w.E
other times as it may deem fit, reports upon the
working of those safeguards;
ƒƒ To make recommendations as to the measures that
Jan 2014, Jains have also been notified as minority community.
The Commission has one Chairperson and five Members
represented five minority communities. At present Chairperson

a
should be taken by the Union or a state for the

syE
effective implementation of those safeguards and
other measures for the protection, welfare and socio-
economic development of the STs; and
is Naseem Ahmad and the post of Vice Chairperson is vacant.
Shri Tsering Namgyal Shanoo, Prof. Farida Abdulla Khan,
Ms. Mabel Rebello, Capt. Praveen Davar and Sh. Dadi E.
Mistry are the present Members.
ƒƒ To discharge such other functions in relation
to the protection, welfare and development and ngi Andhra Pradesh , Assam , Bihar, Chattisgarh, Delhi ,
Jharkhand, Karnataka, Maharashtra, Madhya Pradesh,
advancement of the STs as the President may specify.
•• The Commission presents an annual report to the
President. nee
Manipur, Rajasthan, Tamil Nadu, Uttarakhand, Uttar Pradesh
and West Bengal have also set up State Minorities Commissions
in their respective States. Their offices are located in the State
The President places all such reports before the Parliament.
The President also forwards any report of the Commission
pertaining to a state government to the state governor.
r
capitals. The functions of these Commissions, inter-alia, are

ing
to safeguard and protect the interests of minorities provided in
the Constitution and laws enacted by Parliament and the State
The governor places it before the state legislature.

National Commission for Backward Classes


Legislatures.

National Commission for Women (NCW) .ne


A Statutory Body under the Ministry of Social
Justice & Empowerment
The Supreme Court of India in its Judgment dated 16.11.1992
Commission for Women Act, 1990.
Composition of Commission
t
(NCW is a statutory body for women established in 1992
by Government of India as per provisions made in National

in Writ Petition (Civil) No. 930 of 1990 – Indra Sawhney &


The commission consists of
Ors. Vs. Union of India and Ors., reported in (1992) Supp. 3
•• A Chairperson, to be nominated by the Central
SCC 217 directed the Govt. of India, State Governments and
Government.
Union Territory Administrations to constitute a permanent body
in the nature of a Commission or Tribunal for entertaining, •• Five members to be nominated by the Central Government
examining and recommending upon requests for inclusion and from amongst persons of ability, integrity and standing,
complaints of over-inclusion and under-inclusion in the list of who have had experience in law or legislation, trade
OBCs. unionism, management of an industry or organisation
The Act came into effect on the 2nd April, 1993. Section 3 committed towards increasing the employment potential
of the Act provides that the Commission shall consist of five of women, women’s voluntary organisations (including
Members, comprising of a Chairperson who is or has been women activists), administration, economic development,
a judge of the Supreme Court or of a High Court; a social health, education or social welfare provided that atleast one
scientist; two persons, who have special knowledge in matters member each shall be from amongst persons belonging to
relating to backward classes; and a Member-Secretary, who is the Scheduled Castes and Scheduled Tribes respectively.

Downloaded From : www.EasyEngineering.net


Downloaded From : www.EasyEngineering.net
Miscellaneous Topics P-175

•• Member-Secretary to be nominated by the Central ƒƒ Participate and advise on the planning process of
Government who shall be an expert in the field of socio-economic development of women.
Management Organisational Structure or Sociological ƒƒ Evaluate the progress of the development of women
Movement or an Officer, who is a member of a Civil under the union and any state.
Service of the Union or of All India Service or holds a
ƒƒ Inspect or cause to be inspected a jail, remand home,
Civil Post under the Union with appropriate experience.
women’s institution or other place of custody where
Functions of the Commission women are kept as prisoners or otherwise and take up
••National Commission for Women Act, 1990 has below with the concerned authorities for remedial action, of
mentioned provisions regarding the functions of National found necessary.
Commission for Women. The Commission performs all ƒƒ Fund litigation involving issues affecting a large
or any of the following functions, namely body of women.
ƒƒ Investigate and examine all matters relating to the ƒƒ Make periodical reports to the government on any
safeguard provided for women under the Constitution matter pertaining to women on various difficulties
and other laws. under which women toil.
ƒƒ Present to the Central Government, annually and at ƒƒ Any other matter which may be referred to it by

ww
such' other times as the Commission may deem fit,
reports upon the working of those safeguards.
ƒƒ Make in such reports recommendations for the
Central Government.

Union Public Service Commission


w.E
effective implementation of those safeguards for
improving the conditions of women by the Union or
any State.
(Part XIV, Article 315-323)
•• UPSC is a central recruiting agency. It is independent

a
ƒƒ Review, from time to time, the existing provisions

syE
of the Constitution and other laws affecting women
and recommend amendments there to so as to suggest
remedial legislative measures to meet any lacuna,
constitutional body. Apart from UPSC, Constitution
provides provisions for State Public Service Commission
and Joint State Public Service Commission.

inadequacies or shortcomings in such legislations.


ƒƒ Take up the cases of violation of the provisions of ngi •• JSPSC can be created for two or more states by an Act of
Parliament on the request of the legislatures of the States
concerned.
the Constitution and of other laws relating to women
with the appropriate authorities.
ƒƒ Look into complaints and take suo motu notice of
••
nee
The UPSC can also serve the needs of a State on the
request of the State Governor and with the approval of



matters relating to
(a) deprivation of women’s rights.
(b) non-implementation of laws enacted to provide states.
r
the President.

ing
Article 315 – Public Service Commissions for Union and

protection to women and also to achieve the


objective of equality and development.
.ne
Article 316 – Recruitment and conditions of service of
persons serving the Union or a state
(c) non-compliance of policy decisions, guidelines
or instructions aimed at mitigating hardships
and ensuring welfare and providing relief to
women and take up the issues arising out of
Service Commission
t
Article 317 – Removal and suspension of a member of Public

Article 319 – Prohibition as to the holding of offices by


members of the Commission on ceasing to be such members.
such matters with appropriate authorities. Article 320 – Functions of Public Service Commission
ƒƒ Call for special studies or investigations into specific
Article 321 – To extend functions of public service
problems or situations arising out of discrimination
commission
and atrocities against women and identify the
Article 323 – Reports of Public Service Commissions
constraints so as to recommend strategies for their
removal.
Composition
ƒƒ Undertake promotional and educational research so
as to suggest ways of ensuring due representation of •• Constitution does not specify the strength of the
women in all spheres and identify factors responsible Commission but has left the matter to the discretion of the
for impeding their advancement, such as, lack of President. Usually, the Commission consists of 9 to 11
access to housing and basic services, inadequate members including the Chairman.
support services and technologies for reducing •• Chairman and members have tenure of six years or until
drudgery and occupational health hazards and for age of 65 years, in the case of SPSC or JSPSC age limit
increasing their productivity. is 62 years.

Downloaded From : www.EasyEngineering.net


Downloaded From : www.EasyEngineering.net
P-176 Miscellaneous Topics

Removal Staff Selection Commission


•• Removed on the grounds of insolvent bankrupt, paid •• SSC, established in 1975 by executive resolution.
employment, infirmity of mind. Responsible for:
•• President can also remove him on the grounds for his •• All Group ‘B’ posts having the maximum pay scale of less
misbehaviour. However, in this case, President has to than Rs. 10,500.
refer the matter to the Supreme Court for an enquiry. •• All non-technical Group ‘C’ post for recruiting personnel
If the Supreme Court, upholds the cause President can to middle and lower services of Central Government.
remove him. The advice tendered by the Supreme Court
in this regard is binding on the President. During the Law Commission
course of enquiry by the Supreme Court, the President
Law Commission of India is a non-statutory body constituted by
can suspend the chairman or the member of UPSC.
the Government of India from time to time. The Commission
Independence was originally constituted in 1955 and is reconstituted every 3
years. The tenure of the 19th Law Commission ended on 31st
•• Security of tenure.
August, 2012.
•• Conditions of service determined by the President, cannot
First 1955-1958 MC Setalvad

••
ww
be varied to his disadvantage after his appointment.
Entire expenses are charged on the consolidated fund of
India.
Second
Aiyar
Third
1958-1961

1961-1964
Justice TV Venkatarama

Justice JL Kapur
••

••
w.E
Chairman is not eligible for further employment in the
Government of India or any state.
Members eligible for appointment as the chairman of
Fourth
Fifth
Sixth
1964-1968
1968-1971
1971-1974
Justice JL Kapur
KVK Sundaram, ICS
Justice Dr PB Gajendragadkar

••
UPSC or a SPSC.
a syE
Chairman or members are not eligible for reappointment
for second term.
Seventh
Eighth
Ninth
Tenth
1974-1977
1977-1979
1979-1980
1981-1985
Justice Dr PB Gajendragadkar
Justice HR Khanna
Justice PV Dixit
Justice KK Mathew
Functions
•• The UPSC performs the following functions: ngi Eleventh
Twelfth
Thirteenth
1985-1988
1988-1991
1991-1994
Justice DA Desai
Justice MP Thakkar
Justice KN Singh
••

••
Assists the states (if requested by two or more states so to
do) in framing and operating schemes of joint recruitment.
Serves all or any of the needs of a state on the request of
nee
Fourteenth
Fifteenth
1995-1997
1997-2000
Justice K Jayachandra Reddy
Justice BP Jeevan Reddy

••
the state Governor and with the approval of the President
of India.
It advises the President of India –
Sixteenth

Eighteenth
Nineteenth
r 2000-2001
Seventeenth 2003-2006
2006-2009
2009-2012 ing
Justice BP Jeevan Reddy
Justice M Jagannadha Rao
Justice AR Lakshmanan
Justice PV Reddy
ƒƒ All matters relating to methods of recruitment civil
services and for civil posts.
Twentieth 2012-2015

20th Law commission


DK Jain
.ne
ƒƒ Suitability of candidates for appointments for
promotions.
ƒƒ On all disciplinary matters regarding person serving
under the Government of India.
••
t
The 20th Law Commission was approved for a period
of 3 years from 1st January to 31st August, 2015 by the
Union Cabinet. The 20th Law Commission’s Terms of
Reference will include the following
•• The UPSC presents a report, annually, to the President on ƒƒ Recognition of new laws that are in consonance with
its performance. The President places this report before the current climate of globalised economy.
both the houses of parliament.
ƒƒ To provide equal redress of grievances to the citizens
•• The President can exclude posts, services and matters related to the field of law, exploring of suitable
from the purview of the UPSC. measures.
ƒƒ Conducting socio-economic, post-audits and
Role
examining laws affecting poor.
•• UPSC is only a central recurring agency while the ƒƒ Take measures in the service of poor by harnessing
department of personnel and training is the central the laws and legal processes in this regard.
personnel agency in India. ƒƒ Speedy clearance of arrears of cases, eliminate
•• Role of UPSC is not only limited, but also recommendations delays and reduction in costs.
made by it are only of advisory nature and hence, not ƒƒ Examination of existing laws with a motto to promote
binding on the government. ‘Equality of Gender’ and suggest amendments.

Downloaded From : www.EasyEngineering.net


Downloaded From : www.EasyEngineering.net
Miscellaneous Topics P-177

ƒƒ To probe the impact of globalisation on unemployment •• While IAS and IPS existed at the time of independence
and food security and introduce measures for IFS came into existence in 1966.
protecting the interests of the marginalised. •• Administrative control of different services is as under:
•• With a view to achieve better implementation of Law ƒƒ IAS - Ministry of Personnel
Commission Reports, the 20th Law Commission will ƒƒ IPS - Ministry of Home
have a full time Chairperson who will head it, four full ƒƒ IFS - Ministry of Environment and Forest
time Members Department of Legal Affairs (as Ex- •• An All India Service can be created by parliament
officio Member), Legislative Department (as Ex-officio under Article 312 on the basis of a resolution passed by
Member) and Secretaries and five part time members. The Rajya Sabha. Thus an All India Service can be created
commission will also consult the nodal holders Ministry/ only by an act of parliament and not by the resolution
Department(s) and other stake holders before concretising of Rajya Sabha, though such a resolution is must before
its recommendations. enacting such an Act.

Evolution of the Civil Services in India Constitutional Safeguards to Civil Servants


The beginning of a more organised form of the Civil Services •• Doctrine of Pleasure (English Common Law Doctrine) –
started when Lord Cornwallis in 1793, started covenanted A civil servant holds office during pleasure of crown and

ww
services. Indians were virtually disallowed to join these
services and only lower posts were kept open for the Indians.
Another serious effort was made by Lord Wellesley, who ••
his services can be terminated at any time without giving
reasons. Not bound by any contract of employment.
Same doctrine has been embodied in Article 310

w.E
established a college at Fort William to train civil servants
in India for a period of 3 years before assigning them any
administrative duties. ••
(not absolute) subject to the expressed provisions of
Constitution (Article 311).
Article 310: All persons (members of defense/civil

a
In 1854 Iord Macaulay Committee recommended for

syE
conducting exams for recruitment into the Civil Service.
Consequently, first ever competitive exam was held in 1855
in London. Indians could not find their way into Indian Civil
••
service/ all india service) hold office during the pleasure
of President.
Restrictions on doctrine of pleasure:
ƒƒ Certain constitutional offices have been excluded
Services due to several hurdles like the entry age was kept
very low, exams were not conducted in India, cost of living in
ngi from application of the doctrine like that of SC/
HC Judges, CAG, Chief Election Commissioner,
Chairman and members of UPSC.
London was relatively expensive.
In 1864, Satyendranath Tagore became the first Indian
to qualify for the covenanted Civil Services. The British nee
ƒƒ The doctrine cannot be exercised in derogation of
fundamental rights
Parliament passed an Act in 1870, authorising the appointment
of any Indian (of proved merit and ability) to any office or the
Civil Services without reference to the Act of 1861, which
••
r
ƒƒ Article 311 provides safeguards to civil servants.

ing
Safeguards under Art 311:
(1) There shall be no removal by subordinate authority.

.ne
reserved specific appointments to the covenanted service. Removing authority can be of coordinate rank/higher
Later on various committees (Aitchison Committee, Islington than the appointing authority.
Committee, Lee Committee, etc.) recommended for increasing (2) Reasonable opportunity to defend himself.
the representation of Indians in Civil Services. As provided in
Government of India Act, 1919 (later recommended by the
Lee Commission also) the Federal Public Service Commission
was set-up in 1926. This Commission went on to become the
••

•• Principle of Natural Justice:


ƒƒ No bias,
t
After 42nd Amendment, not heard during punishment
stage.

Union Public Service Commission after Independence. ƒƒ Can’t be Judge in his own case,
Three types of services viz, All India Services Central Services ƒƒ Both parties are heard,
and State Services were created. The original Constitution ƒƒ Notice given.
of India had recognised only two All India Services namely •• Exceptions to natural justice:
Indian Administrative Service and Indian Police Service. The ƒƒ Person dismissed/removed/reduced in rank on
Indian Administrative Service replaced the former Indian account of misconduct which has led to conviction/
Civil Service and similarly in 1951, Indian Police Service was criminal charges.
constituted in place of the Indian Police. In 1966, another ƒƒ Where it is impracticable to give civil servant such
All India Service i.e. the Indian Forest Service was created. an opportunity. But in such a case, the authority
Presently, there are three all India Civil Services namely taking such action has to record reasons.
1. Indian Administrative Service (IAS) ƒƒ If President/ Governor satisfied that in the interest
2. Indian Police Service (IPS) and of security of State, it is expedient to hold such an
3. Indian Forest Service (IFS) inquiry.

Downloaded From : www.EasyEngineering.net


Downloaded From : www.EasyEngineering.net
P-178 Miscellaneous Topics

The Lokpal and Lokayuktas •• Lokpal’s jurisdiction will cover all categories of public
servants including Group ’A’, ‘B’, ‘C’ & D officers and
The Scandinavian institution of Ombudsman created in Sweden employees of government. On complaints referred to
in 1809 is the earliest democratic institution in the world for CVC by Lokpal, CVC will send its report of preliminary
the redressal of citizens’ grievances. The Ombudsman in India enquiry in respect of Group A’ and ‘B’ officers back to
is called as Lokpal/Lokayukta. Lokpal for further decision. With respect to Group ‘C’
The idea of creating an anti-corruption ombudsman, in the and ‘D’ employees, CVC will proceed further in exercise
form of a Lokpal, was first conceptualised in 1968 in the fourth of its own powers under the CVC Act subject to reporting
Lok Sabha. Thereafter in 1971, 1977, 1985, 1989, 1996, 1998 and review by Lokpal.
and 2001 efforts were made to enact legislation to create the •• All entities receiving donations from foreign source in
institution of Lokpal, but these efforts remained unsuccessful. the context of the Foreign Contribution Regulation Act
This has been one of the few pieces of legislation in recent years (FCRA) in excess of ` 10 lakhs per year are brought
which has been extensively debated publicly and received in under the jurisdiction of Lokpal.
depth parliamentary scrutiny. Before the bill was introduced in •• Lokpal will have power of superintendence and direction
Parliament, a joint committee made up of government and civil over any investigation agency including CBI for cases
society representatives had made an attempt to draft the Bill. referred to them by Lokpal.

ww
The historic Lokpal and Lokayuktas Bill, 2011 passed by
Parliament (17th December, 2013 in Rajya Sabha and 18th
December, 2013 in Lok Sabha) paves the way for setting up of
••

••
A high powered committee chaired by the Prime Minister
will recommend selection of the Director, CBI.
Attachment and confiscation of property of public servants

w.E
the institution of Lokpal at the Centre and Lokayuktas in States
by law enacted by the respective State Legislatures within one
year of coming into force of the Act. ••
acquired by corrupt means, even while prosecution is
pending.
Clear time lines for

a
Lokpal and Lokayuktas to inquire into allegations of corruption

syE
against certain public functionaries and for matters connected
therewith or incidental thereto. The new law provides for a
mechanism for dealing with complaints of corruption against
ƒƒ Preliminary enquiry - three months extendable by
three months.
ƒƒ Investigation - six months which may be extended by

public functionaries, including those in high places.

Salient Features of the Act ngi six months at a time.


ƒƒ Trial - one year extendable by one year and, to
achieve this, special courts to be set up.
•• Establishment of the institution of Lokpal at the Centre
and Lokayuktas at the level of the States, thus providing
••
nee
Enhancement of maximum punishment under the
Prevention of Corruption Act from 7 years to 10 years.
The minimum punishment under sections 7, 8, 9 and 12 of

••
a uniform vigilance and anti-corruption road-map for the
nation, both at the Centre and the States.
The Lokpal to consist of a Chairperson and a maximum of
r ing
the Prevention of Corruption Act will now be 3 years and
the minimum punishment under section 15 (punishment
for attempt) will now be 2 years.
eight members, of which 50% shall be judicial Members.
50% of members of Lokpal shall be from amongst SC,
ST, OBCs, Minorities and Women.
Special Lokpal Courts
••
.ne
•• The selection of Chairperson and Members of Lokpal
shall be through a Selection Committee consisting of
ƒƒ Prime Minister;
t
The Central Government shall constitute a number of
special courts as recommended by the Lokpal, to hear
and decide the cases arising out of the Prevention of
Corruption Act, 1988 or under this Act. These special
courts are supposed to ensure completion of each trial
ƒƒ Speaker of Lok Sabha;
ƒƒ Leader of Opposition in the Lok Sabha; within a year of the filing of the case.
ƒƒ Chief Justice of India or a sitting Supreme Court •• The Lokpal is not meant to inquire into any complaint
Judge nominated by CJI; made against the Chairperson or any member and any
complaint against the Chairperson or a member shall be
ƒƒ An eminent jurist to be nominated by the President
made by an application by the party aggrieved, to the
of India.
president. If on trial a public servant is seen to have been
•• A Search Committee will assist the Selection Committee
involved in some corrupt practice, then he or she will be
in the process of selection. 50% of members of the Search liable of making up for the loss to the exchequer if any.
Committee shall also be from amongst SC, ST. OBCs.
•• Whoever makes a false or frivolous complaint under this
Minorities and Women.
Act shall, on conviction, be punished with imprisonment
•• Prime Minister was brought under the purview of the for at least two. The term of imprisonment can extend to
Lokpal with subject matter exclusions and specific process 5 years and with a fine not less than 25 thousand which
for handling complaints against the Prime Minister. can be raised to 2 lakhs.

Downloaded From : www.EasyEngineering.net


Downloaded From : www.EasyEngineering.net
Miscellaneous Topics P-179

ANTI-DEFECTION LAW 3. Deciding Authority


The 52nd Amendment Act of 1985 provided for the Any question regarding disqualification arising out of
disqualification of the members of Parliament and the state defection is to be decided by the presiding officer of the
legislatures on the ground of defection from one political party house. Originally, the act provided that the decision of
to another. For this purpose, it made changes in four articles the presiding officer is final and cannot be questioned in
of the Constitution and added a new schedule (the Tenth any court. The Supreme Court declared this provision as
Schedule) to the Constitution. This Act is often referred to as unconstitutional on the ground that it seeks to take away
the ‘Anti-defection Law’. the jurisdiction of the Supreme Court and the high courts.
Later, the 91st amendment act of 2003 made one change in It held that the presiding officer, while deciding a question
the provisions of the Tenth Schedule. It omitted an exception under the Tenth Schedule, function as a tribunal. Hence,
provision, i.e. disqualification on ground of defection not to his decision like that of any other tribunal, is subject to
apply in case of split. judicial review on the grounds of mala fides, perversity,
Provisions of the Act etc. But, the court rejected the contention that the vesting
of adjudicatory powers in the presiding officer is by itself
The Tenth Schedule contains the following provisions with
invalid on the ground of political bias.
respect to the disqualification of members of parliament and

ww
the State Legislatures on the ground of defection:
1. Disqualification
Members Of Political Parties
4. Rule-Making Power
The presiding officer of a house is empowered to make
rules to give effect to the provisions of the Tenth Schedule.

w.E
A member of a house belonging to any political party
becomes disqualified for being a member of the house, (a)
if he voluntarily gives up his membership of such political
All such rules must be placed before the house for 30 days.
The house may approve or modify or disapprove them.
Further, he may direct that any willful contravention by

a
party; or (b) if he votes or abstains from voting in such

syE
house contrary to any direction issued by his political
party without obtaining prior permission of such party
and such act has not been condoned by the party within

any member of such rules may be dealt with in the same
manner as a breach of privilege of the house.
According to the rules made so, the presiding officer
can take up a defection case only when he receives a
15 days.
From the above provisions it is clear that a member ngi complaint from a member of the house. Before taking
the final decision, he must give the member (against


elected on a party ticket should continue in the party and
obey the party directions.
Independent Members nee
whom the complaint has been made) a chance to submit
his explanation. He may also refer the matter to the
committee of privileges for inquiry. Hence, defection has
An independent member of a house (elected without being
set up as a candidate by any political party) becomes
disqualified to remain a member of the house if he joins
r ing
no immediate and automatic effect.

Evaluation of the Act



any political party after such election.
Nominated Members
A nominated member of a house becomes disqualified for .ne
The Tenth Schedule of the Constitution (which embodies the
Anti-defection Law) is designed to prevent the evil or mischief

being a member of the house if he joins any political party


after the expiry of six months from the date on which he
takes his seat in the house. This means that he may join
any political party within six months of taking his seat in
t
of political defections motivated by the lure of office or
material benefits or other similar considerations. It is intended
to strengthen the fabric of Indian parliamentary democracy by
curbing unprincipled and unethical political defections.
the house without inviting this disqualification. Advantages
2. Exceptions
The following can be cited as the advantages of the Anti-
The above disqualification on the ground of defection
defection Law:
does not apply in the following two cases:
(a) If a member goes out of his party as a result of a (a) It provides for greater stability in the body politic by
merger of the party with another party. A merger checking the propensity of legislators to change parties.
takes place when two-thirds of the members of the (b) It facilitates democratic realignment of parties in the
party have agreed to such merger. legislature by way of merger of parties.
(b) If a member, after being elected as the presiding (c) It reduces corruption at the political level as well as
officer of the house, voluntarily gives up the non-developmental expenditure incurred on irregular
membership of his party or rejoins it after he ceases elections.
to hold that office. This exemption has been provided (d) It gives, for the first time, a clear-cut constitutional
in view of the dignity and impartiality of this office. recognition to the existence of political parties.

Downloaded From : www.EasyEngineering.net


Downloaded From : www.EasyEngineering.net
P-180 Miscellaneous Topics

Criticism natural resources including enforcement of any legal right


relating to environment and giving relief and compensation for
Though the Anti-defection Law been hailed as a bold step
damages to persons and property and for matters connected
towards cleansing our political life and started as new epoch in
therewith or incidental thereto.
the political life of the country, it has revealed many lacunae in
its operation and failed to prevent defections in toto. It came to It is a specialised body equipped with the necessary expertise
be criticised on the following grounds: to handle environmental disputes involving multi- disciplinary
issues. The tribunal shall not be bound by the procedure laid
1. It does not make a differentiation between dissent and
down under the Code of Civil Procedure, 1908, but shall be
defection. It curbs the legislator’s right to dissent and
guided by principles of natural justice.
freedom of conscience. Thus, ‘it clearly puts party
bossism on a pedestral and sanctions tyranny of the party The tribunal’s dedicated jurisdiction in environmental matters
in the name of the party discipline’. shall provide speedy environmental justice and help reduce
the burden of litigation in the higher courts. The tribunal is
2. Its distinction between individual defection and group
mandated to make and endeavour for disposal of applications or
defection is irrational. In other words, ‘it banned only
appeals finally within 6 months of filing of the same. Initially,
retail defections and legalised wholesale defections,5.
the NGT is proposed to be set-up at five places of sittings and

ww
3. It does not provide for the expulsion of a legislator from
his party for his activities outside the legislature.
4. Its discrimination between an independent member and
will follow circuit procedure for making itself more accessible.
New Delhi is the Principal place of sitting of the Tribunal and
Bhopal, Pune, Kolkata and Chennai shall be the other four

w.E
a nominated member is illogical. If the former joins a
party, he is disqualified while the latter is allowed to do
the same.
places of sittings of the tribunal.

Members

a
5. Its vesting of decision-making authority in the presiding

syE
officer is criticised on two grounds. Firstly, he may not
exercise this authority in an impartial and objective manner
due to political exigencies. Secondly, he lacks the legal
The sanctioned strength of the tribunal is currently 10 expert
members and 10 judicial members, although the act allows
for upto 20 of each. The Chairman of the tribunal, who is the

knowledge and experience to adjudicate upon the cases.


ngi administrative head of the tribunal, also serves as a judicial
member. Every bench of the tribunal must consist of atleast
one expert member and one judicial member. The Chairman of
The National Green Tribunal
The National Green Tribunal has been established on 18th nee
the tribunal is required to be a serving or retired Chief justice
of a High Court or a Judge of the Supreme Court of India.
October, 2010 under the National Green Tribunal Act,
2010 for effective and expeditious disposal of cases relating to
environment protection and conservation of forests and other r
On 18th October, 2010, Justice Lokeshwar Singh Panta

ing
became its first Chairman. Currently, it is chaired by Justice
Swatanter Kumar since, 20th December, 2012.

.ne
t

Downloaded From : www.EasyEngineering.net


Downloaded From : www.EasyEngineering.net
Miscellaneous Topics P-181

1. In case of national emergency, the State Government 9. A Joint Public Service Commission may be set up for
(a) cannot legislate two or more states
(b) can legislate only on subjects in the Concurrent List (a) if the Parliament provides for it after Rajya Sahha
(c) can legislate on the subjects in the State List resolves to that effect
(b) only if the States are very small
(d) Legislative power is suspended
(c) if the State Legislatures approve of the decision
2. Chairman of the Finance Commission must be: (d) if the Parliament provides for it after a resolution to
(a) Qualified to be a High Court judge that effect is passed by the State
(b) A person with background of economics 10. Which statement is/ are false ?
(c) A person in public affairs (a) Supreme Court can appoint officers of the Court
(d) All of the above after consulting the UPSC
3. President’s Rule can be declared (b) Joint Public Service Commission member retires at
(a) on the written advice of the Union Cabinet 62 years

ww
(b) only on the recommendation of the Governor of the
concerned State


(c) UPSC may in case of a parliament resolution agree
to serve the needs of a State
(d) Public Service Commissions are to be consulted on

w.E
(c) if the President is satisfied that the government of matters relating to system of recruitment to civil
a state cannot be carried on in accordance with the posts
provisions of the Constitution 11. System of representation used in India for elections of the
(d) if a State ministry is defeated on the floor of the Lok Sabha and State Assemblies is

4.

a
Legislative Assembly
Impact of financial emergency excludes
syE
(a) Union getting the power to issue directions to State
to observe canons of financial propriety



(a) proportional representation
(b) territorial representation
(c) functional representation
(d) none of the above

ngi
(b) President’s power to direct a reduction in the salaries
of Supreme Court and High Court judges
12.

Constitution stipulates that
(a) Election Commission may consist of a Chief
Election Commissioner and not more than three


(c) President’s right to direct States to reserve even
money bills for his consideration
(d) President’s power to suspend fundamental rights in

nee
other Election Commissioners
(b) when any other Election Commissioner is appointed
the Chief Election Commissioner shall act as
5.


Article 19
The Constitution envisages
(a) two types of civil services administrative and police
(b) two types of civil services Indian and provincial


r
Chairman of the Election Commission

ing
(c) Election Commissioners can be removed only on the
recommendation of the Supreme Court
(d) Supreme Court may set aside an election on the
(c) three types of civil services : all-India, central and
regional 13. .ne
ground that the electoral roll was defective
Permission for the use of English for official purposes

6.

(d) three types of civil services : all-India, central and
state
Composition of the UPSC is
(a) laid down in the Constitution



by the Constitution through
(a) a Constitutional amendment
(b) an act of Parliament
(c) President’s order
t
was extended beyond the fifteen years initially allowed

(b) determined by Parliament (d) a government order


(c) determined by the President 14. The Constitution protects the interests of the Anglo-
(d) determined by the Chairman of UPSC Indian community by
7. Members of a State Public Service Commission can be (a) Reserving seats for them in the defence forces
removed by the (b) Reserving seats for them in the services and legislatures
(a) Governor on a report by the Supreme Court (c) Reserving seats for them in the Indian Army
(b) Governor on a resolution passed by the Parliament (d) authorizing the President to nominate two members
(c) President on a report by the Supreme Court of the community to the Lok Sabha
(d) President on a resolution passed by the Parliament 15. Election Commission holds election for the:
8. Resignation letter of a State Public Service Commission (a) Parliament
member is addressed to the (b) Parliament and State Legislative Assemblies
(a) President (c) Parliament, State Legislative Assemblies and State
(b) Governor of the state Legislative Councils
(c) Chief Justice of India (d) Parliament, State Legislatures and elections of
(d) Chief Minister of the state President and the Vice President

Downloaded From : www.EasyEngineering.net


Downloaded From : www.EasyEngineering.net
P-182 Miscellaneous Topics

16. Who, under the anti-defection act, is the final authority (c) only the President on the advice of the Chief Justice
to decide whether a member of Lok Sabha has incurred of the state
disqualification? (d) in the same manner as the Vice President of India
(a) Speaker of the House 24. Single-member constituency system means
(b) President of India (a) a constituency having only one candidate
(c) Central Election Commission (b) there can be only one voter in the constituency
(d) High Court of the State (c) a constituency which elects only one representative
17. After the Constitutional Amendment to the Anti-Defection though there can be many candidates.
Act in 2003, (d) a constituency having only one political party to
(a) a merger is no longer valid even if 2/3 of the contest election
members of a political party join another party 25. For which one of the following reforms was a Commission
(b) a split of a legislature party is invalid even if 1/3 of set up under the Chairmanship of Veerappa Moily by the
the party leaves Government of India?
(c) a defector has to resign and cannot stand for (a) Police Reforms
re-election till the term of the legislature ends (b) Tax Reforms
(d) all the above have been legislated (c) Reforms in Technical Education
18. National Commission for Backward Classes



ww
(a) has been set up by a Constitutional decree
(b) has three members
(c) has the powers of a civil court

26.
(d) Administrative Reforms
The personnel system of any local authority, corporate
body or public institution can be placed within the


year term w.E
(d) is a permanent body whose members enjoy a four-

19. A political party that loses recognition still retains its





jurisdiction of the UPSC by:
(a) President of India
(b) Central Ministry of Personnel
(c) Parliament




symbol:
(a) for two years
(b) for six years
a
(c) till the next elections
syE

27.


(d) Supreme Court
A Joint Public Service Commission can be created by:
(a) An order of the President
(b) A resolution of the Rajya Sabha
(d) only so far as another party does not claim it
20. For recognition as a state party, a political party ngi

(c) An act of Parliament
(d) A resolution of the concerned state legislatures
(a) must secure at least 6% of valid votes in the general
elections in the state plus two seats in the State
Assembly
28.

nee
According to the constitution of India, a new all India
service can be instituted with the initiative taken by
(a) more than two-thirds of the states


(b) must win at least six seats in the State Assembly
(c) must secure at least 2% of the valid votes polled in
the state’s elections to the Lok Sabha



r
(b) The inter-states council
(c) the Lok Sabha
(d) The Rajya Sabha ing
(d) get at least 11 MPs in the Lok Sabha
21. Which statement correctly describes the 4rth Schedule of
the Constitution of India?
29.

.ne
Which one of the following is not explicitly stated in the
Constitution of India but followed as a convention?
(a) The Finance Minister is to be a member of the lower



(a) It lists the distribution of powers between the Union
and the states
(b) It contains the languages listed in the Constitution
(c) It contains the provisions relating to administration


House

in the lower house


t
(b) The Prime Minister has to resign if he loses majority

(c) All the parts of India are to be represented in the


of tribal areas Council of Ministers
(d) It allocates seats in the Council of States (d) In the event of both the President and the Vice-
22. Proclamation of emergency by the President: President deemitting office simultaneously before
(a) Cannot apply to J&K the end of the their tenure, the Speaker of the
(b) Is effective in J&K only with concurrence of the lower house of the Parliament will officiate as the
State Legislature President.
(c) Apply to J&K after endorsement by the Governor 30. The finance commission is constituted to recommend
criteria for
(d) Has to be separately issued
(a) Framing a finance bill
23. State Election Commissioner can be removed from
(b) preparing the annual budget of union government
office:
(c) Distribution of financial resources between union
(a) only by the Governor
and the states
(b) in the same manner and grounds as a Judge of the
(d) Auditing the receipts and expenditures of the union
Supreme Court
government

Downloaded From : www.EasyEngineering.net


Downloaded From : www.EasyEngineering.net
Miscellaneous Topics P-183

31. Which of the following expenditures shall be charged on 37. Five year plan in India is finally approved by:
the Consolidated Fund of India? (a) Union Cabinet
(a) The emoluments and allowances of the President (b) President
and other expenditure relating to his office (c) Planning Commission
(b) The emoluments and allowances of the Prime (d) National Development Council
Minister and other expenditure relating to this office 38. Financial distribution between the Union and the States
(c) The salaries and allowances of the chairman and the takes places on the basis of the recommendation of
deputy chairman of the council of states (a) The Planning Commission
(d) The salaries and allowances of the speaker and the (b) The National Development Council
deputy speaker of the Lok Sabha (c) Inter-State Council
32. Whose duty is it to recommend to the President of India (d) The Finance Commission
on the issue of the distribution and allocation of the net 39. The jurisdiction of the Finance Commission does not
proceeds of taxes in the context of Centre-State fiscal extend to:
relations? (a) recommendation of the allocation of funds among
(a) Planning commission the various heads of expenditure in the Union and
(b) National Development Council State budgets
(c) Union Ministry of Finance (b) recommendation of the distribution between the

ww
(d) Finance Commission
33. With reference to Indian polity, which one of the
following statements is correct?

Union and the States of the net proceeds of taxes
(c) recommendation of the allocation to the States of the
respective shares of such tax proceeds


Parliament
w.E
(a) Planning Commission is accountable to the

(b) President can make ordinance only when either of


(d) recommendation of the principles which govern the
Grants-in-Aid of the revenue of the States out of the
Consolidated Fund of India.


a
the two Houses of Parliament is not in session

syE
(c) The minimum age prescribed for appointment as a
Judge of the Supreme Court is 40 years
(d) National Development Council is constituted of
40.


The strength of the UPSC:
(a) has been permanently fixed by the Constitution
(b) was determined by the Presidential Ordinance in
1952
(c) is determined by the Parliament
Union Finance Minister and the Chief Ministers of
all the States
34. From time to time, the Government of India approves ngi
41.
(d) is determined by the President from time to time
The members of UPSC hold office:
various Foreign Direct Investment (FDI) proposals.
Which among the following bodies recommends this
approval prior to the ministry of Finance/Cabinet

nee
(a) for a term of six years or till they attain the age of
65 years
(b) for a term of five years irrespective of the upper age



Committee on Economic Affairs?
(a) National Development Council
(b) Foreign Investment Promotion Board


r
limit

62 years
ing
(c) for a term of six years or till they attain the age of

(d) during the pleasure of the President




(c) Central Economic Intelligence Bureau
(d) Ministry of Commerce, Govt. of India
35. The Primary function of the Finance Commission in
42.
.ne
The members of the UPSC can be removed from their
office during their tenure by


India is to:
(a) Distribution of revenue between the centre and the
states
(b) Prepare the Annual Budget




(a) the President
(b) the Parliament
t
(c) the President on the report of the Parliament
(d) the President on the report of the Supreme Court of
India
(c) Advise the President on financial matters 43. The functions of the UPSC include
(d) Allocate funds to various ministries of the Union (a) conduct of examination for appointment to services
and State Governments of the Union
36. With reference to Indian polity, which one of the (b) tendering of advice to the President in matters
following statements is correct? relating to the methods recruitment to civil services
(a) National Development Council (NDC) is constituted including promotion and disciplinary actions
of Union Finance Minister and the Chief Minister of (c) tendering of advice to the President regarding the
all the States claims of a person regarding costs incurred by him
(b) Planning Commission is accountable to State in defending legal proceedings instituted against him
Legislature in respect of acts done in execution of duties
(c) President can make ordinance even when both house (d) all the above
44. The UPSC submits an annual report of its work to:
is in session
(a) the President (b) the Cabinet Secretariat
(d) The minimum age to contest election for Member of
(c) the Home Minister (d) Parliament
Parliament is 40 years

Downloaded From : www.EasyEngineering.net


Downloaded From : www.EasyEngineering.net
P-184 Miscellaneous Topics

45. A State PSC member resigns by writing a letter addressed 54. The recommendations of the Finance Commission are:
to the: (a) binding on the President
(a) President (b) UPSC Chairman (b) not binding on the President
(c) Governor (d) Chief Minister (c) generally accepted as a matter of convention
46. Why does the Constitution debar the Chairman of (d) None of the above
the UPSC from further employment either under the
55. Which of the following Central Pay Commission
Government of India or the Government of a State?
recommended that the membership of an unrecognized
(a) Because one cannot take up government post after
association by an employee should not be considered as
the age of sixty-five years
disciplinary offence?
(b) To ensure the independence of the Commission
(a) Single Commission
(c) To ensure the dignity of the office of Chairman,
UPSC (b) Veradachariar Commission
(d) To make sure that the Commission does not become (c) Rabhubir Dayal Commission
corrupt (d) Jagannath Das Commission
47. Which one of the following is not an All India Service? 56. In the constitution of India, the budget is known as the:
(a) Indian Foreign Service (a) Annual Financial Statement
(b) Economic Service (b) Annual Budget Statement

ww
(c) Indian Administrative Service
(d) Indian Police Service
48. The Comptroller and Auditor General acts as friend,


(c) Annual Revenues Statement
(d) Annual Expenditure Statement
57. Who prepares the Appropriation Accounts?



w.E
philosopher and guide of:
(a) the Public Accounts Committee
(b) the Estimate Committee



(a) The Planning Commission
(b) The Public Accounts Committee
(c) The Comptroller and Auditor-General


a
(c) the Committee on Public Undertaking
(d) All of the above
syE
49. The independence of the Election Commission has
(a) by making the removal of the Chief Election


(d) The Finance Minister.
58. Under the Comptroller and Auditor General of India
there is in each state:
(a) Accountant General


Commissioner difficult
ngi
(b) by not permitting any change in the salary and other


(b) Audit Chief General
(c) Comptroller Accountant


service conditions of the Election Commissioner
during his term
(c) both the above provisions

nee
(d) Chief of the state Accounts
59. What are the functions of the Finance Commission?
(a) The distribution between the union and states, on
(d) by none of the above provisions
50. The power to decide an election petition is vested in the
(a) Parliament
r
the net proceeds of taxes which are to be, or may be

ing
divided between them and the allocation between the
states, of the respective shares of such proceeds.



(b) Supreme Court
(c) Administrative Tribunal
(d) Election Commissioner

.ne
(b) Regarding the principles which should govern the
grants-in-aid of the revenue of the states in need of
such assistance out of the consolidated fund of India.
51. The power to set up a Administrative Tribunal for the



adjudication of disputes relating to elections vested in:
(a) presidential proclamation
(b) the appropriate legislature

(d) All of the above


t
(c) Any other matter referred to the Commission by the
President in the interest of sound finance.

60. Which among the following is an extraconstitutional


(c) special ordinances made for the purpose growth in Indian democracy ?
(d) any law made by the Election Commission (a) Attorney General of India
52. The term of office, salaries, allowances and other (b) Autonomous Districts
conditions of the Chairman and other members of (c) Political Parties
Finance Commission: (d) Deputy Chairman of the Council of States
(a) have been specified in the Constitution
61. Who among the following has a right to speak and
(b) are determined by the President
otherwise take part in proceedings of either House of
(c) are specified by the Finance Ministry
Parliament and to be a member of any parliamentary
(d) are determined by the Parliament from time to time
committee but is not entitled to vote ?
53. In which of the following states, it is constitutionally
(a) Chairman, Finance Commission
obligatory for the state to have a Minister for Tribal
Welfare? (b) The Attorney General
(a) Bihar (b) Madhya Pradesh (c) The Comptroller and Auditor General
(c) Orissa (d) All of the above (d) The Chief Election Commissioner

Downloaded From : www.EasyEngineering.net


Downloaded From : www.EasyEngineering.net
Miscellaneous Topics P-185

62. Which one among the following is not a function of the (d) for 5 years or till the age of 60 years, whichever is
Comptroller and Auditor General of India? earlier
(a) Auditing the transactions of Centre and state 70. Which one among the following is the distinguishing
Governments relating to contingency funds and factor between a pressure group and a political party?
public accounts (a) Pressure groups are confined to a few, while political
(b) Compiling the accounts of Defence parties involve larger number of people
(c) Auditing the accounts of institutions financed by the (b) Pressure groups do not seek active political power,
Government political parties do
(d) Compiling the accounts of States (c) Pressure groups do not politically motivate people,
63. Who of the following constitutes a Finance Commission while political parties do
for a State in India? (d) Political parties take political stance, while pressure
groups do not bother about political issues
(a) The President of India
71. Under which law it is prescribed that all proceedings in
(b) The Governor of the State
the Supreme Court of India shall be in English language?
(c) The Union Finance Minister (a) The Supreme Court Rules, 1966
(d) The Union Cabinet (b) A Legislation made by the Parliament
64. Who among the following can be the Chairperson of the (c) Article 145 of the Constitution of India
National Human Rights Commission? (d) Article 348 of the Constitution of India



ww
(a) A Member of either House of Parliament
(b) A Chief Justice of the Supreme Court of India
(c) A social worker actively involved in the promotion
72. Delimitation of constituencies and determination of
constituencies reserved for Scheduled Castes and
Scheduled Tribes are done by


w.E
of human rights
(d) Any sitting Judge of High Court or Supreme Court
65. Financial distribution between the Union and the State



(a) Election Commission
(b) Delimitation Commission
(c) Planning Commission



one of the following?
a
takes place on the basis of the recommendations of which

(a) The National Development Council


(b) The Inter-State Council syE
(d) Election Commission with the assistance of
Delimitation Commission
73. Which one among the following statements about the
functioning of political parties in a democracy is not
correct?


(c) The Planning Commission
(d) The Finance Commission
66. Department of Official Language (Raj Bhasha Vibhag) ngi (a) Political parties give political education to the people
(b) Political parties serve as a link between the
government and the people


comes under which one of the following Ministries?
(a) Ministry of Culture
(b) Ministry of Home Affairs


nee
(c) Political parties fight elections and try to get the
maximum number of their candidates elected
(d) None of the above


(c) Ministry of Human Resource Development
(d) Ministry of Information and Broadcasting
67. Which one of the following Commissions is not provided
in the Constitution of India?
r
74. Electoral disputes arising out of Presidential and Vice-

ing
Presidential Elections are settled by
(a) Election Commission of India

(a) Planning Commission


(b) UPSC



(b) Joint Committee of Parliament
(c) Supreme Court of India
(d) Speaker of Lok Sabha .ne


(c) Finance Commission
(d) Election Commission
68. Which one among the following is not true of the Planning
Commission?



for it
(b) through an Act of Parliament
t
75. The Planning Commission of India has been constituted
(a) under constitutional provision with specific mention

(c) through a cabinet decision in this regard


(a) It is an advisory body and makes recommendations
to the Cabinet (d) through constitutional amendment
(b) It is responsible for the execution of development 76. The power to decide the date of an election to a State
Legislative Assembly rests with the
programmes and plans
(a) President of India
(c) It is responsible for formulation of a plan for the
(b) Chief Minister and his/her Cabinet
most effective and balanced utilisation of the (c) Election Commission of India
country’s resources (d) Parliament
(d) It indicates the factors which tend to retard economic 77. The primary function of the Finance Commission in
development India is to:
69. The Chief Election Commissioner of India holds office (a) distribution of revenue between the Centre and the
for a period of States
(a) six years (b) prepare the Annual Budget
(b) during the pleasure of the President (c) advise the President on financial matters
(c) for 6 years or till the age of 65 years, whichever is (d) allocate funds to various ministries of the Union and
earlier State Governments

Downloaded From : www.EasyEngineering.net


Downloaded From : www.EasyEngineering.net
P-186 Miscellaneous Topics

78. Which Article of the Constitution provides that it shall be by the Election Commission
the endeavour of every state to provide adequate facility (b) The recommendation for election is made by the
for instruction in the mother tongue at the primary stage Election Commission and the notification for election
of education? is issued by the Home Ministry at the Centre and
(a) Article 349 (b) Article 350 Home Departments in the States
(c) Article 350-A (d) Article 351 (c) The recommendation for election is made by the
79. Which one of the following duties is not performed by Election Commission and the notification for
Comptroller and Auditor general of India? election is issued by the President and Governors of
(a) To audit and report on all expenditure from the the States concerned
Consolidated Fund of India (d) Both the exercises of making a recommendation for
(b) To audit and report on all expenditure from the election and that of issuing a notification in respect
Contingency Funds and Public Accounts of it are done by the Election Commission
(c) To audit and report on all trading, manufacturing, 83. Which of the following are the State in which the Lok
profit and loss accounts Ayukta Act includes the Chief Minister in its ambit?
(d) To control the receipt and issue of public money, (a) West Bengal and Kerala
and to ensure that the public revenue is lodged in the (b) Gujarat and Maharashtra
exchequer (c) Madhya Pradesh and Orissa

ww
80. With reference to Indian polity, which one of the
following statements is correct?
(a) Planning Commission is accountable to the

84.
(d) Rajasthan and Karnataka
If in an election to a State legislative assembly, the
candidate who is declared elected loses his deposit, it


Parliament

w.E
(b) President can make ordinance only when either of
the two Houses of Parliament is not in session
(c) The minimum age prescribed for appointment as a



means that:
(a) the polling was very poor
(b) the election was for a multi-member constituency
(c) the elected candidate’s victory over his nearest rival

a syE
Judge of the Supreme Court is 40 years
(d) National Development Council is constituted of
Union Finance Minister and the Chief Ministers of
all the States

85.
was very marginal
(d) a very large number of candidates contested the
election
Proportional representation is not necessary in a country
81. Which of the following bodies does not/do not find
mention in the Constitution? ngi
where:
(a) there are no reserved constituencies

nee
1. National Development Council (b) a two-party system has developed
2. Planning Commission (c) the first past-post system prevails
3. Zonal Councils (d) there is a fusion of presidential and parliamentary



Select the correct answer using the codes given below.
(a) 1 and 2 only
(c) 1 and 3 only
(b) 2 only
(d) 1, 2 and 3
82. Which one of the following is correct in respect of the
86.


r
forms of government

ing
State funding of elections takes place in:
(a) U.S.A and Canada
(b) Britain and Switzerland


commencement of the election process in India?
(a) The recommendation for election is made by the
government and the notification for election is issued


(c) France and Italy
(d) Germany and Austria
.ne
t

Downloaded From : www.EasyEngineering.net


Downloaded From : www.EasyEngineering.net
Miscellaneous Topics P-187

7. Which of the following political parties came into being


Statement Based MCQ
before independence?
1. Communist Party of India (Marxist)
1. Finance Commission recommends:
1. Determination of and principles guiding grant-in-aid 2. Communist Party of India
2. Economy in expenditure 3. Dravida Munnetra Kazhagam
3. Distribution of net proceeds of tax collection 4. Bharatiya Janata Party
between Centre and States Which of the above is/are correct?
Which of the following statements is/are correct? (a) 1 and 2 (b) 2 only
(a) 1 and 2 (b) 1, 2 and 3 (c) 1, 2 and 3 (d) 2 and 4
(c) 3 only (d) 1 and 3 8. The Constitution seeks to protect the interests of the
2. Which one of the following is/ are all-India services?
Scheduled Castes/Scheduled Tribes by
1. Indian Administrative Services
2. Indian Foreign Service 1. reservation of seats for them in the legislatures




ww
3. Indian Police Service
4. Indian Defence Service
Which of the above is/are correct?
(a) 1 and 2 (b) 1 and 3
2. bestowal of special rights on the state to impose
special restrictions on their rights for their
advancement


3.

(c) 2, 3 and 4
w.E
UPSC holds examinations for
1. Central Services
(d) 1, 2, 3 and 4



3. National Commission for the Scheduled Castes and
Scheduled Tribes
Which of the following statements is/are correct?
(a) 1 and 2 (b) 2 and 3





3. services
a
2. All India Services State

syE
4. State services for two or more States jointly
Which of the above is/are correct?
(a) 1, 2 and 3 (b) 2 and 4
(c) 1, 2 and 3 (d) 1 only
9. Consider the following statements about the recent
amendments to the Election Law by the Representation
of the People (Amendment) Act, 1996 :

4.
(c) 1 only (d) 1 and 2
ngi
A person is eligible to vote in the general elections if he or
she
1. Any conviction for insulting the Indian National
flag or the Constitution of India shall entail




1. is a citizen of India
2. is not less than 21 years
3. does not hold any office of profit under the Government
nee
disqualification for contesting elections to Parliament
and state legislatures for six years from the date of
conviction.



4. is not disqualified on grounds of unsound mind
Which of the above is/are correct?
(a) 1, 2 and 3 (b) 1 and 4


r
2. There is an increase in the security deposit which a

Sabha.
ing
candidate has to make to contest the election to Lok

3. A candidate cannot stand for election from more



5.
(c) 1 and 2 (d) 1, 2 and 4
A political party is recognised by the Election Commission
only if .ne
than one parliamentary constituency.
4. No election will now be countermanded on the death


1. it has engaged in political activity continuously for
five years
2. has returned at least one member of the Lok Sabha
for every 25 members of that House or any fraction
of that number elected from that State



of a contesting candidate.
Which of the above statements are correct?
(a) 2 and 3
(c) 1 and 3
(b) 1, 2 and 4t
(d) 1, 2, 3 and 4
10. Which statements are correct about the rights of the
3. polled not less than 6% of total number of valid
votes polled by its candidates during elections District Council of an autonomous district?
4. has contested elections in four or more states in Council may make regulations to
three consecutive general elections 1. license money-lending business
Which of the following statements is/are correct? 2. prescribe the maximum rate of interest to be charged
(a) 1 and 2 (b) 1, 3 and 4 by a money-lender
(c) 3 and 4 (d) 1, 2, 3 and 4 3. prevent non-Schedule Tribe residents from carrying
6. Which of the following are national political parties ? on business without a license
1. Congress (I) 4. do any of the above provided it is passed by a
2. All India Muslim League
majority of 2/3 of the total membership of the
3. Dravida Munnetra Kazhagam
4. All India Forward Bloc District Council
Which of the above is/are correct? Which of the following statements is/are correct?
(a) 1 only (b) 1, 2 and 4 (a) 1 and 2 (b) 1, 2 and 3
(c) 1 and 2 (d) 1, 3 and 4 (c) 2 and 3 (d) 1, 2, 3 and 4

Downloaded From : www.EasyEngineering.net


Downloaded From : www.EasyEngineering.net
P-188 Miscellaneous Topics

11. National Commission of Scheduled Castes: 17. According to the anti-defection law:
1. has a chairperson, a vice chairperson and four other 1. it is not possible for 1/3 of a legislature party to go
members over to an-other party claiming a ‘split’.
2. the tenure of office of the chairperson is fixed as 2. all disputes arising out of interpretation of the
three years anti-defection law are decided by the Election
Commission.
3. conditions of service of the members are as the
Which of the following statements is/are correct?
President may by rule determine
(a) 1 only (b) 2 only
Which of the following statements is/are correct? (c) Both 1 and 2 (d) Neither 1 nor 2
(a) 1 and 2 (b) 2 only 18. The UPSC derives its functions from which of the
(c) 1 and 3 (d) 3 only following sources?
12. National Commission for Scheduled Tribes 1. Constitution
1. is a constitutional body 2. Parliamentary laws
2. inquires into specific complaints with respect to 3. Executive rules and orders
deprivation of rights and safeguards of the Scheduled 4. Conventions
Tribes Which of the above is/are correct?
3. has a chairman, vice-chairman and three other (a) 1 only (b) 1 and 2
members (c) 1 and 3 (d) 1,2,3 and 4


ww
4. is to present to Parliament, an annual report upon
the working of safeguards of the Scheduled Tribes
Which of the following statements is/are correct?
19. Which of the following statements are correct?
1. The Constitution does not fix the number of members
of the UPSC.


(a) 1, 2 and 3
(c) 1 and 2
w.E (b) 2, 3 and 4
(d) 1, 2, 3 and 4
13. Consider the following statements about the minorities in
2. One-half of the members of the UPSC should be
persons who have held office under the Government
of India or of a state atleast for five years.


India:

a syE
1. The Government of India has notified five
communities, namely, Muslims, Sikhs, Christians,
Buddhists and Zoroastrians as Minorities.
2. National Commission for Minorities was given


3. The Chairman and members of the UPSC hold
office for a term of five years or until they attain the
age of 60 years.
4. The salaries and allowances of the members of the
UPSC are determined by the Parliament.
statutory status in 1993.
3. The smallest religious minority in India are
Zoroastrians. ngi


5. The entire expanses of UPSC are charged on the
Consolidated Fund of India.
Which of the following statements is/are correct?
4. Constitution of India recognises and protects


religious and linguistic minorities.
Which of these statements are correct?


nee
(a) 2, 4 and 5
(c) 2, 3 and 4
(b) l and 5
(d) 1, 4 and 5
20. Which of the following statements are correct with regard


(a) 2 and 3
(c) 2, 3 and 4
(b) 1 and 4
(d) 1, 2 and 4
14. Article 164 of the Constitution provides for a minister in
r
to the prohibition as to the holding of offices by members

ing
of Public Service Commissions on ceasing to be such
members?


charge of tribal welfare in:
1. Bihar 2. Chhattisgarh
3. Madhya Pradesh 4. Odisha

.ne
1. The Chairman of the UPSC shall be ineligible for
further employment either under the Government of
India or under the government of a state.



Which of the above is/are correct?
(a) 1, 2 and 3
(c) 2 and 3 only
(b) 2, 3 and 4
(d) 1, 2, 3 and 4
15. A political party becomes a national party if:

t
2. The Chairman of a SPSC shall be eligible for
appointment as Chairman or any other memeber
of the UPSC or as Chairman of any other SPSC,
but not for any other employment either under the
1. it is recognised as a State party in at least three Government of India or under the government of a
States state.
2. it wins 2% of seats in the Lok Sabha from at least 3. A member other than Chairman of the UPSC shall
three different States be eligible for appointment as Chairman of the
Which of the above is/are correct? UPSC or as Chairman of a SPSC, but not for any
(a) 1 and 2 (b) 1 only other employment, either under the Government of
(c) Neither 1 nor 2 (d) 2 Only India or under the government of a state.
16. To be recognised as a State party, a political party should: 4. A member other than Chairman of the SPSC shall be
1. win at least one seat for every 25 Lok Sabha seats, eligible for appointment as Chairman or any other
member of the UPSC or as Chairman of that or
or any fraction thereof.
any other SPSC, but not for any other employment
2. secure 6% of valid votes polled in that State at a
either under the Government of India or under the
general election to the House of the People government of a state.
Which of the following statements is/are correct? Which of the following statements is/are correct?
(a) 1 only (b) 2 only (a) 1 only (b) 2 and 4
(c) Both 1 and 2 (d) Neither 1 nor 2 (c) 1 and 3 (d) 1, 2, 3 and 4

Downloaded From : www.EasyEngineering.net


Downloaded From : www.EasyEngineering.net
Miscellaneous Topics P-189

21. Which of the following statements related to the Central 2. The National Commission for Minorities was given
Administrative Tribunal are correct? statutory status in 1993.
1. It is a statutory body. 3. The smallest religious minority in India are the
2. Its members are drawn from administrative Zoroastrians.
background only. 4. The Constitution of India recognizes and protects
3. It is not bound by the procedure prescribed in the religious and linguistic minorities.
code of civil procedure. Which of these statements are correct?
4. Its jurisdiction covers the members of All India Services (a) 2 and 3 (b) 1 and 4
as well as Central Services and Central Government (c) 2, 3 and 4 (d) 1, 2 and 4
posts. 27. Consider the following statements regarding the National
5. It was setup in 1985. Human Rights Commission of India:
Which of the following statements is/are correct?
1. Its Chairman must be retired Chief Justice of India.
(a) 2, 3 and 5 (b) 1 and 4
2. It has formations in each state as
(c) 1, 3, 4 and 5 (d) 2 and 3
22. Which of the following civil services find mention in the State Human Rights Commission
Constitution? 3. Its powers are only recommendatory.
1. Indian Administrative Service 4. It is mandatory to appoint a woman as a member of
2. Indian Forest Service the Commission.
3. Indian Police Service Which of the above statements are correct?



ww
4. All-India Judicial Service
5. Indian Foreign Service
Which of the above is/are correct?


(a) 1, 2, 3 and 4
(c) 2 and 3
(b) 2 and 4
(d) 1 and 3
28. Consider the following statements regarding the political


(a) 1 and 3
(c) 1, 3 and 5
w.E (b) 1,2 and 3
(d) 1,3 and 4
23. Which of the following are not the concerns of the UPSC?
1. Classification of services

parties in India.
1. The representation of the People’s Act, 1951
provides for the registration of political parties.

2. Promotion
3. Training


a
4. Disciplinary matters
5. Talent hunting syE


2. Registration of political parties is carried out by the
Election Commission.
3. A national level political party is one which is
recognized in four or more states.
4. During the 1999 general elections, there were 6



Which of the above is/are correct?
(a) 2, 4 and 5
(c) 1 and 3
(b) 1, 3 and 4
(d) 1 and 4 ngi
national and 48 state level parties recognized by the
Election Commission.
Which of these statements are correct ?
24. Which of the following are correct with regard to the


functions of the UPSC?
1. To conduct examinations for appointments to the


nee
(a) 1, 2 and 4
(c) 2 and 4
(b) 1 and 3
(d) 1, 2, 3 and 4
29. Consider the following statements with reference to

services of the Union.
2. To assist states, if requested, in framing and
operating schemes of joint recruitment for any
services for which candidates possessing special

India.
r ing
1. The Chief Election Commissioner and other election
commissioners enjoy equal powers but receive


qualifications are required.
3. To advise the Union and state governments on all
matters relating to methods of recruitment to civil

unequal salaries.

.ne
2. The Chief Election Commissioner is entitled to the
same salary as is provided to a judge of the Supreme


services and for civil posts.
4. To present, annually, to the President a report as to
the work done by it.
Which of the following statements is/are correct?

Court.
t
3. The Chief Election Commissioner shall not be
removed from his office except in like manner and
on like grounds as a judge of the Supreme Court.
(a) 1, 2 and 3 (b) 1, 3 and 4 4. The term of office of the Election Commissioner is
5 years from the date he assumes his office or till
(c) 1, 2 and 4 (d) 1, 2, 3 and 4
the day he attains the age of 62 years, whichever is
25. Consider the following functionaries:
earlier.
1. Cabinet Secretary
2. Chief Election Commissioner Which of these statements are correct ?
3. Union Cabinet Ministers (a) 1 and 2 (b) 2 and 3
4. Chief Justice of India (c) 1 and 4 (d) 2 and 4
Their correct sequence, in order of precedence is 30. Which of the following have recommended the
(a) 3, 4, 2, 1 (b) 4, 3, 1, 2 establishment of public sector corporations ?
(c) 4, 3, 2, 1 (d) 3, 4, 1, 2 1. Administrative Reforms Commission
26. Consider the following statements about the minorities in 2. Krishna Menon Committee
India: 3. Estimates Committee
1. The government of India has notified five 4. Arjun Sengupta Committee
communities, namely, Muslims, Sikhs, Christians, (a) 1 only (b) 1 and 3
Buddhists and Zoroastrians as Minorities. (c) 1 and 4 (d) 1 and 2

Downloaded From : www.EasyEngineering.net


Downloaded From : www.EasyEngineering.net
P-190 Miscellaneous Topics

31. Consider the following statements : 37. Which among the following statements with respect to
1. Central Administrative Tribunal (CAT) was set the Comptroller and Auditor General of India is/are
up during the Prime Ministership of Lal Bahadur correct?
Shastri. I. The procedure and grounds for his removal from the
2. The Members for CAT are drawn from both judicial office are the same as of a Judge of Supreme Court.
and administrative streams. II. He prescribes the form in which accounts of the
Which of the statements given above is/are correct ? Union and the States are to be kept.
(a) 1 only (b) 2 only Select the correct answer using the code given below
(c) Both 1 and 2 (d) Neither 1 nor 2 (a) I only (b)  II only
32. Which of the following can be said to be essentially the (c) Both I and II (d)  Neither I nor II
parts of ‘Inclusive Governance’? 38. Assertion (A) The number of the Members of the
1. Permitting the Non-Banking Financial Companies to Union Public Service Commission is preserved in the
do banking Constitution of India.
2. Establishing effective District Planning Committees Reason (R) The Union Public Service Commission
was constituted under the provisions in the Constitution
in all the districts
of India.
3. Increasing the government spending on public health

ww
4. Strengthening the Mid-day Meal Scheme
Select the correct answer using the codes given below :


Codes:
(a) Bath A and R are true and R is the correct explanation
of A


(a) 1 and 2
(c) 2, 3 and 4
w.E
33. Consider the following:
(b) 3 and 4
(d) 1, 2, 3 and 4



(b) Both A and R are true, but R is not the correct
explanation of A
(c) A is true, but R is false
(d) A is false, but R is true




3. Pension cases a
1. Disputes with mobile cellular companies
2. Motor accident cases

syE
For which of the above are Lok Adalats held?
39. Consider the following statements
1. The maximum number of the Judges of the Supreme
Court of India is prescribed in the Constitution of
India.


(a) 1 only
(c) 2 only
(b) 1 and 2
(d) 1, 2 and 3
34. Which of the following is/are extra-constitutional and ngi 2. The maximum number of the Members of the Union
Public Service Commission is prescribed in the
Constitution of India.
extra-legal device(s) for securing cooperation and
coordination between the States in India?


nee
Which of the statements given above is/are correct?
(a) Only 1
(c) Both 1 and 2
(b) Only 2
(d) Neither 1 nor 2




1. The National Development Council
2. The Governor’s Conference
3. Zonal Councils
4. The Inter-State Council


r
40. Consider the following statements

ing
1. The Chairman and the Members of the UPSC are
appointed by the President.
2. The Chairman and the Members of the UPSC
Codes:
(a) 1 and 2 (b) 1, 2 and 3 Government. .ne
are eligible for further employment under the

(c) 1, 2 and 4 (d) 2 only


35. Arrange the following stages in recruitment process in
proper order—
1. Selection 2. Appointment



(a) Only 1
(c) Both 1 and 2
(b) Only 2
t
Which of the statements given above is/are correct?

(d) Neither 1 nor 2


41. Consider the following statements with respect to the
Attorney General of India
3. Placement 4. Certification 1. He is appointed by the President.
5. Probation 6. Orientation 2. He must have the same qualifications as are required
(a) 1, 4, 2, 6, 3, 5 (b) 4, 1, 2, 6, 3, 5 by a Judge of High Court.
(c) 4, 1, 2, 5, 3, 6 (d) 1, 2, 4, 6, 3, 5 3. In the performance of his duties he shall have the
36. Consider the following statements right of audience in all courts of India.
Which of the statements given above is/are correct?
1. Report of the Administrative Reforms Commission,
(a) Only 1 (b)  1 and 3
headed by late Morarji Desai, recommended the (c) 2 and 3 (d)  1, 2 and 3
establishment of Lokpal and Lokayukta institutions. 42. Consider the following statements with respect to the
2. Lokpal is the highest institution in India to investigate Comptroller and Auditor General of India
corruption at higher places in Government. 1. He shall only be removed from office in like manner
Which of the statements given above is/are correct? and on the ground as a Judge of the Supreme Court.
(a) 1 only (b) 2 only 2. He shall not be eligible for further office either under
(c) Both 1 and 2 (d) Neither 1 nor 2 the Government of India or under the Government
of any State after he has ceased to hold his office.

Downloaded From : www.EasyEngineering.net


Downloaded From : www.EasyEngineering.net
Miscellaneous Topics P-191

Which of the statements given above is/are correct? Which of the statements given above is/are correct?
(a) Only 1 (b)  Only 2 (a) 1 only (b) 2 and 3 only
(c) Both 1 and 2 (d)  Neither 1 nor 2 (c) 1 and 3 only (d) 1, 2 and 3
43. Consider the following statements 49. Which of the followings political parties is/are national
1. The Anti-Defection Law bans an elected member political parties?
from voting against the explicit mandate of his/her
1. Muslim League
party.
2. The Anti-Defection provisions do not apply if one- 2. Revolutionary Socialist Party
third of the members of a party disobey the mandate 3. All India Forward Block
of the party and constitute themselves as a separate 4. Peasants and Workers Party of India
party. Codes:
Which of the statement(s) given above is/are correct? (a) 1, 2 and 3 (b) 2 and 4
(a) Only 1 (b)  Only 2 (c) 3 only (d) None
(c) Both 1 and 2 (d)  Neither 1 nor 2 50. Consider the following statements regarding the political
44. Which one among the following is not a constitutional parties in India:
body in India? 1. The Representation of the People Act, 1951 provides
(a) Comptroller and Auditor General for the registration of political parties
(b) National Commissioner for religious and Linguistic



ww Minorities
(c) National Commission for Scheduled Castes
(d) National Human Rights Commission in the eyes of


2. Registration of political parties is carried out by the
Election Commission
3. A national level political party is one which is
the law.
w.E
45. Which of the following statements relating to Comptroller
and Auditor General in India is/are correct?

recognized in four or more states
4. During the 1999 general elections, there were six
national and 48 state level parties recognised by the
Election commission

a
1. He/She is not an officer of the Parliament but an
officer under the President.

syE
2. He/She is an independent constitutional authority
not directly answerable to the House.
Select the correct answer using the codes given



Which of these statement are correct?
(a) 1, 2 and 4
(c) 2 and 4
(b)  1 and 3
(d)  1, 2, 3 and 4
51. Consider the following statements with reference to


below
(a) Only 1 (b)  Only 2 ngi
India:
1. The Chief Election Commission and other Election

nee
(c) Both 1 and 2 (d)  Neither 1 nor 2 Commissioners enjoy equal powers but receive
46. Which of the following is/are extra-constitutional unequal salaries
and extralegal device(s) for securing cooperation and 2. The Chief Election Commissioner is entitled to the




coordination between the States in India?
1. The National Development Council
2. The Governor’s Conference
3. Zonal Councils
r
same salary as is provided to a judge of the Supreme
Court

ing
3. The Chief Election Commissioner shall not be
removed from his office except in like manner and


4. Inter-State Council
Codes:
(a) 1 and 2 (b)  1, 2 and 3

.ne
on like grounds as a judge of the Supreme Court
4. The term of office of the Election Commissioner is
five years from the date he assumes his office or till


(c) 3 and 4

General of India:
(d)  4 only
47. Consider the following statements about the Attorney

1. He is appointed by the President of India




Which of these statements are correct?
(a) 1 and 2 (b)  2 and 3
t
the day he attains the age of 62 years, whichever is
earlier

2. He must have the same qualifications as are required


(c) l and 4 (d)  2 and 4
for a judge of the Supreme Court
3. He must be a member of either House of Parliament 52. Consider the following statements:
4. He can be removed by impeachment by Parliament The function(s) of the Finance commission is/are:
Which of these statements are correct? 1. to allow the withdrawal of the money out of the
(a) 1 and 2 (b) 1 and 3 Consolidated Fund of India
(c) 2, 3 and 4 (d) 3 and 4 2. to allocate between the States the shares of proceeds
48. Consider the following statements: of taxes
1. National Development Council is an organ of the 3. to consider applications for grants-in-aid from States
Planning Commission. 4. to supervise and report on whether the Union and
2. The Economic and Social Planning is kept in the State governments are levying taxes in accordance
Concurrent List in the Constitution of India. with the budgetary provisions
3. The Constitution of India prescribes that Panchayats Which of these statements is/are correct?
should be assigned the task of preparation of plans (a) Only 1 (b)  2 and 3
for economic development and social justice. (c) 3 and 4 (d)  1, 2 and 4

Downloaded From : www.EasyEngineering.net


Downloaded From : www.EasyEngineering.net
P-192 Miscellaneous Topics

53. Consider the following tasks: 2. report on the working of the welfare programmes
1. Superintendence, direction and conduct of free and for the SCs and the STs in the Union Territories.
fair elections Select the correct answer using the codes given below
2. Preparation of electoral rolls for all elections to the (a) Only 1 (b)  Only 2
Parliament, state Legislatures and the Office of the (c) Both 1 and 2 (d)  Neither 1 nor 2
President and the Vice-President
3. Giving recognition to political, parties and allotting Matching Based MCQ
election symbols to political parties and individuals
contesting the election. DIRECTIONS (Qs. 58 and 61) : Match List-I with List-II
4. Proclamation of final verdict in case of election and select the correct answer using the codes given below the
disputes
lists.
Which of the above are the functions of the Election
Commission of India? 58. List-I List-II
(a) 1, 2 and 3 (b)  2, 3 and 4 (A) Schedule (1) Oath or affirmation
(c) 1 and 3 (d)  1, 2 and 4 (B) Schedule (2) Details of seat allotted to
54. Consider the following statements: Three   States in Rajya Sabha
1. The Advocate General of a State in India is appointed (C) Schedule Four (3) Administration and control
by the President of India upon the recommendation of scheduled areas

ww of the Governor of the concerned State.


2. As provided in Civil Procedure Code, High Courts
have original, appellate and advisory jurisdiction at



(D) Schedule Five (4) Salaries of President
(a) A – 1 ; B – 2 ; C – 3 ; D – 4
(b) A – 2 ; B – 3 ; C – 4 ; D – 1


(a) 1 only w.E
the State level.
Which of the statements given above is/are correct?
(b)  2 only


(c) A – 4 ; B – 1 ; C – 2 ; D – 3
(d) A – 4 ; B – 2 ; C – 1 ; D – 3
59. List I (Events)
(A) Formation of the
List II (Years)
(1) 1990
(c) Both 1 and 2

a (d)  Neither 1 nor 2

syE
55. In India, other than ensuring that public funds are
used efficiently and for intended purpose, what is the
importance of the office of the Comptroller and Auditor
General (CAG)?

Party



Bharatiya Janata

(B) Acceptance of
the Mandal
Commission report
(2) 1980

1. CAG exercises exchequer control on behalf of the


Parliament when the President of India declares ngi

(C) Formation of the

government
first communist
(3) 1957

nee
national emergency/financial emergency.
2. CAG reports on the execution of projects or in an Indian state
programmes by the ministries are discussed by the (D) Passing of the 42nd (4) 1976
Public Accounts Committee. Amendment Act (5) 1947
3. Information from CAG reports can be used by
investigating agencies to frame charges against
those who have violated the law while managing
A B C D
(a) 1
(b) 1
(c) 2
r 2
2
1 ing4
3
3
5
4
4

public finances.
4. While dealing with the audit and accounting of
government companies, CAG has certain judicial
(d) 2
60. Match the following–
1 4 5
.ne



powers for prosecuting those who violate the law.
Which of the statements given above is/are correct?
(a) 1, 3 and 4 only
(c) 2 and 3 only
(b)  2 only
(d)  1, 2, 3 and 4
List-I List-II
A. Third Pay Commission 1. Varadachariar
B. First Pay Commission
C. Fourth Pay Commission 3. Jagannath Das
D. Second Pay Commission 4. Singhal Rajamannar
t
2. Raghubir Dayal

56. Consider the following statements : Attorney General of


Codes:
India can A B C D
1. take part in the proceedings of the Lok Sabha (a) 1 3 4 2
2. be a member of a committee of the Lok Sabha (b) 3 2 1 4
3. speak in the Lok Sabha (c) 2 1 4 3
4. vote in the Lok Sabha (d) 2 3 4 1
Which of the statements given above is/are correct?
61. Consider the following statements about the Attorney-
(a) 1 only (b)  2 and 4
General of India?
(c) 1, 2 and 3 (d)  1 and 3 only
1. He is appointed by the President of India
57. The Departmental Committee of the Parliament of India
2. He has the right to take part in the proceeding of the
on the welfare of the Schedule Castes (SCs) and the
Parliament.
Scheduled Tribes (STs) shall
3. He has the right of audience in all courts in India.
1. examine whether the Union Government has secured
due representation of the SCs and the STs in the Which of the statements given above are correct?
services and posts under its control. (a) 1 and 2 (b) 1 and 3
(c) 2 and 3 (d) All of these

Downloaded From : www.EasyEngineering.net


Downloaded From : www.EasyEngineering.net
Miscellaneous Topics P-193

EXERCISE-1 of India. The Planning Commission is neither


1. (d) 2. (c) 3. (c) 4. (d) 5. (d) constitutional nor a statutory body. In India, it is the
6. (c) 7. (c) 8. (b) 9. (d) 10. (c) supreme organ of planning for social and economic
11. (b) 12. (b) 13. (b) 14. (d) 15. (d) development. It is not responsible for taking and
16. (a) 17. (b) 18. (c) 19. (b) 20. (a) implementing decisions. It is only a staff agency, an
21. (d) 22. (b) 23. (b) 24. (c) 25. (d) advisory body and has no executive responsibility.
26. (c) 27. (c) 28. (d) 29. (c) 30. (c) 69. (c) The Chief Election Commissioner holds office for a
31. (c) term of six years or until they attain the age of 65
32. (d) An instrument which the constitution has evolved years, whichever is earlier.
for the purpose of distributing financial resources 70. (b) Pressure groups do not seek active political power,
between the centre and the states is the finance political parties do. Political parties exist to gain
commission. According to Article 280 of the power over governmental policy by winning
constitution, it is to be constituted by the president elections for political office. Interest groups do

ww once five year and consist of a chairman and four


other members appointed by the president.
33. (a) 34. (b) 35. (a) 36. (a) 37. (d)
38. (d) 39. (a) 40. (d) 41. (a) 42. (d)
not necessarily have their members run for office
and they vote in a nonpartisan way, supporting
candidates who promote their point of view.

w.E
43. (d) 44. (a) 45. (c) 46. (b) 47. (b)
48. (a) 49. (c) 50. (c) 51. (b) 52. (d)
53. (d) 54. (c) 55. (d) 56. (a) 57. (c)
58. (a) 59. (d)
71. (d) Article 348 of Indian Constitution mentions the
language to be used in Supreme Court and the High
Courts. According to the article 348, the language
of all proceedings in the Supreme court and in every

a
60. (d) Deputy Chairman of the council of states is an extra-

syE
constitutional growth in Indian democracy.
61. (b) The Attorney General for India is the Indian
government’s chief legal advisor, and its primary
72.
high court shall be English.
(b) Delimitation commission of India is a Commission
established by Government of India under the
provisions of the Delimitation Commission Act. In

ngi
lawyer in the Supreme Court of India. He is appointed
by the President of India under Article 76(1) of the
Constitution and holds office during the pleasure of
India, such Delimitation Commissions have been
constituted 4 times - in 1952 under the Delimitation
Commission Act, 1952, in 1963 under Delimitation
the President. He must be a person qualified to be
appointed as a Judge of the Supreme Court.
62. (b) The Comptroller and Auditor General (CAG) of nee
Commission Act, 1962, in 1973 under Delimitation
Act, 1972 and in 2002 under Delimitation Act,
2002. The main task of the commission is to redraw

r
India is an authority, established by the Constitution the boundaries of the various assemblyand Lok
of India under Chapter V, who audits all receipts
and expenditure of the Government of India and the
state governments, including those of bodies and ing
Sabha constituencies based on a recent census. The
representation from each state is not changed during
this exercise. However, the number of SC and ST seats
authorities substantially financed by the government.
The CAG is also the external auditor of government-
owned companies.
73.
.ne
in a state ischanged in accordance with the census.
(a) “Political parties give political education to the
people” is not the function of political parties in a
63. (b) According to Article 243 (I) the governor of the
state shall set up the Finance Commission within
the period of one year. State Finance Commissions
receive grants from the Finance Commission that is
74.
democracy.
t
(c) All doubts and disputes arising out of or in connection
with the election of a President or Vice-President
shall be inquired into and decided by the Supreme
set up by the central government. Court whose decision shall be final (Article 71(a)).
64. (b) The Human Right Commission consists of a 75. (c) The Planning Commission was established in March
chairman and other four members. The chairman 1950 by an executive resolution of the Government
should be a retired chief justice of India.
of India, on the recommendation of the Advisory
65. (d) The Finance Commission is constituted by the
Planning Board constituted in 1946, under the
President under article 280 of the Constitution,
mainly to give its recommendations on distribution chairmanship of KC Neogi. Thus, the Planning
of tax revenues between the Union and the States Commission is neither constitutional nor a statutory
and amongst the States themselves. body. Planning commission has been replaced by
66. (b) Department of Official language(Raj Bhasha new institution namely NITI Aayog.
Vibhag) comes under Ministry of Home affairs. 76. (c) The power to decide the date of an election a
67. (a) The Planning Commission was set up by a Resolution state legislative assembly rests with the election
of the Government of India in March 1950.It is not commission of India.
provided in the constitution of India. 77. (a) According to Article 280 it shall be the duty of
68. (b) The Planning Commission was established in March the Commission to make recommendations to the
1950 by an executive resolution of the government President as to (1) the distribution between the

Downloaded From : www.EasyEngineering.net


Downloaded From : www.EasyEngineering.net
P-194 Miscellaneous Topics

Union and the States of the net proceeds of taxes in detail, with all aspects of conduct of elections and
which are to be, or may be, divided between them post election disputes.
under this Chapter and the allocation between the 83. (b) In Gujarat and Maharashtra the Chief Ministers
States of the respective shares of such proceeds; (2) come under the ambit of Lok Ayukta Act.
the principles which should govern the grants in aid 84. (d) A defeated candidate who fails to secure more than
of the revenues of the States out of the Consolidated one sixth of the valid votes polled in the constituency
Fund of India; (3) any other matter referred to the will lose his security deposit. When a very large
Commission by the President in the interests of number of candidates contest the election, due to
sound finance; (4) The Commission shall determine distribution of votes, the winning candidate may get
their procedure and shall have such powers in the less than 1/6 th of valid voters.
performance of their functions as Parliament may by 85. (b) Political scientists speculate that proportional
law confer on them. representation leads logically to multi-party systems,
78. (c) Article 350A was inserted by 7th Constitutional since it allows new parties to build a niche in the
Amendment act 1956. It says, it shall be the legislature.
endeavour of every State and of every local authority 86. (d) Party funding in Austria has been subject to public
within the State to provide adequate facilities for regulation and public subsidies since 1975. Party
instruction in the mother-tongue at the primary finance in Germany is the subject of statutory
stage of education to children belonging to linguistic reports, which up to 35 parties file annually with the

ww minority groups; and the President may issue such


directions to any State as he considers necessary or
proper for securing the provision of such facilities.
Article 349 – Special Procedure for enactment of 1. (d)
administration of the German parliament.
EXERCISE-2
2. (b) 3. (d) 4. (b) 5. (c)

w.E
certain laws relating to language; Article 350 –
Language to be used in representations for redress of
grievances; Article 351 – Directive for development
6. (a) 7. (b) 8. (c) 9. (b) 10.
11. (d) 12. (a) 13. (d) 14. (b) 15. (d)
16. (b) 17. (a) 18. (d) 19. (b) 20. (d)
(b)

79. (d)
a
of the Hindi language.

syE
As per provisions under Article 149, the Comptroller
and Auditor General shall perform such duties and
exercise such powers in relation to the accounts of the
Union and of the States and of any other authority or
21. (c) 22. (d)
23. (c) 24. (c) 25. (c) 26. (c) 27. (a)
28. (d) 29. (b) 30. (d) 31. (b) 32. (c)
33. (d) 34. (b)
35. (c)
body as may be prescribed by or under any law made
ngi
by Parliament and, until provision in that behalf is so
made, shall perform such duties and exercise such
36. (c) Lokpal and Lokayukta are established under the
report of the administrative reforms commission
headed by Morarji Desai to investigate corruption at
powers in relation to the accounts of the Union and
of the States as were conferred on or exercisable
by the Auditor General of India immediately before
37. (c)
nee
higher places in government.

38. (d) A is false because the constitution without specifying

80. (b)
the commencement of this Constitution in relation
to the accounts of the Dominion of India and of the
Provinces respectively.
President can issue an ordinance only when both
r
the strength of the Commission has left the matter

ing
to the discretion of the president, who determines
its composition. According to article 315, the UPSC
consists of a chairman and other members appointed
houses of parliament are not in session (Art 123).
No minimum age is prescribed for appointment as
a judge of the Supreme Court in the Constitution. .ne
by the president of India. Articles 315 to 323 of Part
XIV of the constitution provide for a Public Service
Commission for the Union and for each state.
The age of a Judge of the SC shall be determined
by such authority and in such manner as parliament
may by law provide. NDC is composed of the PM
as its head, all Union Cabinet ministers, the CMs of
t
39. (a) Article 124 (a) of the Indian Constitution stipulates
that “There shall be a Supreme Court of India
constituting of a Chief Justice of India and, until
Parliament by law prescribes a larger number, of
all states, CMs/Administrators of all UTs and the not more than seven other Judges”. At present the
members of the Planning Commission. There is no Supreme Court consists of thirty one judges (one
constitutional provision regarding the accountability chief justice and thirty other judges).
of the planning commission the parliament. Under article 315 the UPSC consists of a chairman
81. (d) National Development council is not a constitutional and other members appointed by the president of
body. It is an extra-constitutional body. Planning India. The constitution without specifying the
Commission is a non-constitutional and non-statutory strength of the Commission has left the matter to
body. It was created by the Govt. of India in 1950 by the discretion of the president, who determines its
a resolution. Zonal councils have been recognized in composition.
the 74th Constitutional Amendment Act of 1992. 40. (a) According to Article 316(a)of the Indian Constitution,
82. (a) Elections in India are conducted according to the the chairman and the members of the UPSC are
constitutional provisions, supplemented by laws made appointed by the president. Under Article 316(c) a
by Parliament. The major laws are Representation person who holds office as a member of a Public
of the People Act, 1950, which mainly deals with Service Commission shall, on the expiration of his
the preparation and revision of electoral rolls, the term of office, be ineligible for re-appointment to
Representation of the People Act, 1951 which deals, that office.

Downloaded From : www.EasyEngineering.net


Downloaded From : www.EasyEngineering.net
Miscellaneous Topics P-195

41. (b) The Attorney General of India is the chief legal lays down that the State  shall take  steps  to organize
advisor of Indian government. He is appointed by village panchayats and endow them  with  such powers
the President of Indiaunder Article 76(a) of the and authority as may be necessary to enable them to
Constitution. He must be a person qualified to be function as units of self government. Planning for
appointed as a Judge of the Supreme Court. The economic development and social justice is one such
Attorney General has the right of audience in all power given to village panchayats.
Courts in India as well as the right to participate in 49. (d) All the parties mentioned are state level parties.
the proceedings of the Parliament. He holds office 50. (d) All the statements are correct.
during the pleasure of the President. The current 51. (b) The Chief Election Commissioner and other Election
Attorney General is Mukul Rohatgi(14th Attorney Commissioners enjoy equal powers and salaries.
General). The term of office of the Election Commissioner
42. (c) Under Article 148 of the Indian Constitution is 6 years or till he attains the age of 65 years or
the Comptroller and Auditor General (CAG) of whichever is earlier.
India is an authority who audits all receipts and 52. (b) Article 280 of the Constitution of India provides for
expenditure of the Government of India and the a Finance Commission as a quasi-judicial body. It is
state governments, including those of bodies and constituted by the President of India every fifth year
authorities substantially financed by the government. or at such earlier time as he considers necessary.
The CAG shall only be removed from office in like The commission makes recommendations to the

ww manner and on the like grounds as a judge of the


Supreme Court of India :Article148(a)
The Comptroller and Auditor General shall not be
eligible for further office either under the Government
president with regard to the distribution of the
proceeds of taxes between the union and the states.
The principles which should govern the grants-in-aid
to be given to the states. Any other matter referred

w.E
of India or under the Government of any State after
he has ceased to hold his office :Article 148(d)
43. (a) The grounds for disqualification under the Anti-
to the Commission by the President in the interest of
sound finance.
53. (a) The High Court (and not the Election Commission)


a
Defection Law are as follows:

syE
If he votes or abstains from voting in such House
contrary to any direction issued by his political party
or anyone authorised to do so, without obtaining
prior permission. As a pre-condition for his
is the final authority to give a final verdict in case of
election disputes. In the alternative special election
benches may be constituted in high courts and
earmarked exclusively for the disposal of election
petitions and disputes.
disqualification, his abstention from voting should
not be condoned by his party or the authorised person
within 15 days of such incident. As per the 1985 Act, ngi 54. (d) Statement 1 is incorrect as Advocate General
of the state is appointed by the governor of the
State. Statement 2 is incorrect as High Courts
a ‘defection’ by one-third of the elected members
of a political party was considered a ‘merger’.
Finally, the 91st Constitutional Amendment Act, nee
have Original, Appellate and Writ jurisdiction (not
advisory jurisdiction).
55. (c) Only 2nd and 3rd are correct statements.
2003, changed this. So now at least two-thirds of
the members of a party have to be in favour of a
“merger” for it to have validity in the eyes of the
law.


r
(2) CAG reports on execution of projects or programmes

ing
by the ministries are discussed by the Public Accounts
Committee.
(3) Information from CAG reports can be used by
44. (b) National Commission for Religious and Linguistic
Minorities was constituted by Government on 29
.ne
investigating agencies to press charges against those
who have violated the law while managing public

t
October 2004 to look into various issues related to finance.
Linguistic and Religious minorities in India. It is 56. (c) The Attorney General of India has a post parallel to
also called Ranganath Misra Commission because it any minister in Parliament. He can take part in the
was chaired by former Chief Justice of India Justice proceedings of either house. He can be a member
Ranganath Misra. of any committee of Parliament. He has the right to
45. (b) The Comptroller and Auditor-General of India is speak in the Parliament but he has no right to vote.
appointed by the President of India. CAG is an 57. (a) The main functions of the Committee are to consider
authority that was established by the Constitution of all matters concerning the welfare of the Scheduled
India under article 148. Report of CAG of Union Castes and Scheduled Tribes, falling within the
Accounts to be submitted to President who causes purview of the Union Government and the Union
them to be laid before each house of parliament. Territories, to consider the reports submitted by
46. (a) Best answer is 1, 2 and 3. Because only Inter State the National Commission for Scheduled Castes
Council is a constitutional body under article 263. and Scheduled Tribes and to examine the measures
So option 4 should not be included. taken by the Union Government to secure due
47. (a) Under Article 76, impeachment procedure of the representation of the Scheduled Castes and Scheduled
Attorney-General is not provided. He shall hold Tribes in services and posts under its control.
office during the pleasure of the President. He must 58. (c) 59. (c) 60. (c)
not be a member of either House of Parliament. 61. (d) Attorney General of India is appointed by the
48. (b) Out of 52 items on the concurrent list, Economic President of India under Article 76(a) of the
and Social Planning is placed under Article 40 of the Constitution and holds office during the pleasure of
Constitution. Directive  Principles  of State Policy the President.

Downloaded From : www.EasyEngineering.net


Downloaded From : www.EasyEngineering.net

AMENDMENTS & ARTICLES OF


8
THE INDIAN CONSTITUTION Chapter

ww
Amendment Procedures Under 2. For amending certain provisions a special procedure to be
Indian Constitution
w.E
Indian Constitution is a balanced Constitution. The farmers
of the constitution desired to secure balance and moderate in

followed, e.g.
(i) A Bill for the purpose must be passed in each house
of parliament by a majority of total membership of
the house.

a
incorporating various provision in our Constitution. As far as

syE
the amendments of the Constitution is concerned, a balance
is struck in making the Constitution partly rigid and partly
flexible. A flexible Constitution is one, which can be easily
(ii) By a majority of not less than two-third of the


members of that house present and voting and
(iii) It should be notified by the legislatures of not less
than one-half of the states before the Bill is presented
amended like ordinary law of the land. On the contrary, a
rigid Constitution is one whose amendment is very difficult
and where there is a distinction between the amendment of ngi to the President for assent.
The provisions requiring this special procedure to be
the Constitutional law and ordinary law. Both the types of
Constitutions had their merits and demerits.
The proposal for amending the Constitution can be initiated

nee
followed include:
(a) Manner of the election of the President.
(b) Matters relating to the executive power of the Union
only in the Union Legislature and the state legislatures have
no such power. But the farmers of the Indian Constitution did
not go to the extreme. They incorporated a unique procedure
of amendment which combines both rigidity and flexibility. In

r
and of the state.

ing
(c) Representation of the states in Parliament.
(d) Matters relating to the Union Judiciary and High
this connection, it is worldwhile to quote Pandit Nehru who
clearly stated, “While one wants this constitution to be solid
and permanent as we can make it, there is no permanence in the

Courts in the states.

.ne
(e) Distribution of legislative powers between the Union
and the States.
Constitution. There should be certain flexibility. If you make
anything rigid and permanent you stop the nation’s growth, the
growth of a living vital organic people.


(f) any of the list in the Seventh Schedule.
t
(g) Provisions of Article 368 relating to the procedure
for amendment of the Constitution, etc.
3. There are certain provisions which require simple majority
Methods of Amendments
for amendments. They can be amended by the ordinary
Article 368 of the Constitution deals with procedure of law making process they include:
amendments of the Constitution. It can be amended in three
different ways: (a) formation of new states and alteration of areas,
boundaries or names of existing ones.
1. An amendment of the Constitution may be initiated only
by the introduction of a bill for the purpose in either house (b) creation or abolition of legislative councils in the
of Parliament and when a bill is passed in each house. states.
(i) By a majority of total membership of that house. (c) administration and control of scheduled areas and
(ii) By a majority of not less than two-thirds of the scheduled tribes.
members of that house present and voting, it shall be (d) the salaries and allowances of the Supreme Court and
presented to the President who shall give his assent High Court Judges.
to the Bill and there upon the Constitution shall stand (e) laws regarding citizenship, etc. It is significant that
amended in accordance with the term of the Bill. the laws passed by Parliament to change the above
Most of the provisions of the Constitution can be amended provisions would not be deemed to be amendments
by this procedure. of the Constitution for the purpose of Article 368.

Downloaded From : www.EasyEngineering.net


Downloaded From : www.EasyEngineering.net
Amendments & Articles of the Indian Constitution P-197

Scope of Amendability: Basic Features While no list of ‘basic features’ has been drawn up by the court,
How far can the constitution be amended? Can any of its different judgements lead one to view the ‘basic features’ as –
provisions be amended? Over the years, there has been some (i) Supremacy of the Constitution,
confusion over this issue. Until the Golak Nath Case (1967), (ii) Rule of law,
the Supreme Court had been holding the view that no part of (iii) Separation of powers,
the Constitution is unamendable. In the Golak Nath case it
was held that the Fundamental Rights cannot be amended. The (iv) Objectives mentioned in the Preamble,
decision was over turned in the Keshavananda Bharti Case (v) Judicial review,
(1973). However, it was also held by the Supreme Court that (vi) Federalism,
there are certain basic features of the Constitution that cannot (vii) Securalism,
be altered in the course of Parliament exercising its amending
(viii) Soverign, democratic and republician structure,
power under Article 368. As to what these basic features are,
there seems to be no unanimous opinion. One may only reach (ix) Principle of equality and the essence of other Fundamental
certain broad conclusion on the matter, as pointed out by Rights,
Dr. D. D. Basu in his Introduction to the Constitution of India: (x) Social and economic justice,
– that any part of the Constitution may be amended after (xi) balance between Part III and Part IV,

ww
complying with the procedure laid down in Article 368;
– a reference to a Constituent Assembly or a referendum is
not required to amend any part of the Constitution;
(xii) Concept of free and fair elections,
(xiii) independence of the judiciary,
(xiv) effective access of justice,

w.E
– however, no provision of the Constitution can be amended
if it takes away or destroys any of the basic features of the
Constitution.
(xv) Limitations on the amending power, conferred by Article
368.

C.A. Acts

1st C.A.
a
Coverage
List of Amendments of the Constitution of India

syE Date of Assembly


Enforced since
Amend articles 15, 19, 85, 87, 18 June 1951
Objectives

To fully secure the constitutional validity of zamindari


Act 1951 174, 176, 341, 342, 372 and 376.
Insert articles 31A and 31B. Insert ngi abolition laws and to place reasonable restriction on freedom
to speech. A new constitutional device, called Schedule

2nd C.A.
Schedule 9.

Amend article 81(1)(b). 1 May 1953 nee


Ninth introduced to protect laws that are contrary to the
Constitutionally guaranteed.
Removed the upper population limit for a parliamentary
Act 1952

3rd C.A. Amend schedule 7.


r
constituency by amending Article 81 (1) (b).

ing
22 February 1955 Re-enacted entry 33 of the Concurrent List in the Seventh
Act 1954

.ne
Schedule with relation to include trade and commerce in,
and the production, supply and distribution of 4 classes of
essential commodities, viz, foodstuffs, including edible oil

4th C.A.
Act 1955
5th C.A.
Amend articles 31, 35 and 305. 27 April 1955
Amend 9th Schedule.
Amend article 3. 24
seeds and oils; cat.

in 9th Schedule of the Constitution.


t
Restriction on property rights and inclusion of related bills

December Empowered the President to prescribe a time limit of a State


Act 1955 1955 Legislature to convey its views on proposed Central laws
relating to the formation of new States and alteration of
areas, boundaries or names of existing States.

6th C.A. Amend articles 269 and 286. 11 September Amend the Union and State Lists with respect to raising of
Act 1956 Amend 7th Schedule. 1956 taxes.
Amend articles 1, 3, 49, 80, 81,
82, 131, 153, 158, 168, 170, 171,
216, 217, 220, 222, 224, 230, 231
and 232.
7th C.A. Insert articles 258A, 290A, 298, 1 November 1956 Reorganization of states on linguistic lines, abolition of Class
Act 1956 350A, 350B, 371, 372A and 378A. A, B, C, D states and introduction of Union Territories.
Amend part 8.
Amend schedules I, II, III and IV.

Downloaded From : www.EasyEngineering.net


Downloaded From : www.EasyEngineering.net
P-198 Amendments & Articles of the Indian Constitution

8th C.A. Amend article 334. 5 January 1960 Extended the period of reservation of seats for the Scheduled
Act 1959 Castes and Scheduled Tribes and Anglo-Indians in the Lok
Sabha and State Legislative Assemblies till 1970.

9th C.A. Amend I Schedule. 28 December Minor adjustments to territory of Indian Union consequent
Act 1960 1960 to agreement with Pakistan for settlement of disputes by
demarcation of border villages, etc.
10th C.A. Amend article 240. Amend 1st 11 August 1961 Incorporation of Dadra, Nagar and Haveli as a Union
Act 1961 Schedule. Territory, consequent to acquisition from Portugal.
11th C.A. Amend articles 66 and 71. 19 December Election of Vice President by Electoral College consisting of
Act 1961 1961 members of both Houses of Parliament, instead of election
by a Joint Sitting of Parliament, Indemnify the President
and Vice President Election procedure from challenge on
grounds of existence.
12th C.A. 1st Amend article 240. Amend 20 December Incorporation of Goa, Daman and Diu as a Union Territory,
Act 1962 Schedule. 1961 consequent to acquisition from Portugal.

ww
13th C.A.
Act 1962
14th C.A.
Amend article 170. Insert new
article 371A.
Amend articles 81 and 240. Insert
1 December 1963 Formation of State of Nagaland, with special potection

28
under Article 371A
December Incorporation of Pondicherry into the Union of India and
Act 1962
w.E
article 239A.
Amend schedules 1 and 4.
1962

15th C.A. Amend articles 124, 128, 217, 5 October 1963


creation of Legislative Assemblies for Himachal Pradesh,
Tripura, Manipur and Goa.
Raise retirement age of High court judges from 60 to 62 and

th
a
Act 1963 222, 224, 226, 297, 311 and 316.
Insert article 224A. Amend 7th
Schedule. syE
other minor amendments for rationalizing interpretation of
rules regarding judges, etc.

16 C.A. Amend articles 19, 84 and 173.


Act 1963 Amend 3rd Schedule.
5 October 1963

ngi
Make it obligatory for seekers of public office to swear their
allegiance to the Indian Republic and prescribe the various
obligtory templates.
17th C.A.
Act 1964
Amend article 31A. Amend 9th 20 June 1964
schedule.
nee
To secure the constitutional validity of acquisition of Estates
and place land acquisition laws in 9th Schedule of the
Constitution.
18th C.A.
Act 1966
19th C.A.
Amend article 3.

Amend article 324. 11


r ing
27 August 1966 Technical Amendment to include Union Territories in Article
3 and hence permit reorganisation of Union Territories
December Abolish Election Tribunals and enable trial of election
Act 1966
20th C.A. Insert article 233A.
1966
22
petitions by regular High Courts.
.ne
December Indemnity and validate judgements, decrees, orders and
Act 1966

21st C.A. Amend 8th Schedule.


1966

10 April 1967
were not eligible for appointment under article 233.
Include Sindhi as an Official Language.
t
sentences passed by judges and to validate the appoinment,
posting, promotion and transfer of judges barring a few who

Act 1967
22nd C.A. Amend article 275. Insert articles 25 September Provision to form Autonomous states within the State of
Act 1969 244A and 371B. 1969 Assam.
23rd C.A. Amend articles 330, 332, 333 and 23 January 1970 Discontinued reservation of seats for the Scheduled Tribes
Act 1969 334. in Nagaland, both in the Lok Sabha and State Legislative
Assembly and stipulated that not more than one Anglo-
Indian could be nominated by the Governor to any State
Legislative Assembly. Extend reservation for SC/ST
and Anglo Indian members in the Lok Sabha and State
Assemblies for another ten years, i.e. up to 1980.
24th C.A. Amend articles 13 and 369. 5 November 1971 Enable parliament to dilute fundamental rights through
Act 1971 amendments to the Constitution.

Downloaded From : www.EasyEngineering.net


Downloaded From : www.EasyEngineering.net
Amendments & Articles of the Indian Constitution P-199

25th C.A. Amend article 31. Insert article 31C. 20 April 1972 Restrict property rights and compensation in case the state
Act 1971 Insert article 31C. takes over private property.
26th C.A. Amend article 366. 28 December Abolition of privy purse paid to former rulers of princely
Act 1971 Insert article 363A. 1971 states which were incorporated into the Indian Republic.
Remove articles 291 and 362.
th
27 C.A. Amend articles 239A and 240. 15 February 1972 Reorganization of Mizoram into a Union Territory with a
Act 1971 Insert articles 239B and 371C. legislature and Council of Ministers.
28th C.A. Insert article 312A. Remove article 29 August 1972 Rationalize Civil Service rules to make it uniform
Act 1972 314. across those appointed prior to independence and post
independence.
29th C.A. Amend 9th Schedule. 9 June 1972 Place land reform acts and amendments to these act under
Act 1972 9th Schedule of the Constitution.
30th C.A. Amend article 133. 27 February 1973 Change the basis for appeals in Supreme Court of India
Act 1972 in case of Civil Suits from value criteria to one involving
substantial question of law.

ww
31th C.A. Amend articles 81, 330 and 332.
Act 1973
17 October 1973 Increase size of Lok Sabha from 525 to 545 seats. Increased
seats going to the new states formed in North East India and
minor adjustment consequent to 1971 Delimitation exercise.

w.E
32rd C.A. Amend article 371.
Act 1973 Insert articles 371D and 371E.
Amend 7th Schedule.
1 July 1974 Protection of regional rights in Telengana and Andhra
regions of State of Andhra Pradesh.

Act 1974 a
33rd C.A. Amend articles 101 and 190.

34th C.A. Amend 9th Schedule


syE
19 May 1974 Prescribes procedure for resignation by members of
Parliament and state legislature and the procedure for
verification and acceptance of resignation by house speaker.
7 September 1974 Place land reform acts and amendments to these act under
Act 1974
35th C.A. Amend articles 80 and 81. 1 March 1974 ngi 9th Schedule of the Constitution.
Terms and Conditions for the incorporation of Sikkim into
Act 1974 Insert article 2A.

th
Insert schedule 10.
36 C.A. Amend articles 80 and 81. 26 April 1975
nee
the Union of India.

Formation of Sikkim as a State within the Indian Union


Act 1975 Insert article 371F.
Amend 1st and 4th Schedules.
Remove Schedule 10.
r ing
th
37 C.A. Amend articles 239A and 240.
Act 1975
3 May 1975
.ne
Formation of Arunachal Pradesh legislative assembly.

38th C.A. Amend articles 123, 213, 239B,


Act 1975 352, 356, 359 and 360.
39th C.A. Amend articles 71 and 329.
Act 1975 Insert article 329A.
1 August 1975
ordinances.
t
Enhances the powers of President and Governors to pass

10 August 1975 Amendment designed to negate the judgement of Allahabad


High Court invalidating Prime Minister Indira Gandhi’s
Amend schedule 9. election to Parliament. Amendment placed restrictions on
judicial scrutiny of post of Prime Minister.
40th C.A. Amend article 297. 27 May 1976 Enable Parliament to make laws with respect to Exclusive
Act 1976 Amend 9th Schedule. Economic Zone and vest the mineral wealth with Union of
India.
Place land reform & other acts and amendments to these act
under 9th Schedule of the Constitution.
41st C.A. Amend article 316. 7 September 1976 Raise Retirement age Limit of Chairman and Members of
Act 1976 Joint Public Service Commissions and State Public Service
Commissions for sixty to sixty two.

Downloaded From : www.EasyEngineering.net


Downloaded From : www.EasyEngineering.net
P-200 Amendments & Articles of the Indian Constitution

42nd C.A. Amend articles 31, 31C, 39, 55, 1 April 1977 Amendment passed during internal emergency by Indira
Act 1976 74, 77, 81, 82, 83, 100, 102, 103, Gandhi. Provides for curtailment of fundamental rights,
105, 118, 145, 150, 166, 170, 172, imposes fundamental duties and changes to the basic
189, 191, 192, 194, 208, 217, 225, structure of the Constitution by making India a “Socialist
226, 227, 228, 311, 312, 330, 352, Secular” Republic
353, 356, 357, 358, 359, 366, 368
and 371F.
Insert articles 31D, 32A, 39A,
43A, 48A, 131A, 139A, 144A,
226A, 228A and 257A.
Insert parts 4A and 14A.
Amend 7th Schedule.
43rd C.A. Amend articles 145, 226, 228 and 13 April 1978 Amendment passed after revocation of internal emergency
Act 1977 366 in the Country. Repeals some of the ‘Anti-Freedom’
Remove articles 31D, 32A, 131A, amendments enacted through 42nd Amendment Bill.
144A, 226A and 228A.
th

ww
44 C.A. Amend articles 19, 22, 30, 31A,
Act 1978 31C, 38, 71, 74, 77, 83, 103, 105,
123, 132, 133, 134, 139A, 150,
6 September 1978 Amendment passed after revocation of internal emergency
in the Country. Provides for human rights safeguards and
mechanisms to prevent abuse of executive and legislative

w.E
166, 172, 192, 194, 213, 217, 225,
226, 227, 239B, 329, 352, 356,
358, 359, 360 and 371F.
authority. Annuls some Amendments enacted in 42nd
Amendment Bill 42

a
Insert articles 134A and 361A.
Remove articles 31, 257A and
329A.
Amend part 12.
syE
th
Amend 9th Schedule.
45 C.A. Amend article 334. 25 January 1980ngi Extend reservation for SC/ST and nomination of Anglo
Act 1980

46th C.A. Amend articles 269, 286 and 366 2 February 1983 nee
Indian members in Parliament and State Assemblies for
another ten years, i.e. upto 1990.
Amendment to negate judicial pronouncements on scope
Act 1982 Amend 7th Schedule.
47th C.A. Amend 9th Schedule.
Act 1984
26 August 1984
r
and applicability on Sales Tax.

ing
Place land reform acts and amendments to these act under
9th Schedule of the Constitution.
48th C.A. Amend article 356.
Act 1984
1 April 1985

.ne
Article 356 amended to permit President’s rule up to two
years in the state of Punjab.
49th C.A. Amend article 244.
Act 1984 Amend 5th and 6th Schedules.
50th C.A. Amend article 33.
Act 1984
11
1984
11 t
September Recognize Tripura as a Tribal State and enable the creation
of a Tripura Tribal Areas Autonomous District Council.
September Technical Amendment to curtailment of Fundamental
1984 Rights as per Part III as prescribed in Article 33 to cover
Security Personnel protection property and communication
infrastructure.
51st C.A. Amend articles 330 and 332. 16 June 1986 Provide reservation to Scheduled Tribes in Nagaland,
Act 1984 Meghalaya, Mizoram, Arunachal Pradesh Legislative
Assemblies.
nd
52 C.A. Amend articles 101, 102, 190 and 1 March 1985 Anti Defection Law - Provide disqualification of members
Act 1985 191. from Parliament and assembly in case of defection from one
Insert schedule 10. party to other.
53rd C.A. Insert article 371G. 20 February 1987 Special provision with respect to the State of Mizoram.
Act 1986
54th C.A. Amend articles 125 and 221. 1 April 1986 Increase the salary of Chief Justice of India & other Judges
Act 1986 Amend 2nd Schedule. and to provide for determining future increases without the
need for constitutional amendment.

Downloaded From : www.EasyEngineering.net


Downloaded From : www.EasyEngineering.net
Amendments & Articles of the Indian Constitution P-201

55th C.A. Insert article 371H. 20 February 1987 Special powers to Governor consequent to formation of
Act 1986 state of Arunachal Pradesh.
56th C.A. Insert article 371I. 30 May 1987 Transition provision to enable formation of state of Goa.
Act 1987
57th C.A. Amend article 332. 21 September 1987 Provide reservation to Scheduled Tribes in Nagaland,
Act 1987 Meghalaya, Mizoram, Arunachal Pradesh Legislative
Assemblies.
th
58 C.A. Insert article 394A. 9 December 1987 Provision to public authentic Hindi translation of constitution
Act 1987 Amend part 22. as on date and provision to public authentic Hindi translation
of future amendments.
th
59 C.A. Amend article 356. Insert article 30 March 1988 Article 356 amended to permit President’s rule up to three
Act 1988 359A. years in the state of Punjab, Articles 352 and Article 359A
amended to permit imposing emergency in state of Punjab
or in specific districts of the state of Punjab.
60th C.A. Amend article 276. 20 December Profession Tax increased from a minimum of ` 250/- to a

ww
Act 1988
61st C.A. Amend article 326.
Act 1988
1988
28 March 1989
maximum of ` 2500/-
Reduce age for voting rights from 21 to 18.

Act 1989
w.E
62nd C.A. Amend article 334. 20
1989
December Extend reservation for SC/ST and nomination of Anglo
Indian members in Parliament and State Assemblies for
another ten years, i.e. upto 2000.

a
63rd C.A. Amend article 356.
Act 1989 Remove article 359A.
64th C.A. Amend article 356.
Act 1990
6 January 1990

syE
16 April 1990
Emergency powers application to State of Punjab, accorded
in Article 359A as per amendment 59th repealed.
Article 356 amended to permit President’s rule up to three
years and six months in the state of Punjab.
th
65 C.A. Amend article 338.
Act 1990
12 March 1990
ngi
National Commission for Scheduled Castes and Scheduled
Tribes formed and its statutory powers specified in the
Constitution.
th
66 C.A. Amend 9th Schedule.
Act 1990
7 June 1990
nee
Place land reform acts and amendments to these act under
9th Schedule of the Constitution.
th
67 C.A. Amend article 356.
Act 1990
68th C.A. Amend article 356.
4 October 1990

12 March 1991
r
Article 356 amended to permit President’s rule up to four
years in the state of Punjab.
ing
Article 356 amended to permit President’s rule up to five
Act 1991
th
years in the state of Punjab.

.ne
69 C.A. Insert articles 239AA and 239AB. 1 February 1992 To provide for a legislative assembly and Council of
Act 1991 ministers for Federal National Capital of Delhi. Delhi

th
70 C.A. Amend articles 54 and 239AA.
Act 1992
71st C.A. Amend 8th Schedule.
21
1991
continues to be a Union Territory.
t
December Include National Capital of Delhi and Union Territory of
Pondicherry in electoral college for Presidential Election.
31 August 1992 Include Konkani, Manipuri and Nepali as Official
Act 1992 Languages.
nd
72 C.A. Amend article 332. 5 December 1992 Provide reservation to Scheduled Tribes in Tripura State
Act 1992 Legislative Assembly.
rd
73 C.A. Insert part 9. 24 April 1993 Statutory provisions for Panchayat Raj as third level of
Act 1992 administration in villages.
th
74 C.A. Insert part 9A. 1 June 1993 Statutory provisions for Local Administrative bodies as
Act 1992 third level of administration in urban areas such as towns
and cities.
75th C.A. Amend article 323B. 15 May 1994 Provisions for setting up Rent Control Tribunals.
Act 1993
76th C.A. Amend 9th Schedule. 31 August 1994 Enable continuance of 69% reservation in Tamil Nadu by
Act 1994 including the relevant Tamil Nadu Act under 9th Schedule
of the constitution.

Downloaded From : www.EasyEngineering.net


Downloaded From : www.EasyEngineering.net
P-202 Amendments & Articles of the Indian Constitution

77th C.A. Amend article 16. 17 June 1995 A technical amendment to protect reservation to SC/ST
Act 1995 Employees in promotions.
78th C.A. Amend 9th Schedule. 30 August 1995 Place land reform acts and amendments to these act under
Act 1995 9th Schedule of the Constitution.
79th C.A. Amend article 334. 25 January 2000 Extend reservation for SC/ST and nomination of Anglo
Act 2000 Indian members in Parliament and State Assemblies for
another ten years, i.e. up to 2010.
80th C.A. Amend articles 269 and 270. 9 June 2000 Implement Tenth Finance Commission recommendation to
Act 2000 Remove article 272. simply the tax structures by pooling and sharing all taxes
between states and the Centre.
81st C.A. Amend article 16. 9 June 2000 Protect SC/ST reservation in filling backlog of vacancies.
Act 2000
82nd C.A. Amend article 335. 8 September 2000 Permit relaxation of qualifying marks and other criteria in
Act 2000 reservation in promotion for SC/ST candidates.
83rd C.A. Amend article 243M. 8 September 2000 Exempt Arunachal Pradesh from reservation for Scheduled

ww
Act 2000
84th C.A.
Act 2001
Castes in Panchayati Raj institutions.
Amend articles 55, 81, 82, 170, 21 February 2002 Extend the usage of 1971 national census population figures
330 and 332. for statewise distribution of Parliamentary seats.
85th C.A.
Act 2002
86th C.A.
w.E
Amend article 16.

Amend articles 45 and 51A.


4 January 2002

12
A technical amendment to protect Consequential seniority
in case of promotions of SC/ST Employees
December Provides Right to Education until the age of fourteen and
Act 2002
87th C.A.
Act 2003
88th C.A.
330.
a
Insert article 21A.

Amend article 270.


2002

syE
Amend articles 81, 82, 170 and 22 June 2003

15 January 2004
Early childhood care until the age of six.
Extend the usage of 2001 National Census population
figures for statewise distribution of parliamentary seats.
To extend statutory cover for levy and utilization of Service
Act 2003 Insert article 268A.
Amend 7th Schedule. ngi Tax.

th
89 C.A. Amend article 338.
Act 2003
Insert article 338A.
28 September 2003
nee
The National Commission for Scheduled Castes and
Scheduled Tribes was bifurcated into the National
Commission for Scheduled Castes and the National

90th C.A. Amend article 332.


Act 2003
r
Commission for Scheduled Tribes.

ing
28 September 2003 Reservation in Assam Assembly relating to Bodoland
Territory Area.
91st C.A. Amend articles 75 and 164.
Act 2003
1 January 2004
.ne
Restrict the size of council of Ministers to 15% of legislative
members and to strengthen Anti Defection laws.

nd
Insert article 361B.
Amend 10th Schedule.
92 C.A. Amend 8th Schedule.
Act 2003
7 January 2004
t
Include Bodo, Dogri, Santhali, Maithali as official
languages.
93rd C.A. Amend article 15. 20 January 2006 To enable provision of reservation (27%) for Other
Act 2005 Backward Classes (O.B.C.) in government as well as
private educational institutions.
94th C.A. Amend article 164. 12 June 2006 To provide for a Minister of Tribal Welfare in newly
Act 2006 created Jharkhand and Chhattisgarh States including
Madhyapradesh, Orrissa.
95th C.A. Amend article 334. 25 January 2010 To extend the reservation of seats for SCs and STs in
Act 2009 the Lok Sabha and states assemblies from Sixty years to
Seventy years.
96th C.A. Amend schedule 8. 23 September Substituted “Odia” for “Oriya”
Act 2011 2011

Downloaded From : www.EasyEngineering.net


Downloaded From : www.EasyEngineering.net
Amendments & Articles of the Indian Constitution P-203

97th C.A. Amend Art 19 and added Part IXB. 12 January 2012 Added the words “or co-operative societies” after the word
Act 2011 “or unions” in Article 19(I) (c) and insertion of article 43B,
i.e. Promotion of Co-operative Societies and added Part-
IXB, i.e. the Co-operative Societies.
The amendment objective is to encourage economic activities
of cooperatives which in turn help progress of rural India. It
is expected to not only ensure autonomous and democratic
functioning of cooperatives, but also the accountability of
the management to the members and other stakeholders.
98th C.A. To insert Article 371J in the 2 January 2013 To empower the Governor of Karnataka to take steps to
Act 2012 Constitution develop the Hyderabad-Karnataka Region.
99th C.A. Amend Article 127,128 Insert new 13 April 2015 Amendment provides for the formation of a National
Act 2014 Articles 124 A, 124 B & 124 C. Indician Appointments Commission.
100th Amendment of First Schedule to 1 August 2015 To operationalise the land Boundary Agreement between
C.A. Act Constitution. India and Bangladesh.
2015

ww
Article List
Part /
Schedule
I
Chapter

w.E
1-4
Articles

The Union and its Territory


Subject matter

II
III
IV
a
5-11
12-35
36-51
Citizenship

syE
Fundamental Rights
Directive Principles of State Policy
IVA
V
I
51A
52-151
52-78
Fundamental Duties
The Union
ngi
The Executive – The President and Vice President, Council of Ministers, The Attorney-

nee
General for India, Conduct of Government Business, Duties of Prime Minister as
respects the furnishing of information to the President, etc.
II

III
79-122

123
etc.
Legislative powers of the President. r
Parliament, Conduct of business, Disqualifications of Members, Legislative procedure

ing
IV
V
124-147
148-151
The Union Judiciary.
Comptroller and Auditor-General of India.
.ne
VI
I
II
152-237
152
153-167
The States
Definition
t
The Executive - The Governor, Council of Ministers, The Advocate-General for
the State, Conduct of Government business, Duties of Chief Minister in respects of
furnishing of information to Governor, etc.
III 168-212 The State Legislature, Disqualification of Members, Legislative Procedure, etc.
IV 213 Legislative power of the Governor.
V 214-232 The High Courts in the States.
VI 233-237 Subordinate Courts.
VII 238 Repealed.
VIII 239-242 The Union Territories – administration, High Courts for Union Territories etc.
IX 243-243O The Panchayats.
IXA 243P-243ZG The Municipalities.
X 244-244A The Scheduled and Tribal Areas.
XI 245-263 Relations between the Union and the States

Downloaded From : www.EasyEngineering.net


Downloaded From : www.EasyEngineering.net
P-204 Amendments & Articles of the Indian Constitution

I 245-255 Legislative Relations – distribution of legislative powers, residuary powers of legislation,


etc.
II 256-263 Administrative Relations.
XII 264-300A Finance, Property, Contracts and Suits
I 264-291 Finance – taxes, grants, Finance Commission, recommendations of Finance Commission,
exemption from taxes etc.
II 292-293 Borrowing – by the Government of India and by States.
III 294-300 Property, Contracts, Rights, Liabilities, Obligations and Suits and proceedings.
IV 300A Right to Property.
XIII 301-307 Trade, Commerce and Intercourse within theterritory of India.
XIV 308-323 Services under the Union and the States
I 308-314 Services – recruitment and conditions of service of persons serving the Union or State,
II 315-323 Public Service Commissions.
XIVA 323A-323B Tribunals.
XV
ww
XVI
324-329A
330-342
Elections.
Special Provisions relating to certain Classes- Scheduled Castes and Scheduled Tribes in
House of the People, etc. Scheduled Castes and Scheduled Tribes
XVII
I w.E
343-351
343-344
Official Language
Language of the Union.

XVIII
II
III
IV
a
345-347
348-349
350-351
352-360
Regional Languages.

syE
Language of the Supreme Court, High Court, etc.
Special Directives,
Emergency Provisions,
XIX 361-367
ngi
Miscellaneous – protection of President, Governors and Rajpramukhs, abolition of privy
purses, special provisions as to major ports and aerodromes, etc.
XX
XXI
368
369-392
nee
Amendment of the Constitution - Power of Parliament and procedure.
Temporary, Transitional and Special Provisions.

r
XXII 393-395 Short Title, Commencement, Authoritative text in Hindi and Repeals.
Part I : The Union And Its Territory
1 Name and territory of the Union.
Part III : Fundamental Rights
General
ing
2

Admission or establishment of new States.
2A [Repealed.] 12 Definition.
.ne
13 Laws inconsistent with or in derogation of the
3

4
Formation of new States and alteration of areas,
boundaries or names of existing States.
Laws made under articles 2 and 3 to provide for the
amendment of the First and the Fourth Schedules and
supplemental, incidental and consequential matters.
fundamental rights.

Right to Equality
14 Equality before law.
t
15 Prohibition of discrimination on grounds of religion,
Part II : Citizenship race, caste, sex or place of birth.
5 Citizenship at the commencement of the Constitution. 16 Equality of opportunity in matters of public employment.
6 Rights of citizenship of certain persons who have 17 Abolition of Untouchability.
migrated to India from Pakistan. 18 Abolition of titles.
7 Rights of citizenship of certain migrants to Pakistan.
8 Rights of citizenship of certain persons of Indian origin Right to Freedom
residing outside India. 19 Protection of certain rights regarding freedom of
9 Persons voluntarily acquiring citizenship of a foreign speech, etc.
State not to be citizens. 20 Protection in respect of conviction for offences.
10 Continuance of the rights of citizenship.
21 Protection of life and personal liberty.
11 Parliament to regulate the right of citizenship by law.
22 Protection against arrest and detention in certain cases.

Downloaded From : www.EasyEngineering.net


Downloaded From : www.EasyEngineering.net
Amendments & Articles of the Indian Constitution P-205

Right against Exploitation 48A Protection and improvement of environment and


23 Prohibition of traffic in human beings and forced labour. safeguarding of forests and wild life.
24 Prohibition of employment of children in factories, etc. 49 Protection of monuments and places and objects of
  Right to  Freedom of Religion national importance.
25 Freedom of conscience and free profession, practice 50 Separation of judiciary from executive.
and propagation of religion. 51 Promotion of international peace and security.
26 Freedom to manage religious affairs.
27 Freedom as to payment of taxes for promotion of any
Part IVA : Fundamental duties
particular religion. 51A Fundamental duties.
28 Freedom as to attendance at religious instruction or
religious worship in certain educational institutions.
Part V : The Union
  Cultural and Educational Rights Chapter I. The Executive
29 Protection of interests of minorities.
The President and Vice-President
30 Right of minorities to establish and administer
52 The President of India.
educational institutions.
53 Executive power of the Union.
31 [Repealed.]

ww
54 Election of President.
Saving of Certain Laws 55 Manner of election of President.
31A Saving of Laws providing for acquisition of estates, etc. 56 Term of office of President.

principles.
31D [Repealed.]
w.E
31B Validation of certain Acts and Regulations.
31C Saving of laws giving effect to certain directive
57
58
59
Eligibility for re-election.
Qualifications for election as President.
Conditions of President’s office.

32
a
Right to Constitutional Remedies
syE
Remedies for enforcement of rights conferred by this
Part.
60
61
62
Oath or affirmation by the President.
Procedure for impeachment of the President.
Time of holding election to fill vacancy in the office of
President and the term of office of person elected to fill
32A [Repealed.]
33 Power of Parliament to modify the rights conferred by
this Part in their application to Forces, etc. ngi 63
64
casual vacancy.
The Vice-President of India.
The Vice-President to be  ex officio  Chairman of the
34

35
Restriction on rights conferred by this Part while martial
law is in force in any area.
Legislation to give effect to the provisions of this Part.
65 nee
Council of States.
The Vice-President to act as President or to discharge
his functions during casual vacancies in the office, or

Part IV : Directive Principles Of State Policy


36 Definition.
66
67
r ing
during the absence, of President.
Election of Vice-President.
Term of office of Vice-President.
37
38
Application of the principles contained in this Part.
State to secure a social order for the promotion of
welfare of the people.
68
.ne
Time of holding election to fill vacancy in the office of
Vice-President and the term of office of person elected

39

40
Certain principles of policy to be followed by the State.
39A Equal justice and free legal aid.
Organisation of village panchayats.
41 Right to work, to education and to public assistance in
69
70
71
to fill casual vacancy.

t
Oath or affirmation by the Vice-President.
Discharge of President’s functions in other contingencies.
Matters relating to, or connected with, the election of a
President or Vice-President.
certain cases.
72 Power of President to grant pardons, etc., and to
42 Provision for just and humane conditions of work and suspend, remit or commute sentences in certain cases.
maternity relief.
73 Extent of executive power of the Union.
43 Living wage, etc., for workers.
  Council of Ministers
43A Participation of workers in management of industries.
74 Council of Ministers  to aid and advise President.
44 Uniform civil code for the citizens.
45 Provision for free and compulsory education for 75 Other provisions as to Ministers.
children.   The Attorney-General for India
46 Promotion of educational and economic interests of 76 Attorney-General for India.
Scheduled Castes, Scheduled Tribes and other weaker   Conduct of Government Business
sections. 77 Conduct of business of the Government of India.
47 Duty of the State to raise the level of nutrition and the 78 Duties of Prime Minister as respects the furnishing of
standard of living and to improve public health. information to the President, etc.
48 Organisation of agriculture and animal husbandry.

Downloaded From : www.EasyEngineering.net


Downloaded From : www.EasyEngineering.net
P-206 Amendments & Articles of the Indian Constitution

Chapter II. Parliament 107 Provisions as to introduction and passing of Bills.


General 108 Joint sitting of both Houses in certain cases.
79 Constitution of Parliament. 109 Special procedure in respect of Money Bills.
80 Composition of the Council of States. 110 Definition of “Money Bills”.
81 Composition of the House of the People. 111 Assent to Bills.
82 Readjustment after each census.   Procedure in Financial Matters
83 Duration of Houses of Parliament. 112 Annual financial statement.
84 Qualification for membership of Parliament. 113 Procedure in Parliament with respect to estimates.
85 Sessions of Parliament, prorogation and dissolution. 114 Appropriation Bills.
86 Right of President to address and send messages to 115 Supplementary, additional or excess grants.
Houses. 116 Votes on account, votes of credit and exceptional grants.
87 Special address by the President. 117 Special provisions as to financial Bills.
88 Rights of Ministers and Attorney-General as respects   Procedure Generally
Houses. 118 Rules of procedure.
  Officers of Parliament 119 Regulation by law of procedure in Parliament in relation

90 ww
89 The Chairman and Deputy Chairman of the Council of
States.
Vacation and resignation of, and removal from, the
120
121
to financial business.
Language to be used in Parliament.
Restriction on discussion in Parliament.

w.E
office of Deputy Chairman.
91 Power of the Deputy Chairman or other person
to perform the duties of the office of, or to act as,
Chairman.
122 Courts not to inquire into proceedings of Parliament.

Chapter III. Legislative Powers of the President


123 Power of President to promulgate Ordinances during

consideration.
a syE
92 The Chairman or the Deputy Chairman not to preside
while a resolution for his removal from office is under

93 The Speaker and Deputy Speaker of the House of the


recess of Parliament.

Chapter IV. The Union Judiciary


124 Establishment and constitution of Supreme Court.

94
People .
Vacation and resignation of, and removal from, the ngi 125
126
Salaries, etc., of Judges.
Appointment of acting Chief Justice.

95
offices of Speaker and Deputy Speaker.
Power of the Deputy Speaker or other person to perform
the duties of the office of, or to act as, Speaker.
127
128
nee
Appointment of ad hoc judges.
Attendance of retired Judges at sittings of the Supreme
Court.
96 The Speaker or the Deputy Speaker not to preside
while a resolution for his removal from office is under
consideration. r
129 Supreme Court to be a court of record.
130 Seat of Supreme Court.
ing
131 Original jurisdiction of the Supreme Court.
 97 Salaries and allowances of the Chairman and Deputy
Chairman and the Speaker and Deputy Speaker.
 98 Secretariat of Parliament.
131A [Repealed.]

.ne
132 Appellate jurisdiction of Supreme Court in appeals from
High Courts in certain cases.
  Conduct of Business
 99 Oath or affirmation by members.
100 Voting in Houses, power of Houses to act notwithstanding
vacancies and quorum.
High Courts in regard to Civil matters.
t
133 Appellate jurisdiction of Supreme Court in appeals from

134 Appellate jurisdiction of Supreme Court in regard to


criminal matters.
134A Certificate for appeal to the Supreme Court.
  Disqualifications of Members
135 Jurisdiction and powers of the Federal Court under
101 Vacation of seats.
existing law to be exercisable by the Supreme Court.
102 Disqualifications for membership.
136 Special leave to appeal by the Supreme Court.
103 Decision on questions as to disqualifications of members. 137 Review of judgments or orders by the Supreme Court.
104 Penalty for sitting and voting before making oath or 138 Enlargement of the jurisdiction of the Supreme Court.
affirmation under article 139 Conferment on the Supreme Court of powers to issue
99 or when not qualified or when disqualified. certain writs.
  Powers, Privileges and Immunities of Parliament and 139A Transfer of certain cases.
its Members 140 Ancillary powers of Supreme Court.
105 Powers, privileges, etc., of the Houses of Parliament 141 Law declared by Supreme Court to be binding on all
and of the members and committees thereof. courts.
106 Salaries and allowances of members.  142 Enforcement of decrees and orders of Supreme Court
  Legislative Procedure and orders as to discovery, etc.

Downloaded From : www.EasyEngineering.net


Downloaded From : www.EasyEngineering.net
Amendments & Articles of the Indian Constitution P-207

143 Power of President to consult Supreme Court. 174 Sessions of the State Legislature, prorogation and
144 Civil and judicial authorities to act in aid of the Supreme dissolution.
Court. 175 Right of Governor to address and send messages to the
144A [Repealed.] House or Houses.
145 Rules of Court, etc. 176 Special address by the Governor.
146 Officers and servants and the expenses of the Supreme 177 Rights of Ministers and Advocate-General as respects
Court. the Houses.
147 Interpretation.   Officers of the State Legislature
178 The Speaker and Deputy Speaker of the Legislative
Chapter V. Comptroller and Auditor General of Assembly.
India 179 Vacation and resignation of, and removal from, the
148 Comptroller and Auditor-General of India. offices of Speaker  and Deputy Speaker.
149 Duties and powers of the Comptroller and Auditor- 180 Power of the Deputy Speaker or other person to perform
General. the duties of the office of, or to act as, Speaker.
150 Form of accounts of the Union and of the States. 181 The Speaker or the Deputy Speaker not to preside
151 Audit reports. while a resolution for his removal from office is under

ww
Part VI : The States
Chapter I. General
consideration.
182 The Chairman and Deputy Chairman of the Legislative
Council.

152 Definition.
w.E
Chapter II. The Executive
183 Vacation and resignation of, and removal from, the
offices of Chairman and Deputy Chairman.
184 Power of the Deputy Chairman or other person
The Governor
153 Governors of States. a
154 Executive power of State.
155 Appointment of Governor.
syE
to perform the duties of the office of, or to act as,
Chairman.
185 The Chairman or the Deputy Chairman not to preside
while a resolution for his removal from office is under

156 Term of office of Governor.


157 Qualifications for appointment as Governor. ngi consideration.
186 Salaries and allowances of the Speaker and Deputy
Speaker and the Chairman and Deputy Chairman.
158 Conditions of Governor’s office
159 Oath or affirmation by the Governor.
160 Discharge of the functions of the Governor in certain
  nee
187 Secretariat of State Legislature.
Conduct of Business
188 Oath or affirmation by members.
contingencies.
161 Power of Governor to grant pardons, etc., and to
suspend, remit or commute sentences in certain cases.  
r
vacancies and quorum.
ing
189 Voting in Houses, power of Houses to act notwithstanding

Disqualifications of Members
162 Extent of executive power of State.

Council of Ministers
190 Vacation of seats.
191 Disqualifications for membership. .ne
163
164
 
165
Council of Ministers to aid and advise Governor.
Other provisions as to Ministers.
The Advocate-General for the State
Advocate-General for the State.
t
192 Decision on questions as to disqualifications of members.
193 Penalty for sitting and voting before making oath or
affirmation under article 188 or when not qualified or
when disqualified.
  Powers, privileges and immunities of State Legislatures
  Conduct of Government Business and their Members
166 Conduct of business of the Government of a State. 194 Powers, privileges, etc., of the Houses of Legislatures
167 Duties of Chief Minister as respects the furnishing of and of the members and committees thereof.
information to Governor, etc. 195 Salaries and allowances of members.
Chapter III. The State Legislature   Legislative Procedure
General 196 Provisions as to introduction and passing of Bills.
168 Constitution of Legislatures in States. 197 Restriction on powers of Legislative Council as to Bills
other than Money Bills.
169 Abolition or creation of Legislative Councils in States.
198 Special procedure in respect of Money Bills.
170 Composition of the Legislative Assemblies.
199 Definition of “Money Bills”.
171 Composition of the Legislative Councils.
200 Assent to Bills.
172 Duration of State Legislatures.
201 Bills reserved for consideration.
173 Qualification for membership of the State Legislature.
  Procedure in Financial Matters

Downloaded From : www.EasyEngineering.net


Downloaded From : www.EasyEngineering.net
P-208 Amendments & Articles of the Indian Constitution

202 Annual financial statement. 237 Application of the provisions of this Chapter to certain
203 Procedure in Legislature with respect to estimates. class or classes of magistrates.
204 Appropriation Bills.
Part VII : The States In Part B Of The First
205 Supplementary, additional or excess grants.
206 Votes on account, votes of credit and exceptional grants.
Schedule
207 Special provisions as to financial Bills. 238 [Repealed.]
  Procedure Generally
Part VIII : The Union Territories
208 Rules of procedure.
239 Administration of Union territories.
209 Regulation by law of procedure in the Legislature of the
State in relation to financial business. 239 A Creation of local Legislatures or Council of Ministers
or both for certain Union territories.
210 Language to be used in the Legislature.
239 A Special provisions with respect to Delhi.
211 Restriction on discussion in the Legislature.
239 AA Provision in case of failure of constitutional machinery.
212 Courts not to inquire into proceedings of the Legislature.
239 AB Power of administrator to promulgate Ordinances
Chapter IV: Legislative Power of The Governor during recess of Legislature.
213 Power of Governor to promulgate Ordinances during 240 Power of President to make regulations for certain

ww
recess of Legislature.

Chapter V: The High Courts in the States


Union territories.
241 High Courts for Union territories.
242 [Repealed.]
214
215
216
w.E
High Courts for States.
High Courts to be courts of record.
Constitution of High Courts.
Part IX: The Panchayats
243 Definitions.
217
a High Court.
a
Appointment and conditions of the office of a Judge of

syE
218 Application of certain provisions relating to Supreme
Court to High Courts.
243A Gram Sabha.
243B Constitution of Panchayats.
243C Composition of Panchayats.
243D Reservation of seats.
219 Oath or affirmation by Judges of High Courts.
220 Restriction on practice after being a permanent Judge. ngi 243E Duration of Panchayats, etc.
243F Disqualifications for membership.
221 Salaries, etc., of Judges.
222 Transfer of a Judge from one High Court to another.
223 Appointment of acting Chief Justice. nee
243G Powers, authority and responsibilities of Panchayats.
243H Powers to impose taxes by, and Funds of, the
Panchayats.
224 Appointment of additional and acting Judges.
224A Appointment of retired Judges at sittings of High
Courts.
r
243-I Constitution of Finance Commission to review financial
position.
ing
243J Audit of accounts of Pachayats.
225 Jurisdiction of existing High Courts.
226 Power of High Courts to issue certain writs.
226A [Repealed..]
243K Elections to the Panchayats.
243L Application to Union  territories.
243M Part not to apply to certain areas. .ne
227 Power of superintendence over all courts by the High
Court.
228 Transfer of certain cases to High Court.
t
243N Continuance of existing laws and Panchayats.
243-O Bar to interference by courts in electoral matters.

Part IXA : The Municipalities


228A [Repealed.]
229 Officers and servants and the expenses of High Courts. 243P Definitions.
230 Extension of jurisdiction of High Courts to Union 243Q Constitution of Municipalities.
territories. 243R Composition of Municipalities.
231 Establishment of a common High Court for two or more 243S Constitution and composition of Wards Committees,
States. etc.
243T Reservation of seats.
Chapter VI: Subordinate Courts 243U Duration of Municipalities, etc.
233 Appointment of district judges. 243V Disqualifications for membership.
233A Validation of appointments of, and judgments, etc., 243W Powers, authority and responsibilities of Municipalities,
delivered by, certain district judges. etc.
234 Recruitment of persons other than district judges to the 243X. Power to impose taxes by, and Funds of, the
judicial service. Municipalities.
235 Control over subordinate courts. 243 Finance Commission.
236 Interpretation. 243Z Audit of accounts of Municipalities.

Downloaded From : www.EasyEngineering.net


Downloaded From : www.EasyEngineering.net
Amendments & Articles of the Indian Constitution P-209

243ZA Elections to the Municipalities. 261 Public acts, records and judicial proceedings.
243ZB Application to Union territories.   Disputes relating to Waters
243ZC Part not to apply to certain areas. 262 Adjudication of disputes relating to waters of  inter-
243ZD Committee for district planning. State rivers or river valleys.
243ZE Committee for Metropolitan planning.   Co-ordination between States
243ZF Continuance of existing laws and Municipalities. 263 Provisions with respect to an inter-State Council.
243ZG   Bar to interference by Courts in electoral matters.
Part XIII Finance, Property, Contracts And Suits
Part X : The Scheduled and Tribal Areas
Chapter I. Finance
244 Administration of Scheduled Areas and Tribal Areas.
244A Formation of an autonomous State comprising certain General
tribal areas in Assam and creation of local Legislature 264 Interpretation.
or Council of Ministers or both therefor. 265 Taxes not to be imposed save by authority of law.
Part XI : Relations Between The Union and The 266 Consolidated Funds and public accounts of India and of
the States.
States
267 Contingency Fund.

ww
Chapter I: Legislative Relations
Distribution of Legislative Powers
  Distribution of Revenues between the Union and the
States

w.E
245 Extent of laws made by Parliament and by the
Legislatures of States.
246 Subject-matter of laws made by Parliament and by the
268 Duties levied by the Union but collected and appropriated
by the State.
269 Taxes levied and collected by the Union but assigned to
the States.

a
Legislatures of States.

certain additional courts.


248 Residuary powers of legislation.
syE
247 Power of Parliament to provide for the establishment of
270 Taxes levied and  distributed between the Union and the
States.
271 Surcharge on certain duties and taxes for purposes of
the Union.
249 Power of Parliament to legislate with respect to a matter
in the State List in the national interest. ngi 272 [Repealed.]
273 Grants in lieu of export duty on jute and jute products.
 250 Power of Parliament to legislate with respect to any
matter in the State List if a Proclamation of Emergency
is in operation. nee
274 Prior recommendation of President required to Bills
affecting taxation in which States are interested.
275 Grants from the Union to certain States.
251 Inconsistency between laws made by Parliament under
articles 249 and 250 and laws made by the Legislatures
of States.
277 Savings. r
276 Taxes on professions, trades, callings and employments.

278 [Repealed.] ing


252 Power of Parliament to legislate for two or more States
by consent and adoption of such legislation by any other
State.
279 Calculation of  “net proceeds”, etc.
280 Finance Commission.
.ne
253 Legislation for giving effect to international agreements.
254 Inconsistency between laws made by Parliament and
laws made by the Legislatures of States.
255 Requirements as to recommendations and previous
  Miscellaneous financial provisions
t
281 Recommendations of the Finance Commission.

282 Expenditure defrayable by the Union or a State out of


its revenues.
sanctions to be regarded as matters of procedure only. 283 Custody, etc., of Consolidated Funds, Contingency
Funds and moneys credited to the public accounts.
Chapter II. Administrative Relations 284 Custody of suitors’  deposits and other moneys received
by public servants and courts.
General 285 Exemption of property of the Union from State taxation.
  256 Obligation of States and the Union. 286 Restrictions as to imposition of tax on the sale or
  257 Control of the Union over States in certain cases. purchase of goods.
257A [Repealed.] 287 Exemption from taxes on electricity.
258 Power of the Union to confer powers, etc., on States in 288 Exemption from taxation by States in respect of water
certain cases. or electricity in certain cases.
258A Power of the States to entrust functions to the Union. 289 Exemption of property and income of a State from
259 [Repealed.] Union taxation.
260 Jurisdiction of the Union in relation to territories outside 290 Adjustment in respect of certain expenses and pensions.
India. 290A Annual payment to certain Devaswom Funds.
291 [Repealed.]

Downloaded From : www.EasyEngineering.net


Downloaded From : www.EasyEngineering.net
P-210 Amendments & Articles of the Indian Constitution

Chapter II. Borrowing 317 Removal and suspension of a member of a Public


292 Borrowing by the Government of India. Service Commission.
293 Borrowing by States. 318 Power to make regulations as to conditions of service of
members and staff of the Commission.
Chapter III. Property, Contracts, Rights, 319 Prohibition as to the holding of offices by members of
Liabilities, Commission on ceasing to be such members.
320 Functions of Public Service Commissions.
Obligations and Suits 321 Power to extend functions of Public Service
294 Succession to property, assets, rights, liabilities and Commissions.
obligations in certain cases. 322 Expenses of Public Service Commissions.
295 Succession to property, assets, rights, liabilities and 323 Reports of Public Service Commissions.
obligations in other cases.
296 Property accruing by escheat or laps or as bona vacantia. Part XIVA: Tribunals
297 Things of value within territorial waters or continental 323A Administrative tribunals.
shelf and resources of the exclusive economic zone to 323B Tribunals for other matters.
vest in the Union.
298 Power to carry on trade, etc. Part XV : Elections

ww
299 Contracts.
300 Suits and proceedings.
Chapter IV. Right to Property
324 Superintendence, direction and control of elections to
be vested in an Election Commission.
325 No person to be ineligible for inclusion in, or to claim

of law. w.E
300A Persons not to be deprived of property save by authority
to be included in a special, electoral roll on grounds of
religion, race, caste or sex.
326 Elections to the House of the People and to the

a
Legislative Assemblies of States to be on the basis of
Part XII: Trade, Commerce and Intercourse adult suffrage.
within the Territory of India
syE
301 Freedom of trade, commerce and intercourse.
302 Power of Parliament to impose restrictions on trade,
327 Power of Parliament to make provision with respect to
elections to Legislatures.
328 Power of Legislature of a State to make provision with
commerce and intercourse.
303 Restrictions on the legislative powers of the Union and
of the States with regard to trade and commerce.
ngi respect to elections to such Legislature.
329 Bar to interference by courts in electoral matters.

304 Restrictions on trade, commerce and intercourse among


States.
305 Saving of existing laws and laws providing for State
nee
329A [Repealed.]

Part XVI: Special Provisions Relating to


Relating to Certain Classes
monopolies.
306 [Repealed.]
307 Appointment of authority for carrying out the purposes
330

331
r ing
Reservation of seats for Scheduled Castes and Scheduled
Tribes in the House of the People.
Representation of the Anglo-Indian community in the
of articles 301 to 304.

Part XIV: Services Under the Union and the 332


House of the People.
.ne
Reservation of seats for Scheduled Castes and Scheduled
States
Chapter I. Services
308 Interpretation.
333

334
Legislative Assemblies of the States. t
Tribes in the Legislative Assemblies of the States.
Representation of the Anglo-Indian community in the

Reservation of seats and special representation to cease


after sixty years.
309 Recruitment and conditions of service of persons
serving the Union or a State. 335 Claims of Scheduled Castes and Scheduled Tribes to
310 Tenure of office of persons serving the Union or a State. services and posts.
311 Dismissal, removal or reduction in rank of persons 336 Special provision for Anglo-Indian community in
employed in civil capacities under the Union or a State. certain services.
312 All-India services. 337 Special provision with respect to educational grants for
the benefit of Anglo-Indian Community.
312A Power of Parliament to vary or revoke conditions of
service of officers of certain services. 338 National Commission for Scheduled Castes.
313 Transitional provisions. 338 A National Commission for Scheduled Tribes.
314 [Repeated.] 339 Control of the Union over the Administration of
Scheduled Areas and the welfare of Scheduled Tribes.
Chapter II. Public Service Commissions 340 Appointment of  a Commission to investigate the
315 Public Service Commissions for the Union and for the conditions of backward classes.
States. 341 Scheduled Castes.
316 Appointment and term of office of members. 342 Scheduled Tribes.

Downloaded From : www.EasyEngineering.net


Downloaded From : www.EasyEngineering.net
Amendments & Articles of the Indian Constitution P-211

Part XVII: Official Language 364 Special provisions as to major ports and aerodromes.
365 Effect of failure to comply with, or to give effect to,
Chapter I. Language of the Union directions given by the Union.
343 Official language of the Union. 366 Definitions.
344 Commission and Committee of Parliament on official 367 Interpretation.
language.
Part XX: Amendment of the Constitution
Chapter II. Regional Languages 368 Power of Parliament to amend the Constitution and
345 Official language or languages of a State. procedure therefor.
346 Official language for communication between one State
and another or between a State and the Union. Part XXI: Temporary, Transitional and Special
47 Special provision relating to language spoken by a Provisions
section of the population of a State. 369 Temporary power to Parliament to make laws with
respect to certain matters in the State List as if they
Chapter III. Language of the Supreme Court, were matters in the Concurrent List.
High Courts, Etc. 370 Temporary provisions with respect to the State of
348 Language to be used in the Supreme Court and in the Jammu and Kashmir.

ww
High Courts and for Acts, Bills, etc.
349 Special procedure for enactment of certain laws relating
to language.
371 Special provision with respect to the States of
Maharashtra and Gujarat.
371A Special provision with respect to the State of Nagaland.

w.E
Chapter IV. Special Directives
350 Language to be used in representations for redress of
grievances.
371B Special provision with respect to the State of Assam.
371C Special provision with respect to the State of Manipur.
371D Special provisions with respect to the State of Andhra

a
Pradesh.
350A Facilities for instruction in mother-tongue at primary
stage.
syE
350B Special Officer for linguistic minorities.
351 Directive for development of the Hindi language.
371E Establishment of Central University in Andhra Pradesh.
371F Special provisions with respect to the State of Sikkim.
371G Special provision with respect to the State of Mizoram.

Part XVIII: Emergency Provisions


352 Proclamation of Emergency. ngi 371H Special provision with respect to the State of Arunachal
Pradesh.
371-I Special provision with respect to the State of  Goa.
353 Effect of Proclamation of Emergency.
354 Application of provisions relating to distribution of
revenues while a Proclamation of Emergency is in
nee
372 Continuance in force of existing laws and their
adaptation.
372A Power of the President to adapt laws.
operation.
355 Duty of the Union to protect States against external
aggression and internal disturbance.
r
373 Power of President to make order in respect of persons

ing
under preventive detention in certain cases.
374 Provisions as to Judges of the Federal Court and
proceedings pending in the Federal Court or before His
356 Provisions in case of failure of constitutional machinery
in States.
357 Exercise of legislative powers under Proclamation
Majesty in Council.
.ne
375 Courts, authorities and officers to continue to function
issued under article 356.
358 Suspension  of provisions of article 19 during
emergencies.
359 Suspension of the enforcement of the rights conferred
376 Provisions as to Judges of High Courts.
t
subject to the provisions of the Constitution.

377 Provisions as to Comptroller and Auditor-General of


India.
by Part III during emergencies. 378 Provisions as to Public Service Commissions.
359A [Repealed.] 378A Special provision as to duration of Andhra Pradesh
Legislative Assembly.
360 Provisions as to financial emergency.
379-391  [Repealed.]
Part XIX : Miscellaneous 392 Power of the President to remove difficulties.
361 Protection of President and Governors and Rajprakukhs. Part XXII: Short Title, Commencement,
361A Protection of publication of proceedings of Parliament
and State Legislatures.
Authoritative Text
361B Disqualification for appointment on remunerative In Hindi and Repeals
political post.
393 Short title.
362 [Repealed.]
394 Commencement.
363 Bar to interference by courts in disputes arising out of
certain treaties, agreements, etc. 394A Authoritative text in the Hindi language.
363A Recognition granted to Rulers of Indian States to cease 395 Repeals. 
and privy purses to be abolished.

Downloaded From : www.EasyEngineering.net


Downloaded From : www.EasyEngineering.net
P-212 Amendments & Articles of the Indian Constitution

1. The Constitution (43rd Amendment) Act: 9. Constitutional safeguards to civil servants are ensured
(a) ensured Press freedom by:
(b) restore the Supreme Court and the High Courts (a) Article 310 (b) Article 315
the power to consider the constitutional validity of (c) Article 312 (d) Article 311
Central or State laws 10. Which Article of the Indian Constitution provides for the
(c) prescribed serious limitations on the Government’s setting up of the Consolidated Fund?
power to proclaim internal emergency (a) Article 278 (1) (b) Article 283 (1)
(d) removed the right to property from the Constitution (c) Article 267 (1) (d) Article 301 (1)
2. The majority of the provisions of the Indian Constitution 11. Which one of the following Amendments of the
can be amended: Constitution of India deals with the issue of strengthening
(a) by the State Legislature alone of the Panchayati Raj?
(b) the Parliament alone (a) 42nd (b) 44th
(c) with the joint approval of the Parliament and State (c) 73rd (d) 86th

ww Legislatures
(d) only on ratification of half the States
3. Which of the following is not of the methods for amending
12. Which of the following Constitutional Amendments
have added Article 15 (5) in the Constitution of India
providing for reservation in educational institutions in the


w.E
the different categories of provisions in the Constitution?
(a) Certain provisions may be amended by a simple
majority in Parliament
(b) Certain other provisions may be amended only by a


private sector also ?
(a) 81st Amendment (b) 86th Amendment
(c) 9lst Amendment (d) 93rd Amendment


a
two-thirds majority

fourths majority syE


(c) Certain provisions may be amended only by a three-

(d) Certain amendments require to be ratified by one-


13. The Constitution (74th) Amendment Act makes mention



of the
(a) composition of the National Development Council
(b) structure of the Planning Commission of India
half of the States after being passed in Parliament
4. Which one of the following amendments has been
described as Mini Constitution?
ngi

(c) functions of the State Finance Commission
(d) functions of the Kaveri Water Authority
14. Which one of the following amendments to the Indian
(a) Forty-third
(c) Fifty-second
(b) Forty-second
(d) Thirty-ninth nee
Constitution empowers the President to send back any
matter for reconsideration by the Council of Ministers?
(a) 39th (b) 40th

r
5. Which among the following constitutional amendment
(c) 42nd (d) 44th


provided for state reorganisation on linguistic basis?
(a) Third Amendment
(b) Fourth Amendment

ing
15. The 93rd Constitution Amendment deals with the:
(a) continuation of reservation for backward classes in
government employment


(c) Sixth Amendment
(d) Seventh Amendment
6. Which one of the following may be said to constitute the

.ne
(b) free and compulsory education for all children
between the age of 6 and 14 years




basic structure of the Constitution?
(a) Federal character of Constitution
(b) Secular nature of polity
(c) Mandate to build a welfare state


government recruitments
t
(c) reservation of 30 percent posts for women in

(d) allocation of more number of parliamentary seats


for recently created States
16. Which of the following Constitutional Amendments are
(d) all of them
related to raising the number of Members of Lok Sabha
7. One of the following amendments imposed restrictions
to be elected from the States?
on the fundamental rights of the citizens with a view to
(a) 6th and 22nd (b) 13th and 38th
protect the sovereignty and integrity of India. It was the:
(c) 7th and 31st (d) 11th and 42nd
(a) Sixteenth Amendment
(b) Twentieth Amendment 17. The Constitution (98th Amendment) Act is related to:
(c) Fifteenth Amendment (a) empowering the Centre to levy and appropriate
(d) Forty-second Amendment service tax
8. Simple majority in voting is enough to amend provisions (b) the constitution of the National Judicial Commission
relating to: (c) readjustment of electoral constituencies on the basis
(a) citizenship of the population census 2001
(b) creation and abolition of a State (d) the demarcation of new boundaries between States.
(c) administration of Scheduled Castes and Schedules 18. Which of the following Constitution Amendment Acts
Tribes seeks that the size of the Councils of Ministers at the
(d) all the above Centre and in a State must not exceed 15 per cent of the

Downloaded From : www.EasyEngineering.net


Downloaded From : www.EasyEngineering.net
Amendments & Articles of the Indian Constitution P-213

total number of members in the Lok Sabha and the total (c) Substituted entry 33 of Concurrent List of 7th
number of members of the Legislative Assembly of that Schedule
State, respectively? (d) Act 92A was added to the Union List of Seventh
(a) 91st (b) 93rd Schedule
(c) 95th (d) 97th
26. Which of the following Amendment Act of the
19. Which Article mentions disqualification of members in
Constitution imposed certain restrictions on the right to
the Parliament”
property?
(a) Article 101 to Article 104
(a) The Constitution ( Thirty-seventh Amendment) Act,
(b) Article 101 to Articles 105
1975
(c) Article 102 to Article 106
(b) The Constitution ( Thirty-eighth Amendment) Act,
(d) Article 106 to Article 110 1975
20. Which Constitutional Article lays down the reservation (c) The Constitution ( Forty-First Amendment) Act,
of seats for Scheduled Castes and Scheduled Tribes in the 1976
Lok Sabha ?
(d) The Constitution ( Forty-Second Amendment) Act,
(a) Article 330 (b) Article 332 1976
(c) Article 333 (d) Article 334 27. Which of the following is true about the Constitution
21. Which Constitutional Article deals with `Representation (Twenty Furth Amendment) Act, 1971


?
ww
of the Anglo-Indian Community’ with House of the People

(a) Article 334


(a) Parliament has the power to amend any part of the
Constitution including fundamental rights

w.E
(b) The President is bound to assent Constitution
(b) Article 331 amendment bill
(c) Article 332 (c) Education was transferred to the Concurrent List\
(d) Article 333 (d) All the above
22.

a
Which of the following is correct about the Constitution
(First Amendment) Act, 1950
syE
(a) provides new grounds of restrictions to the right to
freedom of speech and expression
28. Which of the following amendment of the Constitution



increased the elective strength of Lok Sabha from 525 to 545?
(a) 31st amendment (b) 30th amendment
(c) 25th amendment (d) 21st amendment

ngi
(b) Provides new grounds of restrictions to the right to
practice any profession or to carry on any trade or
business
29. Which of the following amendment was passed during
the period of emergency?

(c) Inserted two new Articles 31A and 31B and the
Ninth Schedule to give protection from challenge to
land reform laws


nee
(a) 45th amendment (b) 50th amendment
(c) 42nd amendment (d) 47th amendment
30. Which of the following is true about the Constitution

23.

(d) All the above
The Constitution (Sixth Amendment) Act,1956
(a) Articles 269 and 286 were amended regarding taxes


r
(Forty Second Amendment) Act, 1976?

Rights
ing
(a) Precedence to Directive Principles over Fundamental

(b) Fundamental Duties are included



in the course of interstate trade and commerce
(b) 92A was added to the Union List of Seventh

.ne
(c) Constitutional Amendment should not be questioned
in any court


Schedule
(c) (a) and (b)
(d) Inserted two new Articles 31A and 31B and the
Ninth Schedule to give protection from challenge to
land reform laws


(d) All the above

Sabha is increased from 5 to 6 years?


(a) 40th amendment
t
31. In which of the following amendments the term of Lok

(b) 42nd amendment


24. The Constitution (Seventh Amendment) Act, 1956
(c) 44th amendment
(a) Giving effect to the recommendations of the State
(d) 46th amendment
Reorganisation Commission
32. Which of the following Amendment Act of the constitution
(b) Two fold classification of States and Union
deleted the right to property from the list of fundamental
Territories
rights?
(c) (a) and (b)
(a) The Constitution ( Thirty-seventh Amendment) Act,
(d) Act 92A was added to the Union List of Seventh
1975
Schedule
(b) The Constitution ( Thirty-eighth Amendment) Act,
25. The Constitution (Eighth Amendment) Act, 1960
1975
(a) Article 31(2) of Constitution was amended to
(c) The Constitution ( Forty-Fourth Amendment) Act,
provide state power of compulsory acquisition of
1978
private property
(d) The Constitution ( Forty-Second Amendment) Act,
(b) Extenstion of reservations to SC and ST and Anglo
1976
Indians

Downloaded From : www.EasyEngineering.net


Downloaded From : www.EasyEngineering.net
P-214 Amendments & Articles of the Indian Constitution

1. Any conviction for the offence of insulting the


Statement Based MCQ Indian National flag or the Constitution of Indian
shall entail disqualification for contesting elections
1. Which of the following provisions of the Constitution of to Parliament and State Legislatures for six year
India need the ratification by the legislatures of not less from the date of conviction
than one-half of the states to effect amendment? 2. There is an increase in the security deposit which a
1. The manner of election of the President of India. candidate has to make to contest the election to the
2. Extent of the executive power of the Union and the Lok Sabha
states. 3. A candidate cannot now stand for election from
3. Powers of the Supreme Court and High Courts. more than one Parliament Constituency
4. Any of the Lists in the 7th Schedule. 4. No election will now be countermanded on the death
Select the correct answer using the codes given below of a contesting candidate


ww
(a) 1, 2, 3 and 4
(c) 3 and 4
(b) 1, 2 and 3
(d) 1 and 2
2. There are provisions in the Constitution of India which



Which of the above statements are correct?
(a) 2 and 3
(c) 1 and 3
(b) 1, 2 and 4
(d) 1, 2, 3 and 4

w.E
empower the Parliament to modify or annuel the operation
of certain provisions of the Constitution without actually
amending them.
6.

Consider the following statements:
1. An amendment to the Constitution of India can be
initiated by an introduction of a bill in the Lok Sabha


They include
a syE
1. any law made under Article 2 (relating to admission
or establishment of new states)
2. any law made under Article 3 (relating to formation

only.
2. If such an amendment seeks to make changes in the
federal character of the Constitution, the amendment


of new states)
3. amendment of First Schedule and Fourth Schedule. ngi
also requires to be ratified by the legislature of all
the States of India.
Which of the statements given above is/are correct?
Select the correct answer using the codes given


below
(a) 1 and 2 (b) 2 and 3

nee
(a) 1 only
(c) Both 1 and 2
(b) 2 only
(d) Neither 1 nor 2
(c) 2 and 3 (d) None of these
3. Which of the following are matters on which a
constitutional amendment is possible only with the
7.

r
Consider the following provisions-

14 years of age. ing


1. Free & compulsory education to all children below

ratification of the legislature of not less than one-half of


the states?


2. Right to elementary Education.

.ne
3. Ban on employment of children below the age of 14

t
1. Election of the President in hazardous industries & factories.
2. Representation of states in the Parliament Which provision comes under Article 24?
3. Lists in the 7th Schedule (a) 1 & 2 (b) 2 & 3
4. Abolition of the Legislative Council in a State (c) only 3 (d) None of the above
Codes: 8. The National Green Tribunal Act, 2010 was enacted in
(a) 1, 2 and 3 (b) 1, 2 and 4 consonance with which of the following provisions of the
(c) 1, 3 and 4 (d) 2, 3 and 4 Constitution of India?
4. Consider the following statements: (1) Right to healthy environment, construed as a part of
An amendment to the Constitution of India can be Right to life under Article 21.
initiated by the:
(2) Provision of Grants for raising the level of
1. Lok Sabha 2. Rajya Sabha
3. State Legislature 4. President administration in the scheduled Areas for the welfare
Which of the above statements is/are correct? of Scheduled Tribes under Article 275 (1)
(a) Only 1 (b) 1, 2 and 3 (3) Powers & functions of gram sabha as mentioned
(c) 2, 3 and 4 (d) 1 and 2 under Article 243 (A)
5. Consider the following statements about the recent Select the correct answer using the codes given below:
amendments to the elections law by the Representation of (a) 1 only (b) 2 & 3 only
the People (Amendment) Act 1996: (c) 1 & 3 only (d) 1, 2 & 3

Downloaded From : www.EasyEngineering.net


Downloaded From : www.EasyEngineering.net
Amendments & Articles of the Indian Constitution P-215

9. Which of the following is correct statement? (a) A-1, B-2, C-3, D-4
1. The universal Adult franchise is guaranteed under (b) A-2, B-1, C-3, D-4
the Constitution. (c) A-3, B-2, C-1, D-4
2. The 61st Amendment brought the age for voting to (d) A-3, B-1, C-4, D-2
18 years. 12.
Which of the following articles are correctly matched:
(a) (1) only 1. Election Commission – Article 338.
(b) (2) only 2. Finance Commission – Article 280.
(c) 1 and 2 3. National Commission for SCs – Article. 324
(d) Neither 1 nor 2 4. CAG – Article 148.
10. Article 358 & 359 describe the effect of a National 5. Attorney General of India – Article 76.
emergency on the fundamental right which of the (a) All except 1 & 5.
following statements are correct with regard to article (b) All except 1 & 3.
358 & 359. (c) All except 2 & 5.
1. Article 358 operates only in the case of External (d) None of the above options are correct.
13.
Match List-I with List-II and select the correct answer

ww emergency & not in the case of internal emergency.
2. Article 359 operates in case of both External &
Internal emergency.

using the codes given below the lists:
List-I
the Constitution)
List-II (Amendments to


w.E
3. Article 358 suspends FR under Article 19 for the
entire duration of emergency.
4. Article 358 extends to the entire coutry whereas


A. The Constitution 1. Establishment of state
(Sixty-ninth
Amendment)
level Rent Tribunals


it.
(a) 1 & 2 only
a
Article 359 may extend to entire country or part of

syE





Act, 1991
B. The Constitution 2. No reservations for
(Seventy-fifth
Amendment)
Scheduled Castes in
Panchayats in



(b) 1, 2 & 3 only
(c) 2, 3 & 4 only
(d) 1, 2, 3 & 4 ngi


Act. 1994 Arunachal Pradesh
C. The Constitution 3. Constitution of
(Eighteenth Panchayats in Villages
Matching Based MCQ


nee
Amendment)
Act, 2000
or at other local level

D. The Constitution 4. Accepting the


11. Match the following columns
List-I List-II
A. Reorganisation of 1. Seventh Amendment



r
(Eighty-third
Amendment)
Act, 2000 ing recommendations of
the Tenth Finance
Commission
States


B. Sikkim became
22nd State of Indian
2. Thirty-sixth Amendment



5.
.ne
According the status of
National Capital
Territory to Delhi
Union
C. Ninth Schedule
added

3. First Amendment

D. Abolition of Titles 4. Twenty-sixth Amendment


Codes:



(a) A-5, B-1, C-4, D-2
(b) A-1, B-5, C-3, D-4
(c) A-5, B-1, C-3, D-4
t
of Princes (d) A-1, B-5, C-4, D-2

Downloaded From : www.EasyEngineering.net


Downloaded From : www.EasyEngineering.net
P-216 Amendments & Articles of the Indian Constitution

EXERCISE-1 judges to the higher judiciary and for transferring


High Court Judges.
1. (b) 2. (b) 3. (c) 4. (d) 5. (c) 18. (a) The above provision has been added by 91st
6. (d) 7. (a) 8. (d) 9. (d) 10. (c) constitutional Amendment Act, 2003.
11. (c) The 73rd Amendment (1992) of Indian constitution 19. (a) 20. (a) 21. (b) 22. (d) 23. (c)
provided for constitution of municipalities, 24. (c) 25. (b) 26. (d) 27. (d) 28. (a)
reservation of seats in every municipality for the SC 29. (c) 30. (d) 31. (b) 32. (c)
and ST women and backward classes.
12. (d) The reservation in educational institutions in the EXERCISE-2
private sector was provided in the 93rd Amendment 1. (a) Bills that have to be passed by Special Majority and
under the article 15 (5) of the constitution of India. also to be ratified by not less than one-half of the
13. (c) Article 243(I) of the Indian Constitution prescribes State Legislatures . This comprises of Constitutional
that the Governor of a State shall, as soon as may

ww be within one year from the commencement of


the Constitution (73rd Amendment Act, 1992),
and thereafter at the expiration of every fifth year,
Amendment Bills which seek to make any change in
articles relating to:
· The Election of the President.

w.E
constitute a Finance Commission to review the
financial position of the Panchayats.
14. (d) 44th Before the 42nd amendment, Article 74(1)

· The extent of the Executive Power of the Union
and the States.
· The Supreme Court and the High Courts.

a
stated that, “there shall be a Council of Ministers

syE
with the Prime Minister at the head to aid and advise
the President in the exercise of his functions”.
However, there was a slight ambiguity whether



· Any of the Lists in the Seventh Schedule.
· The representation of States in Parliament.
· The provisions of Article 368 itself.
2. (c) Representation of states in the Parliament does
the advice of the Council of Ministers is binding
on the President. Forty-second Amendment of the
ngi require a constitutional amendment only with the
ratification of the legislature of not less than one-
half of the states. Other given options require the
42nd Constitutional Amendment (1976) made it
explicit that the President shall, “act in accordance
with such advice”. The amendment went into nee
same.
3. (d) An amendment to the Constitution of India can
be initiated by either House of Parliament under
effect from 3 January, 1977. The 44th Amendment
(1978) however added that the President can send
r
article 368. It does not require the President’s
recommendation.
ing
the advice back for reconsideration once. But if the 4. (b) Statement 3 is incorrect as a candidate shall not be
Council of Ministers sends the same advice again to
the President then the President must accept it. The
amendment went into effect from 20 June, 1979. 5. (a)
it also adds options 1 and 2.
.ne
allowed to contest from more than two constituencies,

15. (b) According to 93rd Amendment every Child of the 6. (d) An amendment to the constitution of India is
age group of 6-14 years shall have right to free and
compulsory Education. No child is liable to pay any
kind of fee/ capitation fee/ charges. A collection
t
introduced as a bill in the Parliament. It then must
be approved by both the houses of Parliament. The
amendments must then be ratified by the legislatures
of capitation fee invites a fine up to 10 times the of at least one half of the states (not all the states).
amount collected. Once all these stages are complete the amendment is
16. (c) 7th Amendment Act 1956 provides for composition bound to receive the assent of the President of India.
of the House of the People and re-adjustment after 7. (c) 8. (a) 9. (c) 10. (d) 11. (a)
every census. 31st amendment act 1973 provides 12. (a) Article 2- (Admission or establishment of new
for raising the upper limit for the representation of States) Parliament may by law admit into the Union,
states in the Lok Sabha from 500 to 525 and reducing or establish new States on such terms and conditions
the upper limit for the representation of UTs from 25 as it thinks fit.
to 20. Article 3- Formation of new States and alteration of
17. (b) The Constitution 98th Amendment Bill, 2003, areas, boundaries or names of existing States.
seeks to constitute a National Judicial Commission Amendment in the First and Fourth schedule can be
(NJC) by including Chapter IV-A in Part V of the done through constitution amendment bill.
Constitution which will be in charge of appointing 13. (a)

Downloaded From : www.EasyEngineering.net

You might also like